cuestionario de urología

245
CUESTIONARIO DE FAC. ESP. MÉDICO DE FAMILIA – EAP DEL GRUPO PROFESIONAL DE TÉCNICOS SUPERIORES SANITARIOS PARTE 1 de 3 (872 cuestiones) 1: El enfoque biosico-social del modelo de salud y enfermedad se caracteriza por todo lo siguiente menos por: a) Ser un modelo de participación mutua. b) Basarse en la teoría general de sistemas. c) Poner en duda los diagnósticos orgánicos. d) Ser de utilidad para la clínica y la investigación. 2: En la práctica clínica el profesional aplica el modelo biosico-social cuando: a) No distingue entre demanda de salud y necesidad. b) Considera un objetivo a lograr en la relación asistencial que ésta tenga un componente intrínsecamente terapéutico. c) Evita integrar la prevención y promoción de la salud. d) Considera a determinados pacientes como casos cerrados. 3: De las funciones de un centro de salud ¿cuál ocuparía el núcleo principal?: a) Asistenciales curativas, de rehabilitación y de reinserción social. b) Promoción y prevención. c) Docencia e investigación. d) Vigilancia epidemiológica. 4: ¿Cuál de las siguientes etapas debería realizarse previamente al desarrollo de un programa concreto de salud?: a) Determinación de objetivos. b) Identificación de problemas y necesidades. c) Determinación de actividades. d) Movilización y coordinación de recursos. 5: Dentro de las ventajas de la protocolización en atención primaria se encuentra una de las siguientes: a) Normalmente se pueden reflejar todas las situaciones clínicas. b) Permite aumentar la actividad asistencial. c) Facilita la introducción del control de calidad. d) Requiere revisión periódica. 6: Cuando decimos que un determinado procedimiento, servicio o programa sanitario beneficia a las personas que se someten al mismo en condiciones reales de aplicación hablamos de: a) Eficacia. b) Efectividad. c) Equidad. d) Eficiencia. 7: ¿Cuál de las siguientes técnicas de evaluación económica o de eficiencia utilizaríamos para medir el resultado de dos tratamientos en términos de coste por unidad de beneficio ajustado por calidad?: a) Análisis coste-beneficio. b) Análisis coste-utilidad. c) Análisis coste-efectividad. d) Análisis coste-minimización. 8: ¿Cuál de las siguientes estrategias es útil para prevenir o controlar un sesgo de selección de los pacientes en un ensayo clínico?: a) Uso de placebo. b) Uso de técnicas de enmascaramiento (doble ciego). c) Asignación aleatoria. d) Análisis estratificado. 9: ¿Qué tipo de estudio realizará si desea evaluar que la disminución de las cifras de tensión arterial en pacientes hipertensos causa una disminución en la incidencia de enfermedad cardiovascular?: a) Casos y controles. b) Ensayo clínico. c) Cohortes. d) Transversal. 10: Si usted desea comparar dos métodos de determinación de la colesterolemia que utilizan unidades de medida diferentes, ¿cuál de las siguientes medidas de dispersión le permitiría comparar su variabilidad?: a) Desviación típica. b) Coeficiente de variación. c) Desviación media. d) Varianza. 11: En los programas poblacionales de detección precoz del cáncer de mama se recomienda habitualmente: a) Mamografía bianual a partir de los 35 años y anual a partir de los 50 años. b) Mamografía bianual a partir de los 35 años. c) Mamografía bianual a partir de los 50 años. d) Mamografía anual a partir de los 40 años y bianual a partir de los 65 años.

Upload: roundeyes67

Post on 08-Aug-2015

399 views

Category:

Documents


23 download

TRANSCRIPT

Page 1: Cuestionario de Urología

CUESTIONARIO DE FAC. ESP. MÉDICO DE FAMILIA – EAP D EL GRUPO PROFESIONAL DE TÉCNICOS SUPERIORES SANITARIOS

PARTE 1 de 3 (872 cuestiones)

1: El enfoque biosico-social del modelo de salud y enfermedad se caracteriza por todo lo siguiente menos por:

a) Ser un modelo de participación mutua. b) Basarse en la teoría general de sistemas. c) Poner en duda los diagnósticos orgánicos. d) Ser de utilidad para la clínica y la investigación.

2: En la práctica clínica el profesional aplica el modelo biosico-social cuando:

a) No distingue entre demanda de salud y necesidad. b) Considera un objetivo a lograr en la relación asistencial que ésta tenga un componente intrínsecamente terapéutico. c) Evita integrar la prevención y promoción de la salud. d) Considera a determinados pacientes como casos cerrados.

3: De las funciones de un centro de salud ¿cuál ocuparía el núcleo principal?:

a) Asistenciales curativas, de rehabilitación y de reinserción social. b) Promoción y prevención. c) Docencia e investigación. d) Vigilancia epidemiológica.

4: ¿Cuál de las siguientes etapas debería realizarse previamente al desarrollo de un programa concreto de salud?:

a) Determinación de objetivos. b) Identificación de problemas y necesidades. c) Determinación de actividades. d) Movilización y coordinación de recursos.

5: Dentro de las ventajas de la protocolización en atención primaria se encuentra una de las siguientes:

a) Normalmente se pueden reflejar todas las situaciones clínicas. b) Permite aumentar la actividad asistencial. c) Facilita la introducción del control de calidad. d) Requiere revisión periódica.

6: Cuando decimos que un determinado procedimiento, servicio o programa sanitario beneficia a las personas que se someten al mismo en condiciones reales de aplicación hablamos de:

a) Eficacia. b) Efectividad. c) Equidad. d) Eficiencia.

7: ¿Cuál de las siguientes técnicas de evaluación económica o de eficiencia utilizaríamos para medir el resultado de dos tratamientos en términos de coste por unidad de beneficio ajustado por calidad?:

a) Análisis coste-beneficio. b) Análisis coste-utilidad. c) Análisis coste-efectividad. d) Análisis coste-minimización.

8: ¿Cuál de las siguientes estrategias es útil para prevenir o controlar un sesgo de selección de los pacientes en un ensayo clínico?:

a) Uso de placebo. b) Uso de técnicas de enmascaramiento (doble ciego). c) Asignación aleatoria. d) Análisis estratificado.

9: ¿Qué tipo de estudio realizará si desea evaluar que la disminución de las cifras de tensión arterial en pacientes hipertensos causa una disminución en la incidencia de enfermedad cardiovascular?:

a) Casos y controles. b) Ensayo clínico. c) Cohortes. d) Transversal.

10: Si usted desea comparar dos métodos de determinación de la colesterolemia que utilizan unidades de medida diferentes, ¿cuál de las siguientes medidas de dispersión le permitiría comparar su variabilidad?:

a) Desviación típica. b) Coeficiente de variación. c) Desviación media. d) Varianza.

11: En los programas poblacionales de detección precoz del cáncer de mama se recomienda habitualmente:

a) Mamografía bianual a partir de los 35 años y anual a partir de los 50 años. b) Mamografía bianual a partir de los 35 años. c) Mamografía bianual a partir de los 50 años. d) Mamografía anual a partir de los 40 años y bianual a partir de los 65 años.

Page 2: Cuestionario de Urología

12: La densitometría para la detección precoz de osteoporosis está indicada en mujeres:

a) Nulíparas. b) De más de 40 años. c) Ooforectomizadas antes de los 35 años. d) Obesas.

13: La corrección precoz del estrabismo se aconseja por:

a) El alto riesgo de miopía. b) El alto riesgo de ambliopía. c) El riesgo psicomotor por la duplicidad de imágenes. d) La dificultad para conseguir posteriormente una alineación perfecta de los ojos oculares.

14: La profilaxis preexposición al virus de la hepatitis B está indicada en todos los casos, excepto en:

a) Embarazadas. b) Hemofílicos y talasémicos. c) Hijos de madres portadoras. d) Pacientes VIH positivos.

15: ¿Cuál de las siguientes afirmaciones sobre los indicadores de la utilidad de una prueba diagnóstica es falsa?:

a) La sensibilidad no depende de la prevalencia de la enfermedad. b) La especificidad no depende de la prevalencia de la enfermedad. c) El valor predictivo no depende de la prevalencia de la enfermedad. d) La sensibilidad no depende de la gravedad (o diferentes estadios) en que se encuentra la enfermedad.

16: Si una prueba diagnóstica que tiene una sensibilidad del 90 % Y una especificidad también del 90 % se aplica a una población de 200 individuos con una prevalencia de enfermedad del 50 %. ¿Cuál será el valor predictivo positivo?:

a) 90 %. b) 80 %. c) 70 %. d) 60 %.

17: ¿Cuál de los siguientes estudios no permite calcular la incidencia de una enfermedad?:

a) Cohortes prospectivas. b) Cohortes retrospectivas. c) Casos y controles dentro de una cohorte. d) Casos y controles.

18: La medida epidemiológica que indica la probabilidad de que una enfermedad se desarrolle en un grupo de individuos expuesto a un factor de riesgo comparada con la de un grupo no expuesto, se denomina:

a) Incidencia acumulada. b) Densidad de incidencia. c) Fracción atribuible. d) Riesgo relativo.

19: La historia clínica en la atención primaria es una de las herramientas básicas imprescindibles que responde a todas estas características excepto una:

a) Continuidad. b) Integridad. c) Calidad. d) Unidisciplinariedad.

20: Las condiciones que deben establecerse para el funcionamiento correcto de la técnica "lluvia de ideas o brainstorming", como método de detección de problemas, son todas excepto una:

a) Grupo pequeño. b) Tiempo limitado (30-50 minutos). c) Existencia de un facilitador. d) Posibilidad de emitir opiniones.

21: Los cuestionarios de satisfacción de la población miden el elemento:

a) Proceso. b) Estructura. c) Resultado. d) Competencia profesional.

22: En el tratamiento del dolor en pacientes terminales con narcóticos (morfina): ¿cuál de las siguientes afirmaciones es cierta?:

a) Debe prestarse especial atención a la frecuente tolerancia a los narcóticos. b) En el caso de dolor inestable podemos utilizar preparados de acción retardada. c) El antiemético de elección en las náuseas y vómitos por morfina es la metoclopramida. d) La clorpromacina es especialmente útil en caso de náuseas y vómitos con ansiedad asociada.

23: En la atención domiciliaria de pacientes terminales, ante una crisis de claudicación emocional de la familia, la respuesta terapéutica NO se orientará en el sentido de:

a) Revisar los últimos acontecimientos y esclarecer dudas sobre la evolución. b) Cambiar los objetivos terapéuticos. c) Mostrar disponibilidad.

Page 3: Cuestionario de Urología

d) Instaurar medidas de control de síntomas del paciente.

24: En la atención de la agonía de un paciente terminal, ¿cuál de las siguientes afirmaciones NO es correcta?:

a) Conviene prescindir del uso de narcóticos potentes cuando el paciente entre en coma. b) Ante la falta de ingesta, la aplicación de medidas agresivas (sonda nasogástrica, sueros) no mejoraría la situación. c) Se debe adecuar el tratamiento farmacológico, prescindiendo de los fármacos que no tienen una utilidad inmediata. d) la escopolamina es de gran utilidad para tratar la respiración estertorosa.

25: ¿Cuál de los siguientes factores NO contribuye a aumentar el umbral del dolor?:

a) Descanso. b) Aislamiento. c) Tranquilidad. d) Diversión.

26: Los contraceptivos orales tienen como efectos secundarios beneficiosos todos los siguientes excepto uno:

a) Disminución del riesgo de padecer enfermedad inflamatoria pélvica. b) Disminución del riesgo relativo de padecer cáncer de cuello uterino. c) Disminución de riesgo relativo de padecer cáncer de ovario. d) Disminución de la osteoporosis en la postmenopausia.

27: En una mujer postmenopáusica histerectomizada el tratamiento hormonal sustitutivo de elección es:

a) Estrógenos solos. b) Progesterona sola. c) Estrógenos y progesterona. d) Veralipride.

28: Señale cuál de estas alteraciones no aparece en el embarazo:

a) Disminución del hematocrito. b) Disminución de la hemoglobina. c) Aumento de la VSG. d) Disminución del número de hematíes.

29: ¿Qué es típico de la distimia?:

a) Las ideas suicidas reiteradas. b) La irritabilidad. c) La fobia social. d) Su curso breve.

30: ¿En cuál de los siguientes pacientes podríamos prescribir un antidepresivo tricíclico sin temor a incurrir en una contraindicación relativa o interacción medicamentosa?:

a) Hipertenso en tratamiento con metildopa. b) Embarazada. c) Epiléptico en tratamiento con hidantoínas. d) Enfermo diagnosticado de enf. de Parkinson.

31: En la deshabituación de un paciente cocainómano, el fármaco más indicado es:

a) Imipramina. b) Dextropropoxifeno. c) Naltrexona. d) Tetrabamato.

32: ¿Cuál de estos 4 pacientes presenta mayor riesgo de suicidio?:

a) Varón de 35 años, soltero, vive con sus padres. b) Mujer de 75 años, viuda, vive con su hija. c) Varón de 55 años, alcohólico, viudo, vive solo. d) Varón de 30 años, desempleado, aislamiento social.

33: Un paciente que acude a Urgencias presenta taquicardia, palpitaciones, midriasis, hipertermia y cuadro delirante agudo. El diagnóstico más probable es:

a) Intoxicación por heroína. b) Intoxicación por cannabis. c) Intoxicación por benzodiacepinas. d) Intoxicación por cocaína.

34: ¿Cuál de los siguientes cuestionarios no se utiliza en la detección del abuso del alcohol?:

a) CAGE. b) Escala de BLESSED. c) MALT. d) MAST.

35: ¿Qué fase de Korotk:off establece la presión arterial diastólica (PAD)?:

a) Fase 1 de Korotk:off. b) Fase 4 de Korotk:off. c) Fase 5 de Korotk:off. d) Ninguna fase de Korotk:offpermite establecer la PAD.

Page 4: Cuestionario de Urología

36: En el estudio analítico de un paciente hipertenso: ¿Cuál de los datos analíticos siguientes NO es imprescindible en un primer estudio?:

a) Glucemia. b) Colesterol total. c) Triglicéridos. d) Potasemia.

37: En el tratamiento farmacológico de la HTA con inhibidores de la enzima de conversión de la angiotensina: ¿Cuál de los siguientes efectos adversos puede aparecer con mayor frecuencia?:

a) Edema. b) Tos irritativa. c) Sedación. d) Estreñimiento.

38: La localización electrocardiográfica de ondas Q de necrosis en las derivaciones n, ID y aVF es sugestiva de:

a) Infarto agudo de miocardio inferior. b) Infarto agudo de miocardio lateral. c) Infarto agudo de miocardio anterior. d) Infarto agudo de miocardio septal.

39: Para el registro continuo de ECG (Holter): ¿Cuál de las siguientes NO es una indicación?:

a) Sospecha de angina Prinzmetal. b) Evaluación de la terapia antianginosa. c) Arritmias. d) Angina prolongada.

40: En el tratamiento de una angina estable en un paciente con HTA severa y bradicardia, el fármaco de elección es:

a) Verapamilo. b) Propanolol. c) Nifedipino. d) Nitratos de acción prolongada.

41: ¿Cuál de los siguientes criterios diagnósticos es un criterio mayor de insuficiencia cardiaca?:

a) Ingurgitación de las venas yugulares. b) Tos nocturna. c) Edema de los miembros inferiores. d) Hepatomegalia.

42: ¿Cuál de las siguientes circunstancias se asocia con mayor índice de probabilidad a la diabetes o a la intolerancia a la glucosa?:

a) Isquemia crónica en extremidades inferiores. b) Candidiasis vaginal o balanitis. c) Catarata antes de los 50 años. d) Polineuropatía sensitiva distal.

43: ¿Cuál de los siguientes criterios NO es diagnóstico de diabetes?:

a) Glucemia basal (en ayunas) y en sangre venosa igual o superior a 140 mg/dL. b) Glucemia a las 2 horas de ingesta de 75 g. de glucosa igual o superior a 200 mg/dL. c) Presencia de síntomas clínicos y elevación inequívoca de la glucemia. d) Glucemia basal en sangre capilar igual o superior a 120 mg/dL en más de una ocasión.

44: En el tratamiento de la diabetes tipo 2 con sulfonilureas, ¿cuál de los siguientes fármacos potencian la acción hipoglucemiante?:

a) Rifampicina. b) Fenilbutazona. c) Furosemida. d) Estrógenos.

45: Un índice de masa corporal de 28 en una mujer adulta indica:

a) Normopeso. b) Sobrepeso. c) Obesidad. d) Obesidad mórbida.

46: ¿En cuál de las siguientes enfermedades la alteración lipídica principal es un aumento del colesterol total?:

a) Colestasis. b) Consumo excesivo de illcohol. c) Bulimia. d) Insuficiencia renal crónica.

47: En el tratamiento farmacológico de la hipertrigliceridemia aislada: ¿Cuál de los siguientes fármacos está indicado?:

a) Colestiramina. b) Probucol. c) Lovastatina. d) Gernfibrozilo.

Page 5: Cuestionario de Urología

48: En tratamiento farmacológico de la hipercolesterolemia: ¿Cuál de los siguientes fármacos parece ser más eficaz para reducir las concentraciones de LDL-CT?:

a) Colestiramina. b) Gernfibrozilo. c) Probucol. d) Lovastatina.

49: Los pacientes con encefalopatía hepática crónica recurrente en grados I y II deben tratarse con:

a) Dieta rica en proteínas. b) Diuréticos. c) Lactulosa. d) Sodio.

50: Todas las siguientes pueden ser causa de estreñimiento excepto una:

a) Hiperpotasemia. b) Hipotiroidismo. c) Esclerosis múltiple. d) Rectocele.

51: ¿Cuál de los siguientes tratamientos para la erradicación del Helicobacter pylori en el ulcus ofrece un mayor porcentaje de éxito con menor presencia de efectos secundarios?:

a) Bismuto 120mg/6h + Claritromicina 500mg/12h + Tetraciclina 500mg/6h (durante 1 semana). b) Omeprazol20mg/12h + Claritromicina 500mg/12h + Amoxicilina 1g/12h (durante 1 semana). c) Omeprazol20mg/12h + Claritromicina 500mg/12h (durante 1 semana). d) Bismuto 120mg/6h + Metronidazol500mg/8h + Amoxicilina 500mg/6h (durante 1 semana).

52: Son características del reflujo gastro-esofágico todas las siguientes excepto una:

a) Presión disminuida del esfínter esofágico inferior. b) Aumento del vaciado gástrico. c) Hipersensibilidad de la mucosa esofágica a los ácidos o alcalinos. d) El tabaco y el embarazo son factores predisponentes.

53: ¿Cuál de las siguientes afirmaciones es falsa?:

a) La neuralgia del trigémino afecta en la mayoría de los casos a mujeres mayores de 50 años. b) El síndrome de Costen suele requerir tratamiento quirúrgico maxilofacial. c) El tratamiento de elección de la migraña con áurea es el sumatriptan. d) La cefalea tipo "ciuster" mejora con la inhalación de oxígeno al 100 %.

54: ¿Cuál de las siguientes es una característica más específica del vértigo de origen central?:

a) Asociación con acúfenos. b) Supresión del nistagmus con la fijación visual. c) Presencia de nistagmus vertical. d) La maniobra de Barany es positiva.

55: ¿.cuál de los siguientes gérmenes puede producir neuIÍÍ.onías cavitadas con mayor frecuencia?:

a) Staphilococcus aureus. b) Mycoplasma pneumoniae. c) Haemophilus influenzae. d) Virus.

56: La utilización de corticoides en el tratamiento de la LCFA está indicada en:

a) Únicamente en el tratamiento de las reagudizaciones. b) Pacientes menores de 40 años. c) Pacientes que estén bajo cobertura antibiótica. d) En pacientes con LCFA severa.

57: En la sobreinfección respiratoria de los pacientes con EPOC los agentes causales más frecuentes son:

a) Mycoplasma Pneumoniae. b) Pseudomonas aeruginosa y Streptococcus viridans. c) Klebsiella. d) Streptococcus pneumoniae y Haemophilus int1uenzae.

58: Señale cuál de las siguientes afirmacioes es falsa en relación con la EPOC:

a) La determinación de alfa-antitripsina debe practicarse a todos los pacientes de más de 60 años. b) La teofilinemia debe determinarse si no se obtienen los beneficios terapéuticos esperados. c) Los factores pronósticos fundamentales en la EPOC son la FEV 1 y la edad. d) Los síntomas graves de la EPOC aparecen en la mayor parte de pacientes cuando la FEV] es inferior a 1,5 litros.

59: En relación al asma bronquial es falso que:

a) En el asma crónico la sintomatología es prácticamente diaria. b) El asma inducido por el esfuerzo es más frecuente en los adultos que en los niños. c) El hexaclorofeno puede ser causa de asma bronquial ocupacional. d) Entre el 4-20 % de los asmáticos presentan intolerancia al ácido acetilsalicílico.

60: Se considera que una prueba broncodilatadora es positiva cuando:

Page 6: Cuestionario de Urología

a) La FEV1 aumenta entre un 5 y un 10 %. b) La FEV1 aumenta, al menos, en un 15 %. c) La FEV1 aumenta, al menos, en un 20 %. d) La FEV1 aumenta, al menos, en un 30 %.

61: Las lesiones sobreelevadas de la piel y de contenido sólido generalmente inferiores a 1 cm de diámetro se definen como:

a) Máculas. b) Pápulas. c) Nódulos. d) Quistes.

62: La queratosis seborreica se caracteriza por todas excepto una de las siguientes características:

a) Ser una lesión premaligna. b) Tener una coloración que varía de marrón a marrón negruzco. c) Deriva de la epidermis y tiene superficie de características verrugosas. d) Su distribución no se relaciona con las zonas de exposición solar.

63: ¿Cuál de las siguientes afirmaciones no corresponde al melanoma?:

a) Una de sus variedades clínicas es el léntigo maligno melanoma. b) Tiene un claro componente hereditario. c) Las zonas que se asocian con mejor pronóstico son las zonas acras y las superficies mucosas como la vaginal o la oral. d) La determinación de la profundidad de la lesión sirve de referencia en cuanto al pronóstico.

64: El hallazgo radiológico de nódulos de Heberden es característico de:

a) Artritis reumatoidea. b) Enfermedad ósea de Paget. c) Artrosis. d) Mal de Pott.

65: En referencia al tratamiento de la osteoporosis, ¿cuál de las siguientes afirmaciones NO es correcta?:

a) El tratamiento con flúor puede producir efectos secundarios importantes. b) El tratamiento con calcitonina tiene un coste muy elevado. c) Los estrógenos aumentan la incidencia de enfermedad cardiovascular. d) Los difosfonatos inhiben la reabsorción ósea originada por los osteoclastos.

66: Ante un paciente con lumbalgia aguda, ¿cuál de las siguientes acciones NO es correcta?:

a) Analgésicos. b) Electroterapia (onda corta). c) Reposo absoluto. d) Calor local.

67: En referencia a la escoliosis: ¿Cuál de las siguientes afirmaciones NO es correcta?:

a) La prueba clínica demostrativa es la maniobra de la reverencia. b) Sólo una de cada cinco necesita tratamiento ortopédico. c) En el 80 % de los casos es idiopática. d) En el adolescente es más grave que en la infancia (menores de 3 años).

68: ¿Cuál de los siguientes NO es un criterio diagnóstico de la artritis reumatoidea?:

a) Artritis de tres o más áreas articulares. b) Afectación articular asimétrica. c) Rigidez matutina. d) Artritis de las articulaciones de la mano.

69: En el tratamiento de la hiperuricemia: ¿Cuál de los siguientes fármacos está indicado cuando la excreción de ácido úrico es superior a 1 gr./24 horas?:

a) Benzobromarona. b) Indometacina. c) AlopurinoI. d) Colchicina.

70: Una mujer de 20 años presenta en una analítica de rutina los siguientes hallazgos: Hematocrito 37%, VCM 69 fl, HCM 20 pg, CHCM 32 g/dI, recuento reticulocitario 2,5 % Y ferritina de 100 ug/mI.. Nos hallamos ante un caso de:

a) Fase inicial de anemia ferropénica. b) Talasemia. c) Anemia hemolítica. d) Anemia sideroblástica.

71: ¿Cuál de los siguientes procesos NO es causa de anemia megaloblástica?:

a) Embarazo. b) Anticonceptivos orales. c) Hipotiroidismo. d) Gastrectomía.

72: La incontinencia urinaria establecida en el anciano puede ser debida a:

Page 7: Cuestionario de Urología

a) Uretritis atrófica. b) lmpactación fecal. c) Fármacos. d) Todas las anteriores.

73: La exploración más importante para detectar el cáncer de próstata es:

a) La urografía. b) La cistoureterografía. c) El tacto rectal. d) El antígeno específico prostático.

74: Un paciente de 60 años de edad presenta un aclaramiento de creatinina de 40 mlImin. ¿Cuál de los siguientes fármacos no precisa ajuste de posología?:

a) Digoxina. b) Cimetidina. c) Gentamicina. d) Isoniacida.

75: En relación a las diversas formas etiológicas de las demencias, ¿cuál es la más frecuente?:

a) Demencia multiinfarto. b) Demencia de Alzheimer. c) Demencia de tipo mixto (vascular + degenerativo). d) Demencia de tipo alcohólica.

76: Un paciente de 67 años presenta empobrecimiento progresivo del lenguaje, sustracción ocasional de objetos y episodios de desorientación. Este cuadro es compatible con:

a) Senectud. b) Síndrome arnnésico. c) Demencia. d) Delirio.

77: ¿Cuál de las siguientes pruebas de laboratorio puede ser de utilidad en el diagnóstico diferencial de las demencias?:

a) Serología VIH. b) Ácido fólico. c) Serología luética. d) Todas las anteriores.

78: En el fondo de ojo de un paciente afectado de SillA se observa en el cuadrante temporal superior del ojo derecho exudados perivasculares, coloración blanquecina, grisácea o amarillenta de la retina periférica. Lo más probable es que se trate de una:

a) Infección por Toxoplasma gondiL b) Coriorretinitis por Pneumocistis cariniL c) Infección por Aspergilius. d) Coriorretinitis por Citomegalovirus.

79: ¿En qué grupos de población está indicada la prueba de la tuberculina?:

a) Pacientes infectados por el VIR. b) Pacientes gastrectomizados. c) Personas afectadas por neumoconiosis. d) Todos los anteriores.

80: ¿Cuál de las siguientes afirmaciones es falsa respecto a la evolución de los distintos marcadores de la hepatitis B?:

a) El HBeAg indica replicación activa del virus. b) El HBsAc aparece como único marcador en sujetos que han pasado una infección subclínica. c) El HBeAc es detectable en un 70-80 % tras la vacunación. d) El HBsAg puede detectarse antes de las manifestaciones clínicas.

81: En los pacientes VIH positivos ¿Cuál de los siguientes factores no es predictivo de evolución rápida a SIDA?:

a) Anemia. b) V.S.G. elevada. c) Aumento del nivel de beta2-microglobulina. d) Trombocitosis.

82: ¿Cuál de las siguientes afirmaciones en relación a la sífilis es falsa?:

a) Las pruebas no treponémicas se negativizan en un plazo de dos años en un 90 % de los pacientes tratados. b) Las pruebas no treponémicas tienen una alta sensibilidad en la sífilis primaria. c) La diabetes puede producir un falso positivo de las pruebas no treponémicas. d) Las pruebas treponémicas permanecen positivas durante periodos de tiempo prolongados, en ocasiones de por vida.

83: En un paciente aparece el siguiente patrón serológico de la sífilis: VDRL negativo y FfA-Abs positivo. ¿Cuál de las siguientes respuestas es incorrecta?:

a) Reacción falsamente positiva. b) Sífilis primaria precoz. c) Sífilis tardía. d) Enfermedad de Lyme.

Page 8: Cuestionario de Urología

84: ¿Cuál de los siguientes métodos de selección de pruebas de laboratorio sería el más adecuado como sistema de detección?:

a) Perfiles bioquímicos. b) Petición selectiva. c) Perfiles semiológicos. d) Ninguno de los anteriores.

85: De las siguientes tareas a cumplimentar en la parte exploratoria de la entrevista clínica una no es correcta:

a) Orientar y clarificar la exploración física. b) Elaborar y sintetizar la información. c) Delimitar y clarificar la demanda. d) Compartir con el paciente un plan de actuación.

86: En la parte resolutiva de la entrevista, al transmitir la información obtenida se pretende:

a) Orientar al paciente sobre la naturaleza del problema. b) Estimular el cumplimiento del tratamiento. c) Verificar,la comprensión de las instrucciones dadas. d) Comprehder las expectativas del paciente y estimular su pariticipación.

87: En relación a la clasificación de la OMS del ciclo vital de la familia es cierto que:

a) Distingue 4 etapas. b) Distingue 5 etapas. c) La primera etapa abarca el periodo que corresponde a los dos primeros años del matrimonio. d) La segunda etapa o de extensión finaliza con el nacimiento del último hijo.

88: ¿Qué técnica no es indispensable de cara a iniciar una reanimación cardiopulmonar básica ante un paciente que presenta una parada cardiaca?:

a) Mantener las vías aéreas permeables. b) Iniciar las maniobras de compresión torácica. c) Conocer las cifras de tensión arteria!. d) Controlar el pulso.

89: En un paciente que presenta una asistolia, una vez iniciadas las maniobras de reanimación cardiopulmonar, ¿Qué medidas debemos utilizar en primer lugar?:

a) Establecer una vía intravenosa y administrar lidocaina mg/kg de peso en bolus. b) Epinefrina 1:10000,0,5-1,0 mg. J.V. en bo1us. c) Solución de Bicarbonato 1 mEq/kg de peso vía J.V. d) Colocación de un marcapasos.

90: Un paciente es traído a Urgencias en estado de coma. ¿ Cuál de las siguientes medidas farmacológicas puede ser útil?:

a) Tiamina vía J.V. b) Naloxona vía J.V. c) Flumazenil vía I.V. d) Todas las anteriores.

91: Paciente de 30 años que desde hace dos horas presenta cuadro de inicio brusco de vértigo intenso, con sudoración y náuseas, sin pérdida de audición ni acúfenos. El diagnóstico más probable es:

a) Vértigo posicional benigno. b) Neuronitis vestibular. c) Enfermedad de Méniere. d) Neurinoma del acústico.

92: Si al realizar la prueba de Weber en el estudio de una hipoacusia del oído derecho, el sonido se desplaza hacia el oído izquierdo (el de mejor audición), es indicativo de:

a) Hipoacusia de transmisión en el oído izquierdo. b) Hipoacusia de transmisión en el oído derecho. c) Hipoacusia neurosensorial en el oído izquierdo. d) Hipoacusia neurosensorial en el oído derecho.

93: Una otorrea intermitente y fétida, sin otalgia, es sugestiva de:

a) Otitis externa maligna. b) Colesteatoma. c) Otomicosis por Aspergillus niger. d) Otomicosis por Candida albicans.

94: A un paciente que presenta en ojo derecho secreción -matutina, sensación de cuerpo extraño y enrojecimiento de la conjuntiva tarsal1e prescribiremos un colirio antibiótico:

a) Sólo en el ojo afecto de molestias. b) En ambos ojos. c) En ambos a días alternos. d) En el ojo afecto junto con colirio antifúngico.

95: Un paciente varón de 54 años, diabético de seis años de evolución, miope, y con antecedente familiar de un hermano con glaucoma crónico simple (G.C.S.):

a) Tiene riesgo de G.C.S. si ya tiene retinopatía intraretinal. ' b) Tiene el riesgo correspondiente al grupo de edad.

Page 9: Cuestionario de Urología

c) Tiene un riesgo aumentado de G.C.S. d) Tiene un riesgo aumentado sólo si es insulinodependiente.

96: Ante un paciente que, tras exposición a los rayos ultravioleta (esquí), presenta dolor ocular, fotofobia, blefarospasmo, lagrimeo intenso y enrojecimiento bilateral, el diagnóstico de presunción es:

a) Conjuntivitis bacteriana. b) Uveítis fótica. c) Queratitis actínica. d) Crisis glaucomatocíclica.

97: ¿Cuál de las siguientes circunstancias se asocia a una disminución de la globulina transportadora de la tiroxina?:

a) Cirrosis hepática. b) Anticonceptivos orales. c) Embarazo. d) Metadona.

98: ¿En cuál de las siguientes situaciones es preferible utilizar la técnica de radiología con contraste a la endoscopia?:

a) Evaluación de cicatrización de una úlcera duodenal. b) En los trastornos de la motilidad. c) En pacientes gastrectomizados ante la sospecha de úlcera o neoplasia. d) En el diagnóstico de úlcera gástrica.

99: Señale cual de las siguientes afirmaciones es falsa en relación a la medición del flujo espiratorio máximo:

a) Se realiza mediante el peak flow meter. b) Permite valorar el grado de respuesta terapéutica. c) Ayuda en el seguimiento y control del grado de obstrucción bronquial del paciente en su ámbito diario de forma secuencial. d) No tiene correlación con el FEV1.

100: ¿Cuál de las siguientes exploraciones diagnósticas realizaríamos en primer lugar ante la aparición bruca de ictericia clínica en un paciente de 65 años?:

a) Radiografía simple de abdomen. b) Co1ecistografía. c) Ecografía abdominal. d) Colestitografía.

101: Dentro de los diferentes tipos de planificación, la planificación estratégica se caracteriza por:

a) Perseguir grandes objetivos del sistema como la equidad. b) Asociarse a una organización hospitalaria pero no a la de un centro de atención primaria. c) Tener como producto en esta etapa al programa. d) Formalizar un plan director.

102: La proporción de individuos de una población que tiene una enfermedad en un momento dado de tiempo se denomina:

a) Incidencia acumulada. b) Densidad de incidencia. c) Fracción atribuible. d) Prevalencia.

103: ¿Cuál de los siguientes fármacos no es útil en la situación de la agonía?:

a) Morfina. b) Escopolamina. c) Midazolam. d) Dexametasona.

104: ¿A partir de que cifra debe ser considerada patológica la tensión arterial en una embarazada en el tercer trimestre?:

a) A partir de 120170. b) A partir de 130/80. c) A partir de 140/90. d) A partir de 160/110.

105: ¿Cuál de los siguientes procesos se asocian a depresión?:

a) Hipotiroidismo. b) Tratamiento con corticoides. c) Abstinencia por anfetaminas. d) Todos los anteriores.

106: En referencia a los factores de riesgo de enfermedad cerebrovascular (ECV): ¿Cuál de las siguientes afirmaciones NO es cierta?:

a) El nivel alto de ácido úrico en sangre se relaciona con un mayor riesgo de ECV. b) El alcoholismo es un factor de riesgo de ECV tanto la ingestión crónica como la intoxicación aguda. c) La diabetes mellitus incrementa la posibilidad de ECV, en especial isquémica. d) Las mujeres están algo más expuestas que los varones.

107: ¿Cuál de los siguientes criterios diagnósticos de bulimia, NO lo es por sí solo?:

a) Episodios recurrentes de consumo rápido de gran cantidad de alimentos. b) Humor depresivo y pensamientos autodespreciativos tras los accesos.

Page 10: Cuestionario de Urología

c) Accesos de consumo sin testigos. d) Temor a los accesos y conciencia de padecer un trastorno del comportamiento.

108: ¿Cuál de las siguientes situaciones NO es un criterio de ingreso hospitalario en un paciente que presenta una neumonía extrahospitalaria?:

a) Alcoholismo. b) Falta de sostén familiar. c) Mala respuesta a las 48 h. de iniciado el tratamiento. d) Tratamiento antibiótico previo en los últimos 15 días.

109: El hallazgo radiológico de una sacroilitis en un paciente con poliartritis, es sugestivo de:

a) Artropatía psoriásica. b) Artritis reumatoidea. c) Artropatía asociada a enfermedad inflamatoria intestinal. d) Espondilitis anquilosante.

110: Un paciente acude a su consulta con el siguiente patrón serológico de marcadores de hepatitis B : HBsAg positivo, anti-HBs negativo, anti-HBc de la clase Ig G positivo, HBeAg positivo, anti-HBe negativo. Usted pensará que se trata de:

a) Infección aguda con alta infectividad. b) Inmunización contra el virus de la hepatitis B. c) Infección crónica con alta infectividad. d) Infección aguda con baja infectividad.

111: La incidencia de una enfermedad es:

a) La suma de todos los casos existentes en esa enfermedad. b) Los casos de una enfermedad con una antigüedad superior a 5 años. c) Los casos nuevos de la enfermedad. d) Los casos de la enfermedad relacionados entre sí.

112: En una prueba diagnóstica es importante determinar:

a) La sensibilidad. b) La especificidad. c) El valor predictivo positivo. d) Todas son ciertas.

113: El riesgo relativo se utiliza en los estudios:

a) De cohorte prospectiva. b) De casos-control. c) Transversales. d) De prevalencia.

114: El trastorno del sueño en la depresión endógena se caracteriza por todos los síntomas siguientes excepto:

a) Sueño total disminuido. b) Escasa eficacia del sueño. c) Densidad REM disminuida. d) Latencia REM disminuida.

115: En el síndrome de abstinencia alcohólica todos los síntomas siguientes excepto uno son indicativo de gravedad:

a) Agitación psicomotriz. b) CK superior a 700. c) Alucinaciones visuales persistentes. d) Amnesia lacunar nocturna.

116: La causa cardiaca más frecuente de síncope es:

a) Trastornos del ritmo. b) Mixomade aurícula derecha. c) Estenosis aórtica. d) Estenosis pulmonar.

117: ¿Qué le sugiere un paciente con antecedentes de reumatismo poliarticular agudo, síncope de esfuerzo, angina, disnea y soplo de eyección en área aórtica?:

a) Miocardiopatía Dilatada. b) Infarto de miocardio. c) Miocardiopatía hipertrófica obstructiva. d) Estenosis aórtica valvular.

118: En un paciente que presenta por primera vez historia de angina típica de 45 minutos de duración y ECG inespecífico, ¿qué actitud es correcta?:

a) Enviarle a domicilio y repetir dentro de unos días un ECG. b) Realizar determinaciones enzimáticas y electrocardiogramas seriados. c) Considerar que no es infarto agudo al no tener cambios en el ECG. d) Ninguna de las respuestas anteriores es correcta.

119: La úlcera gástrica se diferencia de la úlcera duodenal en que:

a) Cicatriza mejor.

Page 11: Cuestionario de Urología

b) Precisa la asociación de fármacos para su curación. c) Tiene mayor mortalidad. d) Ninguna de las anteriores es correcta.

120: Del adenocarcinoma gástrico podemos afirmar:

a) Ha disminuido su incidencia e índice de mortalidad en los últimos 60 años. b) Las clases socioeconómicas débiles tienen más riesgo de padecerlo. c) La ingestión a largo plazo de altas concentraciones de nitratos, aumenta el riesgo de sufrirlo. d) Todas las anteriores son verdaderas.

121: Con respecto a la colitis ulcerosa, señale qué afirmación es falsa:

a) Suele recidivar tras la colectomía. b) Puede malignizar en caso de larga evolución. c) Cursa con diarrea y con dolor abdominal. d) Puede ocasionar un megacolon tóxico.

122: La depresión en el anciano puede plantear problemas de diagnóstico diferencial con la demencia. Indique cual de los siguientes datos apoya más la sospecha de que se trata de un síndrome depresivo:

a) Test de supresión por dexametasona positivo. b) Presencia de insomnio, apatía y anhedonía. c) Buena respuesta al tratamiento con antidepresivos. d) Inicio solapado.

123: En la enfermedad coronaria en el anciano (señale la respuesta falsa):

a) Por encima de los 70 años su incidencia es igual en ambos sexos. b) La disnea constituye una forma típica de presentación. c) El dolor es de reposo con mayor frecuencia que en el joven. d) Los bloqueadores beta son el fármaco más adecuado para el tratamiento del dolor anginoso.

124: En la enfermedad de Alzheimer es verdad que:

a) Predominan los trastornos afectivos de tipo depresión. b) El comienzo y progreso de la enfermedad son rápidos. c) Hay trastornos de la memoria y del lenguaje. d) Es un proceso distinto de la demencia senil primaria.

125: La enfermedad de Graves:

a) Afecta más al varón. b) Se presenta con hipertiroidismo con bocio difuso, oftalmopatía y dermaatopatía. c) Se presenta con hipertiroidismo sin bocio difuso, oftalmopatía y dermaatopatía. d) Los antitiroideos la bloquean y evitan las recibidas.

126: El bocio simple o no tóxico:

a) Si es endémico hay agrandamiento generalizado o localizado del tiroides en más del 1 0% de la población. b) No siempre conlleva agrandamiento de la glándula. c) La concentración sérica de TSH está alterada. d) No tiene causa identificable.

127: El hipotiroidismo:

a) Obedece más a causa supratiroidea (hipófisis, hipotálamo) que tifoidea. b) Siempre se acompaña de bocio. c) El cretinismo ya se puede apreciar desde el parto. d) Síntomas precoces en el adulto son: letargias, caída del vello, diarrea, sensación de calor, aumento del peso.

128: En el paciente con anemia hipocroma microcítica, las principales posibilidades diagnósticas son:

a) Deficiencia de hierro, talasemia, anemia por inflamación crónica y anemia sideroblástica. b) Deficiencia de hierro, talasemia, anemia de la uremia y anemia sideroblástica. c) Deficiencia de hierro, anemia drepanocítica, anemia de la uremia y anemia sideroblástica. d) Deficiencia de hierro, talasemia, anemia por inflamación crónica y anemia de la uremia.

129: Señale la frase incorrecta:

a) Los alcohólicos presentan a menudo deficiencia de folatos. b) El déficit de folatos ocasiona la anemia perniciosa. c) Junto al déficit de folatos y cobalarnina, la causa más frecuente de anemia megaloblástica es la ingestión de fármacos. d) La anemia megaloblástica aguda responde al tratamiento con folatos más cobalamina a dosis terapéuticas habituales.

130: Entre las causas no infecciosas de la anemia por inflamación crónica, la más frecuente es:

a) Artritis reumatoide. b) Enteritis regional. c) Tumores. d) Lupus eritematoso sistémico.

131: ¿Cuál de estas anormalidades en los resultados analíticos puede considerarse más específica de la enfermedad de los legionarios?:

a) Hiperpotasemia. b) Hipocalcemia.

Page 12: Cuestionario de Urología

c) Leucopenia. d) Hiponatremia.

132: Con respecto a la intubación traqueal, señale la afirmaciones errónea:

a) La inserción del tubo endotraqueal en un bronquio causa hipoxemia. b) La cabeza del paciente debe sobresalir de la cama o mesa. c) El tiempo máximo disponible para completar la maniobra es de 30 segundos. d) Proporciona una ruta para la administración de fármacos durante la resucitación cardiopulmonar.

133: Con respecto a la desfibrilación, señale la afirmación errónea:

a) La aplicación de descargas iniciales mayores a 200 julios no resulta especialmente beneficiosa. b) La aplicación de material conductor en las palas no mejora el resultado. c) Los choques desfibriladores aplicados previamente reducen la resistencia transtorácica. d) La distancia entre palas influye en la resistencia transtorácica.

134: En la canalización de la vena subclavia, señale la afirmación errónea:

a) El hematoma en un lado del cuerpo invita a no intentar la canalización en el lado opuesto. b) El neumotórax es una complicación frecuente. c) La cabeza y el cuello han de girar hacia el lado contrario de la punción. d) Haremos Rx Tórax P A sólo en caso de duda.

135: Respecto a las benzodiacepinas:

a) Se absorben mejor por vía intramuscular que por vía oral. b) No pasan a la leche materna. c) Las de vida media más corta inducen más fácilmente tolerancia. d) Todas ellas se metabolizan en el sistema microsomaI hepático.

136: De las siguientes acciones atribuibles a los diuréticos una es falsa:

a) Aumentan el volumen de eyección. b) Disminuyen las resistencias periféricas. c) Potencian el efecto de otros fármacos hipotensores. d) Disminuyen el volumen extracelular.

137: Para acercamos al objetivo de una dieta saludable. ¿Cuál de éstas le parece más idónea?:

a) Carbohidratos 70%, grasas 10%, proteínas 20%, sal 3 gIs/día. b) Carbohidratos 50%, grasas 30%, proteínas 20%, sal 5 gIs/día. c) Carbohidratos 60%, grasas 20%, proteínas 30%, sal 1 gr/día. d) Carbohidratos 55%, grasas 30%, proteínas 15%, sal 3 grs/día.

138: Respecto a la diabetes mellitus, señale la afirmación correcta:

a) En los adultos el factor de riesgo más importante es la obesidad. b) La malnutrición proteica puede causarla. c) El riesgo de cardiopatía coronaria es 2-3 veces mayor en los diabéticos. d) Todas las anteriores son correctas.

139: Señale cuál no es una contraindicación absoluta para realizar ejercicio físico:

a) Tromboflebitis. b) Enfermedad infecciosa aguda. c) Prolapso mitral. d) Insuficiencia cardiaca congestiva.

140: Dentro de la atención geriátrica:

a) El objetivo principal del Hospital de Día es conseguir que el anciano viva en su domicilio. b) Las residencias mixtas deben su nombre a que acogen a ancianos de ambos sexos. c) La evaluación y el tratamiento han de ser multidisciplinares. ' d) A Y C son ciertas.

141: Los determinantes principales de los problemas de salud por orden creciente de importancia son:

a) Biología humana, medio ambiente, estilo de vida. b) Estilo de vida, biología humana, medio ambiente. c) Biología humana, estilo de vida, medio ambiente. d) Medio ambiente, biología humana, estilo de vida.

142: Con respecto al clínico orientado al modelo biopsicosocial, ¿cuál de las siguientes no es cierta?:

a) Distingue entre demencia y necesidad de salud. b) Evita localizarse hacia lo biológico o lo psicosocial al elaborar hipótesis diagnósticas. c) Considera a determinados pacientes como "casos cerrados". d) Debe conocer los factores de riesgo de cada consultante, integrando la prevención y la promoción de la salud.

143: El trabajo en equipo tiene una serie de ventajas respecto al realizado de forma aislada. Señala una de ellas:

a) La actividad desarrollada por el conjunto es mayor que la suma de las individuales. b) Los miembros del equipo tienen un mayor grado de satisfacción personal. c) El trabajo en equipo facilita y potencia el desarrollo de actividades de educación para la salud. d) Todas las anteriores.

Page 13: Cuestionario de Urología

144: En el proceso de gestión, para el desarrollo nacional de la salud, la evaluación de la eficacia consiste en:

a) Analizar los resultados de una intervención en condiciones ideales de aplicación. b) Comprobar el impacto o los resultados de un programa o actividad y su coste de producción. c) Comprobar que se han realizado adecuadamente las fases previas de la planificación (identificación de problemas, priorización y establecimientos de objetivos y actividades). d) Comprobar las prestaciones efectivas y las que se habían previsto.

145: En relación a los servicios sanitarios que se prestan habitualmente en los centros de salud. ¿Cuál de los siguientes aspectos no puede ser evaluado?:

a) La eficacia. b) La efectividad. c) La eficiencia. d) La accesibilidad.

146: Si la variable "colesterol total" (mg/dl) sigue una distribución normal de µ 170 mgr/dl y δ de 10 mgr/dl, es decir DN (170; 10), entre qué valores de colesterol total, alrededor de la media, esperarías encontrar al 95% de la población:

a) Entre 150 mg/dl y 190 mg/dl. b) Entre 160 mg/dl y 200 mg/dl. c) Entre 160 mg/dl y 180 mg/dl. d) Ninguna es la correcta.

147: Con respecto a la especificidad de una prueba diagnóstica, señale cuál de los siguientes enunciados es el correcto:

a) Se define como la probabilidad de obtener una prueba negativa en personas sin la enfermedad. b) Su probabilidad complementaria sería la proporción de personas que estando enfermas, la prueba resulta negativa. c) Si su valor es del 90%, la tasa de falsos positivos es del 10%. d) Las respuestas a y c son correctas.

148: ¿Cuáles son las características esenciales que debe reunir la investigación en AP.S.?:

a) Poseer rigor metodológico. b) Analizar el contexto donde se desarrolla. c) Incorporar el análisis y la evaluación de las intervenciones y los servicios de salud. d) Todas las anteriores.

149: ¿Qué se entiende por eficiencia?:

a) La relación entre el impacto actual de un servicio en un sistema operativo y su impacto potencial en una situación ideal. b) La relación entre el impacto actual de un servicio o programa y su coste de producción. c) Es el nivel de aplicación en la práctica de los conocimientos médicos y tecnológicos disponibles. d) Ninguna de las anteriores.

150: Al medir los resultados en salud, ¿en qué criterios podemos influir inmediatamente con nuestras modificaciones y conducir más directamente a la acción correctora?:

a) Criterios de estructura. b) Criterios de Proceso. c) Criterios de Resultado. d) En todas por igual.

151: ¿Qué es la fiabilidad?:

a) La capacidad de que un instrumento de medida mida de manera precisa y reproducible lo que realmente mide. b) La capacidad de un instrumento de medida para detectar diferencias en la variable examinada. c) La capacidad de un instrumento de medida de medir lo que se supone que tiene que medir. d) Ninguna de las anteriores.

152: ¿Cuál de las siguientes afirmaciones es falsa?:

a) Para los profesionales sanitarios la Calidad Científico Técnica es el aspecto más importante de la calidad. b) Para los consumidores lo más importante es la efectividad. c) Para las autoridades sanitarias la eficiencia. d) Para los consumidores la adecuación es el aspecto más importante de la calidad.

153: ¿En cuál de los siguientes casos está absolutamente contraindicada la colocación de un dispositivo intrauterino?:

a) Nuliparidad. b) Alergia al cobre. c) Tratamiento anticoagulante. d) Anemia ferropénica.

154: ¿Cuál de las siguientes afirmaciones es falsa en un embarazo normal?:

a) Los requerimientos de hierro aumentan desde 0,8 mg. día al principio de la gestación hasta 10-12 mg al día en el último mes. b) La O.M.S. aconseja suplementar con 0,3-1 mg. al día de ácido fólico a todas las embarazadas y antes de producirse la concepción. c) Las dietas muy hipocalóricas o poco fraccionadas están contraindicadas por el peligro de cetonemia. d) Las embarazadas aumentan su peso entre 300 y 400 gramos semanales desde el primer mes de gestación.

155: La principal utilidad del registro de edad y sexo es:

a) Conocer los datos de filiación y las características demográficas básicas (edad, sexo) de la población asignada al Equipo de Atención Primaria.

Page 14: Cuestionario de Urología

b) Conocer cuáles son las enfermedades o problemas de Salud por los que acude la población al Centro de Salud. c) Permite la cuantificación global de las actividades desarrolladas. d) Permite elaborar indicadores de rendimiento y de utilización de los recursos asistenciales disponibles.

156: ¿Cuál de estas secuencias forma parte de la parte resolutiva de la entrevista semiestructurada?:

a) Enunciación del problema. b) Propuesta de acuerdo. c) Comprobación. d) Todas las anteriores.

157: ¿Cuál de las siguientes es una desventaja de las entrevistas semiestructuradas?:

a) Permite adaptar las técnicas de entrevista más útiles a cada situación asistencial, estructurándolas de forma más lógica. b) Aseguran un nivel mínimo de calidad, compartido por el conjunto del equipo. c) Una entrevista semiestructurada puede convertirse en estructurada si el profesional la aplica de manera mecanicista. d) Casi siempre conllevan la ampliación de técnicas de comunicación respecto a las desarrolladas espontáneamente por los profesionales antes de su implantación.

158: ¿Qué se entiende por silencio funcional?: .

a) El entrevistador permite un lapsus de tiempo entre la intervención del paciente y la suya. b) Conducta verbal o no verbal que ayuda a iniciar y proseguir el relato sin indicar ni sugerir contenidos. c) Ausencia de comunicación verbal que tiene por efecto proporcionar un tiempo de meditación al paciente, ayuda a su concentración, o actúan como catalizador de determinadas reacciones emocionales en el curso de la entrevista. d) Conducta verbal o no verbal que ayuda a iniciar o proseguir el relato sin indicar ni sugerir contenidos.

159: Para comprender el impacto de la enfermedad en la familia hay que conocer el ciclo evolutivo de la enfermedad. ¿Qué variables lo integran?:

a) Debut. b) Pronóstico. c) Grado de incapacidad. d) Todas las anteriores.

160: ¿Cuál de las siguientes no es una finalidad común de las conferencias familiares?:

a) Apoyar el desarrollo normal de la familia y sus miembros, sobre todo el periodo de cambio. b) Promover hábitos saludables y modificación en el estilo de vida. c) Aumentar las exploraciones y las pruebas complementarias. d) Movilizar los recursos familiares para facilitar la adaptación en enfermedades y problemas de salud de sus miembros.

161: ¿Cuál de los siguientes aspectos no se contempla en un genograma?:

a) Ciclo vital familiar. b) Experiencias vitales. c) Enfermedades agudas por las que ha acudido a la consulta. d) Equilibrios o desequilibrios familiares.

162: En relación con las pruebas diagnósticas de las que se dispone en la actualidad, es cierto que:

a) En general han sido validadas sobre población atendida fuera de los hospitales. b) La prevalencia de un proceso es el factor detenninante de los valores predictivos. c) En pacientes ingresados aumentan los falsos negativos. d) Para procesos de mayor gravedad se prefieren pruebas con una alta sensibilidad, aunque tengan poca especificidad.

163: La técnica de elección en el estudio inicial de una masa abdominal palpable en epigastrio es:

a) El tránsito esofagogastroduodenal. b) La ultrasonografía con contraste doble. c) La ecografía simple. d) La radiología simple de abdomen en doble proyección.

164: ¿Cuál de las siguientes es falsa con respecto a los tumores de las glándulas salivares mayores?:

a) La incidencia de los tumores de mayor malignidad aumenta por encima de los 60 años. b) En la parótida, casi el 70 % de los tumores son benignos. c) Casi el 80 % de los pacientes que presentan carcinomas epidermoides tienen más de 60 años. d) La edad avanzada es un factor de buen pronóstico.

165: La gingivitis marginal crónica.

a) Es un proceso infrecuente relacionado con infecciones micóticas. b) Suele aparecer en niños con inmunodeficiencias. c) Con el tiempo progresa y puede provocar enfermedad periodontal. d) Una vez establecida, y aun en fases iniciales, debe emplearse una combinación de imidazoles y penicilinas para su tratamiento.

166: ¿Cuál de los siguientes movimientos no nos sirve para explorar la función motora del nervio facial?:

a) Fruncir la frente elevando las cejas. b) Inflar los carrillos. c) Sacar la lengua hacia fuera de la boca y moverla de un lado a otro. d) Enseñar los dientes.

167: ¿Cuál de los siguientes patrones de la marcha es característico de la ataxia cerebelosa?:

Page 15: Cuestionario de Urología

a) Piernas separadas, cargando el peso alternativamente sobre una y otra como un pato. Suele existir lordosis y un abdomen prominente. b) Piernas muy separadas, se elevan mucho y caen bruscamente a cada paso. El talón deja huella en el suelo. c) Los pies del paciente tienen una amplia base de sustentación. Los escalonamientos y sacudidas hacia uno y otro lado se acompañan del balanceo del tronco. d) Movimientos de baile, en sacudidas sin dirección aparente.

168: Al explorar el nivel de conciencia, ¿qué se considera delirio?:

a) Confusión con percepción desordenada y disminución de la atención. Respuesta inadecuada a los estímulos. b) Escasa capacidad de atención y memoria. c) No estar despierto, ni alerta. d) Puede despertarse durante periodos cortos ante estímulos visuales, verbales o dolorosos.

169: ¿Cuál de las siguientes es falsa en la fiebre de origen desconocido?:

a) La mortalidad es más alta en los pacientes de mayor edad. b) En los niños la etiología más probable son las infecciones y la artritis crónica juvenil. c) En los ancianos las causas más comunes son leucemia aguda, enfermedad de Hodgkin, infecciones intraabdominales, tuberculosis y arteritis de la temporal. d) El pronóstico es peor en los niños.

170: ¿Cuál de los siguientes atributos no es sugestivo de enfermedad infecciosa aguda?:

a) Inicio repentino. b) Fiebre alta con escalofríos o sin ellos. c) Disuria, polaquiria, y dolor en flancos. d) Episodios febriles comprobados en los últimos dos meses sin encontrar la causa.

171: ¿En qué grupo de pacientes con fiebre sin causa conocida, debemos sospechar fiebre facticia?:

a) Ancianos. b) Mujeres jóvenes trabajadoras en el área de salud. c) Varones de mediana edad. d) Adolescentes.

172: Señala la correcta con relación a la enfermedad artrósica.

a) El dolor es el primer síntoma de la enfermedad. b) El dolor es producido por la degeneración del cartílago articular. . c) Los osteofitos son siempre expresión de Artrosis. d) Los nódulos de Bouchard se localizan en las.' articulaciones interfalángicas proximales.

173: Los síndromes de origen lumbar tienen una frecuencia alta en las consultas de atención primaria, clínicamente el dolor se puede presentar de forma aguda, subaguda o crónica. ¿Cuál de las siguientes respuestas no es correcta?:

a) La lumbalgia subaguda es aquella que tiene una evolución de menos de dos semanas. b) La lumbalgia crónica tiene una evolución clínica de más de tres meses. c) La actitud ante una lumbalgia aguda inicialmente es el reposo, calor y AINE Y si no mejora realizar unas radiografías y analítica. d) La mejoría del corsé en las lumbalgias crónicas es como consecuencia de una reducción de la presión sobre el disco.

174: La osteoporosis involutiva es la más frecuente. En un varón anciano de 80 años que padece osteoporosis, ¿qué diríamos?:

a) Que padece una osteoporosis involutiva tipo l. b) Que las fracturas óseas preferentemente las localizaríamos en los cuerpos vertebrales. c) En este tipo de osteoporosis se afecta principalmente el hueso trabecular. d) Suele ser frecuente encontrar una concentración baja de 25-(OH)-D.

175: ¿Cuál de las siguientes dermatomicosis es producida por levaduras?:

a) Tiña versicolor. b) Tiña pedis. c) Tiña cruris. d) Tiña barbae.

176: En relación al epitelioma basocelular, ¿cuál es la incorrecta?:

a) No se desarrolla en las mucosas. b) No ocasiona metástasis. c) Puede ser pigmentado. d) Es siempre único.

177: Con relación a la psoriasis:

a) La afectación de las uñas es muy poco frecuente. b) La asociación con artritis es aproximadamente del 65 %. c) La forma más frecuente de artritis psoriásica es la oligoarticular. d) La ciclosporina se utiliza en las formas moderadas.

178: Con relación a la determinación del colesterol. sérico como medida de cribaje en la población adulta, ¿qué pauta recomendaría?:

a) Existen claras evidencias de recomendar un cribaje a los mayores de 65 años al menos en una ocasión. b) El consenso para el control de la colesterolemia en España aconseja su determinación al menos una vez tanto a varones como mujeres entre los 35 - 75 años.

Page 16: Cuestionario de Urología

c) Sólo determinaría el colesterol en pacientes de riesgo, aunque trabajara en una zona con suficientes recursos. d) Las evidencias hablan a favor de una determinación de hipercolesterolemia plenamente justificada en los varones en edad media de la vida.

179: El médico de atención primaria juega un papel fundamental en la detección de forma precoz del deterioro cognitivo mediante la sospecha clínica y los test mentales como el mini examen cognitivo, para posteriormente realizar una anamnesis más profunda, una exploración neurológica y otras exploraciones:

a) Analítica que incluya hormonas tiroideas, vitamina B 12, Serología Luética. b) La Analítica anterior y EEG mediante la derivación a Neurología. c) Las pruebas anteriores y un TAC Cerebral, a través de interconsulta con el Neurólogo. d) Las pruebas anteriores informando de la conveniencia de una RMN a todos los enfermos con deterioro cognitivo.

180: En relación al diagnóstico reumatológico por radiología simple, ¿qué señalarías como lo más correcto?:

a) La localización de los síntomas indicados por el paciente son una clara referencia topo gráfica para la solicitud de radiografías. b) En reumatología una radiografía normal, descarta patología. c) En articulaciones periféricas es obligado practicar una radiografía comparativa del lado sano. d) Ante toda lumbalgia aguda es preciso en un estudio inicial una radiografía en dos proyecciones de la zona afecta.

181: Las incontinencias de orina en el anciano representan el síntoma urológico más frecuente. Las incontinencias urinarias pueden ser reversibles o establecidas. Señala la respuesta correcta, con relación a las incontinencias establecidas.

a) La incontinencia establecida más frecuente es la originada por los esfuerzos. b) La incontinencia de esfuerzo es muy frecuente en ancianos con hiperplasia benigna de próstata. c) La inestabilidad vesical es la incontinencia establecida más frecuente en los ancianos. d) Las incontinencias funcionales son originadas por lesiones en la función de los esfínteres.

182: ¿Cuál sería su actitud ante una mujer joven que no mejora a los 4-5 días de instaurarle tratamiento para una infección de orina con un antibiótico que se ha elegido sobre la base de un antibiograrna?:

a) Insistir con el mismo antibiótico hasta que desaparezcan los síntomas. b) Cambiar de inmediato a un antibiótico diferente. c) Realizar un nuevo cultivo de orina. d) Determinaría de urgencia los valores séricos de creatinina y urea.

183: Una de estas medidas no estaría protocolizada en una valoración inicial en un paciente que presente síntomas prostáticos:

a) Cuantificación de los síntomas mediante la escala I-PSS. b) Tacto rectal. c) Análisis de orina mediante tira reactiva o sedimento. d) Determinar el Antígeno Prostático Específico (PSA).

184: Con relación al diagnóstico diferencial de las cefaleas, ¿cuál es cierta?:

a). La anamnesis tiene un escaso valor en este tipo de patología. b) Los escotomas centelleantes son propios de la migraña sin aura. c) En las cefaleas de reciente aparición, el NIH de Estados Unidos recomienda de forma sistemática el CT. d) En la migraña el dolor de cabeza desaparece al dormir.

185: El test de Tinetti se utiliza para la valoración de las posibles causas de caídas en los ancianos:

a) Es un test que explora la visión. b) Valora las Actividades de la Vida Diaria Básica. c) Nos da información sobre el equilibrio estático y dinámico. d) También se le llama get-up and go.

186: El tratamiento preventivo del asma bronquial, con corticoides inhalados está indicado cuando:

a) Existen síntomas que obligan al empleo frecuente de broncodilatadores. b) Se presentan crisis de asma moderadas o graves desencadenadas por estímulos banales. c) Los pacientes presentan escasos síntomas, pero los estudios espirométricos demuestran obstrucción bronquial. d) Todas las anteriores son ciertas.

187: ¿Qué germen causal de neumonías extrahospitalarias, produce más frecuentemente necrosis y abcesos pulmonares?:

a) Klebsiela pneumoniae. b) Streptococo pneumoniae. c) Mycoplasma pneumoniae. d) Legionella pneumophila.

188: En el tratamiento de la disnea en un paciente con cáncer en fase terminal están indicados:

a) Metilprednisolona y Salbutamol. b) Morfina y Diazepan. c) Oxigenoterapia y Salbutamol. d) Oxigenoterapia y Metilprednisolona.

189: En pacientes terminales con pronóstico de vida superior al mes, estaría indicado tratar la caquexia con:

a) Acetato de Megestrol. b) Dexametasona. c) Complejos vitamínicos y minerales. d) Metoclopramida.

190: El síndrome de Meniere se caracteriza por:

Page 17: Cuestionario de Urología

a) Vértigo, acúfenos e hipoacusia. b) Vértigo, acúfenos yotalgia. c) Vértigo, otorrea e hipoacusia. d) Vértigo, otorrea y otalgia.

191: ¿Cuál de los siguientes gérmenes raramente es agente causal de las otitis medias?:

a) Streptococus pneumoniae. b) Haemophilus influenzae. c) Branhamella catarralis. d) Klebsie!a 'pneumoniae.

192: La presencia de un enantema de úlceras, vesículas y pápulas maculares sobre el paladar blando o faringe, sugieren:

a) Faringitis streptococica del grupo A. b) Escarlatina. c) Faringitis por enterovirus. d) Mononucleosis infecciosa.

193: Ante un ojo rojo muy doloroso, con baja o nula agudeza visual, pupila en midriasis, cámara anterior estrechada y abolición de la transparencia de la córnea, debemos pensar en:

a) Queratitis parenquimatosa. b) Episcleritis. c) Glaucoma agudo. d) Iritis.

194: Ante un ojo rojo, doloroso, con fotofobia, sensación de cuerpo extraño, disminución de la agudeza visual y blefarospasmo, debemos:

a) Instilar un colirio con corticoides. b) Instilar un colirio de Fenilefrina al5 %. c) Instilar un colirio con Atropina. d) Instilar un colirio con Fluoresceína.

195: La conjuntivitis de inclusión del recién nacido es causada por:

a) Neisseria gonorrhoeae. b) Chlamydia trachomatis. c) Escherichia coli. d) Staphylococus aureus.

196: Ante un paciente con un cuadro febril de comportamiento y evolución fulminante, ¿cuál de estas bacterias y situaciones le parece más congruente?:

a) Brucella en paciente previamente sano. b) Micobacterium tuberculosis en alcohólico. c) Streptococcus pneumoniae en esplenectomizado. d) Herpes zóster en sujeto con enfermedad de Hodgkin.

197: ¿Cuál de los siguientes profesionales no forma parte del núcleo básico de E.A.P.?:

a) Médicos. b) Trabajadores sociales. c) Técnicos no sanitarios. d) Especialistas en Salud Pública y Medicina Comunitaria.

198: En un varón de 85 años, con isquemia crónica grado IIa, por estenosis crítica de la ilíaca común derecha, realizaremos:

a) Bypass aortobifemoral. b) Bypass iliofemoral derecho. c) Bypass femorofemoral. d) Tratamiento médico.

199: El apoyo precoz en las fracturas de cadera del anciano depende fundamentalmente:

a) Del sexo. b) Del tipo de osteosíntesis. c) De la edad del paciente. d) Del estado mental.

200: ¿Qué indica en el diagnóstico de edema agudo de pulmón la ausencia de opacidades alveolares en la radiografía de tórax?:

a) Que es producido por un neumotórax unilateral. b) Que es de causa urémica. c) Que es por sobredosis de heroína. d) Descarta de entrada el diagnóstico de edema agudo de pulmón.

201: Ante un paciente con dolor precordial que se modifica con la respiración y con los movimientos, la existencia de roce en la auscultación y alteraciones en el ECG debe pensarse:

a) Infarto de miocardio. b) Embolismo. c) Pericarditis. d) Miocarditis.

Page 18: Cuestionario de Urología

202: La primera medida ante un infarto agudo de miocardio es:

a) Colocar un marcapaso. b) Dar lidocaína en presencia de bradiarritmias. c) Dar atropina en presencia de taquiarritmias. d) Traslado urgente a una unidad coronaria.

203: La elevación mantenida del segmento ST al cabo de tres semanas de un infarto de miocardio sugiere:

a) Espasmo coronario mantenido. b) Aneurisma ventricular. c) Es una respuesta normal. d) Ninguna de las respuestas anteriores es correcta.

204: El síntoma más frecuente en el tromboembolismo pulmonar recurrente es:

a) Dolor torácico. b) Hemoptisis. c) Disnea de esfuerzo. d) Ninguna de las respuestas anteriores es correcta.

205: El enfisema centroacinar se asocia fundamentalmente a:

a) Déficit de alfa-l-antitripsina. b) Mucoviscidosis. c) Tabaco. d) Asbesto.

206: La mejor vía de administración de los estimulantes Beta-2-Adrenérgico es la:

a) Oral. b) Intramuscular. c) Intravenosa. d) Aerosol.

207: Un estado socioeconómico bajo se asocia constantemente a consecuencias sanitarias adversas. De las cinco posibilidades que se le muestran, diga cuál constituye una EXCEPCION a esta regla:

a) Cáncer de mama. b) Diabetes Mellitus. c) Enfermedad Pulmonar Obstructiva Crónica. d) Infección Respiratoria Aguda.

208: Un dolor lumbar persistente en un paciente anciano, rebelde al tratamiento habitual, que se acompaña de anemia y afectación del estado general, nos debe 'orientar hacia la presunción diagnóstica de:

a) Aplastamiento vertebral por osteoporosis. b) Linfoma no Hodgkin del anciano. c) Degeneración discal múltiple. d) Mieloma múltiple.

209: ¿Cuál de estos factores no incrementa el riesgo de padecer un cáncer de mama?

a) Menarquia tardía, después de los 12 años. b) Hipercolesterolemia. c) Enfermedad fibroquística de la mama. d) Afectación de familiares de primer grado por la enfermedad.

210: Algunos autores recomiendan la realización periódica de una prueba dada su sensibilidad para la detección de recidivas tumorales en el cáncer de colon intervenido, dicha prueba es:

a) Ecografía transrectal. b) Tomografía axial computarizada (TAC). c) Colonoscopia cada 3 meses. d) Determinación de los niveles plasmáticos del antígeno carcinoembrionario (CEA).

211: En un paciente oncológico en situación terminal que presenta vómitos inducidos por estimulación central (tratamiento con opiáceos), la pauta de actuación más CORRECTA será:

a) Domperidona intravenosa. b) Reducir la dosis de opiáceos. c) Metoclorpramida intravenosa. d) Haloperidol vía oral.

212: En relación al tratamiento de las depresiones con fármaco s antidepresivos, señale la respuesta INCORRECTA:

a) Sus efectos indeseables suelen presentarse antes que la mejoría. b) Su reducción debe ser rápida para evitar la dependencia. c) Una vez conseguida la mejoría se deben mantener semanas o meses. d) En cuadros ansiosos se pueden asociar ansiolíticos.

213: Uno de los siguientes síntomas no se presenta de forma típica en la crisis de angustia:

a) Tristeza. b) Disnea. c) Mareo. d) Palpitaciones.

Page 19: Cuestionario de Urología

214: ¿Cuál de los siguientes no es un criterio para el diagnóstico de la anorexia nerviosa?:

a) Miedo intenso a engordar. b) Alteraciones de la imagen corporal. c) Pérdida de peso de al menos un 10 % del original. d) Negativa a mantener el peso corporal por encima del mínimo normal según la edad y la talla.

215: Uno de los siguientes síntomas no se da en el síndrome de abstinencia a opiáceos, señálelo:

a) Miosis. b) Disnea. c) Diarrea. d) Rinorrea.

216: En los trastornos obsesivo-compulsivos, las obsesiones tienen las siguientes características EXCEPTO una, señálela:

a) Son reconocidas como producto de la propia mente. b) Son interpretadas como impuestas desde el exterior. c) Se experimentan como invasoras y sin sentido. d) Existe el intento de neutralizar los pensamientos con alguna idea o acción.

217: En el caso de hepatopatía la Benzodiacepina indicada es:

a) Diazepán. b) Cloracepato. c) Clordiacepóxido. d) Oxazepán.

218: El primer paso a dar para realizar un abordaje estructurado de la atención familiar es:

a) La Atención Longitudinal. b) La revisión del ciclo vital de la familia. c) El estudio de la estructura familiar. d) Evaluar la función familiar.

219: En el contexto de la entrevista clínica, todos son recursos para aumentar el cumplimiento terapéutico, EXCEPTO uno, señálelo:

a) Dar instrucciones por escrito. b) Involucrar al paciente en la terapia. c) Reconvertir ideas negativas. d) Culpabilizar al paciente de las consecuencias si no sigue los tratamientos prescritos.

220: Una de las técnicas para ayudar a los pacientes a elaborar percepciones, emociones o ideas, consiste en ponerles de manifiesto sus emociones o conductas, y se llama:

a) Interpretación. b) Antagonismo. c) Clarificación. d) Señalamiento.

221: El fármaco betabloqueante Propanolol tiene las indicaciones siguientes, EXCEPTO una. Señálela:

a) Migraña. b) Temblor esencial. c) Insuficiencia cardíaca. d) Miocardiopatía Hipertrófica.

222: En relación con el uso de corticoides, una de las observaciones siguientes es FALSA, señálela:

a) La rifampicina disminuye el efecto de los corticoides. b) El uso de acetazolamida y corticoides puede dar lugar a hiperpotasemia. c) Los corticoides pueden alterar la respuesta a los anticoagulantes orales. d) Los corticoides pueden aumentar el aclaramiento de teofilinas.

223: ¿Con cuál de las siguientes asociaciones es más probable que aparezca un bloqueo aurículo ventricular?

a) Digoxina y procainamida. b) Digoxina y enalapril. c) Digoxina y fenitoína. d) Digoxina y diltiazem.

224: Los siguientes grupos farmacológicos pueden producir retención urinaria en el anciano, EXCEPTO uno, señálelo:

a) Bloqueantes alfa-adrenérgicos. b) Antihistarnínicos. c) Neurolépticos. d) Descongestionantes nasales.

225: ¿Qué fármaco de los que se citan, tiene una absorción aumentada en presencia de comida?

a) Amoxicilina. b) Eritromicina base. c) Isoniacida. d) Nitrofurantoína.

226: Los siguientes fármacos son antidepresivos tricíclicos, EXCEPTO uno, señálelo:

Page 20: Cuestionario de Urología

a) Imipramina. b) Amitriptilina. c) Clomipramina. d) Maprotilina.

227: Una opacidad radiológica sobrepuesta al reborde cardíaco sin difuminarlo puede hallarse localizada en:

a) Lóbulo medio. b) Língula. c) Lóbulo superior izquierdo. d) Lóbulo inferior izquierdo.

228: ¿Cuál es el tratamiento del asma leve?

a) Betamiméticos en pauta fija por vía inhalatoria. b) Glucocorticoides inhalados. c) Bromuro de ipratropio inhalado. d) Betamiméticos a demanda por vía inhalatoria cuando se presenten síntomas.

229: En la enfermedad pulmonar obstructiva crónica con predominio de la bronquitis, uno de los rasgos siguientes no es característico, señálelo:

a) La expectoración es abundante y purulenta. b) Aumento, de imágenes bronquiales, corazón grande. c) Disnea grave. d) Cor pulmonale frecuente.

230: Todas las afinnaciones siguientes acerca de la auscultación pulmonar son correctas, EXCEPTO una, señálela:

a) Las sibilancias se oyen preferentemente en espiración. b) Los estertores crepitantes se oyen en inspiración y no se modifican con la tos y respiración profunda. c) Auscultar un soplo tubárico excluye la existencia de neumonía. d) Los roces pleurales se oyen preferentemente en inspiración y no se modifican con la tos.

231: El síntoma más frecuente de la embolia pulmonar es:

a) Disnea inexplicable de aparición brusca. b) Dolor pleurítico y hemoptisis. c) Intenso dolor retroestemal. d) Episodios repetidos de taquiarritmias supraventriculares inexplicables.

232: Las siguientes enfermedades presentan en la radiografía de tórax un patrón alveolar, EXCEPTO una. Señálela:

a) Hemorragia Pulmonar. b) Neumoconiosis. c) Edema Pulmonar. d) Infarto Pulmonar.

233: Todas las siguientes son causas de fiebre y condensación pulmonar no infecciosa, EXCEPTO una, señálela:

a) Edema pulmonar cardiogénico. b) Hamartoma pulmonar. c) Neumonitis por irradiación. d) Neoplasia pulrnonar.

234: ¿Cuál de los siguientes síntomas no es propio del síndrome de apnea obstructiva durante el sueño?:

a) Ronquido. b) Cefalea. c) Convulsiones. d) Anemia.

235: Si un enfermo presenta presión venosa elevada que no disminuye con la inspiración, hepatomegalia, ascitis, ruidos cardíacos apagados, sin soplos, disminución de la presión arterial con la inspiración y corazón de tamaño normal, se deberá pensar en:

a). Cirrosis hepática. b) Estenosis aórtica calcificada. c) Estenosis pulmonar. d) Constricción pericárdica.

236: La isquemia subepicárdica se manifiesta en el electrocardiograma por:

a) Ondas T altas y picudas. b) Segmento ST ascendido. c) Ondas Q de voltaje superior al 25 % de la onda R. d) Ondas T negativas o aplanadas y simétricas.

237: En una persona que presenta fiebre inexplicable y un soplo cardíaco, que duran más de una semana, debe sospecharse siempre:

a) Endocarditis infecciosa. b) Amigdalitis estreptocócica. c) Tromboembolismo pulmonar. d) Infección urinaria.

238: Sólo uno de los siguientes grupos de fármacos utilizados en el tratamiento de la hipertensión arterial, ha demostrado

Page 21: Cuestionario de Urología

disminución de la mortalidad. Señálelo:

a) Antagonistas del calcio. b) Inhibidores de la enzima conversora de la angiotensina. c) Alfabloqueantes. d) Diuréticos.

239: ¿Cuál de las siguientes medidas tiene mayor eficacia en la prevención primaria de la cardiopatía isquémica?

a) Disminuir el nivel de colesterolemia. b) Suspender el hábito tabáquico. c) Tratar la hipertensión arterial. d) Mantener la nonnoglucemia en pacientes diabéticos.

240: Con respecto a la expresión electrocardiográfica de un infarto de miocardio, una de estas afirmaciones es FALSA, señálela:

a) El infarto agudo posterior se caracteriza por una onda R y elevación de ST en VI y V2. b) En el infarto subendocárdico el segmento ST está deprimido. c) Una onda Q mayor o igual a 0,04 seg. es patológica. d) En presencia de bloqueo de rama izquierda (BRI) generalmente no se puede hacer el diagnóstico de infarto en el ECG.

241: Todas las siguientes afirmaciones acerca de la Estenosis Mitral son ciertas, EXCEPTO una, señálela:

a) En el pulso venoso yugular se pueden observar ondas "a" prominentes. b) Se ausculta: 10 tono fuerte, chasquido de apertura y soplo diastólico. c) La existencia de un soplo sistólico I-II/VI en la punta significa que hay insuficiencia mitral agregada. d) En general, la duración del soplo diastólico (retumbo diastólico) se correlaciona con la gravedad de la estenosis.

242: Respecto a la hipertensión maligna o acelerada, una de las siguientes afirmaciones es FALSA, señálela:

a) Clínicamente se caracteriza por un aumento relativamente brusco de la presión arterial. b) La presión arterial diastólica, se sitúa con frecuencia entre 130 rnmHg Y 170 rnmHg. c) En el laboratorio se encuentra una anemia hemolítica microangiopática. d) Para su diagnóstico es necesaria la existencia de exudados retinianos y edema de papila.

243: ¿Cuál de estas características no es propia del síndrome X o angina microvascular?

a) Dolor anginoso de esfuerzo. b) Se trata de una afección de arteriolas coronarias. c) El tratamiento más eficaz es el tratamiento preventivo. d) La coronariografía siempre es anormal.

244: Todas las afirmaciones siguientes acerca de la parálisis de la neurona motora superior son ciertas, EXCEPTO una, señálela:

a) El reflejo plantar es extensor. b) El electromiograma revela fibrilaciones. c) Existe hiperactividad de los reflejos tendinosos. d) La atrofia muscular es leve.

245: Todas las afirmaciones siguientes acerca de la exploración neurológica son ciertas, EXCEPTO una, señálela:

a) El reflejo comeal es conducido por la rama oftálmica sensitiva del V par. b) El reflejo mandibular es conducido por la porción motora del V par. c) La parálisis del XI par provoca parálisis del músculo trapecio. d) La parálisis del VI par produce parálisis de la addución del ojo y estrabismo divergente.

246: Una persona que presenta de 1 a 3 ataques de dolor periorbitario muy agudo, de carácter explosivo, de corta duración, en un período de 4 a 8 semanas con recurrencias anuales, acompañado de lagrimeo, enrojecimiento ocular, congestión nasal y ptosis palpebral en el mismo lado y nauseas, lo más probable es que tenga:

a) Crisis de Migraña. b) Cefalea tensional. c) Hemorragia subaracnoidea. d) Cefalea en racimos.

247: Todas las siguientes son manifestaciones típicas de la crisis de ausencia o pequeño mal, EXCEPTO una, señálela:

a) Se inician casi siempre entre los 6 y 14 años. b) El EEG es patognomónico. c) Cursan, por lo general, sin actividad muscular convulsiva ni pérdida del control postural' d) Habitualmente se produce un período de confusión postictal.

248: El dolor irradiado a todo el brazo por la región posterior del hombro, cara anterior del brazo y borde radial del antebrazo hasta el dedo pulgar, es característica de:

a) Radiculopatía C3-C4 b) Radiculopatía C5 c) Radiculopatía C6 d) Radiculopatía C7

249: En una persona con una hepatitis aguda que presenta los siguientes marcadores: HBsAg +; IgM antiVHA +; IgM antiHBc +; AntiVHC -; el diagnóstico etiológico que estableceríamos sería de:

a) Hepatitis por virus B. b) Hepatitis por virus A. c) Hepatitis aguda por virus A y B. d) Hepatitis aguda por virus C.

Page 22: Cuestionario de Urología

250: ¿Cuál de las siguientes patologías no tiene relación con un mayor riesgo de padecer cáncer colorrectal?

a) Colitis ulcerosa. b) Pólipos adenomatosos colorrectales. c) Poliposis familiar del colon. d) Diverticulosis colorrectal.

251: En relación al síndrome de Reye, señale la respuesta FALSA:

a) Es más frecuente en la infancia. b) Puede precipitarse con la administración de salicilatos. c) La bilirrubina suele estar elevada. d) Se inicia típicamente con vómitos repentinos e incoercibles tras un proceso vírico.

252: En relación con la hepatitis C, una de las siguientes afirmaciones es FALSA, señálela:

a) El virus tiene una semejanza genómica con los de la hepatitis B y D. b) Por encima del 50 % se relaciona con el uso de drogas IV. c) La fuente de infección en muchos pacientes es incierta. d) Puede ser un factor patogénico en la crioglobulinemia.

253: Señale cuál de los siguientes no está establecido como fármaco causante de pancreatitis aguda:

a) Acido valproico. b) Furosemida. c) Digoxina. d) Estrógenos (anticonceptivos orales).

254: El diagnóstico de colecistitis enfisematosa suele hacerse fácilmente por:

a) Ecografía. b) Resonancia magnética. c) Colangiopancreatografía retrógrada endoscópica. d) Radiografía simple de abdomen.

255: La colitis pseudomembranosa, se trata con:

a) Antidiarreicos. b) Penicilina G. c) Vancomicina. d) Clindamicina oral.

256: De las siguientes afirmaciones, señale aquella que sea INCORRECTA:

a) El Helicobacter pylori se asocia a más del 90 % de las úlceras duodenales. b) La colonización gástrica por Helicobacter pylori va disminuyendo con la edad. c) En ensayos terapéuticos, la erradicación del Helicobacter pylori se asocia a cicatrización de la úlcera duodenal. d) La colonización del Helicobacter pylori en el duodeno se restringe a las áreas de metaplasia gástrica.

257: En relación al cuadro clínico de ulcus duodenal, es FALSO que:

a) El dolor epigástrico es el síntoma más frecuente. b) Muchos pacientes con úlcera duodenal activa no tienen síntomas ulcerosos. c) La sensibilidad epigástrica es el hallazgo anormal más frecuente en la exploración física. d) La hemorragia y la obstrucción son más frecuentes en las úlceras bulbares que en las postbulbares.

258: Es característico del hipotiroidismo subclínico todo lo siguiente, EXCEPTO:

a) Cursa generalmente de forma asintomática. b) Se detectan niveles elevados de TSH (hormona tirotropa). c) Se detectan niveles descendidos de hormonas tiroideas. d) Es más frecuente en el sexo femenino.

259: La acidosis láctica en los diabéticos se puede asociar al uso de uno de los siguientes tratamientos, señálelo:

a) Sulfonilureas. b) Acarbosa. c) Fenforrnina. d) Insulina de acción rápida.

260: En la enfermedad de Addison, todas estas manifestaciones son ciertas, EXCEPTO una, señálela:

a) Aumento de peso. b) Disminución del gasto cardíaco. c) Disminución del flujo vascular renal. d) Astenia.

261: ¿Cuál de las siguientes hiperlipemias es la más frecuente en nuestro medio?:

a) Dislipoproteinemia familiar. b) Hipercolesterolemia familiar. c) Hipertrlgliceridemia familiar. d) Hipercolesterolemia poligénica.

262: ¿Cuál de estos fármacos potencian la acción hipoglucemiante de las sulfonilureas?:

a) Indometacina.

Page 23: Cuestionario de Urología

b) Salicilatos. c) Rifampicina. d) Adrenalina.

263: En relación con la tiroiditis subaguda, todas las afirmaciones siguientes son correctas, EXCEPTO una, señálela:

a) V.S.G. elevada. b) La captación tifoidea de iodo radioactivo es alta. c) El síntoma más frecuente es el dolor local o referido. d) Puede producir tirotoxicosis.

264: En relación con el hallazgo de hipercalcemia, todas las afirmaciones siguientes son correctas, EXCEPTO una, señálela:

a) En pacientes ambulatorios el hiperparatiroidismo primario es la causa más frecuente. b) En caso de hipoalbuminemia sérica, la cifra de calcemia debe ser corregida. c) Suele producir síntomas cuando la calcemia es superior a 12 mg/dl. d) En el ECG puede producir un alargamiento del intervalo QT.

265: En un niño de 7 años de edad, cuya talla se sitúa más de dos desviaciones estándar por debajo de la media en relación con su edad, sexo y grupo de población, y presenta una edad ósea retrasada con respecto a la cronológica, deberemos de pensar en las siguientes entidades, EXCEPTO una, señálela:

a) Retraso constitucional del crecimiento. b) Déficit de hormona de crecimiento (GH). c) Hipocrecimiento secundario a enfermedades crónicas. d) Talla baja familiar.

266: La causa más frecuente de síndrome de Cushing es:

a) Adenoma suprarrenal. b) Carcinoma suprarrenal. c) Hiperplasia suprarrenal secundaria a sobreproducción hipofisaria de ACTH. d) Administración yatrogénica de esteroides.

267: En relación a la pubertad precoz, señale la respuesta CORRECTA:

a) Se define por la aparición de los primeros caracteres sexuales antes de los 8 años en las niñas y 9 en los niños. b) Produce una talla alta en la edad adulta. c) Es más frecuente en los niños. d) El 90 % de los casos tiene una causa conocida.

268: Paciente de 45 años que acude a urgencias por presentar desde hace 24 horas dolor en ojo derecho, con fotofobia, lacrimeo, disminución de la agudeza visual y a la exploración discreta miosis e hiperemia ciliar del ojo afecto. Se debe pensar en:

a) Glaucoma agudo en ángulo abierto. b) Glaucoma agudo de ángulo cerrado. c) Iritis aguda. d) Conjuntivitis aguda.

269: La causa neoplásica más frecuente de anosmia es:

a) Glioma del lóbulo frontal. b) Adenoma hipofisario. c) Meningioma de la región frontal inferior. d) Craneofaringioma.

270: El tratamiento de corticoides tópicos está indicado en:

a) Cataratas. b) Glaucoma. c) Ulceras herpéticas. d) Iritis.

271: Respecto a las sorderas, ¿qué proposición es FALSA?:

a) En la sordera de transmisión, la prueba de Rinné es positiva patológica. b) En la sordera de transmisión, en la prueba de Weber, la vibración resuena más en el lado afecto. c) En la sordera de percepción, están alteradas la conducción aérea y ósea. d) En la sordera de percepción, la conducción aérea es mejor que la ósea.

272: ¿Cuál de los siguientes fármacos produce una queratopatía verticillata?:

a) Vitamina D. b) Digital. c) Amiodarona. d) Clorarnfenicol.

273: Una de las siguientes es causa de eritrocitosis secundaria, señálela:

a) Ingesta de aspirina. b) Administración de anticoagulantes. c) Policitemia rubra vera. d) Intoxicación crónica por monóxido de carbono debido al consumo de tabaco.

274: En el diagnóstico diferencial de una anemia microcítica hipocroma, no debe incluirse una de las siguientes anemias. Señálela:

a) Anemia ferropénica.

Page 24: Cuestionario de Urología

b) Anemia de las enfermedades inflamatorias crónicas. c) Anemia sideroblástica. d) Anemia por gastrectornía total.

275: ¿En cuál de las siguientes zonas del cuerpo no valoraría clínicamente la concentración de hemoglobina en la sangre?:

a) Conjuntivas. b) Encías. c) Lecho ungueal. d) Paladar.

276: En una trombosis venosa en terapia con anticoagulantes orales dicumarínicos, ¿qué valores de la INR (razón normalizada interuacional), se consideran recomendados?

a) 1,5/2,0 b) 2,0/3,0 c) 2,5/3,5 d) 3,0/4,5

277: En las anemias secundarias a trastornos crónicos todas las afirmaciones siguientes son ciertas, EXCEPTO una, señálela:

a) Los niveles de sideremia son inferiores a lo normal. b) La ferritina sérica muestra valores inferiores a lo normal. c) Se clasifican normalmente como anemias normocíticas y normocrómicas. d) El recuento reticulocitario es normal o bajo.

278: El inicio abrupto de hematuria, proteinuria con síntomas de hiperazoemia y retención de sal yagua, es propio de:

a) Síndrome nefrótico. b) Litiasis renal complicada. c) Síndrome nefrítico agudo. d) Infección urinaria.

279: La composición más habitual de los cálculos productores de litiasis renal es:

a) Ácido úrico, b) Fosfato amónico. c) Cistina. d) Oxalato cálcico.

280: ¿Cuál de las siguientes afirmaciones no es importante para el diagnóstico de la insuficiencia renal crónica?:

a) Hiperazoemia de más de 3 meses. b) Síntomas y signos prolongados de uremia. c) Síntomas o signos de osteodistrofia renal. d) Proteinuria y Piuria.

281: ¿Cuál de las afirmaciones siguientes es FALSA?:

a) En el individuo sano la orina contiene men9$ de tres hematíes por campo. b) En el individuo sano la orina contiene algunos cilindros hialinos, células epiteliales y cristales. c) En el individuo sano la orina contiene menos de 5 leucocitos por campo. d) En los defectos de los túbulos renales, los riñones no están nunca agrandados.

282: ¿En qué caso está indicado el tratamiento antibiótico de una bacteriuria asintomática?:

a) En niños menores de 5 años con reflujo vesico ureteral. b) Mujeres embarazadas. c) Pacientes diabéticos. d) Todos los casos anteriores.

283: El reservorio de la Salmonella Typhy lo constituye:

a) Las aves de corral. b) El ganado ovino. c) El agua. d) El hombre.

284: La técnica de Westem-Blot utilizada en el diagnóstico de la infección por VIH permite detectar:

a) Material genético del virus. b) Antígeno P24. c) Transcriptasa reversa. d) Anticuerpos.

285: El hallazgo de una miringitis ampollosa en un paciente con neumonía debe hacer sospechar:

a) Neumonía estafilocócica. b) Neumonía neumocócica. c) Neumonía brucellar. d) Neumonía por Mycoplasma.

286: ¿Cuál de las siguientes localizaciones es la más frecuente en la brucelosis?:

a) Osteoarticular. b) Pulmonar. c) Endocardio.

Page 25: Cuestionario de Urología

d) Neurológica.

287: ¿Qué grupo de virus afecta con más frecuencia al paciente inmuno-deprimido?:

a) Los adenovirus. b) Los enterovirus. c) Los herpesvirus. d) El virus influenzae.

288: Paciente de 20 años, que presenta un síndrome de mononuc1eosis con anticuerpo heterófilo negativo, lo más probable que tenga es:

a) Mononuc1eosis por citomegalovirus. b) Mononuc1eosis por virus de Epstein-Barr. c) Hepatitis por virus B. d) Hepatitis por virus C.

289: La complicación infecciosa bacteriana más frecuente en personas consumidoras de drogas por vía parenteral, es:

a) La infección de piel y partes blandas. b) La tuberculosis. c) La neumonía. d) La osteomielitis.

290: En las neumonías extrahospitalarias, una de las siguientes afirmaciones es FALSA, señálela:

a) El neumococo es el agente bacteriano más frecuente. b) En la forma de presentación "atípica" son más frecuentes los síntomas extrapulmonares. c) La legionella pneumophila puede producir alteraciones hepáticas e hiponatremia. d) Los virus habitualmente presentan un patrón radiológico alveolar.

291: En una faringitis estreptocócica el tratamiento antibiótico no previene:

a) La aceleración de la resolución del proceso y la fiebre. b) La diseminación interpersonal. c) La glomerulonefritis aguda. d) La adenitis y el absceso periamigdalino.

292: ¿Cuál es la manifestación pulmonar más frecuente de la artritis reumatoidea?:

a) Nódulos reumatoides pulmonares. b) Neumopatía intersticial. c) Síndrome de Caplan. d) Derrame pleural.

293: La complicación neurológica más frecuente en las fracturas diafisarias humerales es la lesión del nervio:

a) Mediano. b) Radial. c) Circunflejo. d) Cubital.

294: En una de las siguientes enfermedades suele hallarse líquido articular de tipo inflamatorio, señálela:

a) Artropatías postraumáticas. b) Artrosis. c) Osteocondritis. d) Artritis psoriásica.

295: Dentro de las lesiones traumáticas de la rodilla se conoce como la "triada infeliz" la lesión de:

a) Menisco externo, ligamento cruzado anterior y ligamento lateral externo. b) Menisco externo, ligamento cruzado posterior y ligamento lateral externo. c) Ambos meniscos y ligamento cruzado posterior. d) Menisco interno, ligamento cruzado anterior y ligamento lateral interno.

296: De las siguientes afirmaciones, referidas a la polimialgiareumática, señale aquella que es INCORRECTA:

a) Es un síndrome clínico frecuente en ancianos. b) La velocidad de sedimenta«ión suele estar elevada. c) El dolor suele ser nocturno. d) La fuerza muscular está objetivamente alterada.

297: Todos son hallazgos radiológicos de la artrosis, EXCEPTO uno, señálelo:

a) Formación de osteofitos marginales. b) Esclerosis del Hueso subcondral. c) Pinzamiento del espacio articular. d) Grandes erosiones quísticas del hueso.

298: En el tratamiento de la enfermedad de Paget ósea, es FALSO que:

a) La mayoría de los pacientes no requieren tratamiento, ya que la enfermedad es localizada y no provoca ningún síntoma. b) La indometacina puede aliviar el dolor, sobre todo cuando afecta a cadera. c) La administración de ca1citonina durante un período prolongado, disminuye la fosfatasa alcalina en plasma y la hidroxiprolinuria. d) Los difosfonatos no reducen la resorción ósea.

Page 26: Cuestionario de Urología

299: Para el diagnóstico de la fiebre reumática todos son criterios mayores de Jones (actualización de 1992), EXCEPTO uno, señálelo:

a) Eritema marginado. b) Corea. c) Carditis. d) Cultivo faringeo positivo para streptococcus del grupo A.

300: El patrón de afectación articular más frecuente en el Síndrome de Reiter es:

a) Oligoartritis simétrica. b) Poliartritis simétrica. c) Monoartritis crónica. d) Oligoartritis asimétrica.

301: En la artritis reumatoide, no es frecuente la presencia de cambios radiológicos en una de las siguientes articulaciones, señálela:

a) Sacroiliacas. b) Primera metacarpiana. c) Interfalángicas proximales en manos. d) Metatarsofalángicas.

302: En relación al test de cribado con 50 gr. de glucosa y determinación de glucemia a la hora en gestantes, una de las siguientes afirmaciones es CORRECTA, señálela:

a) Se utiliza para diagnosticar a todas las gestantes diabéticas. b) Si el resultado es positivo, no hace falta confirmarlo con el test de sobrecarga oral de glucosa. c) Se recomienda -sólo si hay factores de riesgo para diabetes gestacional- entre las 24 y 28 semanas de gestación. d) A partir de 150 mg/dl ofrece una sensibilidad del 79 % y una especificidad del 87 %.

303: ¿Cuál de las siguientes es una contraindicación absoluta del uso de anticonceptivos orales combinados?:

a) Hipertensión arterial leve. b) Mal uso reiterado de la píldora. c) Litiasis biliar. d) Diabetes con afectación de órganos diana.

304: El riesgo elevado de padecer una neoplasia del cuello uterino está definido por los siguientes factores, EXCEPTO uno, señálelo:

a) Inicio de las relaciones sexuales antes de los 20 años. b) Vaginitis bacteriana de repetición. c) Infecciones por papilomavirus y virus del herpes simple. d) Tabaquismo.

305: Todas las que siguen son causa de dismenorrea secundaria, EXCEPTO una, señálela:

a) Nuliparidad. b) Endometriosis. c) Adenomiosis. d) DIU.

306: La contraconcepción postcoital de Yuzpe consiste en:

a) Administrar 2 dosis de 100 microgramos de etinil estradiol asociadas al miligramo de norgestrel con un intervalo de 12 horas antes de las 72 horas siguientes al coito no protegido. b) Administrar 300 miligramos de progesterona micronizada en dosis única antes de las 72 horas siguientes al coito no protegido. c) Insertar un D.I.U. antes de las 72 horas siguientes al coito no protegido. d) Administrar 600 miligramos de mifepristona, en dosis única antes de las 72 horas siguientes al coito no protegido.

308: En la cirugía de la "uña incarnata", para proceder a la correcta anestesia del dedo, se debe emplear:

a) Aerosol de lidocaína al 10 %. b) Spray de cloruro de etilo. c) Anestésico local con vasoconstrictor, para disminuir el sangrado. d) Anestésico local sin vasoconstrictor.

309: ¿Cuándo está indicado retirar los puntos de sutura no absorbibles en la cara?:

a) A la semana. b) A las dos semanas. c) A las 48 horas. d) A los 3-5 días.

310: La utilización de la anestesia local, con vasoconstrictor (adrenalina), presenta las siguientes desventajas, EXCEPTO una, señálela:

a) Riesgo de necrosis por vasoespasmo. b) Retraso en la cicatrización de las heridas. c) Prolonga la duración del efecto anestésico. d) Disminuye el ph de la solución (aumenta el dolor al infiltrar).

311: En relación con el uso de suturas, señale cuál de estos ejemplos es inadecuado:

a) Cuero cabelludo -7 seda 2/3 ceros.

Page 27: Cuestionario de Urología

b) Miembro superior -7 seda 4/5 ceros. c) Cara -7 seda 2/3 ceros. d) Tejido celular subcutáneo -7 catgut 3/4 ceros.

312: En un varón de 60 años, fumador, con una lesión ulcerada, indurada, de seis meses de evolución, localizada en labio inferior, el diagnóstico más probable es:

a) Líquen plano de mucosas. b) Epiteloma basocelular. c) Epiteloma espinocelular. d) Queratoacantoma.

313: El tratamiento de la pitiriasis rosada debe realizarse con:

a) Corticoides tópicos. b) Corticoides sistémicos. c) Derivados imidazólicos tópicos. d) Ninguno de los anteriores.

314: En un eczema exudativo la base del tratamiento tópico más adecuada será:

a) Un ungüento. b) Una loción. c) Una crema. d) Una pomada.

315: Con respecto al herpes zoster, todas las siguientes afirmaciones son ciertas, EXCEPTO una, señálela:

a) Es debido a una reactivación del virus latente en la varicela. b) Da lugar a una erupción maculopapulosa eritematosa inicialmente. c) El comienzo de la enfermedad está presidido siempre por lesiones cutáneas. d) Si se afecta la rama sensitiva del facial, hay lesiones en conducto auditivo.

316: El eczema dishidrótico se localiza habitualmente en:

a) Superficies de flexión de antebrazos. b) Grandes pliegues. c) Dorso de pies. d) Superficies laterales de los dedos.

317: Joven de 15 años con madre diagnosticada recientemente de tuberculosis pulmonar y que no fue vacunado con B.c.G.. Se le realiza prueba de tuberculina con lectura a las 48 horas y resulta positiva. La radiografía de tórax es normal. No tiene síntomas sospechosos de tuberculosis. ¿Qué procede hacer?:

a) Quimioprofilaxis secundaria. b) Vacuna B.c.G. c) Tratamiento antituberculoso. d) Vigilancia y observación exclusiva.

318: De las siguientes asociaciones, señale cuál de ellas es INCORRECTA:

a) Vacuna antipoliomielítica tipo Sa1k: Mujer embarazada. b) Vacuna antisarampión: Niños VIH (+) asintomáticos. c) Vacuna antigripal: Niños en tratamiento prolongado con AAS. d) Vacuna antineumocócica: Síndrome nefrótico en un niño de un año.

319: ¿Cuál de las siguientes estrategias se considera mejor para detectar a personas con riesgo elevado de enfermedad cardiovascular?:

a) Screening masivo de colesterol sérico. b) Encuesta poblacional sobre tabaquismo. c) Detección casual de factores de riesgo (case-finding). d) Screening masivo de hipertensión arterial.

320: ¿Cuál de las siguientes vacunas es inactivada?:

a) B.C.G. b) Sarampión. c) Parotiditis. d) Hepatitis B.

321: En relación a la vacuna antirrubeola, todas las siguientes afirmaciones son ciertas, EXCEPTO una, señálela:

a) Es una vacuna de virus vivos atenuados. b) Su eficacia es del 90-95 %. c) Su objetivo principal es prevenir la rubeola congénita. d) La capacidad inmunógena de la vacuna es afectada si se administra simultáneamente con las vacunas DTP (difteria, tétanos, pertussis).

322: Para describir la proporción de personas vacunadas en relación a las que deberían haber sido vacunadas dentro de un programa de prevención, utilizamos:

a) La tasa de protección. b) La tasa de cobertura. c) El grado de inmunización. d) La eficacia del programa. d) La efectividad del programa.

Page 28: Cuestionario de Urología

323: Respecto a la Prevención Primaria, señale cuál de las siguientes afirmaciones es FALSA:

a) Su objetivo es disminuir la probabilidad de aparición de enfermedades. b) Pretende reducir la incidencia de la enfermedad. c) Actúa en el período prepatogénico de la enfermedad. d) Se basa en el diagnóstico precoz de la enfermedad.

324: En relación al método de estandarización indirecto utilizado en la comparación de las tasas de mortalidad de dos poblaciones, es FALSO que:

a) Se utiliza cuando no se puede conocer la clasificación de fallecimientos por grupos de edad. b) Se puede usar cuando conocemos la tasa bruta de mortalidad de las poblaciones a comparar. c) Se utiliza en poblaciones de pequeño tamaño. d) Con este método se obtiene la razón de mortalidad comparativa (RMC).

325: Al analizar la relación entre un factor de riesgo y una enfermedad se utiliza el análisis estratificado para:

a) Emparejar casos y controles respecto a una variable. b) Controlar el efecto de un factor de confusión. c) Obtener estimaciones más precisas del riesgo relativo. d) Aumentar el número de sujetos en cada estrato.

326: Se denomina índice epidémico a:

a) La media de casos ocurridos en una zona durante una semana. b) La relación entre el número de casos de una semana y la mediana del quinquenio anterior para esa semana. c) La relación entre el número de casos del quinquenio anterior y los ocurridos en la semana. d) La relación entre el número de casos de una semana y los fallecidos en esa semana.

327: En un estudio tenemos varias variables cuantitativas y queremos comparar sus medias geométricas. Utilizaremos la prueba de:

a) T. de Student-Fisher. b) Análisis de la varianza. c) Correlación de Yates. d) Coeficiente de correlación de Pearson.

328: Respecto a los estudios de cohortes, ¿Cuál de las siguientes afirmaciones es CIERTA?:

a) Se determina y compara la incidencia de enfermedad en los grupos expuesto y no expuesto. b) Se seleccionan enfermos y no enfermos, y se investigan anteriores exposiciones a posibles factores de riesgo. c) Son estudios de tipo retrospectivo. d) Son los más adecuados para el estudio de enfermedades raras.

329: Si tuviera que elegir el parámetro de mayor relevancia para tomar la decisión de poner en marcha una determinada prueba de cribaje, se inclinaría por:

a) Sensibilidad. b) Especificidad. c) Valor predictivo positivo. d) Valor predictivo negativo.

330: ¿Cuál de los siguientes tipos de estudio es el más adecuado para estimar la magnitud de una enfermedad en una población?

a) Ensayo clínico. b) Cohorte. c) Transversal. d) Casos y controles.

331: Respecto a las zoonosis, uno de los siguientes enunciados es FALSO, señálelo:

a) La anisakiasis se produce por el consumo de pescado que contenga larvas de este parásito. b) El consumo de berros es la causa principal de la fasciolosis. c) La filariasis o wuchereriasis se adquiere por contacto con agua que contiene larvas procedentes de diferentes géneros de caracoles. d) La brucelosis se puede transmitir por vía inhalatoria.

332: De la siguiente tabla 2x2, ¿Cuál sería la sensibilidad de la prueba?

a) l/4 b) l/6 c) l/3 d) 2/4

333: ¿Cuál de los siguientes métodos de identificación de problemas y necesidades no pretende llegar a un consenso?:

a) El enfoque a través de informadores clave. b) La técnica del grupo nominal. c) La técnica Delphi. d) El "brainwriting".

334: De acuerdo a la mortalidad publicada por el Instituto Nacional de Estadística (INd) en 1993. ¿Cuál es el tumor, que tras el

PRUEBA: ENFERMOS SANOS

Positiva 100 200

I Negativa 300 400

Page 29: Cuestionario de Urología

cáncer de pulmón, ocasiona más muertes entre los varones en España?:

a) Próstata. b) Colon. c) Estómago. d) Vejiga.

335: ¿Qué es un PERT? ("Program evaluation and review technique" o técnica de evaluación y revisión de programas)?:

a) Un método de consenso. b) Un tipo de algoritmo diagnóstico. c) Un método de control de actividades con estructura de red. d) Un diagrama de flujo.

336: La biología humana, el entorno y el sistema sanitario, son tres de los cuatro determinantes clásicos de la salud; el cuarto es:

a) Los recursos. b) Los servicios. c) La personalidad. d) El estilo de vida,

337: Las siguientes tasas son utilizadas como indicadores sanitarios útiles en planificación, EXCEPTO una. Señálela:

a) Tasa de letalidad. b) Tasa de mortalidad específica por causas. c) Tasa de mortalidad perinatal. d) Tasa de crecimiento natural.

338: La gravedad de un problema, se puede valorar a través de todos los siguientes parámetros, EXCEPTO uno, señálelo:

a) Incidencia. b) Años de vida perdidos. c) Mortalidad. d) Incapacidad generada por el problema.

339: ¿Qué es una meta de salud en un programa?:

a) Una actividad cuantificada. b) Objetivo de salud cuantificado en cantidad y tiempo. c) Sinónimo de prioridad. d) Sinónimo de objetivo de salud.

340: ¿Cuál es la tasa de mortalidad posneonatal?:

a) Número de recién nacidos vivos, fallecidos antes del mes de vida, en un año. b) Número de fallecidos con más de un mes y menos de un año, en un año. c) Número de fallecidos con más de una semana y hasta un mes, en un año. d) Número de fallecidos en la primera semana, en un año.

341: La planificación de una consulta de enfermería para vacunación antigripal que requiere 8 horas/día de local y de dedicación de enfermería y consigue una media de 6 vacunaciones al día implica fundamentalmente a la:

a) Competencia científico-técnica. b) Continuidad. c) Eficacia-efectividad. d) Eficiencia.

342: Según la Ley General de Sanidad 14/l986, las actuaciones de las Administraciones Públicas Sanitarias estarán orientadas... Señalar la respuesta FALSA:

a) A garantizar la Salud. b) A promover el interés individual, familiar y social por la salud, mediante la adecuada educación sanitaria de la población. c) A garantizar que cuantas acciones sanitarias se desarrollen estén dirigidas a la prevención de las enfermedades y no sólo a la curación de las mismas. d) A garantizar la asistencia sanitaria en todos los casos de pérdida de salud.

343: Todos los siguientes son registros médico legales, EXCEPTO uno, señálelo:

a) Sistema de Enfermedades de Declaración Obligatoria. b) Certificado Médico de Defunción. c) Parte de Incapacidad Laboral. d) Registro de Actividad.

348: Según la Ley 14/1986, General de Sanidad, todos los ciudadanos tienen los siguientes derechos con respecto a las distintas Administraciones Públicas Sanitarias, EXCEPTO uno, señálelo:

a) A la información sobre los servicios sanitarios a que pueda acceder. b) A la confidencialidad de toda la información relacionada con su proceso. c) Al respeto a su personalidad, dignidad humana e intimidad. d) A negarse al tratamiento siempre que lo desee y sin excepciones.

349: En Atención Primaria de Salud, el concepto de continuidad de la asistencia hace referencia a:

a) La asistencia a urgencias. b) La continuidad en el tiempo (a lo largo de toda la vida del individuo, y en todos sus momentos vitales). c) La permanencia física en el Centro. d) La asistencia prestada por una misma persona.

Page 30: Cuestionario de Urología

350: El procedimiento fundamental para el diagnóstico de angina de pecho es:

a) Anarnnesis. b) Electrocardiograma. c) Holter. d) Coronariografía.

351: La causa más frecuente de hemorragia digestiva alta es:

a) Tumores esofágicos. b) Lesiones agudas de la mucosa gástrica. c) Varices esofágicas. d) Ulcera péptica.

352: Respecto a la quimioprofilaxis de enfermedades, es FALSO que:

a) Se aplica sólo a las enfermedades infecciosas. b) Se aplica sólo a individuos sanos. c) Puede inducir la aparición de microorganismos resistentes. d) No preteíÍde curar la enfermedad ya iniciada.

353: ¿Cuál de las siguientes etapas corresponde a la planificación táctica?:

a) Identificación de los problemas. b) Previsión de recursos necesarios. c) Puesta en marcha del programa. d) Fijación de los fines.

354: Nos impide delimitar la demanda del paciente:

a) El saludarle de forma cordial y personalizada. b) El facilitarle la expresión emocional. c) El hacer que se sienta cómodo. d) Interpretar inicialmente el motivo de consulta.

355: Señale la respuesta válida respecto a la forma de adquisición de actitudes positivas para la salud:

a) Experiencia directa con el objeto. b) Experiencia directa con la persona o situación. c) Observación de los efectos de un estímulo sobre tercera persona. d) Todas las respuestas son válidas.

357: Respecto de la erisipela, señale la afirmación CORRECTA:

a) El gennen etiológico es el Corynebacterium minutissimum. b) Raramente produce fiebre. c) El tratamiento de elección es la tetracic1ina. d) En la actualidad la localización más frecuente es en miembros inferiores.

359: ¿Cuál de los siguientes métodos se considera más eficaz en la Educación Sanitaria a Grupos de Población?:

a) La charla a cargo de una persona de elevado prestigio. b) La charla a cargo de un sanitario de la comunidad. c) La utilización de medios audiovisuales. d) La discusión en grupo.

360: La estandarización de tasas es una técnica epidemiológica que permite comparar distintas poblaciones. Señale la respuesta CORRECTA:

a) La estandarización de tasas sirve para eliminar el efecto de las diferencias pe tamaño de las poblaciones que se comparan b) La estandarización de tasas por el método directo consiste en aplicar las tasas específicas de una población estándar a cada una de las poblaciones que se comparan c) La estandarización de tasas se aplica sólo a datos de mortalidad, nunca de prevalencia o incidencia d) La estandarización de tasas corrige el efecto del sexo, la raza, la distribución por edades o el nivel socioeconómico al comparar dos o más poblaciones

361: Se denomina muerte neonatal:

a) La que acontece el primer día de vida (primeras 24 horas) b) La que acontece en la primera semana de vida c) La que acontece en las primeras cuatro semanas de vida (primeros 28 días) d) La que acontece en la primera hora de vida

362: Con relación a la prevalencia como medida de frecuencia de la enfermedad, es FALSO que:

a) SU denominador corresponde a la población total, con independencia de su condición de sanos o enfermos b) Su valor aumenta cuando se incrementa la incidencia de la enfermedad c) Su numerador depende de la duración media de la enfermedad d) Su valor disminuye en caso de tratamientos paliativos que prolongan la supervivencia sin curación

363: ¿Cuál de los siguientes tipos de estudios epidemiológicos aporta mayor nivel de evidencia científica?:

a) Opinión de expertos b) Estudios de cohortes c) Metaanálisis de ensayos aleatorizados y controlados d) Estudios no experimentales

Page 31: Cuestionario de Urología

364: La diferencia entre la tasa de aparición de un proceso entre los expuestos a un factor de riesgo y la tasa en los no expuestos (Te - Tne) se denomina:

a) Fracción etiológica b) Riesgo relativo c) Odds Ratio d) Riesgo atribuible

364: Al inicio de un estudio de cohortes, ¿cómo está la población a estudiar?:

a) Todos afectos del proceso que se estudia b) Todos sanos c) La cohorte expuesta sana y la no expuesta enferma d) La cohorte expuesta enferma y la no expuesta sana

366: Cuando hablamos de la validez de un test de screening sabemos que es CIERTO que:

a) Su valor predictivo positivo indica la probabilidad de que un sujeto con la enfermedad tenga un resultado positivo b) Su valor predictivo negativo no depende de la prevalencia de la enfermedad c) La razón de probabilidad es independiente de la sensibilidad y la especificidad d) La razón de probabilidad compara la probabilidad de obtener un resultado en un sujeto que presenta la enfermedad con la de obtenerlo en un sujeto que no la presenta

367: Cuando existe una asociación causal entre un factor de Riesgo y una Enfermedad, es FALSO que:

a) Existe asociación estadísticamente significativa entre ambos b) La relación entre el factor de riesgo y la enfermedad debe ser específica c) La fuerza de la asociación es elevada si el riesgo relativo es alto d) Existe concordancia con los hallazgos de diferentes estudios

368: Al calcular el tamaño de la Muestra en un estudio de investigación es FALSO que:

a) Debe calcularse en función del tiempo y recursos disponibles b) Depende de la variabilidad del parámetro que se quiere estimar c) SU determinación forma parte de la fase de diseño d) El investigador debe decidir la precisión que desea en su estimación

369: En un ensayo clínico en el que para ver la eficacia de un fármaco, se da a un grupo el fármaco y al otro el placebo con iguales características organolépticas y en el que el investigador desconoce a que grupo pertenece cada enfermo: ¿qué técnica se ha utilizado para evitar el sesgo del observador?:

a) Doble ciego b) Triple ciego c) Simple ciego d) Técnica de la doble lectura

370: En un estudio de casos y controles si se constata asociación estadística mediante la chi cuadrado clásica ¿cómo mediremos la fuerza o magnitud de asociación?:

a) Riesgo relativo b) Riesgo atribuible c) Fracción de riesgo atribuible d) Odds-ratio

371: En relación a las medidas de centralización o localización de un conjunto de datos estadísticos, señale la opción ERRONEA:

a) Los cuartiles dividen el conjunto de datos en cuatro partes iguales b) La media está muy influida por los valores extremos de la variable c) La moda puede coincidir en su valor con la media d) La mediana puede utilizarse con variables cualitativas ordinales

372: Los sistemas de evaluación de un programa o actuación sanitaria tienen distintos componentes, señale la opción CORRECTA:

a) La PERTINENCIA refleja las mejoras de resultados tras la implantación de la actuación sanitaria b) Los PROGRESOS indican la relevancia del problema de salud sobre el que se pretende actuar c) La EFICACIA manifiesta el grado en que el programa o la actuación sanitaria produce mejoras de salud en condiciones reales d) La EFICIENCIA compara el impacto de los resultados de un programa o actuación y su coste de producción

373: Señale los factores que se encuentran MENOS ASOCIADOS ál cumplimiento-observancia de las prescripciones por parte del paciente:

a) Las creencias y actitudes hacia la enfermedad b) El apoyo y la supervisión de la familia c) La motivación para recuperar la salud d) La edad y el sexo

374: Según los criterios de clasificación de los fárma por valor intrínseco y potencial de uso, la hormona de crecimiento tendría como característica:

a) Valor intrínseco elevado y potencial de uso elevado b) Valor intrínseco relativo y potencial de uso elevado c) Valor intrínseco relativo y potencial de uso limitado d) Valor intrínseco elevado y potencial de uso limitado

375: Al realizar un test de cribado, si se aplica a un grupo de población de bajo riesgo, en lugar de a la población general, estamos

Page 32: Cuestionario de Urología

aumentando:

a) La sensibilidad del test b) La especificidad del test c) La aceptabilidad del test d) El valor predictivo negativo del test

376: Señale, con relación al cribado de cáncer de próstata, cuál de estas intervenciones es la recomendada en Atención Primaria:

a) No se aconseja ninguna prueba de cribado poblacional para varones asintomáticos en Atención Primaria b) Realizar cada dos años un tacto rectal a varones asintomáticos mayores de 50 años c) Realizar cada 5 años una ecografía transrectal a varones asintomáticos mayores de 65 años d) Determinar una vez antígeno prostático específico (PSa) a varones asintomáticos mayores de 65 años

377: El PARALENGUAJE es:

a) Un componente de la comunicación verbal b) Un componente de la comunicación no verbal c) Un componente de los gestos d) Lo que se expresa con los movimientos faciales

378: En la entrevista clínica, la "toma de precauciones":

a) Debe realizarse cuando hay acompañantes b) Sólo debe realizarse si el paciente es agresivo c) Al realizarla hay que usar un volumen alto y un tono enérgico de voz d) Debe realizarse al terminar la entrevista, antes de despedimos

379: El genograma es la representación gráfica del desarrollo de una familia. Del geno grama obtenemos información acerca de los siguientes elementos EXCEPTO:

a) Enfermedades y problemas de salud de sus miembros b) Patrones relacionales entre los miembros de una familia c) Relaciones legales y extrarnatrimoniales d) Tipo de intervención familiar realizada

380: Para un Médico de Atención Primaria, cuál de las siguientes actividades constituye el nivel MAS DESARROLLADO de Atención Familiar:

a) Elaborar un geno grama completo a la familia de una paciente hiperconsultadora b) Evaluar la red de apoyo social de un paciente perteneciente a una familia monoparental c) Informar a la familia sobre la actuación clínica ante una descompensación de un diabético d) Prescribir a la familia tareas concretas para mejorar la comunicación entre sus miembros

381: ¿Cuál de los siguientes componentes formativos es el más difícil de mejorar con un programa de formación continuada?:

a) Conocimientos clínicos b) Habilidades instrumentales c) Habilidades de comunicación d) Actitudes profesionales

382: ¿Cuál de los siguientes pasos HABITUALMENTE NO FORMA PARTE del ciclo evaluativo o de un Plan de Mejora de la Calidad?:

a) Detección y priorización de problemas b) Análisis de causas y soluciones c) Ejecución de las medidas correctoras y aplicación de los cambios d) Comunicación a la población de los resultados

383: En el modelo de causalidad deterrninista modificado, el treponema pallidum con respecto a la sífilis es:

a) Causa suficiente b) Causa contribuyente c) Causa necesaria d) Causa componente

384: En cuanto a las características del agente infeccioso transmisible por contacto directo, indique la respuesta CORRECTA:

a) La tuberculosis es una enfermedad de contagiosidad alta y virulencia baja b) La varicela es una enfermedad de contagiosidad baja y de virulencia baja c) La rubeola es una enfermedad de contagiosidad alta y de virulencia alta d) El sarampión es una enfermedad de contagiosidad alta y de virulencia baja

385: Todos los que se citan son indicadores indirectos de contaminación fecal en el agua de consumo público, EXCEPTO:

a) MAGNESIO b) CLORUROS c) NITRATOS d) NITRITOS

386: ¿Cuál de las siguientes enfermedades NO es una EDO? (Enfermedad de Declaración Obligatoria):

a) Sarampión b) Rubeola c) Varicela d) Escarlatina

Page 33: Cuestionario de Urología

387: Varios miembros de una familia, tres horas después del consumo de productos en una pastelería, presentan un cuadro de vómitos intensos, dolores abdominales, diarrea intensa e hipotermia. Pasadas veinticuatro horas el cuadro va remitiendo espontáneamente. ¿Qué germen será, con mayor probabilidad, el responsable del brote de toxiinfección alimentaria?:

a) Shigella b) Estafilococo aureus c) Botulismo d) Echerichia coli enteroinvasiva

388: Las siguientes medidas, se dirigen al control de la fuente de infección de una epidemia EXCEPTO:

a) Diagnóstico precoz b) Tratamiento precoz c) Aislamiento d) Vacunación

389: En un niño VIH+ una de las siguientes vacunas está CONTRAINDICADA. Señale cuál:

a) DPT b) Polio oral c) Triple vírica d) Haemophilus influenzae

390: Ante la aparición de spooting en el primer mes de tratamiento con anticonceptivo s hormonales orales (ACHO) señale cual de las siguientes respuestas es la CORRECTA:

a) Es necesario cambiar el ACHO por otro con mayor dosis de estrógenos b) Es necesario retirar el ACHO c) Es necesario descartar olvido de dosis o interacción medicamentosa d) Es necesario cambiar el ACHO por otro con mayor dosis de gestágenos

391: ¿Qué método anticonceptivo aconsejaría a una adolescente con relaciones sexuales esporádicas?:

a) Método barrera b) Método temperatura basal (método natural) c) Anticoncepción hormonal oral d) Dispositivo Intrauterino (DIU)

392: Una de las siguientes situaciones CONTRAINDICA la utilización del DIU como método anticonceptivo. Señale cual:

a) Tratamiento anticornicial b) Tratamiento anticoagulante oral c) Diabetes insulino-dependiente d) Cardiopatía isquémica

393: Todas las que se citan son contraindicaciones absolutas de la anticoncepción hormonal oral, EXCEPTO:

a) Accidente cerebrovascular previo b) Tabaquismo en mujeres mayores de 25 años c) Hipertensión Arterial no controlada d) Diabetes Mellitus con complicación vascular

394: En el desarrollo psicomotor del niño de 6 meses NO SE CONSIDERA signo de alerta:

a) Hipotonía del cuello b) Persistencia de reflejos arcaicos c) Ausencia de sedestación sin apoyo d) Llanto muy frecuente

395: Una puérpera de 25 años solicita anticoncepción hormonal desde los primeros días de la lactancia. ¿Cuál de las siguientes afirmaciones es CORRECTA?:

a) Está indicada la píldora combinada monofásica b) Está indicado el inyectable de acetato de Medroxiprogesterona c) Está indicada la píldora combinada trifásica d) Está indicado el inyectable de Dihidroxiprogesterona y enantato de estradiol

396: Todas las características que se mencionan son propias de la lactancia materna, excepto una. Señale la opción FALSA:

a) Debe de mantenerse al menos hasta los 4-6 meses de vida b) En prematuros de bajo peso puede ser insuficiente por sí sola para satisfacer los requerimientos calóricos del recién nacido de bajo peso c) El inicio de la menstruación tras el puerperio no constituye una contraindicación para mantener la lactancia materna d) Los lactantes alimentados exclusivamente con leche materna precisan suplementos de hierro oral desde los dos meses

397: De los siguientes fármacos uno está formalmente CONTRAINDICADO en el embarazo. Señale cuál:

a) Amoxicilina b) Metildopa c) Salbutamol d) Misoprostol

398: Uno de los siguientes fármacos está CONTRAINDICADO en el embarazo por su efecto teratógeno demostrado:

a) Cortisona b) Heparina c) Oxprenolol

Page 34: Cuestionario de Urología

d) Prednisona

399: Los objetivos de un programa de embarazo en Atención Primaria son todos los que se citan, EXCEPTO:

a) Preparación de la pareja para el parto domiciliario u hospitalario indistintamente b) Disminución de la mortalidad perinatal c) Disminución de la incidencia de recién nacidos de bajo peso d) Disminución de la morbimortalidad de causa obstétrica

400: Una de las siguientes situaciones NO SE CONSIDERA una causa de alto riesgo obstétrico:

a) Embarazo múltiple b) Embarazo prolongado c) Inserción placentaria anómala d) Gestante mayor de 30 años

401: ¿Cuál de las siguientes actividades NO SE REALIZA de forma rutinaria en los controles de embarazo?:

a) Exploración física de la presencia de edemas b) Medición del perímetro abdominal c) Valoración del riesgo obstétrico d) Curva de glucemia

402: De los siguientes parámetros ecográficos, cuál es el MAS FIABLE para determinar la edad gestacional del feto:

a) Diámetro biparietal b) Circunferencia cefálica c) Circunferencia abdominal d) Longitud del fémur

403: NO CONSTITUYE una contraindicación absoluta de la lactancia materna:

a) SIDA de la madre b) Hepatitis B materna c) Tuberculosis pulmonar activa de la madre d) Mastitis aguda

404: De los siguientes objetivos característicos de la educación en grupo de pacientes crónicos señale cuál es ERRONEO:

a) Prepararse para una situación de cambio o de crisis b) Buscar alternativas a problemas personales y familiares c) Tener respuestas para todas las cuestiones d) Promover el aprendizaje de técnicas

405: ¿Cuál de las siguientes afirmaciones NO ES CIERTA con respecto a la educación sanitaria de la Diabetes Mellitus?:

a) Disminuye el número de amputaciones de extremidades inferiores sólo en un 5% b) Disminuye el número de ingresos hospitalarios por coma diabético c) Mejora la aceptación de la enfermedad y disminuye el temor hacia ella y sus complicaciones d) Disminuye la incidencia y la gravedad de las secuelas

406: Las técnicas de educación en grupo se basan en teorías centradas en el aprendizaje entre iguales, de las siguientes todas son técnicas grupales, EXCEPTO:

a) El roll-playing o dramatización b) La técnica de puzzle o aprendizaje cooperativo c) Método Delphy d) Grupo de discusión

407: El Consejo Médico (counselling), como actividad de educación para la salud en la consulta, debe reunir las siguientes condiciones para ser realmente efectivo, EXCEPTO:

a) Reforzarse con el "papel ejemplar" del profesional y del Centro Sanitario para evitar menajes ambivalentes. b) Incluir elementos informativos motivadores y reforzadores de la conducta en el mensaje que se transmite al paciente. c) Influir en la totalidad de pacientes que tienen contacto con el médico de Atención Primaria. d) Trasladar toda la responsabilidad de la salud al profesional que ejercerá la función de "guía sanitario".

408: Entre los marcadores biológicos del consumo crónico de alcohol el MAS INDICATIVO de consumo mantenido e importante es:

a) Aumento del Volumen Corpuscular Medio (VCM) b) Cociente GOT/GPT mayor de 1,5 c) Aumento de garnmaglutamil transpeptidasa (GGT) d) Aumento de bilirrubina indirecta

409: Con relación al Delirium Tremens, señale la opción FALSA:

a) Es un cuadro que se presenta tras una brusca deprivación alcohólica b) Aparece pasadas 48-72 horas desde la última ingesta alcohólica c) Clínicamente se caracteriza por agitación, desorientación intensa y alucinaciones d) En la exploración se manifiesta gran hipotermia y bradicardia

410: Una de las siguientes situaciones NO CONTRAINDICA el uso del parche de nicotina como terapia sustitutiva en la deshabituación tabáquica. Señale cuál:

a) Embarazo b) Infarto agudo de miocardio

Page 35: Cuestionario de Urología

c) Ictus reciente d) Diabetes

411: La Intervención Mínima sobre tabaquismo en Atención Primaria, consiste en:

a) Explicar el uso del chicle o parche de nicotina b) Prohibir el uso del tabaco en Centros Sanitarios c) La aplicación de técnicas conductuales para dejar de fumar d) El consejo firme y claro sobre la necesidad de dejar el tabaco

412: En el cuadro de sobredosificación de morfina, es característica la aparición de:

a) Diarrea b) Excitación c) Miosis d) Taquicardia

413: En la prevención de la conducta suicida se recomiendan todas las actuaciones que se citan, EXCEPTO:

a) Detectar los factores de riesgo de suicidio precozmente b) Buscar sistemáticamente la ideación suicida en las personas asintomáticas de la población general c) Interrogar al paciente con ideación suicida respecto a los medios y los pasos a realizar para consumarla d) Derivar al paciente de forma urgente al Equipo de Salud Mental en el caso de detectar una idea de suicidio organizada o reiterada

414: Todos los que se citan son factores predictores de patología depresiva en mujeres, EXCEPTO:

a) Pérdida de un progenitor antes de los 17 años b) Tres o más hijos menores de 14 años en el domicilio familiar c) Profesionales con alta cualificación d) Relación conyugal pobre y sin confianza

415: Se considera que un adulto geriátrico presenta un déficit global cognitivo si al practicarle un Mini-examen cognoscitivo tipo MMSE en su versión española (test de Lobo) arroja el siguiente resultado en su puntuación:

a) Menor o igual a 23 puntos b) Menor o igual a 26 puntos c) Menor o igual a 28 puntos d) Menor o igual a 32 puntos

416: ¿Cuál de los siguientes fármacos tiene MENOS UTILIDAD en el tratamiento de la tuberculosis?

a) Isoniazida b) Rifampicina c) Pirazinamida d) Estreptomicina

417: En pacientes Mantoux positivo y descartada la enfermedad tuberculosa, todas las que se citan son indicaciones absolutas para instaurar pauta de .' quimioprofilaxis con Isoniazida, EXCEPTO:

a) Paciente de 50 años sin clínica asociada b) Paciente de 69 años con infección por VIH c) Paciente de 66 años conviviente con varios enfermos tuberculosos d) Paciente de 39 años con adicción a drogas por vía parenteral

418: El tratamiento de elección de la Tuberculosis en los casos iniciales es:

a) Isoniazida + Rifampicina + Pirazinamida 2 meses, seguidos de 4 meses más con Isoniazida + Rifampicina b) Isoniazida + Rifampicina + Pirazinamida 4 meses, seguidos de 2 meses más con Rifampicina + Pirazinamida c) Isoniazida + Rifampicina + Estreptomicina 3 meses, seguidos de otros 3 meses con Rifampicina + Estreptomicina d) Ninguno de los anteriores.

419: La única forma clínica de enfermedad por Mycobacterium Tuberculosis que NO TIENE una respuesta uniformemente favorable frente a la quimioterapia es:

a) La adenitis tuberculosa b) La tuberculosis miliar c) La tuberculosis pulmonar d) La meningitis tuberculosa

420: Ante la sospecha de malnutrición en el anciano, ¿cuál de los siguientes parámetros es el de MENOR VALOR en la evaluación de su estado nutricional?:

a) Concentración de transferrina en estados de ferropenia b) Indice de masa corporal c) Grosor del pliegue cutáneo del triceps d) Pérdida porcentual de peso

421: Señale cuál de las siguientes NO ES CARACTERlSTICA del estado confusional del anciano:

a) Se desarrolla en un corto período de tiempo con fluctuaciones a lo largo del día b) Siempre existe una causa orgánica importante que lo desencadena c) Es un proceso progresivo e irreversible d) Presenta desorientación temporoespacial

422: El Mini State Examination (MMSd) de Folstein se utiliza PARA VALORAR:

Page 36: Cuestionario de Urología

a) Actividades básicas de la vida diaria b) Actividades instrumentales de la vida diaria c) Detección de alteraciones cognitivas d) Detección de alteraciones afectivas

423: El estreñimiento es uno de los síntomas digestivos más frecuentes en el Enfermo Terminal, señale la opción FALSA:

a) Presenta una etiología multifactorial debida a la enfermedad de base, el tratamiento farmacológico y la patología anorectal asociada b) El objetivo terapéutico ha de ser conseguir al menos una deposición cada tres o cuatro días c) En pacientes muy deteriorados puede ocasionar distensión abdominal, náuseas-vómitos, inquietud y estado confusional d) Como tratamiento médico sólo están indicados los laxantes formadores de masa combinados con dieta rica en fibra

424: La Escala de Hachinski SE UTILIZA para:

a) El diagnóstico diferencial entre la demencia tipo Alzheimer y la demencia vascular b) La valoración de la independencia del anciano frente a las actividades de la vida diaria c) La valoración cognitiva del anciano d) La valoración social del anciano

425: El tratamiento antibiótico DE ELECCION en la neumonía extrahospitalaria típica es:

a) Eritromicina b) Cefalosporinas de segunda generación c) Penicilina procaína d) Doxiciclina

426: El germen causante de la mayoría de las neumonías extrahospitalarias en sujetos sanos en nuestro medio es:

a) El Haemophilus influenzae b) Streptococus pneumoniae c) Klebsiella pneumoniae d) Chlarnydia pneumoniae

427: Una de las siguientes situaciones clínicas NO ES un síntoma habitual de la gripe del adulto:

a) Dolor retrocular allateralizar la mirada b) Fiebre con duración menor o igual a siete días c) Enrojecimiento conjuntival d) Diarrea

428: La tos seca irritativa de predominio en primavera y otoño SUGIERE ANTE TODO Y COMO PRIMERA OPCION A DESCARTAR:

a) Asma bronquial b) Fibrosis pulmonar c) Enfermedad pulmonar Obstructiva crónica d) Carcinoma broncopulrnonar

429: La extensión radio lógica de los infiltrados pulmonares pese a un tratamiento antibiótico adecuado, ES TIPICA de la neumonía causada por:

a) Clarnydia b) Legionella pneumophyla c) Pseudomona aeruginosa d) Neumococo

430: El tratamiento de elección en la candidiasis vaginal es:

a) Metronidazol b) Clotrimazol c) Fluconazol d) Mebendazol

431: Con respecto al tratamiento de la Tricomoniasis, indique que RESPUESTA ES FALSA:

a) El tratamiento debe ser oral b) El tratamiento se realiza con Metronidazol2 gramos en una o dos tomas c) El tratamiento con Imidazoles está contraindicado en el embarazo d) No es necesario tratar a la pareja si es asintomática

432: La infección genital en la mujer producida por Chlamydia Trachomatis afecta PRINCIPALMENTE:

a) El cérvix b) La vulva c) La trompa de Falopio d) La trompa de Falopio y el cérvix

433: La solicitud de serología de VIH estaría indicada en todos los casos que siguen, EXCEPTO:

a) Personas que lo soliciten a demanda pese a no reconocer práctica de riesgo b) Personas que realizan práctica deportiva profesional en grupo c) Personas con transfusiones de sangre anteriores a 1986 d) Personas con relaciones heterosexuales con una pareja perteneciente a un grupo de riesgo

434: Varón de 33 años, diagnosticado de SillA, que acude a consulta por presentar durante las dos últimas semanas sensación de

Page 37: Cuestionario de Urología

decaimiento general, tos no productiva y febrícula de 37-38°C. El diagnóstico MAS PROBABLE será:

a) Catarro de vías altas b) Neumonía neumocócica c) Neumonía por Micoplasma d) Neumonía por Pneumocystis carinii

435: Cuál de las siguientes circunstancias clínicas SUGIERE MAS la existencia de una infección VIH:

a) Liquen plano b) Lupus discoide c) Psoriasis rebelde d) Penfigo persistente

436: El germen implicado CON MAYOR FRECUENCIA en la producción de la Diarrea del Viajero es:

a) Salmonella b) Shigella c) E. Coli d) Yersinia

437: El tratamiento DE ELECCION de las fiebres tifoideas es:

a) Cloranfenicol b) Tetraciclinas c) Sulfadiacina d) Estreptomicina

438: Señale la afirmación INCORRECTA en relación con la Fiebre Tifoidea:

a) Los portadores transitorios deben tratarse con cotrimoxazol b) Si hay recidivas el tratamiento es idéntico al de la primera infección c) Los portadores sanos son la principal fuente de infección d) En casos graves pueden utilizarse corticoides en el tratamiento

439: ¿Cuál de los siguientes marcadores de la Hepatitis viral tipo B indica replicación viral activa?:

a) HBsAg b) HBcAg c) AntiHBsAg d) HBeAg

440: La profilaxis postexposición al VHB (virus hepatitis b) está indicada en todos los siguientes casos, EXCEPTO:

a) Recién nacido de madre portadora b) Exposición por drogadicción c) Contacto social esporádIco con un portador d) Contacto sexual íntimo con posible portador

441: ¿Cuál de los siguientes enunciados referidos a la Hepatitis C es VERDADERO?:

a) Es origen del 10% de las Hepatitis postransfusionales b) Con una frecuencia superior al 50% evoluciona fatalmente a Hepatitis fulminante c) La transmisión por vía sexual es actualmente la más importante en nuestro país d) El riesgo de evolución a la cronicidad es muy elevado, superando el 70% de los casos

442: ¿Cuál es el diagnóstico de un paciente con AgHBs( +), AntiHBc( +), AgHBe( +) y bioquímica hepática normal?:

a) Hepatitis B aguda b) Hepatitis B crónica c) Efecto ventana d) Portador sano

443: En el contexto del diagnóstico de infección por el virus de la Hepatitis C (VHC), SEÑALA LA RESPUESTA EN EL SUERO en un paciente inmunocompetente, simple portador del virus desde hace un año:

a) EUSA(+); RlBA(+); ARN VHC(+); ALT-GPT elevada b) EUSA(+); RlBA(+); ARN VHC(+); ALT-GPT normal c) EUSA(+); RlBA(+); ARN VHC(-); ALT-GPT normal d) EUSA(+); RlBA(-); ARN VHC(-); ALT-GPT normal

444: Una pauta eficaz en el tratamiento de la Brucelosis puede ser la asociación de:

a) Doxiciclina y Estreptomicina b) Penicilina procaína y Estreptomicina c) Doxiciclina y cefalosporinas de tercera generación d) Ciprofloxacino y Gentamicina

445: La localización MAS FRECUENTE de la Brucelosis es:

a) Meningitis b) Orquiepididimitis c) Sacroileitis d) Endocarditis

446: Entre las principales manifestaciones clínicas de la FIEBRE BOTONOSA MEDITERRÁNEA están todas las siguientes, EXCEPTO:

Page 38: Cuestionario de Urología

a) Fiebre b) Exantema c) Cefalea d) Parestesias

447: Señale cual de los siguientes NO CONSTITUYE un factor predisponente en la valoración de riesgo tromboembólico de pacientes no ingresados:

a) Puerperio b) Cesárea c) Terapia hormonal sustitutiva d) Anovulatorios orales

448: De los siguientes enunciados, ¿cuál ES CIERTO en la diferenciación entre cianosis central y periférica?:

a) La periférica no desaparece al frotar o calentar la piel b) La periférica aparece en las partes superficiales frías c) En la periférica suele haber poliglobulia compensadora d) La saturación arterial de oxígeno es superior al 85% en la central

449: ¿Cuál es el efecto del Acido Acetilsalicílico en pacientes con angina de esfuerzo estable?:

a) Solo tiene efecto si se asocia a betabloqueantes b) Reduce la incidencia de infarto de miocardio y muerte súbita c) No está indicado en este tipo de pacientes d) Sólo las dosis elevadas (500 mg.) consiguen un efecto beneficioso

450: La angina mixta:

a) Es aquella en la que coexisten la de reposo y la de esfuerzo sin predominio llamativo de ninguna de las dos b) Es la que se presenta de forma espontánea sin relación aparente con los cambios de consumo de oxígeno del miocardio c) Es la que se produce por un mecanismo de vasoespasmo coronario sin lesión anatomopatológica d) Es aquella de reciente comienzo pero que permanece clínicamente estable

451: ¿En cuál de los siguientes procesos NO ESTARIA indicada la profilaxis de Endocarditis Infecciosa?:

a) Prolapso mitral con insuficiencia b) Prótesis valvular c) Infarto agudo de miocardio d) Miocardiopatía hipertrófica obstructiva

452: Entre los factores de riesgo cardiovascular NO MODIFICABLE se incluye:

a) Dislipemia . b) HTA (Hipertensión arterial) c) Tabaquismo d) Menopausia

453: ¿Cuál de los siguientes fármacos es DE ELECCION en la Hipertensión sistólica aislada del anciano?:

a) Atenolol b) Tiazida c) Verapamilo d) Losartán

454: Uno de los siguientes grupos de fármaco s está ABSOLUTAMENTE CONTRAINDICADO en la Hipertensión arterial de la gestante debido a su propensión a producir insuficiencia renal e hipotensión neonatal. Señale cual:

a) Metildopa b) Diuréticos c) Betabloqueantes d) IECAs

455: Un mejor control de la Hipertensión Arterial lleva, ante todo, a disminuir la mortalidad por:

a) Accidentes vasculares cerebrales b) Insuficiencia respiratoria c) Insuficiencia cardiaca congestiva d) Cardiopatía isquémica

456: La actitud terapéutica a seguir en una mujer de 50 años con tensiones arteriales mantenidas de 150/95 mm.Hg. sin afectación de órganos dianas y con dislipemia será:

a) Modificación del estilo de vida y tratamiento con Carvedilol b) Modificación del estilo de vida y seguimiento durante seis meses c) Modificación del estilo de vida y tratamiento con cualquier antihipertensivo de primera línea d) Modificación del estilo de vida y tratamiento con Calcio antagonista

457: Un paciente de 60 años diagnosticado de Hipertensión Arterial hace 10 años, en tratamiento con Captopril a dosis de 50 mg. diarios presenta una TA de 150-90, ECG compatible con Hipertrofia de ventrículo izquierdo, Test de microalbuminuria positivo y hemorragias en el fondo de ojo. Con respecto a la clasificación de la OMS de la HTA respecto al grado de repercusión visceral, este paciente se encuentra en:

a) Fase I b) Fase II c) Fase III

Page 39: Cuestionario de Urología

d) Fase IV

458: ¿Cuál de las siguientes patologías NO SE ASOCIA a la obesidad?:

a) Hipoventilación alveolar b) Hipertensión arterial c) Artrosis d) Ulcus péptico

459: Ante un nódulo tiroideo, ¿cuál de las siguientes características NO ES SUGESTIVA de malignidad?:

a) Crecimiento lento b) Aparición en varones menores de 30 años c) Asociación a disfonía d) Tamaño mayor de 4 cm.

460: De los siguientes, ¿cuál es el síntoma o signo MAS FRECUENTE en un paciente afecto de Hipertiroidismo?:

a) Fibrilación auricular b) Eritema palmar c) Nerviosismo o intranquilidad d) Caída del cabello

461: Respecto a las dietas en las Dislipemias, las siguientes afirmaciones son ciertas, SEÑALE LA OPCION FALSA:

a) Se debe mantener colesterol en la dieta mayor de 1.000 mg al día b) Se recomienda que las grasas saturadas cónstituyan menos del 10% de las calorías de la dieta c) Consumos bajos de alcohol no son peIjudiciales d) El aumento de las fibras puede contribuir a disminuir algo la Dislipemia

462: ¿Cuál de las siguientes circunstancias puede producir un Síndrome de Cushing por producción ectópica de ACTH?:

a) Tuberculosis b) Sarcoidosis c) Abetalipoproteinemia d) Carcinoma pulmonar de células pequeñas

463: La educación sanitaria en la Diabetes es fundamental. Señale cual de las siguientes respuestas le parece INCORRECTA:

a) Tiene como finalidad potenciar el autocuidado y la autonomía del paciente b) Aunque mejora el control, no se ha demostrado que reduzca complicaciones (coma diabético, amputaciones, días de hospitalización) c) Es un proceso que debe ser continuado y evaluado periódicamente d) La educación diabetológica se proporcionará a todas las personas con diabetes y a sus familiares

464: El ejercicio físico constituye uno de los pilares del tratamiento de la Diabetes Mellitus. ¿Cuál de los siguientes enunciados sobre el ejercicio físico en diabéticos es FALSO?:

a) Contribuye a reducir el peso del paciente b) Mejora el control metabólico a medio y largo plazo c) Incrementa las concentraciones basales y postprandiales de insulina d) En diabéticos descompensados debe desaconsejarse hasta conseguir un control razonable

465: ¿Cuál de los siguientes es un edulcorante NO NUTRITIVO muy indicado, por tanto, en la dieta del paciente diabético?:

a) Aspartamo b) Fructosa c) Sorbitol d) Manitol

466: ¿A partir de qué cifra de triglicéridos se dará tratamiento farmacológico en un paciente con diabetes y cardiopatía isquémica?:

a) 150 mg/dl b) 200 mg/dl c) 250 mg/dl d) 300 mg/dl

467: Mujer de 55 años de edad, con diagnóstico de obesidad moderada y diabetes en tratamiento con antidiabéticos orales en la que observamos una discordancia importante entre las cifras de glucemia obtenidas de forma aleatoria (elevadas) y las que reflejan en la cartilla de autocontrol. La forma más adecuada de conocer el nivel de control de su diabetes en los 2-3 últimos meses es:

a) Medir glucosuria y cetonuria en orina de 24 horas b) Medir la concentración de Hemoglobina glicosilada c) Hacer un perfil de seis puntos dos días por semana d) Hacer una determinación plasrnática de péptido C

468: La hiperglucemia reactiva a hipoglucemia subclínica, generalmente nocturna, y que se manifiesta por situaciones de hiperglicemia intensa y aislada en pacientes con control habitualmente estable DEBE HACER PENSAR EN:

a) Alergia insulínica b) Pauta de insulina con dosis escasas de la misma c) Resistencia insulínica d) Fenómeno de Somogyi

469: Señale el factor de riesgo de accidente cerebrovascular MAS IMPORTANTE:

Page 40: Cuestionario de Urología

a) Enfermedad cardiaca b) Obesidad c) Hipertensión arterial d) Hipercolesterolemia

470: De las siguientes afirmaciones sobre el accidente isquémico transitorio (AIT) señale cuál es ERRONEA:

a) 30-40% de los pacientes con AIT desarrollan un ictus establecido en los cinco años siguientes b) La duración del episodio isquémico es mayor de 24 horas c) El pronóstico vital es peor si se asocia a cardiopatía isquémica d) Puede ser preciso el tratamiento quirúrgico

471: Ante un paciente de 32 años que es visto de urgencia en domicilio por presentar pérdida de conciencia tras cefalea intensa. En la exploración se evidencia rigidez de nuca, movimientos oculares a la rotación pasiva de la cabeza y estado estuporoso. El diagnóstico MAS PROBABLE será:

a) Hemorragia subaracnoidea b) Hemorragia intracerebral c) Intoxicación por plomo d) Meningitis bacteriana

472: ¿Dónde situaría la lesión de un paciente cuya mirada conjugada está desviada hacia el lado de los miembros paréticos?:

a) Lesión frontal b) Lesión del área pretectal c) Lesión parieto-occipital d) Lesión del tegmento pontino

473: Un dolor epigástrico de carácter sordo, bien delimitado de predominio postpandrial que cede con la ingesta SUGIERE:

a) Pancreatitis b) Esofaguitis con reflujo c) Hernia de hiato d) Ulcus duodenal

474: Con respecto a la úlcera duodenal ES CIERTO que:

a) Tiene un alto índice de malignización b) El método diagnóstico de elección es la demostración de la infección por Helicobacter pylori c) La edad típica de aparición es entre los 40 y 70 años d) En su etiopatogenia está implicado con frecuencia el Helicobacter pylori

475: Se denomina diarrea crónica a la diarrea que:

a) Se prolonga tras 48 horas de ayuno absoluto b) Dura más de 8 días c) Persiste más de 2-3 semanas d) Recidiva después de haber cesado

476: Varón de 37 años que desde hace doce horas presenta de forma insidiosa dolor epigástrico que ha aumentado de intensidad de manera continua. Hace dos horas antes de la consulta comenzó a presentar náuseas y vómitos. Tiene antecedentes de abuso de alcohol. A la exploración física hay dolor a la palpación en epigastrio. La amilasa sérica es de 1.050U/l. La radiografía simple de abdomen muestra niveles hidroaéreos en intestino delgado. El diagnóstico MAS PROBABLE será:

a) Gastritis aguda b) Pancreatitis aguda c) Ulcera péptica perforada d) Colecistitis aguda

477: Señale que afIrmación ERRONEA con respecto al cáncer colo-rectal:

a) Los pacientes con poliposis familiar tienen una elevada predisposición a desarrollar cáncer b) El cáncer de colon izquierdo se manifiesta generalmente por rectorragias y cambio de ritmo intestinal c) El cáncer de colon derecho suele cursar con anemia crónica por sangrado oculto d) El antígeno carcinoembrionario se considera un buen marcador para el diagnóstico de la enfermedad

478: Las metástasis del cáncer de colon, asientan con MAYOR FRECUENCIA en:

a) Hígado b) Pulmón c) Hueso d) Cerebro

479: En cuanto al cáncer de colon izquierdo, el síntoma MAS FRECUENTE es:

a) Anemia crónica b) Rectorragia c) Diarrea d) Tenesmo

480: ¿Cuál de las siguientes afirmaciones ES CIERTA sobre el diagnóstico de cáncer de mama?:

a) El examen clínico posee una alta sensibilidad b) La autoexploración mamaria ha demostrado utilidad para la detección en edades tempranas c) La mamografía es una técnica indolora cuya sensibilidad es del 100%. d) La ecograffa es la técnica de elección en mujeres menores de 25 años

Page 41: Cuestionario de Urología

481: ¿Cuál de las siguientes medidas contribuye EN MAYOR GRADO a aumentar las expectativas de vida del paciente con EPOC? (Enfermedad Pulmonar Obstructiva Crónica):

a) El abandono del hábito tabáquico b) La oxigenoterapia cuando existe insuficiencia respiratoria crónica c) Los corticoides orales d) Los fárrnacos broncodilatadores

482: ¿Cuál de las siguientes circunstancias NO ES UNA CAUSA de reagudización de la EPOC? (Enfermedad Pulmonar Obstructiva Crónica):

a) Infecciones agudas b) Hipertensión no complicada c) Broncoespasmo grave d) Neumotórax a tensión

483: ¿Cuál de las siguientes pruebas funcionales respiratorias SE UTILIZA para medir la obstrucción bronqnial en un asmático?:

a) Registro domiciliario de flujo espiratorio máximo b) Prueba de broncodilatadores c) Prueba de provocación bronquial d) Espirometría forzada

484: El tratamiento MAS EFICAZ a largo plazo para el Cor Pulmonale es:

a) La corrección de la hipoxia mediante oxigenoterapia domiciliaria b) La utilización de antibióticos para prevenir sobreinfecciones c) La disminución de la precarga con diuréticos d) El incremento de la contractilidad cardíaca mediante digitálicos

485: ¿Cuál de los siguientes índices espirométricos se considera la medida MAS SENSIBLE de obstrucción aérea precoz, especialmente en las pequeñas vías?:

a) FVC capacidad vital forzada b) FEVl volumen espirado en el primer segundo de la espiración forzada c) MEF 50% flujo máximo al 50% de la FVC d) FEF 25-75% flujo espiratorio entre el 25 yel 75% de la FVC

486: De los siguientes antibióticos señale cuál es el MENOS INDICADO para el tratamiento empírico de una Pielonefritis aguda:

a) Trimetropin-sulfametoxazol b) Norfloxacino c) Gentamicina d) Ampicilina

487: Ante un paciente con hematuria confirmada, leucocituria positiva y proteinuria negativa SOLICITARA:

a) Biopsia renal b) Microalbuminuria c) Urocultivo d) Calciuria

488: ¿Cuál de los siguientes procesos NO ES un factor de riesgo para la aparición de Litiasis Renal?:

a) Hipercalciuria idiopática b) Hiperuricosuria c) Enfermedad litiásica cálcica idiopática d) Hipoparatiroidismo primario

489: Respecto a la orquiepididimitis en el contexto clínico de la parotiditis, ES CIERTO que:

a) Mecta fundamentalmente a adultos mayores de 30 años b) En el 90% de los casos es bilateral c) Se instaura al ceder la tumefacción parotídea d) El dolor y la tumefacción tardan más de 2 meses en resolverse

490: La realización de citología vaginal para el screening de carcinoma de cérvix uterino, NO ESTA INDICADA en las mujeres:

a) Mayores de 50 años b) Nulíparas c) Menores de 25 años d) Que no han mantenido relaciones sexuales completas

491: El tratamiento de las vaginitis por tricomonas es:

a) Nistatina b) Amoxicilina c) Amoxicilina yAcido clavulánico d) Metronidazol

492: Una vez descartado el embarazo, ¿cuál de las siguientes situaciones se considera criterio diagnóstico de menopausia?:

a) Mujer asintomática con más de 7 meses de amenorrea b) F.S.H. menor de 20 mulml c) Más de 6 meses de amenorrea y clínica de sofocos d) Reglas muy abundantes en los 6 meses anteriores y amenorrea desde hace 3 meses

Page 42: Cuestionario de Urología

493: A una mujer de 22 años, gestante de 36 semanas, sin antecedentes de interés, sin riesgó obstétrico y asintomática, se le realiza una toma de exudado vaginal y anorectal, para cultivo de estreptococo agalactiae, también llamado estreptococo grupo B (EGB). SEÑALE LA OPCION FALSA entre las siguientes explicaciones sobre el EGB en la gestante:

a) Suele colonizar la vagina y ano-recto en un 10- 15% de las embarazadas. b) Produce sepsis neonatal precoz en un 1-3 por 1.000 recién nacidos c) A todas las embarazadas se debe realizar cultivo de vagina y ano-recto para EGB d) Durante el embarazo se debe erradicar con amoxicilina o eritromicina oral

494: ¿Cuál de las siguientes es la actuación MAS CONVENIENTE ante un paciente que consulta de urgencias por derramamiento de líquido caústico en los ojos?:

a) Derivar urgentemente al oftalmólogo b) Irrigar abundantemente con agua c) Explorar e instilar colirio antiséptico d) Averiguar la composición del producto antes de tratar

495: La faceta MAS IMPORTANTE del tratamiento de la retinopatía diabética es:

a) Diagnóstico precoz b) Adecuado control de la Diabetes c) Laserterapia precoz d) Cirugía de la retina

496: En su consulta valora a un paciente que presenta a la exploración del ojo derecho hiperemia, fotofobia y lagrimeo, sin sensación de cuerpo extraño ni secreción purulenta ni alteraciones pupilares. Su diagnóstico sería:

a) Conjuntivitis b) Iritis c) Queratitis d) Glaucoma agudo

497: ¿Qué signo o síntoma NO ES DE ESPERAR que se presente en un paciente con iritis?:

a) Miosis b) Agudeza visual normal c) Fotofobia moderada d) Dolor ocular moderado

498: Habitualmente, el germen responsable de la Otitis Externa es:

a) Aspergillus Niger b) Streptococcus Pneumoniae c) Pseudomona Aeruginosa d) Haemophilus Influenzae

499: El síntoma más precoz y específico de una Otomicosis es:

a) Otalgia b) Fiebre c) Prurito d) Otorrea

500: ¿En cuál de las siguientes circunstancias NO ESTA INDICADA realizar una amigdalectomía?:

a) Abceso periamigdalar b) Portador de difteria c) Amigdalitis frecuentes de repetición causante de convulsiones febriles d) Amigdalitis aguda asociada a faringitis y fiebre elevada

501: Un varón de 52 años, sin antecedentes de interés, tras una infección vírica respiratoria, presenta de forma aguda un vértigo periférico intenso, continuado, sin pérdida de audición, que se resuelve en unos pocos días sin recidivas. ¿Señale el diagnóstico MAS PROBABLE?:

a) Vértigo posicional paroxístico b) Colesteatoma c) Otitis exudativa d) Neuronitis vestibular

502: ¿Cuál de los siguientes fármacos o sustancias está considerado como un fibrinolítico y NO como un antiagregante o anticoagulante?:

a) Estreptoquinasa b) Vitamina K c) Heparina d) Acido acetilsalicílico

503: Una reacción leucemoide de tipo linfocítico puede aparecer en la siguiente patología:

a) Mononucleosis infecciosa b) Tuberculosis c) Cáncer de mama d) En todas las anteriores

504: Un paciente de 28 años asintomático que presenta un ganglio duro, móvil e indoloro en región supraclavicular le hará sospechar EN PRIMER LUGAR:

Page 43: Cuestionario de Urología

a) Adenitis por germen banal b) Linfogranulomatosis benigna c) Linfoma No Hodgkiniano d) Enfermedad de Hodgkin

505: Las verrugas son lesiones cutáneas causadas por:

a) Hipersensibilidad b) Papovavirus c) Herpes virus d) Estreptococo

506: Con respecto al tratamiento tópico con corticoides, todas las afirmaciones son correctas, EXCEPTO:

a) En los niños es preferible utilizar corticoides de baja potencia b) En adolescentes y ancianos es conveniente utilizar ciclos cortos de potencia intermedia c) No utilizar corticoides de potencia elevada en cara y pliegues d) No utilizar corticoides de potencia elevada en lesiones liquenificadas

507: Señale cuál de las siguientes enfermedades sistémicas NO ESTA RELACIONADA con el prurito generalizado sin lesiones:

a) Enfermedad colestásica b) Insuficiencia Renal c) Diabetes Mellitus d) Policitemia Vera

508: El acné vulgar es una enfermedad frecuente del folículo pilosebáceo, de causa desconocida y patogenia multifactorial. Con relación a su tratamiento señale la opción VERDADERA:

a) Los corticoides de potencia media pueden utilizarse en las lesiones de predominio inflamatorio b) Los antibióticos orales de elección son la doxiciclina y las tetraciclinas c) El peróxido de benzoilo como tratamiento tópico debe utilizarse a una concentración del 20% d) Los derivados del ácido retinoico por vía oral están indicados como tratamiento de inicio del acné leve-moderado

509: ¿En qué patología no esperaría encontrar rigidez de nuca?:

a) Meningitis b) Artrosis cervical c) Hipertensión endocraneal d) Coma hipoglucémico

510: Uno de los siguientes fármacos es un antiepiléptico. Señale cuál:

a) Metisergida b) Metildopa c) Bromocriptina d) Lamotrigina

511: El concepto de epilepsia implica:

a) Crisis epilépticas auténticas recurrentes b) Pérdidas de conciencia sin causa conocida c) Crisis de Gran Mal d) Trastornos de la personalidad

512: Los tres signos cardinales de la Enfermedad de Parkinson son:

a) Temblor intencional, bradicinesia y espasticidad b) Temblor de reposo, hipercinesia y alteraciones del lenguaje c) Cefalea, temblor y rigidez d) Temblor de reposo, bradicinesia y rigidez

513: De los siguientes, ¿cuál es el trastorno psiquiátrico MAS FRECUENTE en Atención Primaria?:

a) Depresión b) Neurosis fóbica c) Trastorno bipolar d) Esquizofrenia

514: Todos los siguientes fármacos son ansiolíticos, EXCEPTO:

a) Clordiazepóxido b) Meprobrarnato c) Propanolol d) Espirolactona

515: Una creencia errónea basada en una apreciación incorrecta de la realidad es:

a) Una alucinación b) Una idea delirante c) Una desorganización del lenguaje d) Un comportamiento gravemente desorganizado

516: ¿Cuál de los siguientes factores NO SE ASOCIA a un mayor riesgo de suicidio?:

a) Edad avanzada

Page 44: Cuestionario de Urología

b) Viudedad c) Alcoholismo d) Trastorno obsesivo-compulsivo

517: ¿Qué se entiende por Contrabalanceo Emocional en la entrevista con un paciente agresivo?:

a) Utilizar sus argumentos erróneos para introducir nuestros puntos de vista con el fin de reconvertir su actitud b) Permitir que vacíe su agresividad mediante la alternancia de sus opiniones con nuestras argumentaciones c) Utilizar aspectos de nuestros encuentros previos donde le hemos podido ayudar para hacede ver lo inadecuado de su actitud d) Mantener una actitud asintótica con su agresividad que favorezca el vaciado completo de la actitud hostil

518: ¿Cuál de los siguientes hallazgos es diagnóstico de gota?:

a) Líquido sinovial inflamatorio b) Artritis con datos clínicos sugestivos., c) Radiografía con gran inflamación de partes blandas d) Tofos

519: Todos los que aparecen a continuación son factores de riesgo de la Osteoporosis, EXCEPTO UNO:

a) Menopausia precoz b) Edad avanzada c) Actividad física d) Abuso de alcohol y/o tabaco

520: Uno de los siguientes fárrnacos NO ESTA INDICADO en el tratamiento de la Osteoporosis. ¿Cuál?:

a) Estrógenos b) Difosfonatos c) Calcitonina d) Sales iodadas

521: En la artrosis ES CIERTO QUE:

a) Es característica la correlación clínico-radiológica b) El dolor mejora con la actividad, empeorando con el reposo c) El tratamiento de elección son los corticoides d) Suele respetar las articulaciones del codo, muñeca y tobillos

522: La localización típica de los nódulos de Heberden y Bouchard es:

a) Columna cervical b) Pies c) Rodilla d) Manos

523: ¿En cuál de estas situaciones está CONTRAINDICADA la Terapia Hormonal Sustitutiva?:

a) Menopausia precoz b) Menopausia fisiológica asociada a cardiopatía isquémica c) Diabetes d) Antecedentes de trombosis venosa espontánea

524: La caries es el elemento patogénico clave de las afecciones endodentales causantes de odontalgia, por ello disminuir su incidencia es el objetivo primordial del médico de Atención Primaria. La MEJOR MEDIDA para prevenida es:

a) Actividades de promoción de la salud b) Fluoración del agua potable c) Sellado de los molares (6-7 años, 12-13 años) d) Administración de gotas de flúor desde el nacimiento hasta la adolescencia

525: Una madre acude a consulta con su hijo de 9 meses por fiebre alta de 3 días de evolución que cede sólo parcialmente con antitérmicos, anorexia e irritabilidad. Hoy y coincidiendo con la desaparición de la fiebre le han salido "unas manchas". La exploración física es normal salvo discreta hiperemia faringoamigdalar y un exantema máculopapuloso de predominio en tronco. El diagnótico MAS PROBABLE de los siguientes será:

a) Sarampión b) Varicela c) Rubeola d) Exantema súbito

526: El tratamiento de elección en la Oxiuriasis de niños mayores de 2 años es:

a) Metronidazol b) Linano c) Clotrimazol d) Mebendazol

527: La sialorrea en el recién nacido debe hacer pensar en la posible asociación de:

a) Atresia bronquial b) Atresia esofágica c) Atresia intestinal d) Atresia bilateral de coanas

528: El período de incubación de la Varicela es:

Page 45: Cuestionario de Urología

a) 1-3 días b) 4-8 días c) 10-21 días d) 24-30 días

529: Ante un traumatismo craneo-encefálico, todos los siguientes son considerados signos de gravedad EXCEPTO:

a) Puntuación entre 10-11 de la escala de Glasgow b) Anisocoria c) Otorragia d) Equimosis en mastoides

530: Con respecto a la fractura de cadera señale la afirmación CORRECTA:

a) La extremidad afectada suele presentar acortamiento y rotación interna b) Constituye un factor de riesgo para el embolismo pulmonar c) Se presenta con mayor frecuencia en la quinta década de la vida d) No suele producir impotencia funcional

531: Ante un paciente heroinómano que presenta coma, miosis y parada respiratoria, LA PRIMERA MANIOBRA a realizar será:

a) Administración de Naloxona vía intravenosa b) Dar dosis sustitutiva de Metadona c) Administración de un bolo de corticoides vía intravenosa d) Ventilación asistida y maniobras de RCP (Reanimación Cardiopulmonar)

532: El síndrome MICROATROPÍNICO se caracteriza por todo lo siguiente EXCEPTO:

a) Alucinaciones b) Miosis c) Delirio d) Dolores abdominales

533: ¿Cuál de los siguientes elementos terapéuticos es el PRIMERO QUE SE UTILIZA en el tratamiento de la parada cardiorespiratoria?:

a) Oxígeno b) Lidocaína c) Atropina d) Adenosina

534: Debido al tono vagal muy incrementado que se asocia a las situaciones de Asistolia se recomienda el uso de un anticolinérgico como la:

a) Metilprednisolona b) Atropina c) Digital d) Ajmalina

535: La primera causa de muerte súbita en la electrocución accidental es:

a) Asistolia b) Fibrilación ventricular c) Insuficiencia renal aguda d) Edema cerebral

536: Una de las siguientes afirmaciones es FALSA en el tratamiento del quemado, señale cuál:

a) Siempre que sea posible se retirarán las ropas b) Se aplicará agua en la superficie quemada durante 5-10 minutos c) En quemaduras de tercer grado circunferenciales pueden ser preciso realizar una incisión lateral d) En quemaduras extensas no se debe utilizar la vía intravenosa para administrar morfina

537: La correcta actuación de urgencias requiere como equipamiento mínimo en los centros de salud que atienden esta eventualidad todo lo siguiente, EXCEPTO:

a) Analizador hematológico y bioquímico b) Material de aspiración orotraqueal c) Material de soporte ventilatorio y oxigenoterapia: d) Material para sondaje gastroduodenal y vesical

538: El fármaco de elección en la intoxicación por opiáceos es:

a) Metadona b) Naloxona c) Adrenalina d) Amiodarona

539: ¿Cuál es la vía de administración MAS ADECUADA para el tratamiento farmacológico de la crisis convulsiva?:

a) Intramuscular b) Subcutánea c) Intravenosa d) Vía oral

540: El fármaco de elección en la TAQUICARDIA PAROXÍSTICA SUPRAVENTRICULAR de ritmo regular, complejo estrecho,

Page 46: Cuestionario de Urología

QRS menor de 0,12 y bien tolerada es:

a) Digoxina b) Mexiletina c) Adenosina d) Adrenalina

541: El tratamiento correcto del STATUS EPILÉPTICO IMPLICA, una vez aplicadas las medidas de soporte vital básico, la administración como PRIMERA ELECCION de:

a) Fenitoína 15 mglkg diluida en suero fisiológico y a una velocidad de 0,75 mglkg/minuto b) Diazepan 20 mg intramuscular y nueva administración pasados 15 minutos si continúa el status c) HaloperidoI2,5-5 mg intravenoso en perfusión lenta a pasar en 5 minutos d) Diazepan intravenoso a razón de 0,2 mglkg a una velocidad de 2 mg/minuto

542: En un ensayo clínico controlado la variable dependiente es la tensión arterial (mm de Hg) y la independiente el tratamiento (nuevo/antiguo/placebo). La prueba estadística adecuada para el análisis es:

a) El contraste de proporciones. b) La X2. c) La regresión lineal. d) El análisis de la varianza.

543: En un estudio se comparan las tasas de mortalidad por cáncer de pulmón con el consumo per capita de cigarrillos anual, para las distintas Comunidades Autónomas. La técnica de análisis adecuada es:

a) Prueba de X2. b) Comparación de dos medias. c) Análisis de la varianza. d) Análisis de correlación.

544: Para comprobar la eficacia de un nuevo fármaco, se distribuyeron al azar hipertensos en dos grupos, uno que recibió el nuevo fármaco y otro que recibió el tratamiento habitual. La tensión arterial fue menor en el primer grupo. Se trata de un estudio:

a) Ensayo clínico controlado. b) Estudio de cohortes. c) Ensayo de intervención comunitaria. d) Ensayo de campo.

545: Se estudió el hábito tabáquico de madres en la consulta prenatal, correlacionando luego con el peso al nacer de sus hijos, para ver la relación bajo peso al nacer-hábito tabáquico de la madre. Se trata de un estudio:

a) Ensayo clínico. b) Estudio transversal. c) Estudio de cohortes. d) Estudio de casos y controles.

546: Cuál de los siguientes test estadísticos estaría mal aplicado:

a) Distribución normal y comparación de dos medias: T de Student. b) Distribución no normal y comparación de dos medias: Test de Wilcoson. c) Distribución normal y comparación de más de dos medias: Análisis de varianza. d) Son todos correctos.

548.: En la investigación de un nuevo fármaco la fase de experimentación clínica Fase 1 se caracteriza por:

a) Realizarse sobre una única especie animal. b) Realizarse sobre enfermos controlados y escogidos. c) Realizarse sobre enfermos frente a placebo. d) Realizarse habitualmente sobre voluntarios sanos.

549: La efectividad de un tratamiento mide:

a) La probabilidad de que una persona se beneficie de él en condiciones ideales. b) La eficacia de ese tratamiento con relación a su coste. c) La eficacia de ese tratamiento con relación a los riesgos que implica. d) La probabilidad de que una persona se beneficie de él en circunstancias reales.

550: El análisis coste-efectividad:

a) Calcula los beneficios en unidades monetarias. b) Calcula los beneficios en unidades naturales ligadas al programa de que se trate, como muertes evitadas o casos diagnosticados. c) Calcula los beneficios en términos de valor o utilidad para el paciente, como los años de vida ajustados por calidad. d) No tiene utilidad para tomar decisiones clínicas.

551: Se considera como óptima en la gradación de la calidad de la evidencia científica la obtenida mediante:

a) Estudios analíticos bien diseñados de cohortes o de caso-control. b) Ensayos controlados, bien diseñados y sin randomización. c) Series múltiples de tiempo con o sin la intervención. d) Al menos un ensayo diseñado apropiadamente, randomizado y controlado.

552: Un valor de 0,5 del índice de Kappa significa:

a) Mayor concordancia que la que cabría esperar por azar. b) Concordancia absoluta.

Page 47: Cuestionario de Urología

c) Igual concordancia que la que cabría esperar por azar. d) Menor concordancia que la que cabría esperar por azar.

553: Las normas de Vancouver son:

a) Normas de citación de referencias bibliográficas. b) Normas para la elaboración de un protocolo. c) Normas de presentación de manuscritos a las revistas biomédicas. d) Normas éticas de investigación.

554: El término outliers se refiere a:

a) La amplitud de la caja en el gráfico "box and whiskers". b) Los límites del intervalo de confianza. c) Los casos con valores muy extremos. d) Los niveles de incumplimiento.

555: Las expectativas de los usuarios que producen una mayor satisfacción cuando se cumplen son:

a) Expectativas de calidad expresadas. b) Expectativas de calidad esperadas. c) Expectativas de calidad inesperadas. d) Expectativas de calidad deseadas.

556: Cuando hablamos de un aspecto de la atención que se escoge como relevante, cuya medida nos proporciona información sobre el grado de calidad de ésta, no referimos a un:

a) Criterio. b) Factor. c) índice. d) Parámetro.

557: Para averiguar la hipotética relación o asociación entre dos variables cuantitativas continuas que siguen una distribución nonnal, se utiliza:

a) El test de correlación de Spearman. b) El test de correlación de Pearson. c) El análisis de contingencia de X2. d) El análisis de la varianza ANOV A.

558: La aplicación del conocimiento epidemiológico a la práctica clínica, nos permite en la prevención de riesgo cardiovascular actuar de forma preventiva sobre aquellos pacientes con mayor posibilidad de enfennar. Así, aplicando la escala de Framinghan, podemos decir:

a) Tienen riesgo cardiovascular alto los pacientes que reúnen una puntuación Framingham de 12 o superior. b) Tienen riesgo cardiovascular alto los pacientes que reúnen una puntuación Framingham de 22 o superior. c) El riesgo de padecer un evento cardiovascular en los próximos 10 años en los pacientes de alto riesgo es superior al 20%. d) El riesgo de padecer un evento cardiovascular en los próximos 10 años en los pacientes de alto riesgo es superior al 40%.

559: Qué tipo de carcinoma pulmonar es el más frecuente entre las personas no fumadoras:

a) Carcinoma escamoso. b) Carcinoma de células pequeñas. c) Adenocarcinoma. d) Carcinoma de células grandes.

560: Qué signos son más evidentes en la cirrosis hepática alcohólica que en las cirrosis hepáticas de otra etiología:

a) Desnutrición e hipovitaminosis. b) Hipertrofia parotídea. c) Retracción palmar de Dupuytren. d) Todas las respuestas son ciertas.

561: El tratamiento con ácido fólico en un paciente en fase de desintoxicación alcohólica en el que sospechemos deficiencia del mismo y que a su vez requiera tratamiento con Vitamina B 12, lo iniciaremos:

a) Al mismo tiempo que la vitamina B 12. b) Cuando en la analítica se nonnalice la garnma-GT. c) 15 días después de sufrir el paciente el último delirium tremens. d) 3 semanas después de iniciar el tratamiento con B 12.

562: Cuál de las siguientes preguntas no forma parte del test de CAGE en el diagnóstico del abuso de alcohol:

a) ¿Se ha sentido alguna vez mal o culpable por su costumbre de beber? b) ¿Alguna vez lo primero que ha hecho por la mañana ha sido beber para calmar sus nervios o para librarse de una resaca? c) ¿Ha tenido usted alguna vez la impresión de que debería beber menos? d) ¿Ha intentado dejar de beber en alguna ocasión?

563: Señale la respuesta incorrecta respecto de la Lidocaína:

a) Es el fánnaco de elección para suprimir arritmias ventriculares cuando se sospecha isquemia aguda de miocardio o angina inestable. b) No se recomienda su uso profiláctico rutinario para prevenir la fibrilación ventricular. c) Puede administrarse por vía endovenosa, intramuscular y endotraqueal. d) Aumenta la pendiente de despolarización de la fase 4 y suprime el automatismo en el sistema His- Purkinje.

564: Qué energía emplearía inicialmente para desfibrilar a un paciente de 80 Kg. de peso que presenta fibrilación ventricular:

Page 48: Cuestionario de Urología

a) 360 julios. b) 4 julios / Kg. de peso. c) 100 julios. d) 200 julios.

565: ¿Cuál de los siguientes no puede incluirse en el apartado de los criterios mayores de diagnóstico de la insuficiencia cardíaca (Estudio de Framingham)?:

a) Cardiomegalia. b) Edema agudo de pulmón. c) Dinea de esfuerzo. d) Galope por tercer ruido.

566: Con respecto a la hipertensión arterial (HTA) como factor de riesgo cardiovascular, ¿cuál de las siguientes afirmaciones le parece la respuesta correcta?

a) El riesgo relativo para el accidente vascular cerebral (AVd) trombótico es del orden de 7,5. b) El riesgo relativo para la insuficiencia cardíaca es del orden de 15. c) El riesgo relativo mayor es para la cardiopatía isquémica. d) No está demostrado que la HTA constituya un factor de riesgo principal para la cardiopatía isquémica.

567: Señale la respuesta correcta con respecto al ECG en la angina de pecho:

a) Un trazado estrictamente normal descarta casi con seguridad la existencia de cardiopatía isquémica. b) El ECG realizado en el momento del dolor anginoso tiene una sensibilidad de aproximadamente el 90%. c) La sensibilidad del ECG fuera de la crisis ,aríginosa es prácticamente cero. d) El ECG en el momento de dolor anginoso tiene poco valor diagnóstico.

568: La triada sintomática más característica del feocromocitoma es:

a) Cefalea, pérdida de peso con apetito conservado y palpitaciones. b) Ansiedad, diaforesis y palpitaciones. c) Dolor abdominal, fiebre y cefalea. d) Cefalea, palpitaciones y diaforesis.

569: Cuál de los siguientes IECAS no requiere reducción de dosis en un paciente hipertenso que presente insuficiencia renal:

a) Cilazapril. b) Fosinopril. c) Lisinopril. d) Quinapril.

570: Señale la respuesta INCORRECTA respecto de la diabetes gestacional:

a) Aumenta el riesgo de sufrimiento fetal. b) Aumenta el riesgo de muerte intrauterina. c) Aumenta el riesgo de macrosomía. d) Aumenta la incidencia de malformaciones congénitas.

571: Señale la causa MÁS FRECUENTE de hipoglucemias en los pacientes tratados con Sulfonilureas:

a) Disminución de la ingesta alimentaria. b) Existencia de una disminución de la función renal. c) Presencia de hepatopatía crónica. d) Toma de fármacos que potencian la acción de las sulfonilureas.

572: El llamado fenómeno isomórfico, cuando hablamos de Psoriasis, consiste en:

a) Aparición en la superficie de las lesiones de un piqueteado hemorrágico después de la eliminación de las escamas por raspado. b) Aparición de las lesiones únicamente en los pliegues. c) Aparición de las lesiones después de una infección faringoamigdalar de origen estreptocócico. d) Aparición de las lesiones en áreas que han sufrido un traumatismo.

573: Señale la respuesta INCORRECTA respecto del carcinoma basocelular:

a) Es el tumor cutáneo maligno más frecuente. b) Es más frecuente en personas de tez clara. c) La forma más frecuente es la esclerosante. d) El carcinoma basocelular metastásico es raro.

574: Señale la respuesta incorrecta respecto del queratoacantoma:

a) Suele aparecer como un tumor solitario. b) Aparece en personas maduras o ancianas. c) Se presenta como un nódulo con un cráter central lleno de material queratínico. d) Crece muy lentamente.

575: ¿Cuál de las siguientes vacunas está CONTRAINDICADA en pacientes con inmunosupresión severa no relacionada con el VIII?:

a) Polio oral. b) Polio parenteral. c) Hepatitis B. d) Neumococo.

Page 49: Cuestionario de Urología

576: En relación con el carcinoma de nasofaringe es cierto que:

a) Puede presentarse como una otitis secretora unilateral. b) Su origen se relaciona con el tabaquismo. c) El tratamiento es mediante radioterapia. d) Las respuestas "a" y "c" son ciertas.

577: Según la clasificación de las etapas del ciclo vital familiar de la OMS modificado, la IV etapa llamada de contracción corresponde a:

a) Desde que el primer hijo tiene 11 hasta el nacimiento del último hijo. b) Desde que el primer hijo abandona el hogar hasta que el último hijo abandona el hogar. c) Desde que el último hijo abandona el hogar hasta la muerte del primer cónyuge. d) Desde el nacimiento del último hijo hasta la separación del matrimonio.

579: Respecto al cuestionario APGAR familiar señale la respuesta incorrecta:

a) Consta de 5 preguntas. b) Se puntúa cada con O, 1 ó 2 puntos. c) Puntuaciones por debajo de 3 puntos, se corresponde con familias normofuncionantes. d) Sirve para poner al médico sobre la pista de una posible disfunción familiar, no para diagnosticar.

580: El índice de Katz es:

a) Una de las escalas utilizadas para valorar el grado de depresión del paciente. b) Una de las escalas utilizadas para valorar la situación de sobrecarga familiar. c) Una de las escalas utilizadas para valorar la capacidad funcional de una persona. . d) Una de las escalas para valorar el deterioro cognitivo de una persona.

581: Señale cuál de las siguientes respuestas no es causa de neutropenia:

a) Corticoterapia oral. b) Colagenosis. c) Fiebre tifoidea. d) Dipirona.

582: El diagnóstico serológico sistemático de la hepatitis B aguda en un paciente con síntomas compatibles se basa en:

a) HBcAc IgM + HBeAc. b) HBsAg + HBcAc IgM. c) HBsAg + HBeAc. d) HBcAc IgG + HBsAg + HBeAc.

583: Según elpeakflow metter, ¿a partir de qué variabilidad diaria se considera que existe obstrucción de vías aéreas?:

a) 5% - 10% b) 10% - 15% c) 15% - 20% d) 35%'

584: ¿Cuál de los siguientes principios es FALSO con respecto al tabaquismo?:

a) El riesgo de cardiopatía isquémica se reduce significativamente al cabo de uno o dos años de dejar de fumar. b) El colesterol de lipoproteinas de alta densidad, generalmente es más bajo en fumadores que en no fumadores. c) Los fumadores tienen más probabilidad que los no fumadores de tener un índice cintura-cadera aumentado. d) No se ha demostrado que los fumadores pasivos sufran un aumento del riesgo de infartos de miocardio.

585: Se acepta que la acción central-ánalgésica de los opioides se debe a su fijación a receptores mu, kappa y delta. Se distinguen, por tanto, los opioides en agonistas puros, agonistas parciales, agonistas-antagonistas y antagonistas puros. ¿Cuál de los siguientes opioides no es un agonista puro?:

a) Morfina. b) Metadona. c) Codeína. d) Naloxona.

586: ¿Cuál es el antiemético de elección para las náuseas y vómitos producidos por la morfina?:

a) Escopolamina. b) Domperidona. c) Haloperidol. d) Benzodiazepinas.

587: Todos los siguientes fármacos inhiben la lactancia, EXCEPTO:

a) Bromocriptina. b) Haloperidol. c) Levodopa. d) IMAO.

588: ¿Cuál de las siguientes situaciones no es una contraindicación para la terapia hormonal sustitutiva en la menopausia?:

a) Antecedentes de enfermedad tromboembólica. b) Hepatopatía activa grave. c) Hipertensión arterial. d) Sangrado genital inexplicado.

Page 50: Cuestionario de Urología

589: Pueden disminuir los niveles plasmáticos de contraceptivos orales los siguientes fármacos EXCEPTO:

a) Cefalosporinas. b) Cotrimoxazol. c) Eritromicina. d) Ácido acetilsalicílico.

590: ¿Cuál de los siguientes criterios se consideran diagnósticos de menopausia?:

a) Estar 9 meses sin la regla. b) Estar 5 meses sin la regla y con sofocaciones. c) Tener FSH inferior a 20 UIJL. d) Ninguna de las anteriores.

591: Una mujer postmenopáusica de 54 años. desarrolla hipertensión arterial. ¿Qué fármaco puede inducir equilibrio positivo de calcio y producir una densidad mineral ósea más elevada?:

a) Las tiazidas. b) Los betabloqueantes. c) Los lECA. d) Los bloqueantes de los canales del calcio.

592: El síndrome de supresión aguda por opiáceos en los pacientes con dolor crónico:

a) No se presenta aunque el paciente esté con dosis de hasta 1 g / día. b) Se evita utilizando opiáceos con características agonistaantagonista. c) Es infrecuente cuando se utiliza una pauta descendente. d) Se previene si se utilizan preparados de liberación prolongada.

593: Una mujer de 37 años presenta a la palpación un nódulo de 2,5 cm. de diámetro en el lóbulo derecho del tiroides, de consistencia dura, no dolorosa, sin adenopatías palpables, eco gráficamente sólido, que presenta calcificación punteada en radiografía de cuello. Marque la falsa entre las siguientes opciones:

a) La existencia de nódulo único es más sospechosa de neoplasia que la presencia de nódulo s múltiples. b) La calcificación punteada sugeriría la presencia de cuerpos de psamoma. c) El crecimiento rápido sin asociarse a dolor apuntaría a neoplasia. d) La elevación de la tiroglobulina sanguínea sería muy específica, para diagnóstico de carcinoma.

594: Una embarazada de 26 años de edad tiene bocio, pero clínicamente es eutiroidea. Las pruebas de función tiroidea revelan elevación leve del índice de tiroxina libre. La prueba que sería más apropiada para determinar si hay eutiroidismo o hipertiroidismo en esta mujer es:

a) Captación de yodo radiactivo. b) Garnmagrafía tiroidea con tecnecio. c) Prueba de supresión de T3. d) Prueba de estimulación con hormona liberadora de tirotropina (TRH).

595: En una mujer de 44 años de edad con una concentración subnormal de tiroxina sérica y antecedentes de tratamiento a los 29 años de edad con yodo radiactivo por enfermedad de Graves, ¿cuál de las siguientes sería la mejor prueba para confirmar una sospecha de hipotiroidismo primario?:

a) Concentración sérica de triyodotironina (T3). b) Concentración sérica de triyodotironina reversa (rT3). c) Concentración sérica de hormona estimulante del tiroides (TSH). d) Captación de yodo radiactivo en 24 horas.

596: La diferencia clínica fundamental entre hipotiroidismo por atrofia tiroidea idiopática y tiroiditis de Hashirnoto reside en:

a) Los títulos de anticuerpos antitiroideos. b) Los niveles de T3 y T4. c) Que la tiroiditis de Hashirnoto cursa con bocio y la atrofia tiroidea idiopática no. . d) Que la atrofia tifOidea idiopática cursa con bocio y la tiroiditis de Hashirnoto no.

597: Un paciente que consulta por disfagia de comienzo brusco, evolución y localización variable, no progresiva, igual para sólidos que para líquidos y que la maniobra de Valsalva logra el paso alimenticio, le sugiere:

a) Divertículo de Zenker. b) Hernia de hiato. c) Disfagia funcional. d) Cualquiera de las anteriores.

598: ¿Cuál de los siguientes criterios es más importante en cuanto al pronóstico de supervivencia en la cirrosis biliar primaria?:

a) Título de anticuerpo s antimitocondriales. b) Niveles de fosfatasa alcalina. c) Niveles de Ig. d) Niveles de bilirrubina.

599: Con respecto a la enfermedad por reflujo gastroesofágico, una de las siguientes afirmaciones es falsa:

a) Existen periodos de relajación transitoria que permiten el reflujo gastroesofágico potencialmente lesivo. b) No todos los pacientes con hernia de hiato tienen reflujo gastroesofágico. c) El Nifedipino puede provocar reflujo gastroesofágico. d) Cuando en el estudio endoscópico se objetiva la existencia de esofagitis, es preciso control endoscópico al finalizar el tratamiento.

600: Entre los fármacos capaces de producir parkinsonismo se encuentran los siguientes, EXCEPTO:

Page 51: Cuestionario de Urología

a) Haloperidol. b) Selegilina. c) Tioridazina. d) Levopromacina.

601: En relación con la migraña, indique cuál de las siguientes opciones es CIERTA:

a) Los anticonceptivos orales están contraindicados en las pacientes que tienen migraña con aura. b) El tratamiento inicial de la migraña debe hacerse con ergóticos. c) El sumatriptán es un fármaco de segunda elección para abortar una crisis de cefalea en racimos. d) El tratamiento de la crisis de migraña con sumatriptán previene las recidivas.

602: Señale la opción FALSA en relación con el tratamiento de la enfennedad de Parkinson:

a) Los fánnacos anticolinérgicos tienen una mayor eficacia sobre la bradicinesia que sobre el temblor. b) La L-Dopa es el fánnaco más eficaz. c) El efecto beneficioso de la amantadina desaparece tras 4 - 6 meses de tratamiento continuado. d) El tratamiento con L-Dopa no altera el curso evolutivo de la enfennedad.

603: En relación con la enfennedad de Parkinson, señale la opción FALSA:

a) El temblor aumenta con la ansiedad y cesa durante el sueño. b) El temblor es el síntoma más incapacitante. c) La rigidez predomina en las regiones proximales de extremidades y en cuello. d) Es frecuente la depresión.

604: En un paciente con sospecha de demencia y con un test de Hachinski de 9 puntos, ¿qué causa es la más probable?:

a) Degenerativa. b) Tóxica. c) Infecciosa. d) Vascular.

605: En la valoración de una migraña, ¿cuál de estos cambios nos ha de hacer sospechar la presencia de un proceso expansivo intracraneal?:

a) Cambios en las características de la cefalea. b) Aumento de la frecuencia de las crisis. c) Presencia de aura. d) Presencia de náuseas o vómitos.

606: En la valoración del deterioro cognitivo, una de las siguientes es FALSA:

a) En la alteración de la memoria asociada a la edad (AMAE), los tests cognitivos son nonnales. b) El deterioro cognitivo asociado a la edad condiciona problemas sólo para actividades complejas antes bien desarrolladas. c) Existe evidencia para recomendar el cribado sistemático en ancianos mediante cuestionarios. d) Debemos hacer una valoración mental y funcional.

607: La causa más frecuente de hirsutismo es:

a) Hirsutismo yatrogénico. b) Deficiencia de 21-hidroxilasa. c) Síndrome del ovario poliquístico. d) Hirsutismo idiopático.

608: Paciente de 33 años, hipertensa conocida en tratamiento con lECA que acude a la consulta por estar embarazada; una vez confumado el embarazo, la actitud más correcta respecto al tratamiento antihipertensivo sería:

a) Mantener el lECA por ser inocuo en el embarazo. b) Suspender el lECA y pautar un calcioantagonista por ser el fánnaco de elección durante el embarazo. c) Suspender el lECA y pautar alfa-metildopa por ser el fánnaco de elección durante el embarazo. d) Suspender el lECA y pautar hidroclorotiacida.

609: ¿Cuál de los siguientes casos no se considera factor de riesgo para padecer un cáncer de mama?:

a) Madre con cáncer de mama. b) Dieta rica en grasas. c) Primer embarazo después de los 35 años. d) Menopausia precoz.

610: ¿Cuál de los siguientes no es factor de riesgo para desarrollar cáncer de endometrio?:

a) Hiperplasia endometrial. b) Antecedente de infección por virus del papiloma humano. c) Hemorragias disfuncionales perimenopáusicas. d) Administración de estrógenos sin gestágenos.

611: En una mujer con hemorragia uterina anonnal, una de las siguientes afirmaciones es falsa:

a) En jóvenes menores de 20 años, la causa más frecuente es la presencia de ciclos anovulatorios. b) Desde la menarquia hasta pasados 1 - 2 años, los ciclos pueden ser irregulares. c) En la mujer perimenopáusica con diagnóstico de hemorragia uterina disfuncional, el tratamiento de elección son los gestágenos. d) La principal causa de la hemorragia en periodo perimenopáusico es el déficit estrogénico.

612: Una de las siguientes no es característica de la dismenorrea primaria:

Page 52: Cuestionario de Urología

a) Aparece poco después de la menarquia. b) Disminuye con la edad y la paridad. c) El tratamiento de elección son los progestágenos. d) La falta de respuesta al tratamiento nos hará sospechar una dismenorrea secundaria.

613: La fármaco-dependencia a opiáceos se caracteriza por una activación del sistema:

a) Péptidos vasoactivos. b) Histaminérgico. c) Doparninérgico. d) Serotoninérgico.

614: La escala de ansiedad y depresión de Goldberg (EADG) está referida a:

a) Ultimo año. b) Ultimo mes. c) Ultimos 15 días. d) Ultima semana.

615: En relación con el tratamiento de litio, es FALSO:

a). Es común iniciar tratamiento con dosis de 300 mg dos veces al día. b) En pacientes estables los niveles de litio deben ser medidos frecuentemente. c) Su administración debe ser mantenida por lo menos durante 6 meses tras la desaparición de síntomas. d) El nivel aconsejable en sangre es de 1,5 a 2 mmol/L.

616: Según CIE-1O para establecer el diagnóstico definitivo de trastorno de pánico es FALSO que:

a) Deben presentarse varios ataques de ansiedad al menos durante un mes. b) Deben presentarse en circunstancias en las que no hay un peligro objetivo. c) No deben presentarse sólo en situaciones conocidas o previsibles. d) Es muy infrecuente la ansiedad anticipatoria leve.

617: Según CIE-1O para el diagnóstico preciso de trastorno de somatización es necesaria la presencia de síntomas somáticos múltiples y variables sin una adecuada explicación somática que persista al menos durante:

a) 1 mes. b) 3 meses. c) 6 meses. d) 2 años.

618: En los resultados de las pruebas para investigar la función pulmonar en una EPOC, una de las siguientes aseveraciones es falsa:

a) Disminución del FEVl con reducción concomitante de la relación FEVIIFVc. b) La FVC puede ser normal o estar reducida. c) Capacidad pulmonar total normal o aumentada. d) Disminución del volumen residual.

619: Los siguientes fármacos pueden ocasionar fibrosis intersticial difusa excepto uno. Señálelo:

a) Indometacina. b) Nilutarnida. c) Metisergida. d) Amiodarona.

620: Una de las siguientes enfermedades no está entre las 5 que motivan, más frecuentemente, imposibilidad en los ancianos:

a) Diabetes. b) Enfermedad de Parkinson. c) Demencia. d) Accidentes cerebrovasculares (ACV).

621: Para valorar el grado de depresión en un anciano usaremos:

a) Miniexamen cognoscitivo (Minimental). b) Indice de Lawton y Brody. c) Escala de Yesavage. d) Indice de Katz.

622: Todos los siguientes son cambios que se producen en las funciones neuropsicológicas durante el envejecimiento, excepto:

a) La personalidad se modifica sustancialmente. b) La memoria sensorial y de fijación disminuye. c) La percepción disminuye. d) El tiempo de reacción se enlentece.

623: Acude una mujer de 73 años a su consulta porque, a veces, tiene necesidad imperiosa de orinar y si no lo hace se le escapa la orina. Señale el diagnóstico probable:

a) Vaginitis atrófica. b) Incontinencia por rebosamiento. c) Incontinencia de esfuerzo. d) Inestabilidad del detrusor.

624: En una anemia microcítica con Ferritina normal, ¿qué determinación analítica solicitaría-a continuación para su estudio?:

Page 53: Cuestionario de Urología

a) Reticulocitos. b) Vit. B12. c) Ac. Fólico. d) HbA2 Y HbF.

625: Ante una anemia normocítica con una cifra de reticulocitos corregidos baja, ¿cuál será su primera sospecha diagnóstica?:

a) Anemia por sangrado agudo. b) Anemia hemolítica. c) Anemia por enfermedad crónica. d) Anemia perniciosa.

626: La vitamina K es un factor necesario para la síntesis de todos los factores siguientes, excepto:

a) Factor VIII b) Factor V. c) Factor IX. d) Factor X.

627: Paciente que tras chasquido en región lumbar, sufre un dolor lumbar que le impide enderezarse, unas horas después, se añade un dolor en miembro inferior dcho. que irradia desde la nalga hasta la planta del pie pasando por la cara posterior de muslo y pierna. A la exploración hay Lassegue (+), existe hipoestesia del trayecto doloroso y reflejo aquíleo abolido. ¿Cuál es la raíz afectada?:

a) L4. b) L5. c) L4 y L5. d) S1.

628: En la enfermedad de Reiter es radiográficamente caracteóstica:

a) La erosión. b) La osteoporosis. c) La periostitis. d) La hiperóstosis.

629: ¿Cuál de las siguientes articulaciones no suele estar afectada en la artritis reumatoide?:

a) Metacarpofalángicas. b) Interfalángicas proximales. c) Metatarsofalángicas. d) Interfalángicas distales.

630: La afectación articular más frecuente en la psoriasis es:

a) Artritis monoarticular u oligoarticular asimétrica afectando pequeñas articulaciones de manos y pies. b) Poliartritis simétrica seronegativa similar a la artritis reumatoide. c) Mección de las articulaciones interfalángicas distales. d) Poliartritis destructiva grave tipo "artritis mutilan te".

631: Con relación a la infección por citomegalovirus (CMV) en los pacientes con SillA, es falso que:

a) La coriorretinitis es la forma más frecuente de presentación. b) La retinitis por CMV es la principal causa de ceguera. c) El CMV raramente está implicado directamente en la muerte de los pacientes. d) Ocasionalmente se prescribe tratamiento antiviral por vía intravítrea directa.

632: ¿Cuál de las siguientes afirmaciones, con respecto al efecto "booster", no es correcta?:

a) Se da en personas de edad avanzada, vacunadas o infectadas por micobacterias. b) El efecto "booster" se debe investigar en los vacunados mayores de 15 años y con una primera prueba de tolerancia menor de 5 mm. c) El efecto "booster" se debe investigar en todos los mayores de 65 años, vacunados o no vacunados, y con una primera prueba de tuberculina menor de 5 mm. d) Siempre se debe descartar este efecto en los VIH (+).

633: ¿Cuál es el patrón sero1ógico más característico de la Hepatitis B crónica, con replicación vírica activa?:

a) AgHBs, AgHBe y ADN-VHB (+). b) AgHBs, AntiHBe y ADN-VHB (+). c) AgHBs, y AntiHBc (+). d) Todos ellos.

634: De los fármacos antituberculosos que se mencionan a continuación, señale cual produce, en ocasiones, alteraciones oculares:

a) Isoniacida. b) Etionamida. c) Rifampicina. d) Etambutol.

635: ¿Cuál es el tumor que aparece con más frecuencia sobre lesiones tuberculosas antiguas?:

a) Carcinoma epidermoide. b) Carcinoma de células alveolares. c) Adenocarcinoma.

Page 54: Cuestionario de Urología

d) Carcinoma de células grandes.

636: La tuberculosis postprimaria se localiza con más frecuencia en:

a) Ganglios mediastínicos. b) Ganglios hiliares. c) Segmento paracardial del lóbulo inferior derecho. d) Segmento apical y posterior de lóbulos superiores.

637: La artritis tuberculosa presenta una de las siguientes características:

a) Poliarticular, afectando a articulaciones más pequeñas. b) Asociada a fiebre alta. c) Casi siempre de predominio vertebral. d) Predominio monoarticular con poca inflamación.

638: A nivel mundial, los primeros casos diagnosticados de SIDA se detectaron en el año:

a) 1980. b) 1981. c) 1982. d) 1983.

639: En referencia a la TBC y otras enfermedades causadas por micobacterias en pacientes con infección por el VIH, es falso que:

a) Un test de tuberculina (PPD) (-) no excluye el diagnóstico de TBc. b) Es frecuente la presentación extrapulmonar, con fiebre y adenopatías. c) El tratamiento clásico con isoniacida, rifampicina y piramicina se acompaña frecuentemente de fracaso terapéutico. d) En algunos pacientes se obtiene el diagnóstico por demostración de granulomas en la biopsia de médula ósea.

640: La realización de una serie de procedimientos previos a la determinación de PSA (antígeno prostático) en suero, alteraría las cifras de éste en suero. Con relación a estos procedimientos, cuál de las siguientes aseveraciones es falsa:

a) Tras una biopsia prostática se debe esperar un mes para realizar la determinación analítica. b) Es mejor realizar la determinación de PSA en suero antes de realizar un tacto rectal. c) La eyaculación no altera las cifras de PSA en suero. d) La resección transuretral aconseja posponer la realización de la determinación analítica un mes.

641: ¿Cuál de los siguientes datos de laboratorio no es un hallazgo frecuente en la insuficiencia renal?:

a) Hiperfosforemia. b) Hipoca1cemia. c) Anemia. d) Leucopenia.

642: Normalmente, una proteinuria es de origen glomerular cuando es:

a) Mayor de 4 gr. / 24 horas. b) Menor de 1 gr. / 24 horas. c) De 1 a 3,5 gr. / 24 horas. d) Menor de 150 mgr. /24 horas.

643: En un cuadro clínico determinado en el que encontramos una piuria estéril mantenida, el diagnóstico más probable será:

a) Pie10nefritis crónica. b) Pielonefrítis aguda. c) Tuberculosis urogenital. d) Todas las anteriores.

644: En relación a los test de Prick cutáneos:

a) Son test de hipersensibilidad inmediata. b) Demuestran la presencia de IgE fijada al tejido. c) Son equivalentes in vivo de la determinación de anticuerpos IgE específicos en el suero, aunque los resultados de ambos test no son siempre paralelos. d) Todas ellas son ciertas.

645: ¿Cuál de las siguientes expresiones es cierta?:

a) Intolerancia alimentaria: describe todas las reacciones anormales frente a los productos alimenticios. b) Enfennedad alérgica alimentaria: describe sólo las reacciones a los alimentos o aditivos alimentarios que se sabe que tienen una base inmunológica. c) Idiosincrasia alimentaria: describe una reacción adversa a los alimentos que se sabe que no es inmunológica. d) La "a", la "b" y la "c" son ciertas.

646: Indique cual de las siguientes aseveraciones respecto al adenocarcinoma prostático es correcta:

a) En el 95% de los casos es un tumor que se origina en las glándulas periuretrales. b) Es el segundo tipo histológico en frecuencia dentro del cáncer prostático precedido por el carcinoma epidennoide. c) En un 70% de los casos se origina en la zona prostática periférica. d) En su control juega un papel fundamental el conocimiento de los niveles de alfafetoproteina plasmática.

647: Respecto al carcinoma prostático es cierto:

a) Es la tercera causa de muerte por cáncer en el varón tras el cáncer de pulmón y el colorrectal. b) Generalmente es honnonorresistente. c) El patrón histológico más frecuente es el carcinoma epidennoide.

Page 55: Cuestionario de Urología

d) La adrenal es la localización metastásica más frecuente.

648: Señale la respuesta correcta respecto a la detección precoz del cáncer colorrectal mediante la determinación de sangre oculta en heces:

a) Es una prueba con una sensibilidad para el cáncer superior al 95% en todos sus estadios. b) Ha demostrado ser eficaz para disminuir la mortalidad por cáncer en algún estudio aleatorio controlado. c) Tiene una alta especificidad y un alto valor predictivo positivo para el cáncer colorrectal en nuestro medio. d) Es de las pruebas de cribado que cuenta con mayor aceptación por parte de los ancianos.

649: Un varón de 52 años, alcohólico, acude a consulta por una historia de diez días de febrícula diaria, pérdida de 3 kilos de peso y expectoración purulenta. En la RX de tórax se observa un infiltrado alveolar en LSD (segmento 3) y una imagen redondeada con nivel hidroaéreo en LID (segmento 6). Auramina en esputo negativa. En relación con el cuadro que presenta es cierto que:

a) Hay que realizar fibrobroncoscopia obligatoriamente para filial el gennen, dado el riesgo del paciente. b) El tratamiento deberá incluir un arninoglucósido y una cefalosporina de tercera generación Lv. c) El gennen causal será el Staphilococus aureus. d) Una pauta de tratamiento con clindamicina como único antibiótico se ha mostrado eficaz en el tratamiento de esta patología.

650: De las siguientes afinnaciones sobre la intoxicación por antidepresivos tricíclicos una es FALSA:

a) No producen convulsiones. b) Presentan efectos anticolinérgicos. c) Producen arritmias. d) No tienen antídoto.

651: En los pacientes diabéticos de larga evolución, especialmente cuando existe neuropatía autonómica, pueden producirse situaciones de profunda hipoglucemia sin que se asocien síntomas adrenérgicos acompañantes. ¿Cuál de los siguientes síntomas suele producirse con más frecuencia en esta situación?:

a) Temblor. b) Inquietud. c) Palpitaciones. d) Confusión.

652: Señale los fánnacos antihipertensivos desaconsejados en pacientes hipertensos con enfennedad vascular cerebral:

a) lECA. b) Diuréticos. c) Calcioantagonistas. d) ARA TI.

653: ¿Cuál de las siguientes combinaciones farmacológicas les parece más adecuada en un paciente de 72 años, diagnosticado de miocardiopatía dilatada con fracción de eyección del ventriculo izquierdo del 30%, en fibrilación auricular crónica, que se encuentra en situación funcional TII de la NYHA, en ausencia de otra patología sistémica asociada?:

a) Digoxina, aspirina y captopri!. b) Dicumarínicos, verapamil, furosemida y captopri!. c) Digoxina, dicumarínicos, espironolactona y captopri!. d) Digoxina, dicumarínicos, furosemida y captopril.

654: Señale la respuesta falsa respecto a la acción sobre la sensibilidad a la insulina de los diversos fármacos usados para el tratamiento de la hipertensión del paciente diabético:

a) Los bloqueadores alfa-adrenérgicos producen mayor insulinorresistencia. b) Los beta-bloqueantes disminuyen la sensibilidad a la insulina. c) Los diuréticos aumentan la insulinorresistencia. d) Los inhibidores de la ECA mejoran la sensibilidad a la insulina.

655: Una paciente de 23 años diagnosticada hace tres meses de diabetes mellitus tipo 1 se presenta por cuadro clínico de astenia, anemia, debilidad muscular e hipotensión. Finalmente encuentra usted un hipotiroidismo primario autoinmune e insuficiencia adrenal autoinmune. ¿Cuál sería su diagnóstico final?:

a) Neoplasia endocrina múltiple tipo L b) Neoplasia endocrina múltiple tipo IIA. c) Neoplasia endocrina múltiple tipo lIB. d) Síndrome de Schmidt.

656: En la nefropatía diabética constituida (excreción renal de albúmina superior a 300 mg. en 24 horas), ¿cuál de las siguientes respuestas es correcta?:

a) El uso de inhibidores de la enzima de conversión (lECA) para el tratamiento de la hipertensión no está indicado. b) La terapia insulínica intensiva produce regresión de afectación renal. c) La restricción proteica moderada en la dieta ha demostrado que produce un enlentecimiento de la evolución hacia la insuficiencia renal. d) Los diuréticos son los fármacos de elección para el tratamiento de la hipertensión en estos pacientes.

657: Un paciente diabético insulinodependiente nos trae a la consulta sus perfiles glucémicos, donde se observa de forma repetitiva la presencia de hiperglucemia basal. Todas las siguientes medidas son correctas excepto:

a) Preguntar por la presencia de síntomas de hipoglucemia en la madrugada. b) Solicitar al paciente que realice monitorización de la glucemia a las 3 a.m. para diferenciar el efecto Somogyi del fenómeno del alba. c) Incrementar la dosis de insulina de acción retardada nocturna, en el caso de que se trate de un fenómeno del alba. d) Incrementar la dosis de insulina de acción rápida matutina para compensar la hiperglucemia.

Page 56: Cuestionario de Urología

658: ¿Cuál de las siguientes asociaciones es más rara en la rosácea?:

a) Colitis. b) Rinofima. c) Conjuntivitis. d) Queratitis.

659: Varón de 30 años que presenta una úlcera genital no -indurada dolorosa en el prepucio que había padecido en otras ocasiones. Es más probable que se trate de:

a) Sífilis. b) Chancroide. c) Granuloma inguinal. d) Herpes genital.

660: La forma clínica más frecuente de melanoma maligno es:

a) Melanoma de extensión superficial. b) Melanoma nodular. c) Melanoma lentiginoso acral. d) Léntigo maligno melanoma.

661: Un varón de 60 años con antecedentes de diabetes mellitus, alcoholismo y gota acude a urgencias por un cuadro clínico de fiebre, escalofríos, hematuria y dolor en flanco izquierdo. En la pielografía se visualiza una imagen de "sombra en anillo". El diagnóstico más probable es:

a) Absceso renal. b) Tumor renal. c) Pielonefritis aguda. d) Necrosis papilar.

662: En la neuronitis vestibular no se produce:

a) Nistagmo. b) Hipoacusia. c) Vértigo. d) Ataxia.

663: Paciente que acude a urgencias por haber presentado fotopsias durante días y a continuación un escotoma progresivo con afectación de la visión central y periférica. ¿Cuál de los siguientes no sería un factor favorecedor de este cuadro?:

a) Miopía magna. b) Senilidad. c) Traumatismo ocular previo. d) Glaucoma crónico simple.

664: Uno de los siguientes no es un hallazgo propio de la exploración inicial de un ojo con un ataque agudo de glaucoma por cierre angular:

a) Presión intraocular muy elevada. b) Edema corneal. c) Contacto del iris periférico con la córnea. d) Pupila miótica débilmente reactiva.

665: Mujer de 46 años que acude a urgencias presentando un ojo rojo.con inyección ciliar, dolor y disminución de agudeza visual. No presentaba secreciones ni defecto epitelial corneal, además tenía una miosis, una presión intraocular de 45 mmHg y Tyndall (+). El diagnóstico más probable es:

a) Glaucoma agudo. b) Uveitis anterior aguda. c) Queratitis herpética. d) Conjuntivitis adenovírica.

666: La mononeuropatía que puede aparecer en enfermos diabéticos se caracteriza por:

a) Ser más frecuente que la polineuropatía periférica. b) La afectación del III par es incompleta, es decir, se produce ptosis y oftalmoplejia pero la respuesta pupilar es normal. c) Rara reversibilidad espontánea. d) Ser bilateral y simétrica.,::

667: En epidemiología general (salvo en cardiovascular) el cribaje es un procedimiento asociado a:

a) Prevención primaria. b) Prevención secundaria. c) Prevención terciaria. d) Atención curativa.

668: El tratamiento fannacológico del alcoholismo con disulfirarn tiene una serie de contraindicaciones absolutas y relativas. Indique cuál de las siguientes es una contraindicación absoluta:

a) Cirrosis hepática. b) Diabetes mellitus. c) Alteraciones cardiovasculares. d) Hipotiroidismo.

669: Paciente varón de 20 años que acude a la consulta con TA d_ 1901 100. En la analítica aparecen los siguientes datos en sangrel orina: urea 40/ 1100, creatinina 1 '2/120, Na 139/45, K 3'21 39, CI 100/20. ¿Cuál de las siguientes proposiciones es

Page 57: Cuestionario de Urología

correcta?:

a) La analítica es incompatible con el síndrome de Liddle. b) Es compatible con la toma de regaliz o de carbenoxolona. c) Es compatible con síndrome de Bartter, debido a las pérdidas inadecuadas de K en la orina. d) Lo más probable es la torna de diuréticos que provocarán hiperactividad simpática post hipovolemia e HTA secundaria.

670: Señalar cuál de los siguientes síntomas no es característico del trastorno de ansiedad generalizado:

a) Temores hipocondríacos. b) Disfunción autonómica. c) Cefalea tensional. d) Insomnio distal.

671: Señale la respuesta incorrecta sobre las acciones de los AINE acídicos:

a) La acción analgésica se realiza fundamentalmente a nivel periférico. b) Disminuyen la temperatura (T".) cuando se encuentra elevada. c) A dosis bajas favorecen la eliminación de ácido úrico. d) Inhiben la agregación plaquetaria.

672: Las úlceras en los pies de pacientes diabéticos son muy frecuentes, teniendo en general dos causas principales (aunque frecuentemente se solapan): angiopática (por macroangiopatía) y neuropática (por neuropatía). ¿Cuál de los siguientes hallazgos no esperaría encontrar en un úlcera exclusivamente angiopática?:

a) Localización en el talón. b) Dolorosa. c) Pulsos distales disminuidos. d) Ausencia de vello en el pie.

673: Los problemas pediátricos claramente asociados al uso de alcohol son los indicados a continuación excepto uno:

a) El síndrome alcohólico fetal. b) La cirrosis hepática infantil c) La deprivación afectiva. d) La mayor proporción de accidentes de tráfico en adolescentes.

674: La traqueotornía de extrema urgencia se debe efectuar a nivel de:

a) Espacio preepiglótico. b) Membrana cricotiroidea. c) Glotis. d) Espacio supraglótico.

675: No se deben usar sustitutivos de nicotina en:

a) Enfermos polimedicados. b) Diabetes Mellitus. c) Insuficiencia renal. d) Ulcera péptica activa.

676: En una mujer de 42 años con un sobrepeso del 60 % que consulta por poliuria, se detecta una glucemia a las 18 h de 300 mgldL, sin cetonuria. ¿Cuál será su actitud?:

a) Diagnosticar una DMNID e iniciar dieta e insulina a dosis bajas. b) Citarla para glucemia basal a fin de confirmar el diagnóstico e iniciar dieta hipocalórica. d) Diagnosticar una DMNID e iniciar una sulfonilurea a dosis bajas. d) Recomendar ingreso para estudio y tratamiento.

677: El alcohol produce diversas formas de hepatopatía. Señale la afirmación falsa respecto a ellas:

a) La más frecuente es la esteatosis hepática. b) Las mujeres son más susceptibles a la toxicidad hepática del alcohol. c) La hepatitis alcohólica no es siempre reversible, y puede conducir al desarrollo de cirrosis. d) La cirrosis alcohólica rara vez se complica con el desarrollo de un hepatocarcinoma.

678: La anemia que se produce en la insuficiencia renal crónica es:

a) Microcítica, hipocrómica. b) Microcítica, normocrómica. c) Macrocítica, hipercrómica. d) Normocítica, normocrómica.

679: ¿Cuál de los siguientes tests serológicos se utilizan como diagnóstico de confirmación de sífilis?:

a) VDRL. b) RPR. c) FTA-ABS. d) Wasserman.

680: ¿Cuál de las siguientes afirmaciones sobre el cribado de cáncer colorectal es CORRECTA?:

a) Los pacientes con una prueba de sangre oculta en heces (Hemoccult) positiva, con una dieta con bajo contenido en carne, presentan probablemente un carcinoma colorectal. b) La gran mayoría de pacientes con cáncer colorectal dan positivo en el test Hemoccult. c) No existen ensayos clínicos en el cribado con Hemoccult que demuestren una reducción de la mortalidad en las personas a las que anualmente se les realiza el screening.

Page 58: Cuestionario de Urología

d) La rehidratación de las tiras de Hemoccult no afectan a la tasa de positividad del test.

681: La cifra de leucocitos en las leucemias agudas se encuentra:

a) Disminuida) b) Aumentada) c) Puede ser normal o aumentada) d) Puede ser normal, disminuida o aumentada)

682: Ante un paciente de 35 años de edad que presenta una adenopatía latero-cervical, sudores nocturnos y pérdida de peso, el diagnóstico más probable es:

a) Macroglobulinemia de Waldestrom. b) Leucemia linfática crónica) c) Linfoma de Hodgkin. d) Mieloma múltiple.

683: ¿Cuál de las siguientes afirmaciones sobre la fisiopatología de la fiebre es cierta?:

a) Entre los principales pirógenos endógenos humanos se encuentra la interleukina 1 (IL-I) y el factor de necrosis de los tumores (TNF). b) La aspirina inhibe la producción de pirógenos endógenos. c) Las bacterias, los protozoos y los hongos producen pirógenos endógenos. d) Los pirógenos endógenos aumentan la temperatura corporal mediante sus efectos sobre los músculos.

684: ¿Cuál de los siguientes no es un efecto adverso del verapamilo?:

a) Estreñimiento. b) Edemas en extremidades inferiores. c) Taquicardia d) Transtomos de la conducción aurículo-ventricular.

685: El diagnóstico más probable en un paciente con impotencia e incontinencia urinaria que desde hace años refiere temblor de reposo, bradicinesia, rigidez, hipotensión ortostática importante y anhidrosis:

a) Síndrome de Landry-Guillain-Barré. b) Intoxicación por guanetidina) c) Enfermedad de Parkinson. d) Síndrome de Shy-Drager.

686: La actividad eritropoyética medular se valora mediante:

a) La cifra de hematíes que contienen restos de ARN. b) El recuento de reticulocitos. c) La amplitud de distribución de los hematíes. d) A Y B son correctas, por el ARN de los ribo sornas.

687: De las siguientes medidas terapéuticas. ¿Cuál de ellas desaconsejaría en el tratamiento de una infección de tracto urinario de vías bajas, en una niña de seis años?:

a) Amoxicilina-Clavulánico. b) Cefalexina) c) Cefaclor. d) Norfloxacino.

688: Todas las siguientes excepto una, son actitudes correctas ante un niño con episodios de otitis media aguda de repetición:

a) Antibioterapia apropiada en cada caso. b) Profilaxis antimicrobiana de mantenimiento. c) Miringotornía y timpanostornía con inserción de tubos de drenaje. d) Administración tópica de antibióticos en cada episodio.

689: Respecto al tratamiento inicial de la hemorragia subaracnoidea, señalar la respuesta INCORRECTA:

a) El paciente debe permanecer en cama y recibir emolientes para evitar el estreñimiento. b) La difenilhidantoína puede tener valor como profiláxis de una crisis convulsiva) c) Los corticoides son útiles en el tratamiento del edema cerebral. d) Ante cefalea o rigidez de nuca se pueden utilizar paracetamol, meperidina y fenobarbital.

690: ¿Cuál de las siguientes cardiopatías congénitas es acianótica?:

a) Tetralogía de Fallot. b) Atresia tricuspídea) c) Fístula arteriovenosa pulmonar. d) Ninguna de las anteriores.

691: No se considera un criterio para el diagnóstico de shock:

a) Sudoración fría) b) Presión arterial sistólica menor de 90 mm Hg en normotensos, mantenida más de 30 minutos. c) Diuresis entre 30 y 60 rnl/h, con alto contenido en sodio. d) Acidosis láctica)

692: En relación al cáncer de ovario todás las siguientes afirmaciones son correctas EXCEPTO:

a) La incidencia del cáncer de ovario es baja) b) Los tumores de células germinales y del estroma son los tipos histológicos más frecuentes.

Page 59: Cuestionario de Urología

c) El cáncer de mama es un factor de riesgo para el posterior desarrollo de un cáncer de ovario. d) La nuliparidad es un factor de riesgo.

693: La disnea cardíaca no se relaciona con uno de los fenómenos siguientes:

a) Aumento de la distensibilidad del pulmón. b) Aumento del consumo de oxígeno en el trabajo respiratorio. c) Elevación de la presión de llenado ventricular izquierdo. d) Reducción del aporte de oxígeno a los músculos respiratorios.

694: El síndrome de Barlow es:

a) Una estenosis mitral más comunicación interventricular. b) Una estenosis subvalvular aórtica) c) Una insuficiencia mitral por prolapso mitra!. d) Un síndrome de QT largo con sordera)

695: La presbiacusia o hipoacusia asociada a la edad puede afectar al 33 % de los pacientes mayores de 75 años. La causa más frecuente de este problema es:

a) Lesiones de los huesos del oído medio. b) Pérdida de células neuroepiteliales. c) Lesiones vasculares en las vías acústicas. d) Exposición a medicamentos ototóxicos como la furosemida)

696: En una audiometría tonal, es falso que:

a) Las frecuencias exploradas son 125, 250, 500, 1.000, 2.000, 4.000, 6.000 y 8.000 Hz. b) El paciente con audición normal debe ser capaz de percibir el sonido por debajo de 10 db) c) La unidad de intensidad utilizada en la audiometría es el decibelio HL. d) La unidad de intensidad utilizada es una unidad biológica)

697: De los siguientes enunciados sobre los hallazgos de laboratorios se correlacionan con la progresión del SIDA todos, excepto:

a) Aumento del nivel de neopterina) b) Aumento de la YSG. c) Aumento del nivel de B2-microglobulina) d) Trombocitosis.

698: Según la clasificación TNM para el cáncer de mama, un tumor Estadio IIA representa:

a) TI NI MO. b) TONl MO. c) T2 NO MO. d) Puede corresponder a cualquiera de las anteriores.

699: Dentro de la Esclerosis Sistémica, la forma Limitada, se caracteriza por:

a) Anticuerpos anticentrómero (70-80%). b) Fenómeno de Raynaud de aparición reciente. c) Capilaroscopia: megacapilares y pérdida capilar. d) Anticuerpos antitopoisomerasa 1 (30%).

700: Qué tratamiento recomendaría a una paciente postmenopáusica diagnosticada de cáncer de mama de más de 2 cm. de diámetro con ganglios linfáticos negativos (determinación de anatomía patológica), receptores hormonales positivos:

a) Poliquimioterapia) b) Tamoxifenio. c) Radioterapia) d) Ninguna de las anteriores.

701: La hiperkaliemia aguda se asocia a todos los siguientes cambios electrocardiográficos, EXCEPTO:

a) Ensanchamientos QRS. b) Aplanamiento de la onda P. c) Ondas T picudas. d) Alargamiento del segmento ST.

702: Sobre la erradicación de "Helicobacter pylori" en la enfermedad u1cerosa, señale la afirmación correcta:

a) Acelera la mejoría sintomática) b) Disminuye la tasa de complicaciones. c) Elimina o retrasa la aparición de recidivas. d) Es útil en el tratamiento del u1cus con hemorragia)

703: En la enfermedad pulmonar obstructiva crónica, todas las características siguientes predominan en el enfermo enfisematoso excepto:

a) La disnea es más grave que la del enfermo con predominio de bronquitis. b) La tos aparece después de la disnea) c) La capacidad de difusión está disminuida) d) El cor pulmonale es frecuente.

704: ¿Cuáles de los siguientes son síntomas muscarínicos, en la intoxicación aguda por insecticidas organofosforados?:

a) Taquicardia e hipertensión. b) Miosis y visión borrosa)

Page 60: Cuestionario de Urología

c) Fasciculaciones de la musculatura estriada) d) Temblor, ataxia y disartria)

705: La displasia evolutiva de cadera es más frecuente en:

a) En niños varones. b) En recién nacidos de parto por cesárea) c) En presencia de otras malformaciones visibles del aparato locomotor. d) En presencia de pabellones auriculares de implantación baja)

706: ¿Con cuál de los siguientes fenómenos, no se relaciona la retención de agua y de sodio en la insuficiencia cardiaca congestiva?:

a) Disminución de la presión venosa renal. b) Activación del sistema renina-angiotensina-aldosterona) c) Aumento de la presión capilar y venosa) d) Aumento de la reabsorción tubular de agua y de sodio.

707: ¿Cuál es el tumor broncopulmonar más frecuente después del carcinoma?:

a) Tumor adenoide quístico. b) Tumor carcinoide. c) Tumor mucoepidermoide. d) Adenoma)

708: La presencia de dolor otomastoideo, con dolor facial y debilidad del recto externo homolateral, es indicativo de trombosis de un seno o vena cerebral. Señálelo:

a) Seno de Gradenigo. b) Seno longitudinal superior. c) Trombosis de vena cortical. d) Trombosis de seno lateral.

709: En los pacientes con insuficiencia cardiaca no es cierto que:

a) La actividad del sistema nervioso adrenérgico está muy aumentada) b) El contenido de noradrenalina en el miocardio de dichos pacientes se halla aumentado. c) Existe un incremento de los niveles plasmáticos de noradrenalina tanto en reposo como durante el esfuerzo físico. d) El pronóstico está en relación inversa con los niveles plasmáticos de noradrenalina)

710: ¿Cuál de las siguientes no es una característica de la capsulitis adhesiva (hombro congelado)?:

a) La palpación del hombro No es dolorosa) b) Están limitados los movimientos activos y pasivos. c) La inmovilidad prolongada del brazo contribuye al desarrollo de la capsulitis. d) La cápsula del hombro está claramente engrosada y puede haber un ligero infiltrado inflamatorio crónico.

711: Ante un paciente con infección por HIV -1 que presenta un aumento de tamaño de las adenopatías en un territorio ganglionar, ¿cuál es el diagnóstico más probable?:

a) Linfoma) b) Sarcoma de Kaposi. c) Tuberculosis. d) Toxoplasmosis.

712: El síndrome de uretritis postgonocócica suele ser debido a:

a) Proliferación uretral de cándidas. b) Dosis inadecuada de antirnicrobianos. c) Infección por Haemopbilus ducrey. d) Infección concomitante por clamidias.

713: Si un paciente en tratamiento con fármacos antituberculosos presenta hiperuricemia de nueva aparición. ¿Cuál es la causa más probable?:

a) El etambutol. b) La pirazinamida) c) La rifampicina) d) No tiene relación con el tratamiento.

714: Al centro de Salud acude un paciente que presenta pérdida de agudeza en ojo derecho, instaurada progresivamente en el curso de 2 días, mantiene aceptable el reflejo pupilar, el examen del fondo de ojo con hemorragias abundantes en retina nos sugiere:

a) Catarata hemorrágica) b) Oclusión de la arteria central de la retina) c) Trombosis de la vena central de la retina) d) Glaucoma agudo con afectación de la retina)

715: Entre las prnebas de laboratorio utilizadas para la valoración rutinaria de la demencia se incluyen todas las siguientes, EXCEPTO:

a) Vitamina B 12 . b) VDRL para el diagnóstico de sífilis. c) Velocidad de sedimentación globular. d) Electrolitos séricos.

Page 61: Cuestionario de Urología

716: De los siguientes signos y síntomas que aparecen en la brucelosis. ¿Cuál de ellos lo hace en menor frecuencia?:

a) Fiebre> 38 °. b) Sudación. c) Hepatomegalia) d) Dolor abdominal.

717: En el tratamiento de la insuficiencia cardiaca, ¿cuál de las medidas siguientes es incorrecta?:

a) Mejorar la función contráctil mediante digital. b) Disminuir la retención de agua mediante diuréticos. c) Disminuir el aporte de sodio. d) Aumentar el aporte de agua en caso de hiponatremia dilucional.

718: ¿Cuál de los siguientes fármacos está indicado en el tratamiento de las infecciones por citomegalovirus en los pacientes con SIDA?:

a) Estavudina) b) Lamivudina) c) Zidovudina) d) Ganciclovir.

719: Que persona de las siguientes no es subsidiaria de quimioprofilaxis secundaria:

a) Contacto íntimo, con Mantoux negativo y sin enfermedad. b) Convertores. c) VIH positivo, infectado. d) Inmunodeprimido, infectado.

720: En la escarlatina suelen hallarse:

a) Pequeños elementos purpúricos de distribución circular en pliegues. b) Amigdalitis. c) Exantema micromaculoso que respeta" los pliegues. d) Aftas en las mucosas bucal y lingual.

721: No es indicación prioritaria de profilaxis secundaria de la TBC una de las siguientes afirmaciones:

a) Inmigrantes procedentes de países con elevada endemia) b) Portadores de VIH. c) Los convertores recientes. d) Los contactos y convivientes de un enfermo tuberculoso.

722: En el examen del fondo de ojo. ¿Cuál de los siguientes hallazgos no se relaciona con la retinopatía hipertensiva?:

a) Hemorragias. b) Edema de papila) c) Exudados. d) Cruces arteriolares.

723: Le acude a urgencias de su Centro de Salud un paciente de 60 años, hipertenso, con un déficit motor moderado, déficit de memoria, disartria y síndrome depresivo con labilidad emocional de instauración subaguda) ¿En qué diagnóstico pensaría necesariamente?:

a) AIT (Ataque isquémico transitorio). b) Demencia vascular. c) Hemorragia subaracnoidea) d) Ninguna de las anteriores.

724: Una de las siguientes afirmaciones, respecto al tratamiento de los AIT, es falsa:

a) La edad puede limitar algunos tratamientos agresivos. b) En las estenosis carotídeas del 50% y asintomáticas está indicada la endarterectomía. c) Si hay indicación de tratamiento antiagregante puede usarse el AAS a dosis de 200 mg/día) d) Depende de factores como edad, origen y número de episodios.

725: Ante un paciente con cefalea y diplopia de diez días de evolución, y un LCR con 200 células, 70% de linfocitos, 130 mg de proteínas y 30 mg de glucosa) ¿Cuál es el diagnóstico inicial?:

a) Meningitis purulenta) b) Meningitis vírica) c) Meningitis tuberculosa) d) Meningitis bacteriana decapitada)

726: En el diagnóstico diferencial entre síncope y crisis convulsiva) ¿Cuál de los siguientes datos clínicos sugiere la existencia de una crisis convulsiva?:

a) El episodio se desencadena tras un estrés emocional. b) El episodio se desencadena tras una maniobra de Valsalva) c) Palidez cutánea facial durante el episodio. d) Cianosis facial durante el episodio.

727: En una mujer de 35 años que se encontraba previamente bien, el dedo pulgar derecho empieza a contraerse repetidas veces, a continuación se producen movimientos en su mano derecha) En un minuto su brazo y hemifacies derechos también empiezan a contraerse. Dos minutos después la paciente presenta una crisis convulsiva motora difusa con pérdida de conciencia de alrededor de 5 minutos de duración. Tras recuperar la conciencia la paciente presenta amnesia retrógrada y debilidad en su brazo derecho de 6 horas de duración. ¿Cuál de las siguientes es la causa más probable de esta convulsión?:

Page 62: Cuestionario de Urología

a) Epilepsia del lóbulo temporal. b) Abceso o tumor en el hemisferio izquierdo. c) Embolismo cerebral. d) Epilepsia mioclónicajuvenil.

728: Señale la localización más frecuente de herpes genital en la mujer:

a) Vulva) b) Uretra) c) Vagina) d) Cérvix.

729: Señale el tipo de sintomatología más frecuente en la psicosis anfetamínica:

a) Esquizofreniforme. b) Fóbica) c) Depresiva) d) Confusional.

730: Todos los siguientes han sido identificados como factores de riesgo de suicidio en un paciente terminal EXCEPTO:

a) Alcoholismo. b) Enfermedad de Parkinson. c) Pacientes en hemodiálisis. d) Esquizofrenia)

731: ¿Cuál de las siguientes determinaciones es más útil para el diagnóstico inicial del hipotiroidismo primario?:

a) T3 total. b) T4 libre. c) T.S. H. Basa!. d) Captación de T4 por resina)

732: Señale cuál es falsa de la tiroglobulina:

a) Es una proteína sintetizada por los tirocitos de tejido tiroideo normal. b) Es una proteína sintetizada por los tirocitos de tejido tiroideo tumoral. c) Es una globulina transportadora de tiroxina) d) El ascenso de los valores de tiroglobulina puede indicar la presencia de recidiva local tras la ablación quirúrgica y/o radioterápica del tiroides con carcinoma tiroideo.

733: Respecto al tratamiento de la diabetes mellitus (DM) es cierto todo lo siguiente excepto:

a) La DM tipo 1 (DM1) debe ser tratada siempre con insulina y una dieta ajustada a la edad, caracteósticas y actividad física del individuo. b) En la DM tipo 2 la cifra de glucemia inicial es indicativa de necesidad de tratamiento farmacológico. c) Las personas con DM tipo 2 sin sobrepeso que no se controlan con dieta y sulfonilurea son tributarias de tratamiento insulínico. d) La DM tipo 2 asociada a obesidad debe ser siempre tratada con dieta hipocalórica)

734: De los métodos que se citan a continuación para establecer prioridades en los problemas de salud. ¿Cuál de ellos es un método cuantitativo?:

a) El método Delphi. b) El método de Bainbridge y Sapirie. c) El método Cendes. d) El grupo nominal.

735: Entendemos como fiabilidad de un sistema de información sanitaria:

a) La ausencia de errores en la recogida de datos. b) El grado en que incluye toda la información necesaria) c) A y B son correctas. d) Ninguna es correcta)

736: Todas las siguientes son etapas del proceso de Planificación en Salud, según Nutt, excepto:

a) Formulación. b) Búsqueda de recursos. c) Conceptualización. d) Explicitación.

737: ¿Cuál de las siguientes calificaciones de invalidez permanente (IP) NO EXISTE?:

a) I.P. Parcial, para la profesión habitual. b) I.P. total para la profesión habitual. c) I.P. Absoluta para la profesión habitual. d) I.P. Absoluta para cualquier trabajo.

738: ¿Cuál de los siguientes procesos no es considerado de baja incidencia a efectos del Boletín Epidemiológico?:

a) Paludismo. b) Hidatidosis. c) Triquinosis. d) Difteria)

Page 63: Cuestionario de Urología

739: ¿Cuál de los siguientes cuadros no puede considerarse enfermedad profesional?:

a) Necrosis del hueso semilunar (Enfermedad de KienbOck). b) Brucelosis. c) Tularemia) d) Todas las anteriores son consideradas enfermedades profesionales.

740: Los objetivos propios de la educación para la salud se definen en términos de:

a) Información. b) Conducta) c) Nivel de Salud. d) Tecnología de Salud.

741: Ante un caso de hepatitis B, ¿Qué se debe hacer?:

a) Declararla cuando se tenga la confirmación del diagnóstico. b) Declararla urgentemente en menos de 24 horas. c) Declararla semanal individualizada y numérica con encuesta epidemiológica) d) Declaración exclusivamente numérica)

742: Se denomina a la proporción de enfermos que presentan un resultado positivo de un método de diagnóstico:

a) El valor predictivo positivo del método. b) El valor predictivo negativo del método. c) La especificidad del método. d) Ninguna de las anteriores.

743: Para calcular el índice de envejecimiento de la población. ¿Cuál sería el DENOMINADOR que deberíamos utilizar?:

a) Menores de 15 años. b) Menores de 65 años. c) Entre 15 y 65 años. d) Menores de 35 años.

744: Si tenemos el peso de 5 pacientes cuya media es de 69,6 Kg. Y su desviación típica 10,44 y la media de la tensión arterial sistólica de los mismos es de 166 mmHg Y su desviación típica de 21,30. ¿Qué distribución es más dispersa, el peso o la tensión arterial?:

a) El peso. b) La tensión arterial. c) Peso y tensión arterial tienen la misma dispersión. d) No se puede calcular.

745: Leemos en una revista médica que "la frecuencia cardiaca media de los varones fumadores es 70 más menos 5 ppm, con p < 0,01". Esto significa que:

a) El 99% de los individuos de la muestra tienen entre 65 y 75 ppm. b) La probabilidad de que la frecuencia cardiaca media de los varones fumadores esté entre 65 y 75 es menor de 0,01. c) La probabilidad de que la frecuencia cardiaca media de los varones fumadores esté entre 65 y 75 es menor de 0,01. d) La probabilidad de que la frecuencia cardiaca media de los varones fumadores no esté entre 65 y 75 es menor de 0,01.

746: ¿Qué tipo de estudios tienen como objetivo principal estudiar la prevalencia que presenta una enfermedad en una población?:

a) Casos y controles. b) Cohortes. c) Transversal. d) Ensayo clínico.

747: Todos los siguientes son programas informáticos que utilizan técnicas estadísticas en investigación, EXCEPTO:

a) SPSS-Pc. b) POWERPOINT. c) BMDP. d) STATVIEW.

748: ¿Cuándo elegirían un estudio prospectivo en detrimento de uno retrospectivo?:

a) Escasos recursos disponibles. b) Precisar largo tiempo de exposición para desarrollar el proceso. c) Mala calidad de los registros disponibles. d) Ninguna de las anteriores.

749: ¿Cuál de los siguientes fármacos ha demostrado reducir la mortalidad en pacientes con insuficiencia cardiaca congestiva?:

a) Digoxina) b) Furosemida) c) Enalapril. d) Acido acetil-salicílico.

750: ¿Cuál de las siguientes afirmaciones es falsa en relación a la fiebre tifoidea?:

a) Ofrece buenas posibilidades de erradicación. b) Se transmite por vía oral-fecal. c) Afecta, además de al hombre, a ciertos animales domésticos, como las gallinas. d) El agente etiológico es, casi siempre, Samonella typhi.

Page 64: Cuestionario de Urología

751: Entre las consecuencias de la exposición crónica al asbesto se incluyen todas las siguientes EXCEPTO:

a) Fibrosis pulmonar. b) Derrame pleural. c) Mesotelioma peritoneal. d) Carcinoma de células pequeñas.

752: En la colestasis crónica, ¿Cuál de las siguientes hipovitaminosis no acostumbra a observarse?:

a) Hipovitaminosis A e Hipovitaminosis K. b) Hipovitaminosis b) c) Hipovitaminosis D e Hipovitaminosis a) d) Hipovitarninosis E.

753: La primoinfección por Mycobacterium tuberculosis se caracteriza por:

a) Lesiones pulmonares con diseminación, asintomáticas y se desarrolla en 3-10 años. b) Lesiones pulmonares sin diseminación y se desarrolla en 3-10 días. c) Lesiones pulmonares con diseminación, asintomáticas y se desarrolla en 3-10 semanas. d) Lesiones pulmonares sin diseminación y se desarrolla en 3-10 meses.

754: En relación a los efectos que produce el alcohol sobre el corazón, cuál de las siguientes frases es correcta:

a) La ingesta crónica de alcohol puede producir una miocardiopatía restrictiva) b) Cuando se desarrolla una insuficiencia cardiaca, la abstinencia de alcohol no afecta de forma importante la evolución natural de la enfermedad. c) La taquicardia ventricular es la arritmia más frecuente tras una borrachera) d) Si un paciente con insuficiencia cardiaca secundaria a la ingesta excesiva de etanol continúa bebiendo es poco probable que viva más de tres años.

755: El síndrome serotoninérgico producido por la utilización de inhibidores selectivos de la recaptación de serotonina (ISRS), se caracteriza por presentar los siguientes síntomas, excepto:

a) Hipotermia. b) Vómitos. c) Mioc1onias. d) Confusión.

756: En el tratamiento con antidepresivos tricíc1icos, hay que realizar los siguientes controles (antes y durante el mismo), excepto:

a) Tensión arteria!. b) TSH. c) Hemograma. d) Electrocardiograma.

757: La hipertensión en el embarazo puede ser tratada con los siguientes fármacos, excepto:

a) Alfa-betabloqueantes. b) Hidralazina. c) lECA. d) Nifedipino.

758: Usted nunca emplearía la digital en:

a) En una disfunción sistólica con clínica de insuficiencia cardiaca. b) Cuando un paciente presente ritmo de galope R3. c) En la disfunción diastólica. d) En la insuficiencia cardiaca en ritmo sinusal.

759: Es típico del síndrome de la arteria cerebral media todo, excepto:

a) Hemiplejia de predominio facio-braquial, hemianestesia y hemianopsia contralateral b) Masia, si se trata del hemisferio dominante. c) Signos apraxo-agnósicos si no es el hemisferio dominante. d) Amaurosis unilateral.

760. En una taquicardia supraventricular que no cede a las maniobras de valsalva ni al masaje del seno carotídeo, el siguiente tratamiento de elección será:

a) Digoxina v.o. b) Propanolol Lv. c) Amiodarona i.v. d) Adenosín trifosfato L v.

761. Mediante la HBAIC podemos valorar con gran fiabilidad el grado de control glucémico de un paciente diabético durante un tiempo antes de su determinación, ¿cuánto tiempo es éste?

a) De 2-4 semanas antes. b) De unos 6 meses antes. c) De 4-8 semanas antes. d) De 8-16 semanas antes.

762. Una de las siguientes vitaminas no es hidrosoluble:

a) Tiamina. b) Colecalciferol. c) Piridoxina.

Page 65: Cuestionario de Urología

d) Ü.Cobalamina.

763. Un índice de masa corporal de 28 en una mujer adulta indica:

a) Normopeso. b) Sobrepeso. c) Obesidad. d) Obesidad mórbida.

764. Imagine que se encuentra ante un paciente diabético en tratamiento con insulina NPH y que presenta una amigdalitis pultácea, ¿cuál sería su actitud respecto al tratamiento de su diabetes?

a) Mantendría la pauta de insulina y añadiría (según controles) suplementos de insulina rápida. b) Cambiaría a insulina rápida. c) Reduciría la dosis de insulina y aumentaría la frecuencia de los controles mientras tuviera fiebre. d) Mantendría la misma dosis de insulina.

765. ¿En cuál de las siguientes situaciones está claramente indicada la colecistectomía en la colelitiasis asintomática?

a) Presencia de cálculos biliares radioopacos. b) La asociación de litiasis con "barro" biliar. c) Vesícula en porcelana. d) Enfermos diabéticos.

766. ¿Cuál de las siguientes afirmaciones es falsa en relación a la hepatopatía alcohólica?

a) La esteatosis alcohólica tiene un pronóstico bueno. b) En la hepatitis alcohólica suele haber un cociente ALT/AST mayor de 2. . c) La hepatitis alcohólica puede cursar cm! fi.eo:bre. d) La presencia de hiperbilirrubinemia intensa ensombrece el pronóstico de la hepatitis alcohólica.

767. ¿Cuál es el periodo de incubación aproximado de la salmonella?

a) 1-5h. b) 6-48 h. c) 3-5 días. d) 48-72 horas.

768. En la arteritis de la temporal es falso que:

a) La fiebre es un síntoma frecuente. b) Encontrar pulso en arteria temporal excluye el diagnóstico. c) Es más frecuente en mujeres. d) Los glucocorticoides son el tratamiento de elección.

769. No es característico de la migraña común:

a) Existencia de antecedentes familiares. b) Náuseas y vómitos. c) Más frecuente en mujeres. d) Existencia de "aura".

770. No es típico del vértigo periférico:

a) Náuseas y vómitos. b) Diplopia. c) Nistagmo con fase rápida contralateral alIado afecto. d) Hipoacusia.

771. Señale cuál de los siguientes parámetros espirométricos es más útil en la valoración pronóstica y gravedad de los enfermos con EPOC avanzado:

a) Volumen espiratorio forzado en el primer segundo (FEVl). b) Volumen de reserva espiratOlia (VRE). c) Capacidad vital forzada (FVC). d) Índice mesoespiratorio (F'EF' 25-75%).

772. ¿Cuál de los siguientes tratamientos sería el más indicado en un paciente al que acaban de diagnosticar una bronquitis aguda y no tiene otros factores de riesgo?

a) Sintomático. b) Amoxicilina. c) Amoxicilina-ácido clavulánico. d) Cefuroxima.

773. Teniendo en cuenta la clasificación del asma según niveles de gravedad, la presencia de síntomas nocturnos más de una vez por semana pero no diarios, correspondería a:

a) Asma persistente grave. b) Asma persistente leve. c) Asma persistente moderada. d) Asma intermitente leve.

774. ¿Qué forma histológica de neoplasia pulmonar suele manifestarse por la presencia de derrame pleural o dolor pleurítico?

a) Carcinoma epidermoide. b) Adenocarcinoma.

Page 66: Cuestionario de Urología

c) Carcinoma de células pequeñas. d) Tumor de Pancoast.

775. Entre las dermatofitosis se encuentran las siguientes afecciones, menos una:

a) Tiña del pie. b) Tinea capitis. c) Tiña versicolor. d) Herpes circinado.

776. Con relación a la psoriasis:

a) La afectación de las uñas es muy poco frecuente. b) La asociación con artritis es aproximadamente del 40%. c) La forma más frecuente de artritis psoriásica es la oligoarticular. d) La ciclosporina se utiliza en las formas moderadas.

777. ¿Cuál de las siguientes es la manifestación pulmonar más frecuente de la artritis reumatoide?

a) Afectación pleural, con o sin derrame. b) Neumonitis intersticial. c) Adenopatías mediastínicas. d) Síndrome de Caplan.

778. Señale cuál de los mecanismos que se citan a continuación no es causante de dolor articular en la gonartrosis:

a) Destrucción del cartílago yuxtaarticular. b) Contractura antiálgica periarticular. c) Estrechamiento de la interlínea articular. d) Estasis venoso intraepifisario.

779. ¿Cuál de los siguientes síntomas no es típico de un ataque de gota?

a) Inicio nocturno. b) No hay secuela articular una vez pasada la crisis. c) Mectación poliarticular. d) Puede presentar fiebre.

780. Señale un signo radiológico impropio de la artrosis:

a) Esclerosis del hueso subcondral. b) Disminución de la interlínea articular. c) Erosiones. d) Geodas.

781. En relación al tratamiento de la anemia ferropénica, es falso que:

a) El tratamiento de elección es la administración oral de hierro en forma de sales ferrosas. b) La pauta de tratamiento con hierro debe aportar entre 100-200 mgldía en adultos y de 3-5 mglkg.ldía en niños. c) La respuesta favorable al tratamiento se pone de manifiesto por un aumento de reticulocitos que se inicia al cuarto día. d) El tratamiento se debe interrumpir una vez se hayan alcanzado niveles de hemoglobina aceptables, a fin de evitar una hemosiderosis.

782. No es característico encontrar en una anemia ferropénica uno de los siguientes parámetros hematológicos. ¿Cuál?

a) Hemoglobina corpuscular media (HCM) baja. b) Índice de saturación de la transferrina (1ST) disminuido. c) Disminución de la concentración de hemoglobina corpuscular media (CHCM). d) Amplitud de distribución eritrocitaria (ADE) disminuida.

783. Mujer de 60 años diagnosticada de anemia microcítica e hipocroma con los siguientes parámetros de laboratorio: sideremia disminuida, TIBC (Total Iron Binding Capacity) descendida, ferritina sérica aumentada y protoporfirina eritrocitaria libre incrementada. ¿Cuál es su diagnóstico?

a) Talasemia. b) Ferropenia. c) Anemia sideroblástica. d) Anemia de enfermedades crónicas.

784. Respecto a la anemia de las enfermedades crónicas, las siguientes afumaciones son ciertas, excepto una:

a) Es la anemia más frecuente después de la anemia ferropénica en Atención Primaria. b) La mayoría de las veces son normocíticas, si bien en ocasiones se presentan como microcíticas. c) Los niveles de sideremia son inferiores a los normales. d) El ADE o RDW suele estar ligeramente bajo.

785. La localización más frecuente de metástasis a distancia del cáncer de próstata es:

a) Pelvis. b) Costillas. c) Hígado. d) Pulmón.

786. En los casos en que está indicada una terapéutica profiláctica de la infección del tracto urinario, el tratamiento debe ser:

a) Nitrofurantoina 50 mg/12 h durante 4 meses. b) Ácido pipernídico 800 mg en dosis única nocturna durante 6 meses.

Page 67: Cuestionario de Urología

c) Cotrimoxazol (40 mgl200 mg) a (80 mg/400 mg) en dosis única nocturna, durante 6 meses. d) Nitrofurantoina 50 mg/12 h durante 6 meses.

787. Señale de entre los siguientes, ¿cuál no sería el antibiótico de elección en una cistitis simple sin factores de riesgo?

a) Cefalosporina de 18 generación. b) Cefalosporina de 28 generación. c) Amoxicilina - clavulánico. d) Fosfomicina trometamol.

788. En el tratamiento de la litiasis renal, ¿en qué entidad utilizaría usted los diuréticos tiacídicos?

a) Hiperoxaluria. b) Acidosis tubular renal distal. c) Litiasis. cálcica idiopática. d) Ninguno de los anteriores.

789. Ante un anciano que presenta incontinencia deberemos pensar en primer lugar como causa más frecuente:

a) Incontinencia por inestabilidad del detrusor o de urgencia. b) Incontinencia funcional. c) Incontinencia de estrés. d) Incontinencia por rebosamiento.

790. Nos consulta el hijo de una paciente de 70 años porque desde hace 6 meses su madre ha comenzado con dificultad para realizar algunas actividades domésticas (coser, hacer la comida). Se acompaña de temblor en miembros superiores y cara menos expresiva. En la última semana ha tenido episodios alucinatorios en los que dice ver a su marido con ella en el lugar donde vivían antes. ¿En qué enfermedad debemos pensar en primer lugar?

a) Enfermedad de Alzheimer. b) Demencia multiinfarto. c) Enfermedad de Parkinson. d) Demencia por cuerpos de Lewy difusos.

791. ¿Cuál de estas escalas evalúa el estado funcional del anciano?

a) Índice de Barthel. b) S.P.M.S.Q de Pfeiffer. c) Escala de ansiedad de Goldberg. d) Ninguna de ellas.

792. Respecto a la vaginosis bacteriana, señale la respuesta correcta:

a) El flujo es amarillo-verdoso, espumoso y maloliente. b) Se debe a un sobrecrecimiento de los lactobacilos. c) Su tratamiento de elección es el clotrimazol. d) En la exploración no hay signos inflamatorio s en vulva y vagina característicamente.

793. Según la clasificación de la CDC para la infección por el VlH un paciente VlH (+) asintomático con un número de CD 4 entre 200-499 cel.lrnm 3, se encuadraría como:

a) Al. b) Bl. c) Al. d) B3.

794. Los siguientes significados de los marcadores de la hepatitis B son correctos, excepto:

a) El HBs-Ag indica infección actual (aguda, crónica o estado de portador). b) Anti-HBe indica baja infecciosidad. c) El HBe-Ag indica evolución favorable y ausencia de contagiosidad. d) El anti-HBs indica infección anterior con evolución a la curación y/o inmunización frente al virus.

795. ¿Qué nos indica el hallazgo de hiperuricemia en el primer control analítico de un enfermo tratado con tuberculostáticos?

a) Que hemos de reducir la dosis. b) Que hay que retirar preventivamente el tratamiento. c) Que toma correctamente la pirazinamida. d) Que debemos poner Alopurinol.

796. La garnma-glutamiltranspeptidasa (GGT) es útil al médico de familia para ver si un alcohólico ha dejado de ingerir alcohol. ¿A cuántas semanas de abandonar la toma de alcohol se normaliza esta prueba?

a) A las 6-8 semanas. b) A las 2-4 semanas. c) A las 10-12 semanas, d) A las 16-18 semanas.

797. Para detectar un derrame pericárdico, el método diagnóstico más sensible es:

a) La tomografía axial computerizada. b) La radiografía de tórax. c) La ecocardiografía. d) El electrocardiograma.

798. La progresiva implantación de la espirometría en los centros de salud, hace necesario conocer correctamente la clasificación del grado de severidad de las alteraciones ventilatorias. La SEP AR (Sociedad Española de la Patología del Aparato Respiratorio),

Page 68: Cuestionario de Urología

las clasifica en varios grados, señale la correcta:

a) Ligera (leve): FEVl hasta el 85%. b) Moderada: FEVl entre el 85 y 75%. c) Severa (grave): FEVl entre el 75 y 35%. d) Muy severa (muy grave): FEVI menor del 35%.

799. Ante la sospecha de una enfermedad tiroidea, un médico de familia debiera pedir de primera elección:

a) TSH y FT4. b) TSH. e) TSH y Fr3. d) FT4 Y FT3.

800. El grupo de estudio canadiense sobre exámenes periódicos de salud recomendaba en su informe, con respecto al chequeo anual sistemático:

a) Variar su frecuencia haciéndolo cada tres años en lugar de anualmente. b) Extender su aplicación a grupos cada vez más numerosos de población. c) Abandonar toda forma de detección mediante exámenes periódicos, por su demostrada ineficiencia. d) Sustituirlo por un método selectivo determinado en función de la edad y sexo del individuo.

801. Decimos que un entrevistador tiende a hacer focalizaciones cuando:

a) Transmite sólo la información que cree que el enfermo puede elaborar emocionalmente. b) Infravalora determinados elementos de tipo biológico o psicosocial del paciente. c) Medicaliza al enfermo creando motivos de consulta en diversos órganos y aparatos inexistentes. d) No sabe emitir buenos diagnósticos sobre determinadas patologías al tener "agujeros negros" en su formación médica.

802. El tono y el timbre de voz son elementos importantes de la comunicación no verbal y se denominan:

a) llustradores. b) Paralenguaje. c) Reguladores. d) Adaptadores.

803. Para delimitar la demanda en la entrevista clínica realizada en atención primaria la habilidad más importante es:

a) Realizar un uso racional del tiempo en cada consulta. b) Negociar con el paciente una lista de sus problemas anteriores. c) Tener un plan de entrevista previo y habilidades comunicacionales. d) Clarificar el motivo (o motivos) por el que consulta el usuario.

804. Señale la definición correcta del término "proxémica", empleado en entrevista clínica.

a) Elementos comunicativo s relacionados con el orden y la distancia de los sujetos que se están comunicando. b) Mensajes emitidos mediante gestos o expresiones ajenas al lenguaje hablado. c) Movimientos que muestran el esfuerzo del paciente para asumir una situación desagradable. d) TonalÍdad y timbre de la voz, así como todos los sonidos que no constituyen propiamente palabras.

805. Señale cuál de las siguientes características es propia de la familia nuclear:

a) Está constituida por la pareja, los abuelos y los hijos. b) Es aquella que está conformada por todos los miembros que viven en el hogar. c) Está definida por el parentesco primario de esposos y uno a más hijos. d) Es aquella en la que se establecen exclusivamente interacciones conyugales.

806. Cuando una familia sufre un acontecimiento vital estresante (AVE) Y como consecuencia, sus miembros se encuentran incapaces para cumplir sus roles y tomar decisiones o solucionar los problemas, decimos que:

a) Recibe una Retroalimentación negativa. b) Intenta normalizar sus fronteras. c) Mantiene alerta sus mecanismos homeostáticos. d) Tiene una crisis familiar.

807. ¿Cómo se denomina a la etapa en la que los hijos han abandonado el hogar y vive la pareja sola en el hogar familiar?

a) Etapa IV del CVF. b) Etapa ID del CVF. c) Etapa VI del CVF. d) Etapa V del CVF.

808. El cuestionario APGAR familiar tiene utilidad para conocer:

a) El apoyo social. b) La estructura familiar. c) La función familiar. d) Los acontecimientos vitales estresantes.

809. El empleo de bicarbonato sódico en situaciones de paro cardíaco, debe quedar reservado para los siguientes casos, excepto:

a) Cuando exista una acidosis grave en sangre arterial. b) Hiperpotasemia. c) Intoxicaciones por antidepresivos tricíclicos. d) Debe valorarse siempre como una primera medida.

810. Con respecto a la utilización de una vía intraósea es falso que:

Page 69: Cuestionario de Urología

a) Existe amplia experiencia en su empleo en adultos. b) Es una buena alternativa a la venosa. c) Es más eficaz que la vía endotraqueal. d) Las dosis que se deben emplear son las mismas que para las vías venosas.

811. Tenemos a un paciente en parada cardiorrespiratoria que ha podido ser intubado y aún no se ha canalizado una vena. ¿Qué medicación puede ser administrada vía endotraqueal?

a) Adrenalina. b) Atropina. c) Lidocaina. d) a, b y c son correctas.

812. Indique el enunciado falso respecto a la acumetría (diapasón de 500 H):

a) En la sordera de conducción el Weber está lateralizado alIado enfermo. b) El Rinne negativo indica que la audición aérea es peor que la audición ósea. c) En la sordera de transmisión el Rinne es positivo. d) El Weber compara la conducción ósea entre los dos oídos.

813. Ante un paciente con perforación timpánica central con otorrea episódica no fétida, que acude a consulta por otalgia y otorragia. Usted actuaría:

a) Tratamiento tópico con gotas de antibiótico ante la sospecha de infección aguda de una otitis crónica media secretora. b) Tratamiento con amoxicilina-clavulánico oral ante la sospecha de infección aguda de una otitis crónica media secretora. c) Derivación para estudio de posible colesteatoma.. d) Derivación para descartar carcinoma de oído medio.

814. Acude a urgencia una mujer de 48 años con dolor intenso en cara y ojo derecho, náuseas, vómitos, halos coloreados alrededor de puntos luminosos y disminución de agudeza visual. Refiere haber empezado hace media hora estando en el cine. En la exploración hiperemia conjuntival, edema corneal y anisocoria con pupila derecha arreactiva y parcialmente dilatada. ¿Cuál de las siguientes medidas estaría indicada?

a) Colirio ciclopléjico y oclusión ocular. b) Manitol intravenoso. c) Sumatriptán subcutáneo. d) Oxigenoterapia al 100%. .

815. Es falso respecto a la catarata:

a) Produce presbicia. b) La técnica quirúrgica con más ventajas es la facoemulsificación (se implanta una lente intraocular dentro de la cápsula posterior). c) Miopización. d) Molestias con la luz brillante del día.

816. Varón de 35 años que consulta por dolor ocular bilateral con sensación de cuerpo extraño, fotofobia, lagrimeo y blefarospasmo. Refiere antecedente esta mañana de haber esquiado en la sierra. En la exploración no se observa cuerpo extraño y tras instilar fluoresceína se observa un punteado epitelial en la zona expuesta de la córnea. ¿Cuál de los siguientes tratamientos NO estaría indicado?

a) Analgésico oral según dolor. b) Colirio de ciclopentolato: l gota cada 6-8 horas. c) Colirio de fenilefrina: l gota cada 8-12 horas. d) Vendaje semicompresivo durante 24 horas.

817. ¿Qué fármaco utilizaría para dilatar la pupila en exámenes del fondo de ojo?

a) Fenilefrina al 10%. b) Pilocarpina al 2%. c) Homatropina al 2%. d) Tropicamida al 1%.

818. Debemos sospechar hipertiroidismo en el anciano ante la presencia de todos, excepto:

a) Taquiarritmia auricular. b) Insuficiencia cardíaca. c) Apatía. d) Hiperfagia.

819. La tormenta tirotóxica NO se caracteriza por:

a) Delirium o coma. b) Hipertermia. c) Hipertensión. d) Requiere rehidratación, glucosa, vitamina B corticoides.

820. ¿Cuál de estas afirmaciones sobre bocio simple es cierta?

a) La etiología es el déficit de yodo en la dieta, no incluye defectos de la vía biosintética hormonal. b) Endémico indica afectación de más del 50% de la población. c) La clínica depende sólo de la tiromegalia, el estado metabólico es eutiroideo. d) El tratamiento de elección es la ablación quirúrgica.

821. En el tratamiento del bocio multinodular tóxico se incluye:

Page 70: Cuestionario de Urología

a) Propanolol. b) Yodo radiactivo en pequeñas dosis. c) Ablación quirúrgica. d) Corticoterapia.

822. A la hora de establecer un tratamiento analgésico con morfina se debe tener en cuenta que:

a) Existe techo analgésico. b) Se puede asociar con otro analgésico opioide. c) Se puede comenzar con medio comprimido de liberación retardada. d) No suele producir depresión respiratoria en un paciente con dolor oncológico severo.

823. Señale cuál de las siguientes afirmaciones respecto a la herida por mordedura, NO es cierta:

a) Se consideran sistemáticamente como infectadas y son candidatas a cierre secundario. b) El antibiótico de primera elección para su tratamiento sería la cloxacilina. c) Si el cierre primario evitara secuelas mutilantes, se derivaría al cirujano plástico. d) Se considera imprescindible la profilaxis antitetánica.

824. El consentimiento informado, en la actual situación de litigio creciente a sanitarios, y para evitar situaciones de desamparo legal, es imprescindible en la consulta de cirugía en el centro de salud. Señale la afirmación, de las que siguen, que es INCORRECTA:

a) Para intervenir a menores o a pacientes incapaces (déficit mental) se solicitará consentimiento a los padres, los familiares responsables o al tutor legal. b) En el caso de que el interventor fuese un médico residente en período de formación, se hará constar este hecho al solicitar el consentimiento del paciente, garantizando la adecuada supervisión de su responsable docente. c) Elconsentimiento informado debe ser custodiado por el propio paciente, haciéndose responsable de la pérdida o deterioro del mismo. d) El consentimiento de cirugía debe ser expreso por parte del paciente (no solo implícito).

825. Señale lo FALSO en la intoxicación por acetaminofén:

a) En el adulto son graves las ingesta a partir de 200 mg/Kg. b) La toxicidad hepática suele aparecer a las 36 - 48 horas de la ingesta. c) El fármaco de elección a utilizar en el tratamiento es el succinato sódico. d) Si no se conoce el tiempo de ingesta ni la dosis debe instaurarse el tratamiento con el antídoto.

826. Todas las siguientes asociaciones incrementan sus efectos secundarios excepto:

a) Clorfeniramina-benzodiazepinas. b) Astemizol-macrólidos. c) Loratadina-pseudoefedrina. d) Astemizol-antifúngicos.

827. Señale lo que NO deberíamos hacer ante una intoxicación por antidepresivos tricíclicos:

a) Fisostigmina para los efectos anticolinérgicos. b) Betabloqueantes para las arritrnias. c) Diacepam para las convulsiones. d) Noradrenalina si aparece hipotensión.

828. Señale la asociación INCORRECTA entre tóxico y antídoto:

a) Opiáceos - Naloxona. b) Benzodiacepinas - Flumazenil. c) Betabloqueantes - Atropina. d) Antagonistas del calcio - Gluconato cálcico.

829. El diagnóstico diferencial de la candidiasis persistente en la boca y en el esófago en el adulto, incluiría cada uno de los siguientes, excepto:

a) Una enfermedad de la inmunidad celular. b) Tratamiento con corticoides. c) Candidiasis mucocutánea crónica. d) Linfoma.

830. Con respecto a la caries dental, el índice CAO se refiere a:

a) Número de dientes cariados y obturados en la dentición de leche por niño examinado. b) Número de dientes cariados y obturados en la dentición permanente por niño examinado. c) Número de dientes cariados, obturados y ausentes en la dentición permanente por niño examinado. d) Número de dientes cariados y obturados menos los ausentes en la dentición de leche por niño examinado.

831. La lesión maligna más frecuente en la cavidad bucal es el:

a) Adenocarcinoma. b) Carcinoma epidermoide. c) Melanoma. d) Fibrosarcoma.

832. Todas las siguientes manifestaciones clínicas corresponden a lesiones cerebelosas, excepto:

a) Palabra escándida. b) Asinergia. c) Espasticidad.

Page 71: Cuestionario de Urología

d) Hipotonía.

833. En la parálisis facial periférica todo es cierto, excepto:

a) La causa más frecuente es idiopática. b) Alteración en la secreción lacrimal. c) La motilidad de la lengua está dificultada. d) La masticación está dificultada.

834. Acerca del diagnóstico de muerte cerebral es falso que:

a) Dos EEG con silencio eléctrico cerebral practicados con seis horas de intervalo establece el diagnóstico. b) La práctica de una arteriografía permite asegurarlo. c) En intoxicación por hipnóticos o sedantes se requiere un período de observación de 24 horas. d) Los reflejos espinales pueden permanecer presentes.

835. ¿Qué dato de la exploración neurológica de un paciente nos hará dudar del diagnóstico de polineuropatía?

a) Hiperreflexia. b) Reflejo cutáneo - plantar flexor. c) Atrofia muscular. d) Disminución de la fuerza muscular.

836. Un lactante de doce meses presenta un cuadro de fiebre de 24 horas de evolución acompañado de catarro nasal. A la exploración se aprecia exantema morbiliforme generalizado, rosado y poco confluente. Se palpan numerosas adenopatías de predominio occipital. El diagnóstico más probable es:

a) Mononucleosis infecciosa. b) Exantema súbito. c) Rubéola. d) Escarlatina.

837. ¿Cuál de las afirmaciones siguientes es falsa, en la fiebre de origen desconocido?

a) La tuberculosis es la infección más a menudo responsable de fiebre de origen desconocido en España. b) En los niños la etiología más probable son las infecciones y la artritis crónica juvenil. c) En los ancianos las causas más comunes son leucemia aguda, enfermedad de Hodgkin, infecciones intraabdominales, tuberculosis y arteritis de la temporal. d) El pronóstico es peor en los niños.

838. El mecanismo más frecuente de contagio de la fiebre exantemática mediterránea es:

a) Por picadura de piojos. b) Por aplastamiento de piojo. c) Por picadura de garrapata del perro. d) Por picadura de pulgas.

839. Dentro de las características del trabajo en equipo están:

a) Trabajar en el mismo edificio. b) Que el equipo esté constituido por distintos tipos de profesionales. c) Que existan objetivos comunes. d) Todas son correctas.

840. Respecto al PERT (Program Evaluation and Review Technique), señale la respuesta correcta:

a) Es un método de establecimiento de prioridades que acentúa el peso relativo de los criterios seleccionados para evaluar alternativas. b) Es un indicador sintético del estado de salud de la población. c) Es un método que se utiliza para establecer los objetivos de un programa. d) Permite representar y analizar una secuencia de tareas.

841. La eficacia de un tratamiento mide:

a) La probabilidad de que una persona se beneficie de él en condiciones ideales. b) La eficacia de ese tratamiento con relación a su coste. c) La eficacia de ese tratamiento con relación a los riesgos que implica. d) La probabilidad de que una persona se beneficie de él en circunstancias reales.

842. ¿A través de qué test sabemos si dos variables cuantitativas están relacionadas?

a) El test de la Chi cuadrado. b) El test de la T de Student. c) El test de la U de Mann-Whitney. d) El test de la Correlación de Pearson.

843. ¿Con qué periodicidad recomienda el PAPPS tomar la tensión arterial a los mayores de 40 años?

a) Cada 5 años. b) Cada año. c) Cada 2 años. d) Cada 3 años.

844. Respecto a la ClAP-2 (Clasificación Internacional de la Atención Primaria). ¿Cuál de los siguientes enunciados es correcto?

a) La ClAP posee una estructura biaxial: el primer eje, con un código numérico, el segundo eje está formado por un código alfabético de un dígito.

Page 72: Cuestionario de Urología

b) Es incompatible con La CIE-9. c) La ClAP se guía por el principio de que la localización de la enfermedad tiene preferencia sobre la etiología. d) La CIAP surge para facilitar el registro del proceso en la atención primaria.

845. Un equipo de atención primaria mide el porcentaje de pacientes que tienen una toma" de tensión arterial registrada en su historia clínica durantl el último año. ¿Qué tipo de indicador están utilizando?

a) De eficiencia. b) De resultados. c) De estructura. d) De proceso.

846. ¿Cuál de los siguientes métodos de identificación de problemas y necesidades no pretende llegar a un consenso?

a) El enfoque a través de informadores clave. b) La técnica del grupo nominal. c) La técnica Delphi. d) El "brainwriting".

847. Al método de priorización en el que un grupo formado por expertos redactan individualmente sus opiniones y luego tienen una fase reflexiva común para hacer un listado de problemas identificados, se designa como:

a) Método Delphi. b) Grupo nominal. c) Brainwriting. d) Brainstorming.

848. ¿En cuál de los siguientes estudios existe mayor evidencia respecto a la efectividad de una intervención?

a) En la obtenida a partir de estudios de cohortes o casos controles por varios grupos de investigación. b) En la obtenida a partir de ensayos controlados no randomizados. c) En la obtenida a partir de un solo ensayo controlado y randomizado. d) En la obtenida a partir de la opinión de expertos de reconocido prestigio, basado en su experiencia.

849. Los siguientes fárrnacos se incluyen en el 2° escalón de la escalera analgésica de la OMS excepto:

a) AlNES. b) Dihidrocodeína. c) Codeína. d) Dextropropoxifeno.

850. Son efectos adversos frecuentes tras la administración de cloruro mórfico todos los siguientes excepto:

a) Hipotensión. b) Depresión. c) Sudoración. d) Somnolencia.

851. Una mujer de 35 años que sigue anticoncepción hormonal oral (AHO) como método de planificación familiar, acude a su consulta por sangrado intermenstrual. Está en tratamiento con amoxicilinaclavulánico por infección respiratoria. Señale lo correcto en relación a este supuesto:

a) Se debería, en este caso, suspender la toma de anticonceptivos. b) Recomendaría añadir un método de barrera hasta una semana después de acabar el tratamiento antibiótico. c) No es necesario modificar la pauta, pues se trata de un fenómeno normal al asociar amoxicilina a la anticoncepción hormonal. d) Recomendaría el añadir un método de barrera mientras se mantiene el tratamiento con amoxi-clavulánico.

852. De los siguientes antieméticos ¿cuál utilizaremos en la hiperémesis gravídica?

a) Tietilperazina. b) Doxilamina-Piridoxina. c) Metocloprarnida. d) Domperidona.

853. El feto tiene más riesgo de adquirir toxoplasmosis cuando la madre ha sufrido la infección:

a) Días antes del embarazo. b) En el primer trimestre. c) En el segundo trimestre. d) En el tercer trimestre.

854. Se consideran factores de riesgo de la osteoporosis postmenopaúsica todos menos:

a) Nulípara. b) Hipertiroidismo. c) Obesidad. d) Tabaquismo.

855. ¿Qué afirmación es cierta de las que se hacen respecto a la metadona?

a) Es un analgésico opiáceo mayor agonista parcial. b) Es empleado como coadyuvante en la anestesia. c) La administración oral se distingue de la parenteral en una menor duración del efecto. d) El síndrome de abstinencia que produce es más prolongado y de inicio más lento que la morfina.

856. Para organizar un programa de control de la hipertensión en una zona básica de salud en un plazo de tres meses, el

Page 73: Cuestionario de Urología

procedimiento más eficiente para valorar la magnitud del problema es:

a) Acudir a los registros de las consultas externas de los hospitales próximos. b) Inferir de las tasas de prevalencia de comunidades similares, la esperada para la Zona Básica. c) Realizar una encuesta que incluya la toma de la tensión arterial sobre una muestra significativa de la población de la Zona Básica. d) Tomar la tensión arterial a todos los pacientes que acudan en ese plazo al Centro de Salud.

857. En administración sanitaria, la dotación de recursos para realizar un determinado programa se mide en términos de:

a) Eficacia. b) Efectividad. c) Eficiencia. d) Análisis de coste.

858. Para cuantificar nuestro trabajo como residentes en el centro de salud, hemos diseñado un estudio que consiste en pasar un cuestionario a unos usuarios antes de iniciar la rotación, y otro al acabar la misma, cuyos items se puntúan de 1 a 5. ¿Con qué test podemos analizar este estudio?

a) Mann Whitney. b) Mac Nemar. c) Wilcoxon. d) Correlación de Pearson.

859. En una población de 10.000 mujeres en edad fértil, se proyecta realizar un examen citológico a todas ellas. Suponiendo que la sensibilidad de la citología es del 75% y la especificidad del 97%, ¿cuántas mujeres con cáncer NO serán diagnosticadas por citología, si la prevalencia de cáncer de cuello uterino es del 2 por 1O00?

a) Veinte. b) Cinco. c) Diez. d) Quince.

860. Un hombre de 63 años diabético, con úlcera gastroduodenal crónica fallece por peritonitis a los pocos días de una operación por perforación duodenal. La causa básica de la muerte es:

a) Peritonitis. b) Perforación duodenal. c) Diabetes. d) Úlcera duodenal crónica.

861. Los cuestionarios de satisfacción de la población miden el elemento:

a) Proceso. b) Estructura. c) Resultado. d) Calidad asistencial.

862. No es causa de HTA secundaria:

a) Enfermedad de Cushing. b) Hipotiroidismo. c) Hipertiroidismo. d) Hipoparatiroidismo.

863. ¿Cuál de las siguientes situaciones NO tiene indicación de tratamiento con alopurinol?

a) Gota tofácea. b) Hiperuricemia con ataques recurrentes. c) Nefropatía por ácido úrico. d) Hiperuricemia asintomática.

864. Una de las siguientes afmnaciones es falsa en relación con el colon irritable:

a) La localización más frecuente del dolor es el flanco derecho. b) Puede afectar a intestino delgado. c) Hay una motilidad intestinal alterada. d) Mayor frecuencia en mujeres.

865. Ante un paciente diagnosticado hace 3 días de gripe que comienza con tos y disnea, apareciendo un infiltrado bilateral en la radiografía de tórax, el tratamiento más adecuado sería:

a) Iniciar tratamiento con amoxicilina-c1avulánico a dosis de 875/125 cada 8 horas. b) Remitir el paciente al hospital. c) Administrar tratamiento sintomático. d) Tratar con azitromicina, 500 mg durante 5 días.

866. En una dermatitis aguda con eritema, edema y vesiculación, ¿cuál de los siguientes vehículos es más apropiado para el tratamiento?

a) Líquidos. b) Pomadas. c) Polvos. d) Cremas.

867. ¿En cuál de las siguientes situaciones el diagnóstico de espondilitis anquilosante es más probable?

Page 74: Cuestionario de Urología

a) Durante 5 años, un hombre de 32 años ha presentado dolor en región dorsolumbar, que se exacerba con la actividad, pero mejora con el reposo en cama. b) Durante 10 años, un hombre de 65 años ha sufrido dolor en región dorsolumbar que se irradia a parte posterior de muslos y rodillas. c) Durante 15 años, un hombre de 72 años ha presentado dolor de intensidad progresiva en región dorsolumbar, que se exacerba con la marcha y mejora con el reposo y con la flexión del tronco hacia delante. d) Durante 10 años, un hombre de 28 años ha presentado dolor y rigidez en la región dorsolumbar, ambos síntomas se exacerban por la noche y mejoran con la actividad.

868. Ante un enfermo con hepatitis B aguda, ¿qué prueba valoraría para envío a un hospital?

a) Grado de elevación de las transaminasas. b) Grado de la hepatomegalia. c) Grado de alteración del tiempo de protrombina. d) Grado de elevación de la bilirrubina total.

869. Una de las tareas que se describen a continuación no pertenece a la parte exploratoria de la entrevista clínica. Señálela.

a) Elaborar y sintetizar la información. b) Orientar y ejecutar la exploración física. c) Realizar una toma de precauciones. d) Delimitar y clarificar la demanda.

870. En un paciente diagnosticado de vértigo periférico por lesión vestibular derecha no esperaría encontrar:

a) Hipoacusia y acúfenos en oído derecho. b) Nistagmo horizontal puro. c) Nistagmo hacia la izquierda. d) Romberg positivo con lateralización hacia la derecha.

871. Hay que tener precaución en la utilización de la indometacina en los siguientes casos, EXCEPTO:

a) Epilepsia. b) Ductus arterioso persistente en prematuros. e) Parkinsonismo. d) U1cus.

872. ¿Qué signo clínico NO encontramos en un enfermo de Parkinson?

a) Temblor que mejora con el reposo. b) Bradicinesia. c) Alteración de la marcha. d) Sialorrea.

PARTE 2 de 3 (1015 cuestiones)

1: Varón de 50 años pendiente de cirugía de hernia inguinal. En el preoperatorio la gasometría basal muestra los siguientes resultados: pH 7.37, p02 41 nunHg, pC02 46 mmHg, saturación de O2 74%. La RX de tórax es normal, el paciente se encuentra asintomático y la exploración es anodina. ¿Cuál de las siguientes situaciones es la más probable? :

a) Embolia pulmonar. b) Enfisema pulmonar. c) Bronquitis crónica. d) Muestra venosa.

2: Mujer de 35 años en tratamiento con anticonceptivos orales que acude a Urgencias por disnea. A la exploración se detecta disminución del murmullo vesicular en base de hemitórax derecho y T de 37,8°C. El ECG revela taquicardia sinusal y en la RX de tórax se observa derrame pleural derecho sin condensaciones parenquimatosas. La toracocentesis demuestra líquido sero-hemorrágico con pH 7.33, proteínas 3.6 g/dL, glucosa 78 mg/dL, LDH 320 UI/L, Gram y Zielh negativos, ANAs negativos y ADA de 7 UIL. ¿Qué tratamiento es más oportuno en este contexto? :

a) Macrólidos. b) Inserción de tubo de tórax. c) Dicumarínicos. d) Heparina sódica.

3: El análisis del líquido pleural de un paciente muestra: proteínas en líquido pleural/proteínas en suero <0.5 y LDH en líquido pleural/LDH en suero <0.6. ¿Cuál de los siguientes diagnósticos es más probable? :

a) Mesotelioma. b) Tuberculosis. c) Neumonía. d) Síndrome nefrótico.

4: Un paciente ingresado por pancreatitis aguda comienza con taquipnea, taquicardia, sudoración y cianosis progresivas. La Pa02 es de 55 mmHg, la RX de tórax muestra infiltrados alveolares bilaterales y la presión de enclavamiento capilar pulmonar es normal. El aporte de oxígenó suplementario no mejora la situación. ¿Qué diagnóstico, entre los siguientes, es el más probable?

a) Neumonía nosocomial. b) Insuficiencia cardíaca. c) Distrés respiratorio del adulto. d) Tromboembolismo pulmonar.

5: Un paciente acude a Urgencias por disnea. La exploración física es normal. Se le realiza una gasometría arterial que muestra los siguientes valores: pH 7.47, p02 66 mmHg, PC02 29 mmHg y HCO3 22 mmol/L. Se sospecha crisis de ansiedad. ¿Cuál de las

Page 75: Cuestionario de Urología

siguientes pruebas apoyaría la sospecha de hiperventilación psicógena?:

a) Electrocardiograma con taquicardia sinusal. b) Recuento leucocitario normal. c) Gradiente alveolo-arterial de oxígeno normal. d) Espirometría normal.

6: Una paciente de 35 años acude a Urgencias por disnea. Refiere una historia de asma bronquial, pero nunca ha requerido asistencia ventilatoria. En los últimos días ha presentado disnea sibilante y en las últimas 12 horas ha utilizado al menos 12 veces su inhalador de salbutamol. A la exploración, está bien perfundida y coloreada, con una FC de 110 Ipm y una FR de 35 rpm, con tiraje supraclavicular e intercostal. Su TA es de 140/80 mm Hg y se objetiva un pulso paradójico de 20 mm Hg. A la auscultación pulmonar hay una disminución global del murmullo vesicular y prácticamente no se escuchan sibilancias. La auscultación cardíaca es rítmica, sin soplos. El resto de la exploración es anodina. La gasometría arterial basal muestra un pH 7.46, pC02 44 mmHg, p02 58 mmHg Y bicarbonato 16 mmol/L. ¿Cuál, de los siguientes planteamientos, es el más adecuado?:

a) Es una crisis asmática leve y hay que añadir esteroides inhalados, remitiéndola para control por su médico. b) Se trata de un abuso de betarniméticos. Hay que desaconsejar su uso de esa forma y asociar una xantina o ipratropio. c) Es un cuadro de ansiedad por abuso de betarniméticos. Hay que ajustar las dosis a las recomendadas y añadir una benzodiacepina. d) Es una crisis asmática severa que puede requerir asistencia ventilatoria de forma inmediata.

7: Un varón de 70 años diagnosticado de EPOC de grado moderado, estable, con tratamiento correcto de su patología pulmonar, ingresa por fibrilación auricular rápida cuya reversión a ritmo sinusal no se consigue. La ecocardiografía muestra ausencia de hipertensión pulmonar y cavidades derechas normales. El hematocrito es de 45 %. La Pa02 basal en revisiones anteriores está alrededor de 52 mm Hg y la PaC02 de 45 mmHg. A su tratamiento habitual se deben añadir fármacos frenadores de la frecuencia cardíaca:

a) Exclusivamente. b) Más oxigenoterapia domiciliaria nocturna. c) Más oxigenoterapia domiciliaria durante 16 horas/día mínimo. d) Más analépticos.

8: Ante un paciente con sospecha de sarcoidosis pulmonar estadío ID, ¿qué prueba diagnóstica, de las que a continuación se refieren, tiene mayor rendimiento? :

a) Lavado broncoalveolar. b) TAC torácica de alta resolución. c) Garnmagrafía pulmonar con galio. d) Biopsia transbronquial.

9: En un paciente se muestra una insuficiencia ventilatoria con fallo globál tanto en la inspiración como en la espiración. Señale en qué situación clínica de las siguientes NO se produce esta circunstancia:

a) Espondilitis anquilosante. b) Parálisis diafragmática. c) Miastenia gravis. d) Síndrome de Guillain-Barré.

10: En un paciente con EPOC se precisa diferenciar si el componente enfisematoso predomina sobre el bronquítico. ¿Cuál de las siguientes características es más definitoria del componente enflsematoso? :

a) Disnea importante. b) Expectoración purulenta abundante. c) Incremento de las marcas pulmonares en RX. d) Hipercapnia.

11: Cuando en un traumatismo torácico se descubre radiológicamente un ensanchamiento del mediastino superior, ha de pensarse en una posible lesión:

a) Pulmonar. b) Traqueobronquial. c) Esofágica. d) De los grandes vasos.

12: La práctica de una radiografía posteroanterior de tórax en espiración máxima es de gran utilidad para el diagnóstico de:

a) Un derrame pleural. b) Un neumotórax mínimo. c) Un hemotórax. d) Una atelectasia pulmonar.

13: Ante un paciente que acusa dolor torácico y cervicobraquial, que en la exploración física presenta miosis y enoftalmos ipsilaterales y en cuya RX de tórax se observa erosión de las primeras costillas, ¿cuál, de las siguientes, es la causa más probable? :

a) Tumor de Pancoast. b) Tumor de mediastino. c) Mesotelioma pleurallocalizado. d) Tumor de lóbulo medio.

14: En un enfermo de 65 años que presenta un carcinoma broncogénico no microcítico señale, de las complicaciones siguientes, la que se considera causa de irresecabilidad:

a) Síndrome de Pancoast. b) Afectación de pared torácica. c) Invasión de pericardio.

Page 76: Cuestionario de Urología

d) Síndrome de vena cava superior.

15: Varón de 54 años que ingresa con cuadro de dolor torácico prolongado y presenta en el ECG elevación del segmento ST en derivaciones II, III, aVF, V3R y V4R. A la exploración está sudoroso, con TA 90/50 mmHg, FC 98 lpm y aumento importante de la presión venosa yugular con signo de Kussmaul positivo. ¿Quétratamiento, de los siguientes, debería EVITARSE? :

a) Infusión de líquidos i.v. b) Inotropos. c) Diuréticos. d) Antiagregantes plaquetarios.

16: Mujer de 53 años, hipertensa en tratamiento con diuréticos y sin patología coronaria conocida previa, que ingresa por infarto agudo de miocardio sin onda Q, con cambios de la repolarización en derivaciones V2 a V6, cursando sin complicaciones. Se realiza ecocardiograma que demuestra fracción de eyección del ventrículo izquierdo del 40% y prueba de esfuerzo que resulta negativa para isquemia, con un tiempo de esfuerzo de 3 minutos, alcanzando el 65 % de la frecuencia cardíaca máxima prevista. ¿Qué actitud, de las siguientes, recomendaría? :

a) Tratamiento con bloquean te s de los canales del calcio. b) Tratamiento con betabloqueantes y aspirina, y prueba de esfuerzo al mes. c) Realización de coronariografía inmediata. d) Realización de ecocardiograma de esfuerzo.

17: A un paciente se le diagnostica miocardiopatía hipertrófica obstructiva (MHO). La mayor preocupación consiste en valorar el riesgo que tiene de presentar muerte súbita. ¿Qué factor, de los siguientes, NO se asocia a un mayor riesgo de muerte súbita? :

a) Historia familiar de MHO con muerte súbita. b) Taquicardia ventricular sostenida. c) Diagnóstico en la juventud. d) Severidad del gradiente intraventricular.

18: En un Centro de Salud requiere atención urgente un varón de 50 años, con antecedentes de dos infartos de rniocardio en los últimos 5 años. Refiere un dolor torácico similar al de los infartos previos, con cortejo vegetativo, que comenzó hace 90 minutos y no cede a pesar de la ntilización reiterada de uitroglicerina sublingual. Al explorar al enfermo el dolor ha cedido parcialmente, pero el paciente refiere encontrarse mal. Presenta una TA de 90/70 mmHg, FC 96 lpm, está sudoroso, con aceptable perfnsión periférica y buena coloración. Sn presión venosa central está discretamente elevada. En la exploración pulmonar se objetivan crepitantes en ambas bases. La auscultación cardíaca es rítmica, con cuarto tono. Los pulsos son normales. El abdomen es normal y no tiene edemas. Se le canaliza una vena, se administra oxígeno y se organiza su traslado inmediato a un Centro hospitalario. Dentro de las posibilidades del Centro de Salud y hasta poder trasladado, ¿cuál de las signientes medidas terapéuticas adicionales está indicada?:

a) Iniciar tratamiento con dosis bajas de un betabloqueantes de vida media corta. b) Administrar ácido acetilsalicílico. c) Iniciar digitalización. d) Administrar un expansor plasmático.

19: Paciente de 55 años, que consulta al mes de haber sufrido un infarto agudo de miocardio no complicado. Refiere fiebre y dolor precordial de características pleuríticas. En la exploración física se ausculta roce pericárdico. La RX de tórax muestra aumento del índice cardio-torácico y pequeño derrame pleural bilateral. El manejo más adecuado en este caso sería:

a) Anticoagulación y realizar coronariografía urgente. b) Tratamiento con betabloqueantes y realizar ergometria. c) Realizar una TAC de tórax. d) Tratamiento con salicilatos.

20: Varón de 60 años con historia de insuficiencia ventricular quierda secundaria a cardiopatía isquémica que acude por disnea invalidante de 2 horas de duración. La exploración física y la RX de tórax son compatibles con edema agudo de pulmón. Se observa TA 170/105 mmHg y 36 rpm. En el ECG hay taquicardia de la unión con complejos ventriculares estrechos a 130 lpm y descenso del ST de 1 mm en precordiales izquierdas. Seguía tratamiento con antagonistas de los canales del calcio, digoxina y diuréticos tiazídicos. De las siguientes medidas terapéuticas, señale cuál NO está indicada:

a) Oxigenoterapia. b) Nigroglicerina sublingual c) Furosemida intravenosa d) Digoxina intravenosa

21: Enfermo de 56 años que ingresa en el Hospital por infarto de miocardio agudo de cara inferior. A las 4 horas está bradicárdico (ritmo sinusal a 38 lpm) e hipotenso (80/50 mmHg) sin nuevos cambios en el ECG ni enzimáticos. ¿Cuál es, de las siguientes, la medida terapéutica más adecuada? :

a) Administración de suero salino isotónico. b) Colocación de marcapasos externo temporal. c) Administración i.v. de sulfato de atropina. d) Administración i.v. de dobutamina.

22: ¿Cuál de las siguientes afirmaciones, relativa al pulso arterial paradójico, es FALSA? :

a) Es una exageración de la tendencia a la disminución del pulso arterial en la inspiración mayor de 10 mmHg. b) Se produce por la disminución del volumen de eyección del ventriculo izquierdo y transmisión de la presión negativa intratorácica a la aorta. c) Aparece en la mitad de las pericarditis constrictivas. d) Es un hallazgo exploratorio habitual en la miocardiopatía hipertrófica obstructiva.

23: Un enfermo presenta en la exploración física ondas "a" cañón regulares. ¿Cuál de los siguientes diagnósticos haría? :

a) Bloqueo A-V de tercer grado. b) Ritmo de la unión A-V.

Page 77: Cuestionario de Urología

c) Bloqueo A-V de primer grado. d) Bloqueo A-V tipo Wenckebach.

24: ¿Cuál de las siguientes afirmaciones es correcta, en relación con la insuficiencia cardíaca congestiva? :

a) El pulso alternante es debido a la variación en el volumen de eyección como consecuencia de la recuperación incompleta de las células miocárdicas contráctiles de un latido a otro. b) En la insuficiencia cardíaca por miocardiopatía restrictiva habitualmente hay cardiomega1ia. c) El pulso alternante sólo existe si hay insuficiencia mitral y/o aórtica. d) La febrícula acompañada por vaso constricción indicada siempre infección sobreañadida.

25: Varón de 72 años que, desde hace 2, presenta dolor retroesternal opresivo que cede con el reposo y, ocasionalmente, síncopes de esfuerzo. En el último año tiene disnea de medianos esfuerzos sin ortopnea, disnea paroxística nocturna, ni edemas. ¿Qué hallazgos de los siguientes, esperaría encontrar en la exploración fisica? :

a) Pulso arterial céler, soplo de eyección pulmonar y refuerzo del componente pulmonar del 20 tono. b) Pulso arterial bisferiens, primer tono fuerte y chasquido de apertura mitral. c) Pulso arterial dícroto, soplo de eyección pulmonar y desdoblamiento fijo del 20 tono. d) Pulso arterial anácroto, soplo de eyección aórtico y desdoblamiento invertido del 20 tono con componente aórtico disminuido en intensidad.

26: Enferma de 45 años con antecedentes de fiebre reumática que presenta una historia clínica de disnea progresiva, palpitaciones y ocasional expectoración hemoptoica. La auscultación en el foco mitral muestra primer tono fuerte, chasquido de apertura y soplo de llenado mesodiastólico. Se plantea la posibilidad de cirugía o valvuloplastia con balón. Para inclinarse por una u otra actuación sería imprescindible conocer si hay:

a) Crecimiento importante de la aurícula izquierda. b) Alteraciones de la repolarización del ventriculo izquierdo. c) Trombos en la aurícula izquierda. d) Fibrilación auricular crónica.

27: ¿Cuál de los siguientes supuestos es correcto respecto a la angina de pecho? :

a) La historia clínica es el dato clave para el diagnóstico. b) Un electrocardiograma basal normal excluye el diagnóstico. c) La exploración física normal excluye el diagnóstico. d) Durante el dolor siempre hay galope izquierdo.

28: ¿Cuál de los siguientes medicamentos sería el de elección para controlar la hipertensión arterial en un paciente con varios episodios de insuficiencia cardíaca congestiva? :

a) Diltiazén. b) Propranolol. c) Clortalidona. d) Enalapril

29: Mujer de 58 años con historia de fiebre de 3 semanas de evolución. Hace 6 meses tuvo un episodio de amaurosis derecha de unos 2 minutos de duración y, desde entonces, refiere disnea y febrícula. La disnea mejora cuando descansa tumbada en la cama. Exploración: frecuencia cardíaca 92 Ipm regular y rítmico, TA 110/70 mmHg, T 39.3°C y lesiones petequial es en piel. Auscultación cardíaca: Primer tono reforzado con 2° tono normal, sonido diast6lico precoz de baja frecuencia con soplo diastólico en ápex que se atenúa o desaparece con el decúbito, Con mayor probabilidad la paciente tiene:

a) Estenosis mitral reumática crítica. b) Endocarditis bacteriana subaguda. c) Mixoma auricular izquierdo. d) Prolapso de la válvula mitral.

30: Señale, entre las siguientes, la indicación más adecuada de la valvuloplastia mitral percutánea con balón:

a) Estenosis mitral severa asintomática. b) Lesión mitral combinada con insuficiencia severa. c) Estenosis mitral severa extensamente calcificada. d) Estenosis mitral reumática severa sintomática con fusión comisural.

31: ¿Cuál es, en la actualidad, el tratamiento de elección de la estenosis pulmonar congénita? :

a) La valvuloplastia pulmonar percutánea, con sonda-balón. b) El uso de vasodilatadores. c) El reemplazamiento valvular con prótesis metálica. d) El reemplazamiento valvular con prótesis biológica.

32: Ante un cuadro clínico de edema global del miembro inferior desde la raíz del muslo, en una paciente encamada por postoperatorio traumatológico, ¿qué prueba, de las siguientes, solicitaría en primer lugar? :

a) Angiorresonancia. b) Flebografía ascendente y/o cavografía retrógrada. c) Eco-Doppler venoso. d) Estudio con fibrinógeno marcado 1125.

33: Diagnosticado casualmente, en una ecografia abdominal, un aneurisma de aorta infrarrenal de 4.2 cm de diámetro en un varón de 85 años, con coronariopatía isquémica no revascularizable y EPOC con FEVl menor de un litro, la actitud más adecuada, de las siguientes, será:

a) Cirugía urgente. b) Cirugía electiva. c) Conducta expectante.

Page 78: Cuestionario de Urología

d) Seguimiento ecográfico cada 6 meses.

34: La clínica del síndrome de robo de la subclavia, en pacientes sin lesiones asociadas en vasos extracraneales, se presenta habitualmente como:

a) Claudicación de brazo e isquemia hemisférica homolateralo b) Ataques isquémicos transitorios homolaterales. c) Ictus vertebrobasilar. d) Asintomática.

35: Señale, en relación con las disecciones de aorta tipo B, o tipo III, cuál de las siguientes afirmaciones, es FALSA:

a) Un tercio de los pacientes que sobreviven suelen ser intervenidos de forma tardía por progresión del aneurisma. b) Sólo el 70% requieren cirugía de urgencia. c) Se tratan médicamente con vigilancia intensiva y fármacos hipotensores, como primera opción. d) Pueden romperse en cavidad pleural.

36: ¿Qué afirmación de las siguientes es cierta, respecto a la peritonitis espontánea del cirrótico? :

a) Más de la mitad de los casos no tienen ascitis cuando se inicia el cuadro. b) Sólo el cultivo positivo del líquido peritoneal permite un diagnóstico cierto. c) Aunque no se trate, es un proceso leve reversible espontáneamente. d) Una proporción significativa de los casos cursa sin fiebre.

37: Un alcohólico de 52 años viene presentando, desde hace cuatro, episodios repetidos con frecuencia creciente de dolor epigástrico irradiado a espalda; últimamente, el dolor es diario, inmediatamente tras la ingesta. Ha perdido unos diez kilos de peso en cuatro años. Señala que sus heces son muy abundantes, grasientas, espumosas y flotan sobre el agua. En una placa se ven calcificaciones anómalas en el abdomen superior. ¿Qué prueba haría, a continuación, para completar el diagnóstico?

a) Una determinación de grasa fecal. b) No haría ninguna prueba más. c) Una prueba secretoria con secretina. d) Una colangio-pancreatografía endoscópica.

38: Repetidos estudios recientes han probado que el 95 % de 'os ulcerosos duodenales, no tomadores de AINEs, ni portadores del síndrome de Zöllinger-Ellison (Z-E), están infectados por Helicobacter pylori (HP), que, entre los sanos, la proporción de infectados aumenta con la e4ad, hasta ser de alrededor del 60 % a los setenta años, y qúe, con tratamiento antibiótico correcto, se erradica el germen en alrededor del 90 % de los ulcerososo Como consecuencia de lo anterior, señale, entre los siguientes, el planteamiento más eficiente ante un varón de 35 años, no tomador de AINEs, con historia de diez años de dolores epigástricos ritmados, que acude a su consulta por un nuevo brote doloroso, sin complicaciones y al que se encuentra un bulbo duodenal deformado, con nicho, a RX:

a) Hacer gastroscopia para confirmar el diagnóstico y tomar biopsia para demostrar HP, con prueba de ureasa, antes de iniciar tratamiento erradicador. b) Hacer gastroscopia y biopsia para confirmar el diagnóstico y demostrar el HP, con cultivo y antibiograma, para elegir el tratamiento erradicador. c) Iniciar tratamiento con omeprazol + claritromicina + amixocilina, por ejemplo, y no hacer más estudios, salvo falta de respuesta o recaída. d) lnicar tratamiento sintomático con bloqueadores H2 y hacer una prueba de aliento para HP, antes de iniciar el tratamiento erradicador.

39: Una mujer de 35 años, más bien obesa, acude a la consulta por sensación de contractura de los músculos periorales, especialmente cuando hiperventila, desde hace unos cuatro meses. Últimamente, también nota espasmos de las manos. No tiene historia de hipermenorrea, polimenorrea, ni ninguna otra pérdida de sangre. Hace una deposición al día. Preguntada por el aspecto de sus heces, afirma que "son como toda la vida". Tan sólo tuvo un embarazo y parto normales hace nueve años. Su padre padeció de cólicos nefríticos y su madre de litiasis biliar. La exploración muestra discreta palidez, obesidad discreta, signos de Trousseau y de Chvosteck positivos, resto normal. Las heces de 24 horas pesan 300 g. La analítica muestra Hb 10.1 g/dL, Hto 32%, VCM 70 f.l3, sideremia 20 f.lg/dL, transferrina 450 f.lg/dL, saturación 13% y ferritina 3 ng/dL, albuminemia 4 g/dL y globulinas normales. Función hepática normal. Resto no relevante. ¿Cuál de las siguientes pruebas le conduciría más directamente al diagnóstico?:

a) Hemorragias ocultas en heces y radiología del tubo digestivo con bario. b) Determinación de hormona paratiroidea en sangre y calciuria de 24 horas. c) Estudio de absorción de hierro con radioisótopos. d) Determinación de anticuerpos antiendomisio en plasma.

40: Señale, entre las siguientes, la afirmación correcta respecto a la pancreatitis secundaria a litiasis biliar:

a) Es excepcional que el cálculo responsable se elimine espontáneamente y con ello ceda el cuadro. b) En enfermos con pancreatitis severa no debe hacerse colecistectomía en las primeras 48 horas. c) En pacientes con pancreatitis leve es preferible hacer colecistectomía en las primeras 48 horas. d) No existe indicación de colecistectomía tras el primer episodio de pancreatitis leve.

41: Sobre la infección por Helicobacterpylori (IIP) señale cuál de las siguientes afirmaciones es FALSA:

a) La infección por HP se asocia a la mayor parte de las úlceras pépticas. b) La prevalencia de la infección se relaciona con factores socioeconómicos. c) La presencia de HP en la mucosa duodenal es imprescindible para provocar la lesión ulcerosa. d) En los enfermos ulcerosos duodenales curar la infección supone reducir drásticamente la posibilidad de recurrencia de la úlcera.

42: Señale, del grupo de enfermedades que se relacionan, en cuál puede la seudoobstrucción intestinal cróuica formar parte, con mayor frecuencia, de su síndrome paraneoplásico correspondiente:

a) Linfomas viscerales. b) Carcinoma de ovario.

Page 79: Cuestionario de Urología

c) Carcinoma pulmonar de células pequeñas. d) Carcinoide diseminado.

43: En relación con el seguimiento mediante colonoscopia de los enfermos de colitis ulcerosa y el riesgo mayor de aparición de un carcinoma colo-rectal en ellos, señale cuál de las siguientes afirmaciones es correcta:

a) Antes de las colonoscopias debe investigarse la presencia de sangre oculta en heces, pues es un dato que ayudará al diagnóstico de degeneración neoplásica. b) Las revisiones colonoscópicas pueden limitarse al colon izquierdo porque es en esta localización donde aparecen la gran mayoría de los tumores. c) La presencia de displasia confirmada por dos patólogos expertos es suficiente para indicar una proctocolectomía total. d) Las biopsias se tomarán únicamente en las zonas donde se observen áreas con datos macroscópicos de tumor.

44: Varón de 70 años que consulta por disfagia progresiva que se acompaña, tres meses después de su inicio, de regurgitación alimenticia posingesta. Refiere pérdida de 10 kg de peso. En relación con estos datos, señale cuál de las siguientes afirmaciones NO es correcta:

a) Ha de indicarse una endoscopia oral para descartar el cáncer de esófago. b) Si en la endoscopia se encuentra un esófago dilatado y sin peristaltismo, está excluído el cáncer porque se trata de una acalasia. c) Si en la endoscopia no se ha encontrado tumor, debe indicarse una manometría esofágica porque podría tratar se de un trastorno motor primario. d) La manometría normal excluye el diagnóstico de acalasia.

45: Una mujer de 85 años ingresó con insuficiencia cardíaca y diarrea sanguinolenta. La exploración abdominal no mostraba signos de irritación peritoneal y había ruidos. Un enema opaco mostró imágenes "en huella digital" y la colonoscopia colitis en colon izquierdo respetando la mucosa rectal. ¿Cuál sería, entre las siguientes, la actuación correcta?:

a) Realizar arteriografía. b) Practicar hemicolectomía izquierda. c) Prescribir dieta absoluta, tratamiento de insuficiencia cardíaca y observación. d) Instaurar anticoagulación con heparina.

46: En un paciente alcohólico sin evidencia alguna de compromiso hemodinámico, que desarrollo insuficiencia hepática aguda tras una semana de cefalea inespecífica tratada con analgésicos comerciales, un valor de GPT (ALT) mayor de 8.000 UI/L debe sugerir:

a) Obstrucción completa de la vía biliar principal. b) Toxicidad por Acetaminofén (Paracetamol). c) Hepatitis isquémica aguda. d) Ingesta accidental de metanol.

47: Señale qué afirmación, entre las siguientes, es correcta respecto al tratamiento de las hepatitis virales con Interferón alfa:

a) Está indicado en todas las hepatitis agudas y crónicas. b) Puede ser eficaz en las hepatitis crónicas por sus acciones antiproliferativa y antioxidante. c) Inhibe la replicación vírica y suprime la citolisis a través de sus efectos antivíricos e inmunomoduladores. d) Ocasiona aplasia medular reversible con frecuencia.

48: Señale en cuál de las siguientes enfermedades la manometría esofágica estacionaria constituye la prueba diagnóstica definitiva:

a) Enfermedad por reflujo gastroesofágico. b) Achalasia. c) Esófago de Barrett. d) Enfermedad esofágica por cáusticos.

49: Un paciente de 62 años con etilismo crónico, ingresa por pancreatitis aguda evolucionando favorablemente. Tres semanas después de su ingreso, se realiza una ecografía apreciándose una colección de 3 cm de diámetro, quística, en contacto con la pared posterior del estómago. ¿Entre las siguientes, cuál es la actitud inmediata más correcta? :

a) Drenaje por vía percutánea. b) Tratamiento quirúrgico del seudoquiste. c) Drenaje endoscópico de la colección. d) Seguimiento con ecografías seriadas.

50: Respecto a la enfermedad por reflujo gastroesofágico señale, de las siguientes afirmaciones, cuál es la correcta:

a) El estudio esófago-gastro-duodenal baritado ayuda a cuantificar mejor la enfermedad. b) La manometria esofágica estacionaria no ofrece informa ción pronóstica cuando hay daño fisiológico. c) La indicación para tratamiento quirúrgico son los síntomas que impiden al paciente hacer una vida normal. d) La disfunción del esfínter esofágico inferior con esofagitis es indicación de cirugía inmediata.

51: Un varón de 48 años sin antecedentes digestivos de interés, tratado con AINEs, es intervenido con carácter urgente por hemorragia digestiva alta provocada por una úlcera gástrica solitaria en el fundus del estómago. ¿Cuál de los tratamientos siguientes es el más adecuado? :

a) Sutura de la úlcera, vagotornía y piloroplastia. b) Sutura de la úlcera, antrectornía y Bi1lroth l. c) Biopsias múltiples y cierre de la úlcera. d) Excisión en cuña y biopsia, vagotornía y piloroplastia.

52: ¿Cuál de los siguientes constituye el tratamiento más efectivo y seguro a largo plazo en los pacientes con achalasia? :

a) Esofagomiotornía. b) Nitroglicerina sublingual antes de las comidas.

Page 80: Cuestionario de Urología

c) Agentes bloqueantes de los canales del calcio. d) Dilatación neumática.

53: Mujer de 63 años que acude a Urgencias por fiebre y cefalea. Desde 36 horas antes presenta tos no productiva y artralgias y en el mes previo ha aquejado otalgia derecha. Ha sido diagnosticada de cérvicoartrosis y quince años atrás fue esplenectomizada tras un trauma tismo por accidente de circulación. Exploración: consciente y orientada, febril, sin referir dolor localizado alguno; TA 150/100 mmQg, frecuencia cardíaca 96 lpm regular y rítmico; lesiones purpúricas sobre la piel del abdomen y flancos y cierto grado de resistencia a la flexión de columna cervical. Durante la exploración, súbitamente, la paciente pierde la conciencia, objetivándose entonces TA. .sistólica de 40 mmHg, con pulso arterial a 140 lpm, rítmicamente y débil. Señale cuál de los siguientes diagnósticos es el más probable:

a) Hemorragía subaracnoidea de origen hipertensivo. b) Trombosis arterial cerebral en progresión. c) Disección de la aorta torácica. d) Meningitis aguda por neumococo.

54: Varón de 30 años que acude a consulta por presentar, desde hace unos dos meses, cefalea, náuseas, vómitos y diplopia. En la exploración neurológica se observa una pérdida del reflejo pupilar a la luz y parálisis de la convergencia ocular con conservación de la acomodación. ¿En cuál de las siguientes estructuras se localiza la lesión?:

a) Nervio óptico. b) Quiasma óptico. c) Porción posterior del 3° ventriculo. d) Suelo del 4° ventriculo

55: Paciente que acude a su médico porque, desde hace aproximadamente un mes, presenta cefalea de carácter sordo y dolor a nivel ocular y retroocular derecho y de forma progresiva desarrolla visión doble. En la exploración neurológica encontramos edema de papila del ojo derecho y parálisis del IV y VI par derechos con afectación de la primera rama del trigémino ipsilateral. ¿En cuál de las siguientes localizaciones se sitúa la lesión?

a) Espacio retroesfenoidal. b) Vértice del peñasco. c) Fisura orbitaria superior. d) Región paraselar.

56: Mujer de 23 años que acude a su médico de cabecera por presentar debilidad muscular generalizada. El cuadro se inicia unos meses antes al notar dificultad para subir al autobús, añadiéndose al mes dificultad para elevar los brazos. Con el diagnóstico de miastenia gravis es referida al neurólogo. ¿Cuál de los siguientes tratamientos es el adecuado de manera inmediata? :

a) Timectornía. b) Piridostigmina. c) Azatioprina. d) Prednisona.

57: Paciente de 45 años con antecedentes familiares de enfermedad renal poliquística. Acude a Urgencias por cefalea, diplopia y ptosis del ojo derecho. ¿Cuál es el diagnóstico a descartar en primer lugar de los siguientes?:

a) Migraña oftalmopléjica. b) Arteritis de la arteria temporal. c) Cefalea en racimos. d) Aneurisma de la arteria comunicante posterior.

58: Paciente de 50 años, diabético, que acude a Urgencias por presentar pérdida súbita de la visión del ojo derecho con pérdida del campo visual superior. En la exploración neurológica se objetiva edema de papila del ojo derecho. ¿Cuál, de los siguientes, es el diagnóstico más probable?:

a) Neuritis óptica. b) Neuropatía óptica isquémica anterior. c) Neuritis isquémica secundaria a arteritis de células gigantes. d) Papilitis diabética.

59: Varón de 55 años que padece desde hace tres meses debilidad muscular a nivel proximal de las extremidades, sequedad de boca, dolores musculares y parestesias en los cuatro miembros. Durante la exploración se comprueba debilidad de los músculos proximales. La sensibilidad está conservada y los reflejos osteotendinosos están disminuidos en miembros superiores y abolidos en los inferiores. ¿Qué prueba complementaria de las siguientes ayudaría a establecer el diagnóstico? :

a) Estudio del LCR. b) Biopsia de nervio afecto. c) Biopsia de músculo afecto. d) RX de tórax.

60: Paciente de 60 años que presenta fiebre elevada, dolor de espalda y paraparesia. Se realiza resonancia nuclear magnética de columna que muestra masa epidural que comprime la médula dorsal. ¿Cuál, de los siguientes, es el diagnóstico más probable? :

a) Meningioma dorsal. b) Metástasis epidural. c) Hematoma epidural. d) Absceso epidural.

61: ¿Cuál es el tratamiento antibiótico empírico de elección, entre los siguientes, en un paciente de 30 años con meningitis bacteriana? :

a) Cefotaxima. b) Vancomicina. c) Imipenem. d) Penicilina.

Page 81: Cuestionario de Urología

62: Señale, de las propuestas a continuación, cuál es la causa más frecuente de hidrocefalia congénita:

a) Malformación de Dandy-Walker. b) Quistes congénitos. c) Meningitis perinatal. d) Estenosis del acueducto de Silvio.

63: Señale, de los cuadros clínicos signientes, cuál considera más característico de la hidrocefalia normotensa en el anciano:

a) Crisis convulsivas, alteraciones de la marcha y pérdida de visión. b) Demencia, pérdida de visión e incontinencia de esfínteres. . c) Incontinencia de esfínteres, pérdida de visión y alteraciones de la marcha. d) Demencia, incontinencia de esfínteres y alteraciones de la marcha.

64: Señale, entre las siguientes, cuál es la causa más frecuente en la práctica clínica de compresión medular:

a) Metástasis. b) Neurinoma. c) Ependimoma. d) Astrocitoma.

65: ¿Qué tumor de los siguientes, se origina en el suelo del 4° ventrículo siendo por esta razón dific il su extirpación total?:

a) Oligodendroglioma. b) Ependimoma. c) Neurinoma. d) Hemangioblastoma.

66: ¿Cuál de las siguientes afirmaciones sobre la resistencia insulínica es FALSA?:

a) Es un problema muy raro, desarrollándose en menos de un 0.1 % de los pacientes tratados con insulina. b) Se define arbitrariamente como el requerimiento de 300 ó más unidades de insulina diarias para controlar la hiperglucemia y evitar la cetoacidosis. c) Se desconoce el defecto molecular asociado en muchos de los casos. d) Se encuentran anticuerpos anti-insulina Ig G en muchos sujetos dentro de los primeros 60 días de inicio de la terapia insulínica.

67: ¿Cuál de los motivos siguientes es el fundamental para tratar a los pacientes con acromegalia?:

a) Corregir la hiperprolactinemia asociada. b) Corregir las alteraciones estéticas del síndrome. c) Evitar los efectos secundarios de la disminución de la absorción de calcio y de glucosa. d) Evitar el riesgo de padecer cáncer de colon, pólipos premalignos y trastornos cardiovasculares.

68: Mujer de 63 años con historia de diarrea episódica y pérdida de peso de 2 años de evolución, junto a crisis súbitas de enrojecimiento facial acentuada con la ingesta de alcohol. Menopausia a los 50 años. La exploración muestra rash consistente en telangiectasias lineales sobre un fondo rojo-violáceo sobre mejillas y dorso de la nariz. Datos de laboratorio: serotouina plasmática y ácido 5-hidroxiindolacético urinario (5-HIAA) normales. 5-hidroxitriptófano (5-HTP) e histamina plasmáticos elevados. Calcitonina en sangre, VIP plasmático y V AM urinario dentro de límites normales. Señale, entre los siguientes, el diagnóstico más probable:

a) Somatostatinoma. b) Carcinoma medular tifoideo. c) Feocromocitoma. d) Síndrome carcinoide.

69: Un paciente diabético ingresa en Urgencias presentando bajo nivel de conciencia, glucemia de 350 mgldL, Na 150 mEq/L, K 3.1 mEq/L, bicarbonato 12 mEq/L, pH 7.14 Y cetonuria. ¿Cuál de las siguientes medidas NO es adecuada?:

a) Administración de insulina rápida en bomba de infusión. b) Administración de suero salino fisiológico. c) Administración de cloruro potásico. d) Administración de expansores plasmáticos.

70: En relación con la oftalmopatía de Graves-Bassedow, señale la respuesta correcta:

a) Frecuentemente evoluciona con una importante pérdida de agudeza visual. b) Afecta fundamentalmente al humor vítreo. c) Puede cursar con normofunción tiroidea. d) El tratamiento de elección en sus fases iniciales son los corticoides.

71: Con respecto a los tumores hipofisarios, señale, de las propuestas, la afirmación INCORRECTA:

a) La acromegalia suele ser secundIDia a un macroadenoma hipofisario secretor de GH. b) La enfermedad de Cushing suele deberse a un microadenoma hipofisario secretor de ACTH. c) El macroprolactinoma es más frecuente en los varones. d) Los adenomas. hipofisarios no funcionantes son menos frecuentes que los funcionantes.

72: Señale cuál de las siguientes opciones NO está indicada en el tratamiento del síndrome de secreción inadecuada de ADH (SIADH) crónico:

a) Restricción hídrica. b) Dieta rica en sodio. c) Demeclotetraciclina. d) Espironolactona.

73: Varón de 45 años, gran fumador, sin antecedentes previos de enfermedad, diagnosticado dos meses antes de problema pulmonar no precisado. Ingresa de urgencias por deterioro progresivo, durante las dos últimas semanas, de sus funciones

Page 82: Cuestionario de Urología

cognitivas. La familia niega traumatismo previo. La exploración muestra normotensión arterial y ausencia de edemas. Datos de laboratorio: Na+ plasmático 120 mEq/L, osmolaridad plasmática 245 mOsmlkg, glucemia normal, urea 20 mgldL. Orina: 250 mOsmlkg, Na+ 35 mEq/L. ¿Cuál de los siguientes cuadros es el más probable? :

a) Insuficiencia suprarrenal crónica. b) Diabetes insípida central. c) Polidipsia compulsiva. d) Síndrome de secreción inadecuada de hormona antidiurética (SIADH).

74: Ante una paciente de mediana edad, que en la exploración presenta una glándula tiroidea moderadamente aumentada de tamaño, de consistencia pétrea, sin afectación de nódulos linfáticos regionales, con ausencia de fiebre, con pulso y recuento leucocitario normales, deberemos sospechar la existencia de:

a) Carcinoma folicular. b) Carcinoma medular. c) Carcinoma anaplásico. d) Tiroiditis de Riedel.

75: Un varón de 35 años consulta por un nódulo en el lóbulo derecho tiroideo, de unos 2 cm de diámetro, que se moviliza con la deglución y no produce sintomatología adicional alguna. El estudio de función tiroidea es normal. Se le somete a ecografía y punción aspiración con aguja fina. ¿Qué actitud, entre las siguientes, recomendaría en función del resultado de estas pruebas? :

a) Cirugía, si es sólido y se informa como nódulo coloide. b) Cirugía, si es un quiste, aunque haya desaparecido tras la punción. c) Cirugía, si es sólido y se observan numerosas células foliculares. d) Repetir la citología a los dos meses, si es un quiste con citología sospechosa.

76: ¿Cuál de los siguientes tumores endoI crinos del páncreas incluye en su complejo sintomático la presencia de rash cutáneo, denominado eritema migratorio necrolítico? :

a) Glucagonoma. b) Vipoma. c) Insulinoma. d) Gastrinoma.

77: Una mujer de 42 años consulta por bocio asintomático. Tras el pertinente estudio, se le diagnostica de enfermedad de Hashimoto. ¿Cuál de las siguientes afirmaciones respecto a esta paciente NO es cierta? :

a) El riesgo de linfoma tifoideo es mayor que en la población general. b) Tendrá títulos altos de anticuerpos antitiroideos. c) En los cortes histológicos se observarán células de citoplasma oxifílico. d) El tratamiento será quirúrgico, mediante tiroidectomía subtotal.

78: Un paciente de 35 años, con antecedentes de radiación cervical a bajas dosis por un proceso benigno en la infancia, presenta un nódulo tiroideo en lóbulo derecho. La citología (PAAF) de dicho nódulo es concordante con carcinoma papilar de tiroides. ¿Qué tipo de intervención quirúrgica, entre las siguientes, está indicada? :

a) Tiroidectomía total. b) Hemitiroidectomía derecha. c) Hemitiroidectomía derecha e istmectomía. d) Enucleación de nódulo.

79: ¿Cuál de los siguientes parámetros es de menor utilidad como monitor de la actividad de la artritisreumatoide?:

a) Velocidad de sedimentación. b) Factor reumatoide. c) Proteína C reactiva. d) Otros reactantes de fase aguda.

80: En relación con los hallazgos de laboratorio en la panarteritis nodosa (PAN), señale la afirmación correcta:

a) En más del 90% de los casos hay anticuerpos antiribonucleoproteínas. b) En más del 90% de los casos hay antígeno de superficie del virus de la hepatitis B. c) El hallazgo de título5 elevado5 de anticuerpos antihistonas es muy específico. d) No existe ninguna prueba que sea específica.

81: Señale en cuál de las siguientes manifestaciones clínicas del lupus eritematoso sistémico NO está indicado el tratamiento con corticoesteroides:

a) Anemia hemolítica. b) Glomerulonefritis. c) Artritis aislada. d) Miositis.

82: Mujer de 53 años que padece, desde hace 20, un cuadro no diagnosticado caracterizado' por episodios de dolor en manos, con cambios de coloración y aparición de lesiones ulceradas recidivantes en pulpejo de los dedos. Dos meses antes su TA era de 110/80 mmHg. Consulta por oliguria, con orina de aspecto normal en los días previos y T A de 190/130 mmHg. Niega ingesta previa de fármacos y se objetiva, junto a secuelas de las lesiones referidas en dedos, hinchazón de las manos y cierto endu-recimientos cutáneo en brazos y antebrazos. Con mayor probabilidad la paciente tendrá:

a) Crioglobulinemia mixta esencial. b) Granulomatosis de Wegener. c) Crisis renal de la esclerodermia. d) Nefrosclerosis arterial maligna.

83: Indique cuál de las siguientes afIrmaciones en relación con los ataques de gota es FALSA:

Page 83: Cuestionario de Urología

a) En los niños suelen afectarse las rodillas. b) Se asocian a tratamiento con diuréticos. c) Pueden ocurrir con ácido úrico normal. d) Ceden con la administración de antiinflamatorios.

84: Un paciente de 28 años consulta por dolor lumbar de 9 meses de duración, de predominio nocturno, que le despierta durante el sueño. Actualmente' se ha asociado dolor en la parte inferior del talón derecho, notando en los últimos días hinchazón y dolor en rodilla derecha. Indique cuál de las siguientes actuaciones le puede proporcionar datos que apoyen el diagnóstico que debe sospechar en primer lugar:

a) RX ántero-posterior de pelvis. b) Exploración física de caderas. c) Análisis del líquido sinovial de la rodilla. d) RX lateral de calcáneos.

85: Un paciente de 34 años afecto de esclerosis múltiple ha requerido dosis altas de corticoides. Meses después presenta dolor en pliegne inguinal derecho irradiado a muslo, acentuándose con la deambulación, asociándose a limitación de la movilidad de la cadera. Señale, de las propuestas, cuál es la causa más probable:

a) Distrofia simpático refleja. b) Fractura patológica de cadera. c) Artritis por pirofosfato cálcico. d) Necrosis aséptica de cabeza femoral.

86: Una paciente de 42 años de edad presenta dolor agudo en la cara posterior del miembro inferior izquierdo, junto con abolición del reflejo aquíleo. ¿Cuál, de los que se enumeran, es el diagnóstico más probable?:

a) Escoliosis idiopática tóraco-lumbar. b) Enfermedad de Pott a nivel L1. c) Hernia discal L5-S1. d) Espondilitis infecciosa.

87: Una paciente de 55 años, muy reivindicativa, nos consulta tras haber sido tratada hace 3 meses en otro Centro de una fractura de muñeca. La mano y muñeca afectas muestran piel tumefacta, enrojecida y con sudor frío y se queja de dolor al contacto, que aumenta con los intentos de movilización activa o pasiva. La RX muestra osteoporosis moteada. ¿Cuál de las siguientes causas debe sospecharse en primer lugar? :

a) Síndrome compartimental evolucionando (contractura isquémica de Volkmann). b) Distrofia simpática refleja (atrofia ósea de Südeck). c) Rigidez articular dolorosa secundaria a exceso de inmovilización. d) Consolidación en mala posición de la fractura con defectuosa rehabilitación posterior.

88: Paciente de 45 años con dolor lumbar de características mecáuicas, muy intenso, que le llega a producir crisis vagales al intentar forzar determinadas posturas, sin irradiación a miembros inferiores, de 24 horas de evolución. El signo de Lassegue, es negativo y no hay alteraciones motoras ni sensitivas, ni de los reflejos ni de los esÍmteres. ¿Cuál es la actitud más adecuada ini-cialmente? :

a) Indicar reposo, analgésicos, antiinflamatorios y/o relajantes musculares y vigilar evolución. b) Solicitar resonancia magnética para confirmar hernia discal y valoración quirúrgica. c) Enviarlo a un Servicio de urgencias para valoración por Medicina Interna y Traumatología. d) Solicitar TAe abdominal urgente para descartar patología neoplásica abdominal.

89: La positividad de la prueba de Trendelemburg es reveladora de la debilidad de:

a) El psoas ilíaco. b) Los rectos anteriores del abdomen. c) Los abductores de la cadera. d) El sartorio.

90: A partir del segundo año de la vida puede encontrarse hueso fibrilar o inmaduro en:

a) Los cuerpos vertebrales. b) El calcáneo. c) La cabeza femoral. d) El callo de fractura.

91: Un hombre de 30 años sufre en un átcidente de tráfico fractura del tercio medio del fémur izquierdo, con rotura de la arteria femoral. Una vez ingresado en el hospital, se procede a hacer la reducción y fIjación de la fractura junto con la reparación arterial. Horas después se instaura un edema en el miembro lesionado, dolor intenso de carácter pulsátil y parestesias a nivel del pie. El pulso periférico es débil. ¿Cuál de los siguientes diagnósticos debe plantearse en primer lugar?:

a) Gangrena isquémica. b) Síndrome compartimental. c) Neuropraxia del nervio ciático. d) Distrofia simpático-refleja.

92: Un muchacho de 7 años sufre un traumatismo al caerse de una bicicleta, produciéndose una fractura diafisaria espiroidea del fémur derecho. ¿Cuál de los siguientes tratamientos es un principio el más indicado?:

a) Tracción continua con el miembro en posición 90°-90°. b) Tracción al cénit. c) Osteosíntesis con placa y tomillos. d) Enclavijamiento endomedular.

93: ¿Cuál, de los siguientes, es con más frecuencia el germen causal de espondilodiscitis piogénea en pacientes con SIDA,

Page 84: Cuestionario de Urología

adictos a drogas por vía parenteral?:

a) Salmonella. b) Haemophilus influenzae. c) Pseudomonas aeruginosa. d) Streptoeoeeus granulosus.

94: Un adolescente es apuñalado y golpeado en una reyerta callejera. Está insconsciente, presenta 3 heridas por las que entra y sale aire en hemitórax izquierdo y 2 heridas incisas en cara anterior y medial de muslo derecho por las que sangra pulsátil y abundantemente. ¿Cuál de las propuestas es la actitud a adoptar de forma inmediata antes de la llegada al hospital?:

a) Taponamiento parcial de las heridas torácicas seguido de compresión de las heridas de extremidades y traslado. b) Traslado inmediato a un centro hospitalario, intubándolo durante el transporte y perfundiendo líquidos a presión. c) Control del sangrado arterial seguido de exploración neurológica detallada y después intubación y traslado. d) Control del sangrado arterial seguido de colocación de tubo de Guedel y ventilación con Ambú y traslado.

95: Un paciente de 32 años con infección VIR, ingresa con un cuadro de pancreatitis aguda. No es consumidor de alcohol. La ecografía abdominal descarta litiasis biliar. Sigue tratamiento antirretroviral con zidovudina (AZT), didanosina (DDI) e indinavir. ¿Qué se debe suprimir de su tratamiento? :

a) Didanosina (DDI). b) Zodovudina (AZT). c) fudinavir. d) Los tres fármacos.

96: Un paciente de 43 años, con una prótesis mitral implantada 3 semanas antes, acude a Urgencias por fiebre de 5 días de evolución. Dos días más tarde el laboratorio de Microbiología informa del crecimiento en 5 de los 6 frascos de hemocultivo de cocos Gram positivos en racimo. ¿Cuál, de los siguientes, es el tratamiento de elección hasta conocer los resultados definitivos? :

a) Vancomicina + gentamicina + rifampicina. b) Cloxacilina + gentamicina + rifampicina. c) Penicilina + gentamicina + rifampicina. d) Ceftriaxona.

97: Una paciente de 45 años presenta de forma progresiva en los últimos 3 días un cuadro de cefalea, deterioro del nivel de conciencia y fiebre de 39.5°C. Entre sus antecedente s destaca una enfermedad de Crohn que ha requerido tratamiento de forma irregular en los últimos 3 años. Desde hace 6 meses está tomando mesaIazina y 15 mg de prednisonaldía. La exploración clínica no presenta datos significativos, salvo discreto signos de irritación meníngea. El hemograma muestra 15.600 leucocitos, con una fórmula normal. La bioquímica elemental es normal. La RX de tórax no presenta alteraciones significativas. Se realiza una punción lumbar, obteniéndose un LCR con proteínas de 560 mg/dL, glucosa 25 mg/dL y 325 células/rnL (85% células mononucleares). Las tinciones de Gram y de Ziehl-Neelsen en LCR son negativas. ¿Cuál es la actitud más correcta? :

a) Sospechar una meningoencefalitis auto inmune y subir la dosis de prednisona a 1 mg/kg de peso, añadiendo qui-mioprofilaxis con isoniacida. b) Sospechar una meningitis por gérmenes de origen gastrointestinal. fuicar tratamiento antibiótico empírico que cubra gérmenes Gram (-) y anaerobios. c) Sospechar una meningitis tuberculosa. fuicar tratamiento inmediato con tres tubercu10státicos. d) Sospechar una encefalitis herpética. fuiciar tratamiento empírico inmediato con aciclovir.

98: El germen causal más frecuente de artitis séptica entre pacientes entre 15 a 40 años es:

a) Staphylococcus aureus. b) Haemophilus influenzae. c) Neisseria gonorrhoeae. d) Streptococcus pyogenes.

99: Un paciente, diagnosticado de infección por VIR, se presenta por primera vez en la consulta con candidiasis oral, aportando la siguiente analítica: linfocitos CD4: 123 células/µL; carga viral: 50.000 copias/mL (mediante reacción en cadena de la polimerasa, PCR). ¿Cuál de las siguientes actitudes NO es correcta? :

a) Iniciar tratamiento con tres drogas, dos inhibidores de la transcriptasa inversa y un inhibidor de la proteasa. b) Iniciar profilaxis para neumonía por Pneumocistis carinii. c) Realizar un Mantoux. . d) Iniciar tratamiento con zidovudina y según la respuesta, decidir la ampliación o no del tratamiento.

100: Un paciente ingresa con fiebre de más de 15 días de evolución, observándose en la RX de tórax un infiltrado pulmonar con cavitación en el lóbulo superior derecho. ¿Cuál de las siguientes afirmaciones es correcta? :

a) Un Mantoux negativo excluye el diagnóstico de tuberculosis. b) El paciente tiene una tuberculosis miliar. c) Se debe realizar broncoscopia al día siguiente. d) Debemos recoger al menos tres muestras de esputo para búsqueda de bacilos ácido-alcohol resistentes.

101: ¿Cuál de los siguientes hechos es preciso para el diagnóstico definitivo de una infección por Aspergillus?:

a) Elevación al cuádruple de los títulos de anticuerpos sérico s en un segundo control. b) Aislamiento del microorganismo en cultivos de esputo. c) Demostración de invasión tisular. d) Demostración de material genético en muestras tisulares mediante reacción en cadena de la polimerasa.

102: Un paciente, fumador, de 60 años, acude a Urgencias con un cuadro de fiebre, tos y expectoración amarillenta de 36 horas de evolución. Se le objetiva una condensación con broncograma aéreo en la base pulmonar derecha en la RX de tórax. ¿ Cuál de las siguientes pruebas complementarias es de mayor utilidad para decidir su hospitalización?:

a) Realización de tinción de Gram en una muestra de esputo. b) TAC torácica.

Page 85: Cuestionario de Urología

c) Hemocultivos seriados. d) Gasometría arterial basal.

103: La mayoría de las neumonías producidas por gérmenes Gram negativos son:

a) Comunitarias. b) Producidas tras accidentes de tráfico. c) Habituales de ambientes ganaderos. d) Adquiridas en el hospital.

104: El antibiótico de elección en el tratamiento de una neumonía producida por Legionella pneumophila es:

a) Eritromicina. b) Penicilina G. c) Cotrimoxazol. d) Cefotaxima.

105: Un paciente ingresa en Urgencias con disminución del nivel de conciencia, fiebre de 39°C y TA de 7 0/40 mmHg. ¿Cuál, de las siguientes, considera la actitud inicial más correcta?

a) Iniciar tratamiento antibiótico empírico de amplio espectro. b) Realizar una punción lumbar. c) Bajar la fiebre. d) Aportar líquidos intravenosos.

106: Señale, entre las siguientes, la conducta más adecuada ante un paciente leucopénico que comienza con fiebre:

a) Observarle para vigilar si aparecen síntomas respiratorios. b) Tomar los cultivos adecuados y comenzar tratamiento antibiótico empírico cuanto antes. c) Tomar los cultivos adecuados y esperar a los resultados antes de iniciar ningún tratamiento. d) Tratarle exclusivamente con factor estimulante de colonias de granulocitos.

107: Es conocido el riesgo potencial de transmisión de vm o hepatitis desde un paciente portador al equipo médico-quirúrgico que le atiende. La política recomendada actualmente para evitar estos contagios, consiste en aplicar una serie de medidas especiales para evitar el contacto con sangre o líquidos orgánicos del paciente. Para ello se debe:

a) Llevar a cabo pruebas serológicas sistemáticamente a todos los pacientes y tomar medidas en los que den algún resultado positivo. b) Solicitar consentimiento previo y llevar a cabo pruebas serológicas a los pacientes que lo hayan otorgado, tomado medidas en los portadores. c) Averiguar el nivel de riesgo individual mediante historia clínica y aplicar las medidas a los pacientes con elevada probabilidad de ser portadores. d) Considerar potenciales portadores a todos los pacientes y tomar medidas en todos ellos.

108: Un paciente presenta un cuadro compatible con abdomen agudo quirúrgico. Se le practica una laparotomía exploradora con carácter urgente, sin encontrar nada patológico. Desde el punto de vista del riesgo de infección, esta intervención se considera:

a) Limpia. b) Limpia-contaminada. c) Contaminada. d) Sucia.

109: En relación con el forúnculo, señale cuál de las afirmaciones siguientes es la correcta:

a) Su agente causal suele ser el Streptococcus pyogenes. b) En los de la cara, hay peligro de que la infección afecte a estructuras meníngeas. c) Mecta simultáneamente a varios aparatos pilosebáceos. d) La infección suele comenzar a nivel del tejido celular subcutáneo.

110: Señale, de las siguientes, cuál es la afirmación correcta en relación con la hidrosadenitis supurativa:

a) Es una infección de las glándulas sudoríparas ecrinas. b) Se origina simultáneamente en varios folículos pilosebáceos. c) Suelen participar gérmenes de predominio anaerobio. d) Tiende a la recidiva tras el tratamiento.

111: ¿Qué actitud terapéutica debe tomarse ante un mieloma múltiple quiescente? :

a) Agentes alquilantes (clorambucil, ciclofosfamida, ….) orales en pauta continua. b) Poliquimioterapia tipo CHOP. c) Fludarabina. d) Abstención y observación.

112: Un paciente presenta el siguiente hemograma: Hb 9 gr/dL, VCM 120 fL, HCM 34 pg, reticulocitos 12%, leucocitos 9.500/mm3, plaquetas 320.000/mm3. En el frotis se observa hipersegmentación de neutrórdos. El diagnóstico es de anemia:

a) Aplásica. b) Hemolítica. c) Ferropénica. d) Megaloblástica.

113: Enferma de 78 años, enviada al hospital por astenia progresiva y anemia. Los datos hematológicos son: VSG 90; Hb 9 g/dL; Hto 25%; VCM 102 fL; HCM 30 pg; CHCM 34 g/dL; reticulocitos 1 %; leucocitos 7.300/mm3 (con reparto normal) y plaquetas 230.000/mm3. Las cifras de ácido fólico y vitamina B12 son normales. El diagnóstico más probable será:

a) Anemia por deficiencia de hierro. b) Anemia aplásica.

Page 86: Cuestionario de Urología

c) Síndrome mielodisplásico. d) Leucemia linfocítica crónica.

114: ¿Cuál es el método más fiable para diagnosticar la hemoglobinuria paroxística nocturna? :

a) Test de hemólisis en medio ácido (Ham-Dacie). b) Test de hemólisis en sacarosa. c) Acetil-colinesterasa eritrocitaria. . d) Detección de flujo de CD55 y CD59 eritrocitario y/o leucocitario.

115: ¿Cuál de los siguientes factores NO implica un peor pronóstico en la evolución de la leucemia linfoblástica aguda (LAL)? :

a) Edad superior a 20 años. b) Ausencia de remisión completa a las 4 semanas del tratamiento de inducción. c) Ausencia de CDlO (antígeno común de LAL). d) Hiperdiploidía con más de 50 cromosomas.

116: Señale cuál de los siguientes hallazgos hace muy improbable el diagnóstico de policitemia vera:

a) Masa eritrocitaria >36 mg/kg en el varón y >32 mg/kg en la mujer. b) Saturación de oxígeno arterial >92%. c) Eritropoyetina elevada. d) Esplenomegalia.

117: Paciente de 20 años, con dolor de garganta, fiebre, subictericia y adenopatías cervicales y polo de bazo palpable. En la analítica presenta leucocitosis con linfocitosis, "células linfoides activadas" y elevación discreta de transaminasas. ¿Cuál es el diagnóstico más probable?

a) Infección por virus de Epstein-Barr. b) Hepatitis A aguda. c) Infección por citomegalovirus. d) Hepatitis B aguda.

118: Enferma de 20 años, que sufre una hemorragia moderada después de una extracción dentaria. Se le practica un estudio de hemostasia y se encuentra un tiempo de sangría alargado. ¿En cuál de estos procesos pensaría en primer lugar? :

a) Hemofilia A. b) Anticoagulante circulante. c) Enfermedad de van Willebrand. d) Hepatopatía.

119: Señale cuál de las siguientes pruebas NO tiene ninguna utilidad en el estudio de un caso de mieloma múltiple:

a) Determinación de proteinuria de Bence Jones. b) Estudio de proteína monoclonal en suero. c) Medición de la calcemia. d) Estudio de sobrecarga férrica.

120: En las leucemias agudas no linfoblásticas, la translocación (8,21) es un rasgo de buen pronóstico, que se asocia habitualmente a:

a) Coagulación intravascular diseminada. b) Hipertrofia gingival. c) Subtipo citológico FAB M4 con eosinofilia. d) Subtipo citológico FAB M2

121: En un niño de 5 años, en anasarca de 20 días de evolución y con una analítica de proteinuria muy selectiva de 6 g/24 horas, sedimento urinario normal, hipoalbmninemia e hiperlipidemia, acompañando a una creatinina plasmática de 0.6 mg/dL, el diagnóstico más probable será:

a) Hialinosis focal. b) Nefropatía de cambios mínimos. c) Glomerulonefritis aguda postestreptocócica. d) Nefropatía membranosa.

122: La hipocomplementemia persistente acompaña generalmente a la:

a) Glomerulonefritis membranosa idiopática. b) GIomerulonefritis membrano-proliferativa. c) Enfermedad de Wegener. d) Nefropatía membranosa.

123: Un paciente de 68 años presenta progresivo deterioro del nivel de conciencia en las horas siguientes a haber sido operado de herniorrafia inguinal con anestesia general, de la que despertó normalmente. Su situación hemodinámica es estable, no hay signos de insuficiencia cardíaca, no tiene fiebre ni focalidad neurológica. El hemograma y el estudio de coagulación son normales. La bioquímica sérica muestra: urea 36 mg/dL, Glu 120 mg/dL, Na 118 mEq/L, K 4.5 mEq/L, úrico 2.3 mg/dL, osmolaridad urinaria 500 mOsm/kg, Na urinario 35 mEq/L. Señale, de las siguientes, la causa más probable de este cuadro:

a) Insuficiencia hepática aguda por anestésicos. b) Deshidratación. c) Hiponatremia ADH-dependiente. d) Tubulopatía pierde-sal.

124: Ante el cuadro descrito en la anterior, la actitud más adecuada, de las siguientes, sería:

a) Restricción de líquidos a 800-1000 rnL/día. b) Tratamiento antibiótico empírico en espera de los cultivos.

Page 87: Cuestionario de Urología

c) Reposición de líquidos con suero salino isotónico ajustando el ritmo a la diuresis. d) Reposición de líquidos con suero salino hiperosmolar.

125: ¿En cuál de los siguientes cuadros clínicos hay que tener especial cuidado para evitar el desarrollo de insuficiencia renal aguda después de una exploración con medios de contraste? :

a) Litiasis renal bilateral. b) Ictericia obstructiva. c) Mieloma múltiple. d) Hipercalciuria.

126: El tratamiento correcto de un paciente con hiponatremia severa y disfunción neurológica secundaria es la:

a) Corrección rápida de la hiponatremia en 6 horas con suero salino hipertónico. b) Inducción de coma barbitúrico y manitol i.v. c) Corrección paulatina de la hiponatremia en más de 24 horas para evitar la desmielinización osmótica. d) Administración de vasopresina i.v.

127: Un varón de 45 años llega comatoso a Urgencias. Presenta PaCO2 basal 25 mmHg, pO 7.15, hipocalcemia moderada con Gap aniónico y osmolar elevados, leucocitosis y cristaluria. Con más probabilidad este paciente tiene una intoxicación por:

a) Barbitúricos. b) Monóxido de carbono. c) Etilenglicol. d) Benzodiazepinas.

128: Varón de 45 años con historia de rinorrea purulento, fiebre, artralgias, infiltrados pulmonares nodulares y glomerulonefritis rápidamente progresiva. ¿Cuál de los siguientes hallazgos es más probable encontrar? :

a) Anticuerpos antinucleares. b) Anticuerpos anticitoplasma de neutrófilo. c) Anticuerpos antimembrana basal glomerular. d) Crioinmunoglobulinas.

129: En el tratamiento de la hiperpotasemia grave de un paciente urémico NO está indicado administrar:

a) Resinas de intercambio catiónico (sodio, calcio), b) Bicarbonato sódico i.v. c) Glucosa con insulina i.v. d) Tiazidas por vía oral.

130: La litiasis renal cálcica secundaria a una hipercalciuria idiopática de origen renal se trata mediante abundante ingestión de líquidos, asociada a:

a) Fosfato de celulosa. b) Ortofosfatos. c) Tiazidas. d) Allopurinol.

131: En un varón de 60 años con antecedentes de hematuria autolimitada, la urografía i.v. muestra un defecto de replección en la pelvis renal derecha de 2 cm de diámetro y superficie irregular, sin apreciarse en la ecografía imagen hiperecogénica con sombra acústica posterior. ¿Qué diagnóstico es el más probable, de los que a continuación se relacionan?:

a) Litiasis de ácido úrico. b) Litiasis cistínica. c) Carcinoma urotelial de pelvis renal. d) Amiloidosis piélica.

132: Ante un paciente sin obstrucción uretral, diagnosticado mediante biopsia transrectal de un carcinoma de próstata en el que el tacto rectal pone de manifiesto una próstata totalmente indurada y fija y la gammagrafía ósea evidencia afectación metastásica, ¿cuál, entre los siguientes, es el tratamiento más adecuado? :

a) Prostatectomía radical. b) Resección transuretral del tumor. c) Seguimiento periódico. d) Castración por radiación o por cirugía.

133: Ante un paciente con litiasis renal recidivante, que presenta en la RX simple de abdomen afectación litiásica renal bilateral y cuyo estudio metabólico muestra hipercalciuria, hipocitruria, hiperkaliuria, asícomo un pH urinario alcalino mantenido, debemos pensar en:

a) Hiperparatiroidismo primario. b) Hiperaldosteronismo primario. c) Acidosis tubular renal. d) Sarcoidosis.

134: La exploración más adecuada para diagnosticar el reflujo vésico-ureteral es la:

a) Cistografía retrógrada miccional. b) Cistografía funcional de la urografía i.v. c) Ureteropielografía retrógrada. d) Ecografía renal perimiccional.

135: Si un paciente con acné, en tratamiento con clindamicina por vía oral, presenta un cuadro de colitis con moco, sangre y pus, además de suspender la cIindamicina, ¿qué tratamiento de los siguientes es, en principio, el más indicado? :

a) Dieta astringente.

Page 88: Cuestionario de Urología

b) Prednisona. c) Eritromicina. d) Vancomicina.

136: Ante un niño que acude a una guardería, a quien le han aparecido lesiones vesiculosas y costrosas de color miel en los antebrazos y, cuatro días más tarde, en la cara, sin afectación general, una de las siguientes consideraciones es correcta. Señálela:

a) El diagnóstico se hará por inmunofluorescencia indirecta. b) El cuadro probablemente se deba a sobreinfección herpérica. c) Los títulos de antiestreptofisina O estarán muy probablemente elevados. d) Es muy probable que en el cultivo de las lesiones crezcan varios gérmenes Gram positivos.

137: En el caso descrito en la anterior, la confirmación del diagnóstico clínico nos lleva a tomar una de estas actitudes:

a) Tratar con corticoides. b) Descartar afectación renal secundaria. c) Tratar con aciclovir. d) Tratar con tetraciclinas.

138: Un varón de 30 años, bebedor, refiere la aparición reciente de ampollas en dorso de manos y erosiones frente a mínimos traumatismos. En la exploración destacan: hepatomegalia de 4 cm. no dolorosa; lesiones vesiculosas, áreas erosionadas y cicatrices con quiste de milium en dorso de manos y antebrazos. También se aprecia hipertricosis facial. ¿Cuál de las siguientes pruebas realizaría para confirmar el diagnóstico? :

a) Determinación de anticuerpos antinucleares. b) Determinación de HBSAg. c) Biopsia cutánea y realización de inmunofluorescencia directa. d) Determinación de porfuinas (uroporfirina y coproporfuina) en orina.

139: ¿Cuál es el principal factor condicionante del pronóstico de un melanoma maligno sin metástasis en tránsito, ganglionares ni hematógenas? :

a) La edad del paciente. b) Su desarrollo sobre un nevus previo. c) Su localización en zonas acras. d) El espesor de la lesión medido en milímetros.

140: ¿Cuál de las entidades siguientes representa ya, o puede evolucionar hacia, un carcinoma epidermoide cutáneo?

a) Queratosis seborreica. b) Queilitis actínica. c) Queilitis granulomatosa. d) Nevus displásico.

141: ¿Cuál de los siguientes es el diagnóstico más probable en un paciente adulto que presenta una asociación de otorrea purulenta e hipoacusia, que cursa de forma indolora y unilateral? :

a) Supuración congénita del adulto. b) Otitis media crónica. c) Sordera otógena. d) Otitis media aguda.

142: Señale, de las propuestas, cuál es la conducta más conveniente ante un adulto que presenta una disfonÍa de más de 15 días de duración:

a) Laringoscopia sin demora. b) Antiinflamatorios y revisión al mes. c) Estudio bacteriológico. d) Ecografía cervical.

143: Señale, de lo siguiente, lo más importante en el diagnóstico de los trastornos del equilibrio:

a) Audiometría. b) hnpedanciometría. c) Videonistagmografía. d) Anarnnesis y exploración clínica.

144: En un adulto la insuficiencia respiratoria nasal unilateral, progresiva y acompañada de secreción serosanguinolenta es más sugerente de:

a) Rinitis crónica inespecífica. b) Ocena. c) Cuerpo extraño. d) Tumor de fosas nasales o senos paranasales.

145: Señale cuál sería, de las siguientes, la primera .opción diagnóstica a considerar en un paciente que presenta, durante la ingesta, disfagia severa o afagia que mejora con la regurgitación espontánea o provocada de alimentos ingeridos:

a) Tumor de orofaringe. b) Tumor de hipofaringe. c) Divertículo faringo-esofágico. d) Esclerosis lateral amiotrófica.

146: El ronquido en el niño es un signo que debe orientar a uno de los siguientes diagnósticos:

a) Retraso de maduración neurológica.

Page 89: Cuestionario de Urología

b) Labio leporino. c) Agenesia de úvula. d) Hipertrofia adenoidea.

147: Una mujer de 27 años de edad, miope de 6 dioptrías negativas en ambos ojos, acude a Urgencias refiriendo visión de "moscas volantes" y "puntos brillantes" a lo largo de las últimas 3 semanas en su ojo derecho, así como la aparición reciente de una especie de "cortina" que le impide ver con su campo visual nasal en ese ojo. ¿Qué afirmación, de las siguientes, es la correcta respecto a esta enfermedad? :

a) Lo más probable es que tenga un desprendimiento de retina de tipo traccional. b) Se puede descartar que se trate de una uveítis. c) Se le debe practicar un test de Jones. d) La paciente necesitará probablemente tratamiento quirúrgico.

148: Un paciente acude a consulta por pérdida de visión en un ojo. En la exploración del fondo del ojo se observan hemorragias en llamaradas, venas dilatadas y edema de retina. ¿Cuál es el diagnóstico más probable? :

a) Oclusión de la arteria central de la retina. b) Neuritis óptica. c) Oclusión de la vena central de la retina. d) Neuritis retrobulbar.

149: En un paciente con un glaucoma agudo de ángulo estrecho, ¿cuál de los siguientes tratamientos NO es adecuado? :

a) Pilocarpina all %. b) Atropina. c) Manitol al 20%. d) Acetazolamida.

150: Ante un niño con estrabismo y con ambliopía en un ojo, el tratamiento más eficaz para recuperar su agudeza visual es:

a) Penalización óptica. b) Pleóptica. c) Tratamiento con prismas. d) Oclusión.

151: Una mujer de 78 años acude a consulta por una disminución brusca de agudeza visual en su ojo derecho de una semana de evolución. Refiere que, en la zona central de su campo visual, aparece una mancha grisácea y que las líneas rectas parecen estar torcidas y deformadas. En el fondo de ojo se aprecia una lesión sobreelevada de la retina de color verde grisáceo, rodeada de tres hemorragias intrarretinianas puntiformes. La lesión tiene localización extrafoveal. En el ojo izquierdo se observan drusas blandas confluentes e biperpigmentación macular. ¿Cuál de las siguientes afirmaciones es correcta respecto a esta paciente? :

a) El riesgo de que la visión del ojo izquierdo disminuya es muy pequeño. b) Está indicado realizar una angiografía fluoresceínica del ojo derecho. c) El tratamiento mediante láser de argón no consigue reducir el riesgo de pérdida visual grave. d) Lo más probable es que se trate de una membrana epirretiniana.

152: Acude a Urgencias un paciente con intenso picor y lagrimeo en ambos ojos, hiperemia conjuntival, fotofobia, exudado más o menos viscoso y formación papilar en la conjuntiva tarsal. ¿Cuál es el diagnóstico probable? :

a) Chalazión. b) Conjuntivitis bacteriana. c) Conjuntivitis vírica. d) Conjuntivitis alérgica.

153: Un médico de 40 años ingresó como consecuencia de una fractura de fémur producida en un accidente de tráfico. Se practicó cirugía ortopédica sin complicaciones y, durante los dos primeros días del postoperatorio, el paciente estuvo inquieto pero bien orientado. Al tercer día, el paciente se mostró de repente confuso y temeroso y refirió visión de serpientes y escorpiones que se movían por las sábanas. En la exploración se observó midriasis, temblor de oscilaciones amplias de las manos y los párpados, sudoración profusa, taquicardia con latido hiperquinético y temperatura de 39°C. Los antecedentes pers onales carecían de interés. Se consideraba un bebedor social moderado y negaba el abuso de otras drogas. De los siguientes, ¿cuál es el diagnóstico que más sugiere esta situación? :

a) Delirio por deprivación alcohólica. b) Delirio postanestesia. c) Septicemia. d) Hematoma subdural.

154: En relación con el ataque de pánico, señale cuál de las proposiciones siguientes es la correcta:

a) Está en relación con una situación estresante masiva. b) Cursa con grave desestructuración de la conciencia. c) No está ligado a sucesos claramente identificables. d) Es muy sugerente de ciertos estados depresivos.

155: ¿Qué afirmación de las siguientes, respecto a la naltrexona es correcta? :

a) Es un fármaco sustitutivo de la heroína utilizado en programas de mantenimiento. b) Es un agonista opiáceo, utilizado en la desintoxicación de heroína. c) Es un sedante que se aplica como paliativo en el síndrome de abstinencia de opiáceos. d) Es un antagonista de los opiáceos de acción prolongada, que se utiliza después de la cura de desintoxicación.

156: Señale cuál de estas proposiciones es FALSA respecto a la conducta suicida:

a) El número de intentos de suicidio es mayor en los hombres que en las mujeres, mientras que los suicidios consumados se dan más en las mujeres.

Page 90: Cuestionario de Urología

b) El riesgo de suicidio puede surgir en cualquier categoría de diagnóstico psiquiátrico. c) El riesgo de suicidio aumenta estadísticamente con la edad. d) Las tentativas "chantajistas" de suicidio son menos graves pero no exentas de riesgo.

157: Señale cuál de las siguientes características diferenciadoras de la depresión endógena (DE) y la depresión neurótica (DN), es FALSA:

158: Señale cuál de las siguientes afirmaciones sobre los tratamientos de los procesos afectivos con carbonato de litio es ERRÓNEA:

a) Se elimina casi en su totalidad por la orina. b) Se recomienda, entre otras pruebas, hacer ECG y creatininemia antes de iniciar el tratamiento. c) Se consideran niveles terapéuticos óptimos de litemia los que oscilan entre 0,8 y 1,2 mEq/L. d) Es más eficaz en las fases depresivas que en las maníacas.

159: Señale, de las siguientes, cuál es la defiuición más correcta de esquizofrenia simple:

a) Un proceso esquizofrénico asociado a retraso mental leve o moderado. b) Una forma de esquizofrenia caracterizada por su buen pronóstico. c) Una forma de esquizofrenia donde sólo se detectan síntomas productivos, sin sintomatología negativa alguna, y que no evoluciona hacia un deterioro profundo. d) Una forma de esquizofrenia caracterizada por la ausencia de síntomas productivos y la presencia de escasa respuesta emocional.

160: Las reacciones adversas de los antidepresivos tricíclicos es más probable que tengan significado clínico en los pacientes afectos de uno de los siguientes procesos. Señálelo:

a) Jaqueca. b) Parkinson. c) Insomnio. d) Hipertrofia benigna prostática.

161: Todas las situaciones siguientes, EXCEPTO una, suponen un peligro de que los niveles de litio sean más altos de lo esperado. Señálela:

a) Uso de diuréticos tiazídicos. b) Dieta hipersódica. c) Dieta hiposódica. d) Deshidratación

162: Se ha detectado positividad del HbeAg en un control serológico practicado a una gestante de 32 semanas sin antecedentes de hepatopatía y que presentaba dicho marcador negativo en el primer trimestre. Indique qué afirmación NO es cierta en relación con este caso:

a) El riesgo de transmisión perinatal sería muy elevado si presentase positividad frente al HbeAg. b) Se precisa una especial vigilancia del crecimiento fetal. c) El riesgo de curso clínico grave está aumentado por tratarse de una gestante. d) El riesgo de que el feto infectado se convierta en portador crónico es importante.

163: Después de un expulsivo normal y tras 60 minutos de período de alumbramiento, no se aprecian signos de desprendimiento placentario, a pesar de haberse aplicado masaje uterino y de haber incrementado moderadamente la dosis de oxitocina. Se indica una extracción manual de placenta, que resulta imposible por no existir plano de separación entre la placenta y la pared uterina. ¿Cuál es el diagnóstico más probable?:

a) Placenta incarcerada. b) Engatillamiento placentario. c) Placenta succenturiada con cotiledón aberrante. d) Placenta adherente por acretismo placentario.

164: La curva de temperatura basal en una mujer de 28 años muestra 36,5°C a 36,6°C durante todo un cic lo de 32 días. Debe considerarse que es un ciclo:

a) Anormal por ser excesivamente largo. b) Totalmente normal. c) Anormal por posible anovulación. d) Normal pero con ovulación algo atrasada.

165: Ante una mujer de 60 años que padece una hiperplasia endometrial atípica, ¿cuál de los siguientes tratamientos es el más adecuado? :

a) Legrado fraccionado. b) Histerectomía con doble anexectomía. c) Histerectomía radical. d) 25 mg/d de gestágeno durante l mes.

166: ¿Cuál de los siguientes síntomas clínicos aparece máz precozmente en una paciente con cáncer invasor de cuello uterino? :

a) Dolor hipogástrico.

DE DN

a) Etipatogenia Factores hereditarios y constitucionales

Trastornos del desarrollo afectivo y factores psicosociales

b) Curso Fásico Continuo (fluctuante)

c) Ritmo diurno Empeoramiento vespertino Mejoría vespertina

d) Psicoterapia Relativamente inoperante Indicada

Page 91: Cuestionario de Urología

b) Flujo blanco-verdoso. c) Compresión de la vejiga. d) Metrorragia.

167: Una gestante de 10 semanas tiene contacto con un niño que, 6 días más tarde, desarrolla un cuadro de exantema y síndrome general infeccioso sugerente de infección por virus de rubéola. En el primer control serológico gestacional se detectó la negatividad de la IgG específica. ¿Cuál de las siguientes afirmaciones es la correcta? :

a) La aparición de IgM materna positiva constituye indicación para la detenninación de la IgM fetal. b) No existe posibilidad de contagio dado que el niño ya no se hallaba en fase de eliminación viral. c) En este período de la gestación, el riesgo de infección y de afectación embrionaria es mínimo. d) La administración inmediata de gammaglobulina hiperinmune específica reduce considerablemente el riesgo de infección embriofetal.

168: Una gestante de 37 semanas diagnosticada de preeclampsia leve presenta un cuadro súbito de convulsiones tónico-clónicas seguidas de coma. La presión arterial es de 180/120 mmHg. En función del diagnóstico más probable, ¿qué medicación estaría contraindicada como tratamiento inicial en esta situación? :

a) Labetalol. b) Diazóxido. c) Sulfato de magnesio. d) Hidralazina.

169: Una paciente primigesta sin antecedentes de interés y con controles analíticos previos normales, presenta glucemias basales de 148 y 152 mg/dL en dos determinaciones realizadas en la 22ª semana de gestación. ¿Cuál es la actitud más correcta en este caso? :

a) Realizar una prueba de tolerancia a la glucosa con sobrecarga oral de 100 g. b) Realizar una prueba con sobrecarga oral de glucosa de 75 g y, en caso de normalidad, repetida con 100g. c) Realizar una prueba de O'Sullivan entre la 24ª y la 28ª semanas de gestación d) Instaurar tratamiento dietético y controlar el perfil glucémico.

170: Una paciente con antecedente de aborto tardío anterior, ingresa a la 18ª semana de gestación por dolor hipogástrico leve no acompañado de hemorragia. En la exploración se aprecia cérvix dilatado y bolsa íntegra prominente. Dos horas más tarde se produce un aborto consumado incompleto. ¿Cuál es la etiología más probable de este cuadro? :

a) Síndrome antifosfolípido primario. b) Insuficiencia cérvico-ístmica. c) Mioma uterino de localización submucosa. d) Síndrome de Asherman.

171: Una paciente sometida a cesárea por inducción fallida tras amniorrexis prematura, presenta en su tercer día de puerperio malestar general, fiebre de 38,5°C, loquios fétidos , útero subinvolucionado y doloroso a la palpación. ¿Cuál es el diagnóstico más probable? :

a) Endometritis puerperal. b) Dehiscencia de la histerotomía. c) Retención de restos placentarios. d) Necrosis isquémica de mioma uterino.

172: Entre las citadas, ¿qué respuesta es FALSA con respecto a la enfermedad celíaca del niño? :

a) La biopsia intestinal es indispensable para hacer un diagnóstico de certeza. b) La talla baja es motivo de sospecha de intolerancia al gluten. c) La positividad de los anticuerpos antiendomisio guarda escasa relación con la actividad de la enfermedad. d) Puede haber historia familiar en la intolerancia al gluten.

173: Un lactante de 5 meses presenta deshidratación con los siguientes datos analíticos en sangre: Na 142 mEq/L; K 4.5 mEq/L; Cl 115 mEq/L; pH 7.25; CO3H 15 mEq/L; PaCO2 28 mEq/L. El diagnóstico preciso es una deshidratación:

a) Isonatrémica aislada. b) Hipernatrémica aislada. c) Isonatrémica con alcalosis metabólica. d) Isonatrémica con acidosis metabólica.

174: ¿Cuál de los siguientes hechos es infrecuente en la anorexia nerviosa? :

a) Amenorrea. b) Distorsión de la imagen corporal. c) Importante pérdida de peso (>20%). d) Actividad física disminuida.

175: ¿Cuál de las siguientes afirmaciones referentes al reflujo vesico-ureteral grado I en el niño es INCORRECTA? :

a) Se diagnostica por cistografía. b) Se debe tratar con profilaxis antibiótica. c) Tiene tendencia a desaparecer. d) En un niño menor de 1 año debe corregirse quirúrgicamente de forma precoz.

176: En un niño de 3 años con síndrome nefrótico, ¿cuál de las siguientes circunstancias recomienda la práctica de biopsia renal? :

a) Hipoproteinemia marcada. b) Proteinuria selectiva. c) Edemas intensos. d) Hipocomplementemia.

Page 92: Cuestionario de Urología

177: ¿Cuál de los siguientes signos NO es característico del hipotiroidismo congénito? :

a) Llanto ronco. b) Estreñimiento. c) Somnolencia. d) Bajo peso al nacimiento.

178: ¿Cuál de los siguientes hechos es indicación de corticoterapia en la púrpura reumatoide (SchOnlein - Henoch) en el niño? :

a) Melenas. b) Síndrome nefrótico. c) Manifestaciones purpúricas extensas. d) Edemas en extremidades.

179: ¿Cuál es el agente o mecanismo presuntamente implicado en la producción de una enfermedad de Kawasaki? :

a) Estreptococo Beta-hemolítico grupo A. b) Agentes no identificados que actúan como superantígenos. c) Diferentes entero virus. d) Una reacción injerto contra huésped.

180: ¿Cuál de los siguientes medicamentos parece aumentar el riesgo de que un niño con varicela se complique con un síndrome de Reye? :

a) Ibuprofeno. b) Acido acetilsalicílico. c) Paracetamol. d) Ampicilina.

181: ¿Cuál de los siguientes parásitos provoca diarrea más habitualmente? :

a) Giardia lamblia. b) Ascaris lumbricoides. c) Oxiuros. d) Tenia saginata.

182: ¿Cuál de los siguientes hechos NO suele estar presente en la estenosis hipertrófica de píloro del lactante? :

a) Vómitos copiosos. b) Comienzo desde el nacimiento. c) Estreñimiento. d) Pérdida de peso.

183: ¿Cuál de los siguientes hallazgos en heces se encuentra en una malabsorción de lactosa? : Cuerpos reductores (prueba del Clinitest) PH

a) Más de 0.5% <5.5 b) Más de 0.5% >6.0 c) Menos de 0.5% <7.0 d) Cualquier valor >6.0

184: Entre los factores causantes de la aceleración secular del crecimiento que se citan, ¿cuál parece tener mayor influencia? :

a) Mayor secreción actual de GH en la especie humana. b) Cambios en el patrón genético con el tiempo. c) Mejor equilibrio en el aporte proteico-energético. d) Aumento de la actividad deportiva en los últimos 25 años.

185: ¿Cuál de las siguientes afirmaciones, relativa a la prueba de Apgar, es FALSA? :

a) Una frecuencia cardíaca de menos de 100; suma 1 punto. b) La flacidez muscular suma 0 puntos. c) El color completamente rosado, suma 2 puntos. d) Una puntuación superior a 6, significa asfixia.

186: Se ha detectado un caso de meningitis meningocócica en un niño de 5 años que acude a un colegio. Por el momento no se conoce el serogrupo causante de la enfermedad. La actuación correcta en este momento, con los niños y profesores del colegio, es administrar:

a) Quimioprofilaxis a todos. b) Quimioprofilaxis + vacunación a todos. c) Quirnioprofilaxis a los de la misma clase. d) Quimioprofilaxis + vacunación a los de la misma clase.

187: Una mnjer de 20 años, correctamente vacunada contra tétanos y que recibió la última dosis a los 16 años, acude a Urgencias porque se ha caído y se ha clavado un clavo. Muestra una herida de aproximadamente dos centímetros de profundidad, con orificio de entrada pequeño. Después de limpiar la herida y de curarla, ¿qué hay que hacer para prevenir la aparición de tétanos? :

a) Administrar una dosis de vacuna. b) Administrar una dosis de vacuna e inmunoglobulina. c) Iniciar una nueva pauta de vacunación completa (3 dosis). d) No es necesario hacer nada.

188: En un estudio que evaluaba la eficacia de un tratamiento hipolipemiante comparado con placebo para prevenir episodios coronarios, se observó que el grupo tratado presentó una disminución relativa del riesgo del 37% (intervalo confianza 95% = 21% - 50%). ¿Cómo interpreta este resultado? :

Page 93: Cuestionario de Urología

a) No es estadísticamente significativo. b) El número de personas que es necesario tratar para prevenir un episodio es de 37. c) El número mínimo de personas que es necesario tratar para prevenir un episodio es de 21. d) El riesgo de presentar un episodio relativo al del grupo placebo es de 0.63.

189: Si en un ensayo clínico se especifica que el análisis de los datos se ha efectuado siguiendo la estrategia de "intención de tratar" se debe interpretar que:

a) Sólo se han incluido los sujetos que han cumplido con el protocolo. b) Se han excluido a los sujetos perdidos durante el seguimiento. c) Se han excluido a los sujetos que no respondían al tratamiento. d) Se analizan todos los sujetos para mantener la premisa de aleatoriedad.

190: Un varón de 45 años, que fuma 20 cigarrillos/día desde los 14 años, acude a una consulta por una bronquitis crónica reagudizada y se le aconseja dejar de fumar. Para valorar si tiene una dependencia importante de la nicotina, que se beneficiaría de la utilización de sustitutos de la misma, ¿qué se le debe formular? :

a) ¿Cuánto tiempo pasa entre que se levanta y fuma el primer cigarrillo? b) ¿Qué tipo de tabaco fuma: rubio o negro? c) ¿En qué grado quiere dejar de fumar, puntuando del 1 al l0? d) ¿Cuánto piensa que depende de los cigarrillos?

191: Para valorar el impacto de la mortalidad prematura en una comunidad, ¿cuál de los siguientes indicadores utilizaría?:

a) Años potenciales de vida perdidos. b) Indice de mortalidad. estandarizado. c) Razón de mortalidad comparativa. d) Mortalidad por causas.

192: En un ensayo clínico que compara la eficacia de un tratamiento frente a un placebo, se observa que, en el grupo tratado, curan un 15 % de los individuos, mientras que con placebo, sólo un 10%. Con estos datos se puede estimar que el número de personas que es necesario tratar para conseguir una curación es de:

a) 5 b) 10 c) 15 d) 20

193: Si una prueba diagnóstica tiene una sensibilidad del 80% y una especificidad del 90%, la razón de probabilidad positiva es:

a) 2 b) 8 c) 10 d) 20

194: Paciente de 55 años fumador, hipertenso e hipercolesterolémico que consulta por dolor precordial de ciertas características. Por un estudio reciente se estima que la probabilidad de que un individuo con este tipo de dolor tenga un infarto agudo de miocardio es del 50%. Usted aplica una prueba diagnóstica con una sensibilidad y especificadad del 80 %. Si el resultado de la prueba es positivo, ¿cuál sería su valor predictivo positivo? :

a) 80%. b) 50%. c) 40%. d) 20%.

195: Señale cuál de las siguientes afirmaciones es correcta:

a) Para llegar a un diagnóstico de certeza se requiere una prueba de alta sensibilidad aunque tenga una especificidad menor. b) Para descartar, en principio, un diagnóstico debe utilizarse una prueba de alta sensibilidad, aunque no sea muy específica. c) El poder predictivo de una prueba depende de la sensibilidad y especificidad de la misma, y es independiente de la prevalencia de la enfermedad. d) La sensibilidad de una prueba en una población depende sobre todo de la prevalencia de la enfermedad estudiada, más que de las características de la prueba en sí.

196: En un estudio que evalúa una nueva prueba para el diagnóstico de un determinado tipo de cáncer se informa que, en el grupo de pacientes diagnosticados de cáncer, la probabilidad de encontrar un resultado positivo es 8 veces mayor que en los individuos en los que se ha descartado la enfermedad. Este resultado se refiere a la:

a) Sensibilidad. b) Especificidad. c) Cociente de probabilidad positivo. d) Valor predictivo positivo.

197: ¿Cuál de los siguientes criterios NO se aplica para establecer una posible relación causa-efecto entre un potencial factor de riesgo y una enfermedad? :

a) Existencia de una asociación estadísticamente significativa. b) Que la presunta causa preceda al efecto. c) Que el lapso exposición-comienzo de la enfermedad sea compatible con su historia natural. d) Plausibilidad biológica.

198: En un estudio que compara la eficacia de dos fármacos no se observa una diferencia estadísticamente significativa entre ellos. Si en realidad ambos tratamientos fueran diferentes, todos los siguientes factores podrían explicar por qué se ha obtenido un resultado "falso negativo", EXCEPTO uno. Señálelo:

Page 94: Cuestionario de Urología

a) Error tipo alfa. b) Error tipo beta. c) Potencia estadística insuficiente. d) Número de sujetos estudiados insuficiente.

199: En un estudio transversal sobre una muestra de sujetos representativos de una comunidad, se ha observado una prevalencia de hipertensión arterial (HTA) del 20% (intervalo de confianza del 95%: 15% 25%). ¿Cuál de las siguientes afirmaciones es cierta?:

a) Se tiene un 95% de confianza de que entre un 15% y un 25% de los sujetos de la muestra son hipertensos. b) La prevalencia de HTA en la comunidad es del 20%. c) Se tiene un 95% de confianza de que el intervalo 15-25% incluye el verdadero valor de la prevalencia de HT A en dicha comunidad. d) El número de sujetos estudiados es insuficiente para estimar la prevalencia de HTA en dicha comunidad.

200: Para calcular el número mínimo de sujetos que es necesario incluir en un estudio sobre la prevalencia de un problema de salud se han de realizar las siguientes asunciones, EXCEPTO:

a) Proporción de sujetos que se espera encontrar con el problema de salud. b) El error tipo alfa. c) El número de no respuestas esperado. d) El error tipo beta.

201: En un estudio transversal de correlación entre las cifras de colesterol ligado a lipoproteínas de baja densidad y la reducción del diámetro de las arterias coronarias se ha observado una r de 0.34 con p<0.01 . Debe interpretarse que:

a) Existe una fuerte correlación entre ambas variables. b) Existe una correlación débil entre ambas variables. c) La correlación entre ambas variables es inversa. d) No existe correlación entre ambas variables.

202: En una muestra aleatoria extraída de población sana se encuentra que una variable bioquímica tiene como media 90 y desviación típia 10. La afirmación: "aproximadamente el 95 % de los individuos sanos tienen un valor de esa variable comprendido entre 70 y 110" es correcta:

a) Siempre. b) Nunca. c) Sólo si la variable tiene distribución normal. d) Sólo si la variable tiene distribución normal y la muestra es suficientemente grande.

203: Se pretende comparar la tasa de complicaciones de dos preparados distintos de un mismo fármaco. Se observó un 5% de complicaciones con un preparado y un 3% con el otro, siendo esta diferencia significativa (p=0.045). la interpretación correcta de este resultado es:

a) Si ambos preparados tuvieran la misma tasa de complicaciones, la probabilidad de encontrar una diferencia igual o mayor a la observada es 0.045. b) La probabilidad de que ambos preparados tengan la misma tasa de complicaciones es 0.045. c) Los dos preparados tienen distintas tasas de complicaciones. d) Los dos preparados no tienen la misma tasa de complicaciones.

204: En un contraste de hipótesis, la potencia es la probabilidad de:

a) Rechazar la hipótesis nula siendo cierta. b) Rechazar la hipótesis nula siendo falsa. c) Aceptar la hipótesis alternativa. d) Que la hipótesis alternativa sea verdadera.

205: La prevalencia de una enfermedad no transmisible en una población suficientemente extensa es 0.01 . La probabilidad de que elegidos 3 individuos distintos al azar, los 3 estén enfermos es:

a) 0,01 b) 0,03 c) 0,003 d) 0,000001

206: Un estudio informa que la mediana de supervivencia de los pacientes sometidos a cierta intervención quirúrgica es de 5 años. Ello quiere decir que:

a) El valor esperado del tiempo de supervivencia es de 5 años. b) La mitad de los pacientes sobreviven más de 5 años. c) No hay ningún paciente que sobreviva menos de 5 años. d) La mitad de los pacientes sobreviven aproximadamente 5 años.

207: Un ensayo clínico evalúa tres pautas terapéuticas en pacientes hipertensos. Para comparar las cifras de presión arterial sistólica observadas en los tres grupos a los 6 meses de tratamiento, ¿qué prueba estadística le parece más adecuada? :

a) Análisis de la varianza. b) t de Student para datos independientes. c) Coeficiente de correlación d) Chi-cuadrado.

208: Un ensayo clínico evalúa dos pautas terapéuticas en pacientes hipertensos. Para comparar los porcentajes de pacientes controlados observados en cada grupo a los 6 meses de tratamiento, ¿qué prueba estadística le parece más adecuada?:

a) t de Student para datos independientes.

Page 95: Cuestionario de Urología

b) Análisis de la varianza de medidas repetidas. c) Chi-cuadrado. d) t de Student para datos apareados.

209: Señale, de las siguientes proteínas plasmáticas, cuál NO interviene en el transporteconservación del hierro corporal.

a) Haptoglobina. b) Hemopexina. c) Albúmina. d) Transcortina.

210: Señale de cuál de las siguientes situaciones es característica una gasometría arterial con valores de pH 7.34, pCO2 60 mm Hg, HCO3 32 mmoI/L:

a) Acidosis respiratoria crónica. b) Alcalosis metabólica compensada. c) Acidosis respiratoria aguda. d) Acidosis metabólica aguda.

211: Para la determinación del flujo plasmático renal efectivo se utiliza el aclaramiento renal de:

a) Manitol. b) Inulina. c) Pararninohipurato. d) Creatinina

212: En condiciones normales el ingreso de 600 ml de líquido en el estómago provoca un aumento de presión intragástrica de unos 12 cm de H2O. Después de una vagotomía es de esperara que el ingreso del mismo volumen de líquido provoque una de las reacciones siguientes. Señale cuál:

a) Un aumento igual de presión. b) Un aumento menor de presión. c) No aumentará la presión. d) Un aumento mayor de presión.

213: Si se determinan a lo largo del tiempo la glucemia y la insulinemia de un joven normal que ingiere 75 g de glucosa en ayunas, lo que se debe esperar, respecto a los valores previos a la ingesta es:

a) Glucemia e insulinemia prácticamente sin cambios a una hora de la ingesta. b) Glucemia disminuida e insulinemia aumentada a una hora de la ingesta. c) Glucemia aumentada e insulinemia muy disminuida a las dos horas de la ingesta. d) Glucemia e insulinemia elevadas a una hora de la ingesta y prácticamente normales a las dos horas.

214: Señale la respuesta correcta en relación con la gonadotropina coriónica humana durante el embarazo:

a) Aumenta hasta la 10ª semana y luego se mantiene constante hasta el parto. b) Aumenta constantemente durante todo el embarazo. c) Tiene un máximo alrededor de la 10ª semana. d) Tiene su valor llÚnimo al final del embarazo.

215: Un adulto normal tiene un volumen de líquido extracelular de 15 L, volumen de líquido intracelular 25 L, agua corporal total 40 L, Y la osmolalidad de los compartimentos líquidos es de 300 mOsm/Kg H2O. Se inyecta por vía intravenosa 1,5 L de cloruro sódico con 340 mOsm/Kg H2O de osmolalidad. ¿Cuál será, una vez alcanzado el equilibrio osmótico, la osmolalidad del líquido extracelular?:

a) 300 mOsm/Kg H2O. b) 301 mOsm/Kg H2O. c) 340 mOsm/Kg H2O. d) 640 mOsm/Kg H2O.

216: En los alvéolos de una zona normalmente ventilada y no perfundida, la presión parcial de oxígeno será igual a la de:

a) El aire inspirado. b) El aire espirado. c) La sangre venosa. d) La sangre arterial.

217: Una muestra de sangre contiene 15 gr/dl de hemoglobina normal y tiene una presión parcial de oxígeno de 50 mm Hg (muestra A). Otra muestra con 7,5 gr/dl de hemoglobina normal tiene una presión parcial de oxígeno de 100 mm Hg (muestra B). El tipo de hemoglobina de las dos muestras es la misma, están a la misma temperatura y con las mismas condiciones metabólicas. Señale cuál de las afirmaciones siguientes es la correcta, en relación con el contenido total de oxígeno de la sangre en esta situación:

a) Es igual en ambas muestras. b) Es mayor en la muestra B. c) Es mayor en la muestra A. d) Es el doble en la muestra B que en la A.

218: En una persona con un déficit intenso de producción o de liberación de la hormona antidiurética, es razonable esperar que la orina del sujeto no tratado tenga: Osmolalidad/Flujo

a) Alta/Alto b) Normal/Alto c) AltalBajo d) Baja/Alto

Page 96: Cuestionario de Urología

219: Ante una arteritis segmentaria de arterias pequeñas y medianas que cursa con una inflamación aguda y crónica con formación de trombos que, en la fase aguda, presentan microabscesos, debe pensarse en una de las siguientes enfermedades:

a) Vasculitis reumatoide b) Enfermedad de Raynaud. c) Tromboangeitis obliterante. d) Granulomatosis de Wegener.

220: Un paciente de 52 años aqueja dolor torácico, disnea y derrames pleurales recidivantes, presentando una tumoración difusa en la pleura visceral, que histológicamente está constituida por papilas revestidas por células cuboideas dotadas con largos y delgados microvillis sin positividad para el antígeno carcinoembrionario. ¿Cuál de los signientes diagnósticos debe hacerse en primer lugar? :

a) Mesotelioma benigno. b) Metástasis pleural de adenocarcinoma pulmonar. c) Metástasis pleural de un linfoma. d) Mesotelioma maligno.

221: Paciente varón de 45 años que presenta una clínica de malabsorción de años de evolución, asociada a poliartritis, linfadenopatías y trastornos neurológicos mal definidos, con una biopsia intestinal que muestra voluminosos macrófagos en la lámina propia con gránulos PAS positivos y bacilos. ¿Cuál de los siguientes diagnósticos debe realizarse en primer lugar? :

a) Esprúe celíaco. b) Linfoma intestinal. c) Agarnmaglobulinemia. d) Enfermedad de Whipple.

222: ¿Cuál de las siguientes puede ser la causa de un edema no inflamatorio? :

a) Un aumento de la presión hidrostática intravascular. b) Un aumento de la presión coloidosmótica del plasma. c) Una eliminación excesiva de sal yagua por el riñón. d) La disminución de la presión hidrostática intravascular.

223: Señale cuál de los siguientes hechos histológicos NO aparece, generalmente, en las adenomegalías del SIDA:

a) Hiperplasia folicular florida. b) Depleción linfocitaria. c) Respuesta B-monocitoide sinusal. d) Hiperplasia angiofolicular tipo CastIeman.

224: ¿Cuál de los siguientes fármacos, o grupo de los mismo, que a continuación se relacionan, se asocia con la producción de tos persistente?:

a) Calcio-antagonistas. b) Captopril. c) Verapamil. d) Inhibidores selectivos de la recaptación de serotonina.

225: Un varón de 60 años acude a consulta para control evolutivo. Es hipertenso conocido desde hace más de 10 años, y está en tratamiento con 50 mg/d de clortalidona. El paciente se encuentra asintomático. La TA es de 140/80 mmHg y la exploración clínica es normal. Aporta un ECG y un hemograma normales. En la bioquímica de rutina se objetiva una función renal y un ionograma normal y el único dato reseñable es un ácido úrico de 8.5 mg/dL (normal: 3 - 7.5). El paciente no ha presentado episodios de artritis ni nefrolitiasis. ; ¿Cuál sería su recomendación?:

a) Continuar con el mismo tratamiento. b) Suspender la clortalidona. c) Mantener la clortalidona y añadir alopurinol. d) Mantener la clortalidona y añadir alopurinol y colchicina.

226: ¿De cuál de los siguientes fármacos NO hay demostración de que sea útil en el tratamiento del Warto agudo de miocardio? :

a) Acido acetilsalicílico. b) Inhibidores de la glucoproteína Iib/IIIa. c) Inhibidores de la enzima convertidora de la angiotensina. d) Antagonistas del calcio.

227: Ante una mujer joven con hipertensión esencial, que padece frecuentemente migrañas, el fármaco antihipertensivo más indicado sería:

a) Alfabloqueante. b) Diurético. c) Inhibidor de la enzima de conversión. d) Betabloqueante.

228: Ante un paciente hipertenso con insuficiencia cardíaca añadida, además de tratarle con diuréticos, ¿cuál de los siguientes sería el antihipertensivo de elección y más eficiente? :

a) Betabloqueantes. b) Inhibidores de la enzima de conversión. c) Alfabloqueantes. d) Antagonistas del calcio.

229: ¿Cuál de los siguientes fármacos administrado por vía sistémica NO produce toxicidad ocular?:

a) Cloroquina.

Page 97: Cuestionario de Urología

b) Fluconazol. c) Amiodarona. d) Etambutol.

230: ¿Cuál de los siguientes fármacos es más útil en un paciente con edema cerebral? :

a) Acetazolamida. b) Amilorida. c) Acido etacrínico. d) Manitol.

231: ¿Cuál, de las propuestas, es la acción principal del cromoglicato? :

a) Relajación de la musculatura lisa bronquial. b) Estimulación de la secreción de cortisol por las suprarrenales. c) Bloqueo de los canales de calcio en los linfocitos. d) Bloqueo de la liberación de mediadores de las células cebadas.

232: Paciente de 30 años con tratamiento por problemas psiquiátricos. Durante este período relata que aumenta de peso; está siempre sediento y orina con frecuencia. ¿Cuál de los siguientes fármacos hay que suponer es el responsable de estas reacciones adversas? :

a) Clorhidrato de flufenacina. b) Clozapina. c) Clorhidrato de tioridacina. d) Carbonato de litio.

233: La acción de la lidocaína, utilizada como antiarrítmico, consiste en:

a) Reducir la automaticidad anormal. b) Reducir el potencial de reposo. c) Aumentar la duración del potencial de acción. d) Aumentar el intervalo PRo

234: Un paciente de 72 años, diagnosticado de enfermedad de Parkinson, presenta pies enrojecidos, hinchados, sensibles y muy dolorosos. Señale cuál de los siguientes fármacos es capaz de producir esta sintomatología:

a) Levodopa. b) Seligilina. c) Bromocriptina. d) Benzotropina.

235: Un paciente de 63 años, con insuficiencia cardíaca recibe tratamiento de base con digoxina y diuréticos tiacídicos. Señale cuál de las siguientes posibles situaciones puede aumentar el riesgo de la toxicidad de la digoxina:

a) Hipocalcemia. b) Hipopotasemia. c) Hipermagnesemia. d) La administración conjunta de captopril.

236: ¿Por cuál de los siguientes efectos colaterales es potencialmente más peligrosa la administración de antimuscarínicos en el anciano? :

a) Aumento de la tensión intraocular en individuos con glaucoma. b) Bradicardia sinusal. c) Retención urinaria en las mujeres. d) Hipertermia maligna.

237: En el tratamiento de la miastenia grave, ¿cuál de los siguientes fármacos se utiliza para realizar el diagnóstico diferencial entre una crisis miasténica, por terapéutica insuficiente, y una crisis colinérgica ?:

a) Atropina. b) Fisostigmina. c) Ecotiofato. d) Edrofonio.

238: La distribución de los fármacos por el organismo y su llegada y posible fijación a tejidos específicos es:

a) Independiente del flujo sanguíneo en dicho tejido. b) Independiente de la solubilidad del fármaco en el tejido. c) Dependiente del desnivel de concentración de fármaco entre la sangre y el tejido. d) Acelerada para aquellos fármacos que se unen fuertemente a las proteínas plasmáticas.

239: El fármaco pindolol produce incremento de la frecuencia cardíaca por activación de receptores beta-adrenérgicos. Sin embargo, en presencia de estimulantes beta-adrenérgicos eficaces, el pindolol reduce sus efectos sobre la frecuencia cardíaca de forma reversible y dosis dependiente. Por lo tanto, es probable que el pindolol se comporte como:

a) Antagonista irreversible. b) Antagonista fisiológico. c) Antagonista químico. d) Agonista parcial.

240: Las vacunas de polisacárido purificado utilizadas en la inmunización frente a infecciones producidas por algunas bacterias capsuladas, producen:

a) Respuesta de anticuerpo s sin memoria inmunitaria. b) Estimulación de la producción de linfocitos T.

Page 98: Cuestionario de Urología

c) Respuesta de anticuerpo s con memoria inmunitaria. d) Una respuesta de IgA exclusivamente.

241: ¿Cuál de los siguientes mecanismos es el responsable de la resistencia de Streptococcus pneumoniae a penicilina? :

a) Modificación de las proteínas fijadoras de penicilina. b) Producción de beta-lactamasas cromosómicas. c) Producción de beta-lactamasas plasrnídicas. d) Imperrneabilidad de la membrana externa.

242: ¿Cuál de las siguientes afirmaciones, respecto a la faringoamigdalitis por Streptococcus pyogenes es correcta?

a) El tratamiento de elección es la penicilicina. b) El cultivo permite distinguir entre pacientes con infección aguda y portadores. c) Las técnicas para la detección rápida del antígeno de S.pyogenes son muy sensibles. d) No se han descrito resistencias a los nuevos macrólidos.

243: ¿En cuál de los siguientes cuadros clínicos NO hay evidencia de asociación con las infecciones producidas pro Yersinia enterocolítica?:

a) Enterolitis aguda. b) Adenitis mesentérica e ileítis terminal. c) Poliartritis reactiva. d) Síndrome urémico-hemolítico.

244: Un paciente de 65 años, con claudicación intermitente consulta porque, en una revisión casual, se le han objetivado cifras tensionales elevadas. En sus antecedentes destaca una diabetes mellitus tipo II . Su TA es de 180/105 mmHg en el brazo derecho y 150/90 mmHg en el izquierdo. Se palpan mal los pulsos pedios y el resto de la exploración es anodina. El hemograma es normal, y en la bioquímica sérica destaca: Glu basal 188 mg/dL, urea 68 mg/dL y creatinina 1.5 mg/dL. El ECG y la RX de tórax son normales. Se le indicó inicialmente tratamiento con 10 mg. de enalapril/d . Una semana después, el paciente presenta una TA de 120/70 mmHg y la analítica muestra una creatinina de 2.8 mg/dL. La causa más probable de esta situación es:

a) Una disminución excesiva de la TA. b) Una nefropatía intersticial medicamentosa. c) El origen renovascular de la hipertensión. d) La evolución rápida de una nefropatía diabética condicionada por el tratamiento hipotensor.

245: La actitud más correcta en el caso plantado en la anterior, debe ser:

a) Retirar el enalapril, indicar esteroides y controlar la evolución de la función renal. b) Sustituir el enalapril por otro hipotensor de distinto mecanismo de acción. c) Iniciar tratamiento dialítico. d) Sustituir el enalapril por ramipril

246: Mujer de 32 años con antecedentes de fiebre reumática y disnea de esfuerzo desde hace 6 años, actualmente en fibrilación auricular con disnea de pequeños a moderados esfuerzos y ocasional ortopnea de 2 almohadas. Recibe tratamiento con digoxina y acenocumarol. En el estudio ecocardiográfico se objetiva estenosis mitral aislada con área valvular de 0.9 cm2 y valvas flexibles, fusionadas, sin calcio y sin presencia de trombos en las aurículas. Presión sistólica de arteria pulmonar 55 mmHg. ¿Qué actitud, de las propuestas, es más conveniente? :

a) Añadir diuréticos y valorar la evolución de la paciente. b) Practicar comisurotomía mitral abierta. c) Realizar cateterismo para valorar las lesiones valvulares y la anatomía coronaria. d) Realizar valvuloplastia mitral percutánea.

247: Paciente de 60 años con antecedentes, desde hace tres, de dolor en las plantas de los pies, junto con sensación de acorchamiento en ambas piernas. Un año antes de consultar presentó un brote purpúrico con petequias en ambos miembros inferiores. Unos meses antes notó que arrastraba los pies y tendencia a tropezar. El estudio neurofisiológico mostró una polineuropatía desmielinizante. ¿Cuál de las siguientes es la causa más probable del cuadro? :

a) Amilodosis. b) Paraproteinemia. c) Diabetes. d) Polineuropatía desmielinizante crónica.

248: Respecto a la ginecomastia, señale la afirmación FALSA, entre las siguientes:

a) En la secundaria a tumores testiculares, el mecanismo patogénico es un aumento de la secreción de testosterona. b) La digoxina puede provocarla. c) La fisiológica puberal suele involucionar espontáneamente. d) La fisiológica del recién nacido es secundaria al aumento transitorio de los estrógenos matemos o placentarios.

249: En un paciente que presenta una tumoración de 3 x 3 cm en región parotídea con ulceración en la piel y parálisis facial homolateral, ¿cuál de los siguientes diagnósticos se debe considerar en primer lugar? :

a) Carcinoma adenoide quístico. b) Carcinoma epidermoide. c) Tumor de Warthin. d) Adenoma pleomorfo.

250: ¿Cuál es el primer signo de desarrollo puberal en las niñas? :

a) Aparición del vello pubiano. b) Aparición del vello axilar. c) Aparición de acné facial. d) Desarrollo mamario.

Page 99: Cuestionario de Urología

251: ¿Cuál de las siguientes afirmaciones sobre el consentimiento informado en un ensayo clínico aleatorio es FALSA? :

a) Los pacientes que no lo otorgan no deben participar en el ensayo. b) Debe solicitarse después de que los pacientes han sido asignados aleatoriamente. c) Debe solicitarse.a todos los participantes, independientemente del grupo al que puedan ser asignados. d) Los pacientes que no lo otorgan limitan la generalización de los resultados.

252: El ensayo clínico aleatorio es el estudio que proporciona una mejor evidencia causa-efecto porque existe:

a) Asignación aleatoria de los individuos a los diferentes grupos de tratamiento. b) Un tratamiento alternativo. c) Posibilidad de realizar un análisis estadístico multivariante. d) Una muestra suficientemente amplia.

253: De las siguientes afirmaciones referentes a la hemocromatosis idiopática, señale cuál es INCORRECTA:

a) Es una enfermedad hereditaria, cuyo locus está en el cromosoma 6. b) Es una causa bien definida de artropatía. c) La resonancia magnética hepática es una técnica útil para su diagnóstico. d) Una de las pautas de tratamiento es la deferroxamina por vía oral, para quelar el Fe e impedir su absorción.

254: Señale cuál de las manifestaciones siguientes NO se relaciona con la positividad del HbsAg:

a) Urticaria. b) Panarteritis nodosa c) Crioglobulinemia d) Cirrosis biliar primaria

255: A una paciente de 42 años, que refería tener pirosis desde hacía cuatro, se le realizó una endoscopia digestiva alta, en la que no se observó ninguna lesión esofágica, gástrica, ni duodenal. ¿Cuál de las siguientes afirmaciones, respecto a este cuadro, es FALSA?:

a) Hasta en el 40% de los pacientes que refieren pirosis no hay signos de esofagitis. b) Hay una mala correlación entre la intensidad de los síntomas y la magnitud de las lesiones endoscópicas. c) Si existen dudas del origen de los síntomas, 10 mejor es realizar una manometría esofágica. d) La pHmetría esofágica de 24 horas podría ser útil para establecer la relación temporal entre el reflujo y los síntomas

256: Acude a consulta un paciente de 72 años por presentar disfagia para sólidos y líquidos, desde hace 4 meses. Además, aqueja adelgazamiento de 8 Kg. En el estudio radiológico con contraste se observa un esófago ligeramente dilatado que se estrecha en su parte distal. ¿Cuál de los siguientes debe ser el próximo paso?:

a) Iniciar el tratamiento con un inhibidor de la bomba de protones para mejorar la esofagitis. b) Realizar una manometría esofágica para confirmar la existencia de una acalasia. c) Comenzar un tratamiento con un fármaco procinético para mejorar la motilidad del esófago. d) Indicar la realización de una endoscopia digestiva para descartar la existencia de patología orgánica del esófago.

257: Una paciente de 36 años acude a consulta porque, desde hace 5, padece episodios recurrentes de dolor en la parte inferior del abdomen junto con cambios en el hábito deposicional: períodos de estreñimiento y períodos de diarrea. Los resultados de los análisis de sangre y de la colonoscol1ia son normales. ¿Qué actitud de las siguientes tomaría?:

a) Investigar Helicobactar pylori para descartar enfermedad ulcerosa. b) Solicitar una colangiografía endoscópica para descartar coledocolitiasis. c) No realizar más exploraciones y tranquilizar a la paciente ya que posiblemente padece un síndrome del intestino irritable. d) Solicitar una TAC para descartar cáncer de páncreas.

258: Señale cuál de las siguientes afirmaciones es correcta respecto a la administración de antiinflamatorios no esteroideos (AINEs):

a) Su administración junto con corticoides no aumenta el riesgo de lesiones gastroduodenales. b) La edad no influye en el riesgo de desarrollar lesiones digestivas. c) Es preferible la administración rectal, ya que su efecto lesivo es fundamentálmente local. d) Hasta en el 15-30% de los pacientes que los toman de forma crónica se demuestran úlceras gástricas y/o duodenales en la endoscopia digestiva alta.

259: Una paciente de 58 años acude a la consulta por diarrea de 3 meses de evolución, con dolores cólicos abdominales, síndrome anémico y edemas en miembros inferiores. Fue diagnosticada de enfermedad celíaca hace 15 años mediante biopsia intestinal, realizando dieta sin gluten durante 3 meses. ¿Cuál de las siguientes afirmaciones es correcta?:

a) La paciente posiblemente no padecía una enfermedad celíaca. b) La nueva instauración de una dieta sin gluten mejorará el cuadro clínico. c) La determinación en suero de anticuerpo s antigliadina nos ayudará a conocer si la enfermedad está en actividad. d) Es necesario descartar la presencia de un linfoma intestinal.

260: Una paciente de 66 años padece dolores abdominales recurrentes desde hace 8. Desde hace 2 días se queja de intenso dolor en fosa ilíaca derecha que se alivia parcialmente con la deposición. No tiene fiebre, astenia, anorexia ni refiere adelgazamiento. Los análisis de sangre son normales. En el enema opaco se observa la existencia de múltiples divertículos. ¿Cuál de las siguientes afirmaciones es verdadera?:

a) Posiblemente la paciente tiene una diverticulitis. b) Es muy probable que el tratamiento termine siendo la resección de la zona colónica con divertículos. c) Lo más adecuado será la instauración de un tratamiento antibiótico. d) En la mayor parte de los casos los divertículos de colon son asintomáticos.

261: La cirrosis biliar primaria se asocia a:

a) Anticuerpos antimitocondriales en alrededor del 50% de los casos.

Page 100: Cuestionario de Urología

b) Títulos elevados de anticuerpos antinucleares en más del 90% de los casos. c) Síndrome de CREST (calcinosis, Raynaud, esclerosis cutánea y telangiectasias). d) Anticuerpos tipo IgG contra la proteína Ed)

262: El diagnóstico de hemocromatosis hereditaria puede realizarse con seguridad mediante:

a) Determinación de niveles de ferritina séricos. b) El hallazgo de cirrosis hepática, diabetes e hiperpigmentación cutánea. c) Tipificación HLA. d) El hallazgo de sustitución homocigota Cys282Tyr en el gen HFE.

263: Los enfermos de Crohn que han sufrido una amputación de 50 cm de íleon están abocados a padecer:

a) Síndrome de sobrecrecimiento bacteriano. b) Síndrome de Dumping. c) Anemia mega1ob1ástica. d) Diarrea de tipo osmótico.

264: Un enfermo de 45 años sufre una elevación de los niveles sanguíneos de fosfatasa alcalina hasta 3 veces la cifra normal. ¿Qué prueba diagnóstica le recomendaría a continuación para aclarar el origen de su alteración enzimática?:

a) Ecografía hepatobiliar. b) Radiografía de cráneo. c) Co1angiografía endoscópica retrógrada. d) Determinación de y glutami1 transpeptidasa.

265: En relación con la infección por el virus de la hepatitis C, ¿cuál de las siguientes afirmaciones es FALSA?:

a) La determinación de los anticuerpos anti- VHC en donantes ha eliminado casi por completo el riesgo de adquirir una hepatitis postransfusional por VHC. b) Entre el 60-70% de los adictos a drogas por vía parenteral son anti- VHC positivo. c) Para el diagnóstico de hepatitis C en sujetos inmunodeprimidos se requiere la determinación del RNA- VHC. d) El 70% de las hepatitis agudas C evolucionan espontáneamente a la curación.

266: ¿Cuál de los siguientes hechos reduce la presión del esÍInter esofágico inferior?:

a) Dieta grasa. b) Administración de acetilco1ina. c) Dieta proteica. d) Existencia de reflujo duodeno-gástrico.

267: El trasplante hepático está CONTRAINDICADO en pacientes con:

a) Hepatocarcinoma de 3 cm de diámetro. b) Edad igual o superior a 65 años. c) Tuberculosis pulmonar en tratamiento desde hace 3 semanas. d) Trombosis de la vena porta.

268: Se opera a un paciente de 65 años con diverticulitis cólica perforada, practicándose una hemicolectomía izquierda más esplenectomía por desgarro capsular. Al 6° día del p ostoperatorio presenta fiebre mantenida de 38°C, li gero enrojecimiento facial y oliguria a pesar de sueroterapia correcta. El día anterior tuvo tres deposiciones escasas y diarreicas, con abdomen ligeramente distendido, no doloroso, TA 150/85,. Pulso: 851pm. ¿En cuál de los siguientes procesos hay que pensar en primer lugar?:

a) Peritonitis aguda por contaminación operatoria. b) Endocarditis infecciosa secundaria a diverticulitis. c) Sepsis postesp1enectomía. d) Deshiscencia anastomótica e iniciación de sepsis.

269: Ante la palpación de una vesícula distendida e indolora en un paciente ictérico y con síndrome constitucional asociado, ¿qué diagnóstico, de los sigu,ientes, hay que sospechar en primer "lugar?:

a) Carcinoma vesicular. b) Coledocolitiasis. c) Cólico hepático simple. d) Carcinoma de cabeza de páncreas.

270: Señale la afirmación FALSA acerca del síndrome de Mallory-Weiss:

a) El diagnóstico se hace mediante endoscopia. b) El estudio radio1ógico ayuda poco al diagnóstico, en general. c) Los lavados con suero salino helado pueden ser útiles. d) La cirugía será necesaria en un 25% de los casos.

271: En un paciente de 50 años se encuentra un nicho uIceroso en curvatura menor gástrica, con aspecto radiológico benigno, al practicar un estudio baritado para esclarecer un cuadro de dispepsia inespecífica. Entre las siguientes opciones, ¿qué es obligado hacer?:

a) Endoscopia para ver el aspecto de la lesión, sin biopsia. b) Endoscopia con toma de biopsias y su estudio histo1ógico. c) Estudio radiográfico baritado con doble contraste. d) Estudio de la secreción gástrica con pentagastrina.

272: ¿Cuál de las siguientes pruebas es más adecuada para diagnosticar la causa de un cuadro de melenas de repetición sin otra sintomatología?:

a) Estudio radiológico con doble contraste de esófago, estómago y duodeno. b) Enema opaco.

Page 101: Cuestionario de Urología

c) Arteriografía mesentérica. d) Panendoscopia oral.

273: Un hombre de 43 años con colitis uIcerosa de 10 años de evolución, ingresa en el hospital por deposiciones muy sanguinolentas y frecuentes, distensión abdominal, vientre doloroso, náuseas y fiebre de 39°C. A la exp loración física se objetiva mal estado general, timpanismo y deshidratación. En la analítica de urgencia existe anemia y leucocitosis con desviación izquier-da. ¿Qué exploración diagnóstica, entre las siguientes, debe realizar en primer lugar?:

a) Enema opaco. b) Radiografía simple de abdomen. c) EcografÍa abdominal. d) Rectosigmoidoscopia.

274: Un hombre de 50 años con pancreatitis alcohólica es intervenido quirúrgicamente para tratamiento de pseudoquiste a nivel de cola de páncreas. Al explorar la cavidad quÍstica aparece una lesión sospechosa que se biopsia, revelando cistoadenocarcinoma. No se observaron ganglios linfáticos. ¿Cuál, de los ennmerados, es el tratamiento más apropiado?:

a) Drenaje externo. b) Quisto-yeyunostomía en Y de Roux. c) Excisión radical. d) Biopsia solamente.

275: Señale cuál de las siguientes afirmaciones respecto al síndrome de apneas-hipopneas obstructivas del sueño es FALSA:

a) Afecta entre ell y e15% de la población general adulta. b) El tratamiento quirúrgico (uvulopalatofaringoplastia), es una opción terapéutica eficaz en la mayoría de los pacientes. c) El diagnóstico de certeza se establece mediante la realización de un estudio polisornnográfico completo. d) En la mayoría de los pacientes, los únicos hallazgos en la exploración clínica son la obesidad y la hipertensión arterial sistémica.

276: Con relación a la insuficiencia respiratoria aguda, sólo uno de los siguientes enunciados es verdadero. Señálelo:

a) El diagnóstico se establece fundamentalmente mediante la exploración clínica. b) La cianosis es una manifestación clínica específica. c) El tratamiento consiste fundamentalmente en la corrección de la hipoxemia y de la causa desencadenante. d) La diferencia alveolo-arterial de oxígeno es normal o poco elevada.

277: ¿Cuál de las siguientes afirmaciones, respecto a la determinación de D-dímero para el diagnóstico del tromboembolismo pulmonar, es correcta?:

a) Tiene un alto valor predictivo negativo. b) No tiene valor como prueba de despistaje inicial. c) Tiene una alta especificidad. d) Es menos sensible,que la gasometría arterial basal.

278: A una mujer de 64 años, sin síntomas re_iratorios ni generales, no fumadora ni bebedora, se le pniCtica una radiografía de tórax, como estudio previo a una intervención por hallux valgus. Se encuentran adenopatÍas hiliares bilaterales, con parénquimas pulmonares normales y sin otros hallazgos, lo que se confirma mediante una TAC de gran resolución. Los resultados del estudio del frotis sanguíneo y de la bioquÍmica rutinaria son normales. El enzima de conversión de la angiotensina en suero está dos veces por encima del límite superior de la normalidad. La prueba cutánea con tuberculina con concentración alta es negativa. La espirometrÍa es normal. ¿Qué afirmación de las siguientes es correcta? :

a) Podemos afirmar que la enferma nunca tuvo infección tuberculosa. b) Es imprescindible hacer una biopsia transbronquial de entrada. c) Debemos empezar tratamiento con corticoides desde este momento. d) Es bastante probable que el cuadro remita espontáneamente.

279: Entre las siguientes afirmaciones relativas al adenoma bronquial tipo carcinoide, señale la que NO es correcta:

a) Se origina en las células del sistema APUD. b) La resección quirúrgica es el tratamiento de elección. c) Se asocia a síndrome carcinoide en raras ocasiones y especialmente si hay metástasis hepáticas o diseminadas. d) Es una lesión poco vascularizada.

280: Ante un joven de 16 años que ha estado conviviendo con un enfermo con tuberculosis pulmonar activa y tiene una prueba de la tuberculina negativa, la actitud más correcta, de las siguientes, es realizar:

a) Quimioprofilaxis durmite 6 meses. b) Vigilancia clínica y prueba de tuberculina a los 3 meses. c) Quimioprofilaxis durante 2-3 meses y luego repetir la prueba de la tuberculina. d) Tratamiento con rifampicina, isoniacida y piracinamida durante 6 meses.

281: Entre las siguientes afirmaciones relativas a la hipoxia, señale la que NO es correcta:

a) En la anémica, hay disminución de la saturación de la hemoglobina. b) En la hipoxémica, hay disminución de presión parcial de oxígeno en sangre arterial. c) En la circulatoria, hay aumento de la diferencia arteriovenosa del oxígeno. d) En la cito tóxica, el contenido de oxígeno en sangre arterial es normal.

282: Señale, entre las siguientes afirmaciones relativas al enfisema pulmonar, la que NO es correcta:

a) En el centroacinar es típica la existencia de infiltrado inflamatorio y cierto grado de fibrosis en los bronquiolos respiratorios. b) El panacinar es más frecuente y grave en los lóbulos superiores. c) El centroacinar y el panacinar suelen coexistir en el mismo pulmón. d) El panacinar es típico de los pacientes con déficit de al antitripsina.

Page 102: Cuestionario de Urología

283: Si una enferma con lupus eritematoso diseminado presenta un infiltrado pulmonar, lo más probable es que se trate de:

a) N eumonitis intersticial con fibrosis. b) Neumonitis aguda lúpica. c) Edema pulmonar. d) Infección pulmonar.

284: Un enfermo con neumonía, perteneciente a un brote epidémico de varias personas que ocasionalmente conviven en un edificio, presenta un cuadro confusional desproporcionado a la fiebre, diarrea, hiponatremia y ligero ascenso de las enzimas hepáticas. Entre las siguientes opciones, ¿cuál se debe incluir en el tratamiento?:

a) Cefalosporinas y aminoglucósidos a las dosis adecuadas. b) Vancomicina a las dosis adecuadas. c) Doxicic1ina 1O0mg/12 h. durante 14 días. d) Eritromicina 2-4 g/día durante 14 días.

285: Respecto a la capacidad de difusión pulmonar medida con el test de transferencia alveolocapilar de monóxido de carbono (DLCO), señale qué afirmación, de las siguientes, NO es correcta:

a) Aumenta en el enfisema. b) Disminuye en el embolismo recurrente. c) Disminuye en la fibrosis pulmonar idiopática. d) Aumenta en el síndrome de Good-Pasture.

286: La etiología más frecuente de la mediastinitis aguda es:

a) Cáncer de pulmón. b) Neumonía necrotizante. c) Perforación esofágica. d) Traumatismo torácico abierto.

287: Señale qué afirmación, de las siguientes, relativas a la capacidad pulmonar total es correcta:

a) Es el volumen de aire que permanece atrapado en los pulmones al final de una espiración normal. b) Se incrementa característicamente en las neumopatías intersticiales difusas. c) En personas sanas oscila entre 20 y 30 Vmin. d) Es la suma de la capacidad vital y el volumen residual.

288: Una de las siguientes aseveraciones NO es correcta en la tuberculosis miliar. Señálela:

a) Se debe a la diseminación hematógena del bacilo. b) La tinción de Ziehl del esputo suele ser negativa. c) La prueba de la tuberculina es positiva en el 80% de los casos. d) La biopsia de médula ósea puede proporcionar el diagnóstico.

289: La distensibilidad, adaptabilidad o "compliance" pulmonar estudia las propiedades elásticas del pulmón, relacionando:

a) El volumen pulmonar con el flujo espirado a cada nivel volumétrico. b) El volumen pulmonar con la presión transpulmonar en condiciones estáticas. c) El flujo espirado con el gradiente de presión alveolo boca. d) El volumen residual con la capacidad total.

290: Una mujer de 32 años presenta una ascitis moderada y un derrame pleural derecho. No tiene fiebre y su estado general está conservado. Tanto ellíquido pleural como el peritoneal tienen características bioquímicas de trasudado. La prueba de tuberculina es negativa y, tanto la citología pleural como la peritoneal, son negativas en una primera determinación. ¿ Cuál de los siguientes procedimientos diagnósticos sería el más adecuado? :

a) Biopsia hepática con aguja. b) Biopsia pleural con aguja. c) Repetir citologías pleural y peritoneal. d) Hacer ecografía abdominal.

291: Respecto a la insuficiencia cardíaca con ventrículo izquierdo dilatado y fracción de eyección extremadamente reducida, señale la afirmación correcta:

a) Es muy rara en pacientes con cardiopatía isquémica. b) Nunca es secundaria a lesiones valvulares. c) Aparece con frecuencia en la pericarditis constrictiva. d) Puede aparecer como efecto secundario de la administración de adriarnicina.

292: En un paciente con miocardiopatía hipertrófica obstructiva, la auscultación de un soplo sistólico eyectivo en borde esternal izquierdo, suele reflejar la existencia de obstrucción dinámica en el tracto de salida del ventrículo izquierdo. De las siguientes maniobras sólo una disminuye la intensidad del soplo. Señálela:

a) Maniobra de Valsalva. b) Inhalación de nitrito de amilo. c) Infusión de isoproterenol. d) Decúbito supino con piernas elevadas.

293: Un enfermo de 43 años, con típica angina de esfuerzo, tiene una prueba de esfuerzo normal en cinta rodante. Por este motivo, se repite la prueba con la inyección de un isótopo de talio (TI-20l) encontrándose un área de actividad reducida en la cara anterior del ventriculo izquierdo. La exploración, repetida 4 horas más tarde en reposo, muestra una actividad homogénea en toda la cara anterior. Este hallazgo es sugerente de:

a) Un infarto inferior antiguo. b) Un infarto reciente que compromete la cara anterolateral.

Page 103: Cuestionario de Urología

c) Patología de la arteria coronaria descendente anterior. d) Un infarto anterior antiguo con isquemia residual.

294: ¿Qué diagnóstico, entre los siguientes, establecería ante un ECG con QRS de anchura superior a 0,12 seg., con morfología "rSR'" con "R'" ancha en V1 y "qRS" con "S" ancha en V6?:

a) Bloqueo completo de rama izquierda. b) Bloqueo completo de rama derecha. c) Síndrome de preexcitación tipo W.P.W. d) Hemibloqueo anterior izquierdo.

295: Ante un paciente con gran sospecha clínica de tromboembolismo pulmonar, ¿cuál de las siguientes pruebas tiene un mayor valor predictivo negativo?:

a) Una arteriografía pulmonar por inyección directa normal. b) Una garnmagrafía de ventilación y otra de perfusión de baja probabilidad. c) Una garnmagrafía de perfusión de baja probabilidad y Rx de tórax normal. d) Una flebografía de ambas piernas negativa con Rx de tórax normal.

296: ¿Cuál de los siguientes mecanismos NO contribuye a la aparición de anoxia en el embolismo pulmonar?:

a) Redistribución del flujo sanguíneo pulmonar. b) Cortocircuitos arteriovenosos intrapulmonares. c) Cortocircuitos derecha-izquierda intracardíacos. d) Bloqueo alveolo-capilar en el área afecta.

297: ¿Cuál de los siguientes razonamientos clínicos NO es correcto ante un paciente de 30 años que acude a Urgencias con dolor precordial intenso de tres horas de duración y en cuyo ECG se observa elevación del ST en V1, V2 y V3?:

a) Un diagnóstico probable es pericarditis aguda viral. Buscaremos roce auscultatorio e indagaremos sobre enfermedad catarral o viral previa. b) Sin duda, se trata de un infarto agudo de miocardio anterior y debemos instaurar fibrinolisis. c) Es posible que se trate de una angina de Prinzmetal y debemos observar los cambios del ECG al ceder el dolor. d) Puede tratarse de un infarto agudo anterior y debemos solicitar CPK y CPK-MB.

298: ¿Cuál es, entre las siguientes, la explicación más probable para el cuadro de un paciente de 75 años, hipertenso, con disnea desde hace cuatro meses, sin soplos, con hipertrofia ventricular izquierda en el ECG y silueta cardíaca normal en la radiografia?:

a) Disfunción diastólica crónica del V.I. por hipertensión. b) Disfunción sistólica crónica del V.I. por hipertensión. c) Insuficiencia mitral por dilatación del anillo valvular. d) Infarto de miocardio antiguo, sin onda "q".

299: La mejoría de la capacidad funcional que sigue al entrenamiento fisico, en un individuo de edad media, se debe a:

a) Mejoría de la fracción de eyección. b) Estímulo a la circulación colateral. c) Hipertrofia ventricular izquierda fisiológica d) Disminución de la frecuencia cardíaca y de la tensión arterial como respuesta al ejercicio.

300: Todos los siguientes cuadros pueden ser causa de hipertensión pulmonar precapilar aislada, EXCEPTO uno. Señálelo:

a) Arteriopatía pulmonar plexogénica. b) Arteriopatía pulmonar trombótica. c) Enfermedad pulmonar veno-oclusiva. d) Insuficiencia ventricular izquierda.

301: ¿Cuál de las siguientes situaciones fisiopatológicas es condición "sine qua non" para que exista clínicamente cianosis central?:

a) Insuficiencia cardíaca derecha de cualquier grado de severidad. b) Insuficiencia cardíaca izquierda de cualquier grado de severidad. c) Cantidad de hemoglobina reducida por encima de 5 g/dL. d) Insuficiencia respiratoria de cualquier grado de severidad.

302: Un paciente de 55 años, fumador importante, acude a Urgencias porque lleva dos horas con dolor intenso retrosternal, que comenzó en reposo, acompañado de cortejo vegetativo. A la auscultación cardíaca hay taquicardia y galope y, a la pulmonar, crepitantes en bases. El ECG muestra "Q" de nueva aparición y elevación de S-T en DII, DIII y a VF. La CPK está tres veces por encima de lo normal. ¿Cuál sería su diagnóstico?:

a) Infarto agudo anterior poco extenso. b) Infarto agudo anterior muy extenso. c) Pericarditis aguda posterior. d) Infarto inferior transmural.

303: Se diagnostica HTA moderada a una mujer de 49 años, mtmopáusica desde hace c) Tiene anteI cedentes de migraña desde los 20 años y asma intrínseco desde los 41. ¿Cuál de los siguientes fármacos NO estaría indicado en el tratamiento de su HTA?:

a) Betabloqueantes. b) Diuréticos. c) Antagonistas del calcio. d) IECA.

304: ¿Cuál de los siguientes agentes se ha demostrado capaz de reducir la mortalidad y la aparición de un nuevo infarto, cuando se administra a pacientes que han sufrido un infarto de miocardio?:

a) Nifedipino.

Page 104: Cuestionario de Urología

b) Veraparnilo. c) Nitroglicerina d) Betabloqueantes

305: ¿Qué respuesta es correcta en relación con el taponamiento cardíaco?:

a) Habitualmente se palpa el latido del ápex. b) La presión venosa yugular está elevada. c) El retorno venoso al corazón derecho disminuye en inspiración. d) La frecuencia cardíaca habitualmente es normal.

306: ¿Cuál de las siguientes complicaciones NO es probable que aparezca en el curso de la disección aórtica aguda?:

a) Accidente cerebro-vascular. b) Tromboembolismo pulmonar agudo. c) Hemotórax d) Insuficiencia aórtica aguda.

307: ¿Cuál de las siguientes aseveraciones es INCORRECTA en el contexto del tratamiento de diversos grupos de enfermos con disfunción cardíaca diastólica?:

a) Los diuréticos alivian la congestión pulmonar en los pacientes con insuficiencia cardíaca porque reducen la precarga. b) Los β-bloqueantes aislados pueden ser útiles en casos de HTA esencial porque reducen la postcarga. c) Los digitálicos son útiles por su efecto inotrópico positivo y porque disminuyen el riesgo de arritmias ventriculares. d) El verapamil y el diltiazén mejoran la relajación del ventriculo izquierdo en los pacientes de miocardiopatía hipertrófica.

308: Señale cuál de las siguientes complicaciones se ha demostrado que es significativamente más frecuente, antes del alta del paciente, cuando se realiza cirugía de puenteo coronario (CPC) que cuando se realiza angioplastia coronaria transluminal percutánea (ACTP) para el tratamiento de la cardiopatía coronaria:

a) La muerte del enfermo. b) La aparición de un ataque isquémico cerebral. c) La aparición de tromboembolismo pulmonar. d) La aparición de un infarto agudo con "Q".

309: ¿Cuál es la complicación más frecuente en los aneurismas arteriosclerosos de aorta abdominal mayores de 6 cm de diámetro?:

a) La oclusión de arterias mesentéricas. b) La embolización distal. c) La comprensión ureteral con hidronefrosis secundaria. d) La ruptura.

310: Respecto a la cirugía de la estenosis mitral, es FALSO que:

a) Puede reproducirse la lesión tras la valvulotornía por procesos independientes de la cicatrización. b) La incidencia de embolia sistémica se reduce con la valvulotornía. c) Si aparece insuficiencia mitral severa postvalvulotornía se precisará un recambio valvular. d) La fibrilación auricular es más frecuentemente reversible si la aurícula izquierda no está muy dilatada.

311: Un hombre de 23 años consulta por disminución de la visión en el ojo derecho, instaurada en 24 horas, y dolor en ese ojo que empeoró con los movimientos oculares. En la exploración, se constata la pérdida de agudeza visual con normalidad del fondo de ojo. El diagnóstico más probable entre los siguientes es:

a) Esclerosis múltiple. b) Neuritis óptica retrobulbar. c) Desprendimiento de retina. d) Enfermedad de Devic.

312: El tratamiento de elección para un paciente de 75 años, hipertenso, bien controlado con 25 mg de captopril y con fibrilación auricular crónica no valvular, que ha sufrido un accidente isquémico transitorio con disfasia y hemiparesia, es:

a) Aspirina, 300 mg/día. b) Clopidogrel o ticlopidina. c) Acenocumarol ajustando el I.N.R. entre 2 y 3 d) Aspirina entre 1.000 y 1.500 mg/día.

313: Señale, entre las siguientes, la afirmación INCORRECTA respecto a la neuralgia idiopática del trigémino:

a) Hay episodios repetidos de dolor de breve duración. b) Hay anestesia de una o varias ramas del trigémino del lado afecto. c) Hay respuesta terapéutica a carbamacepina. d) Es más prevalente en mayores de 50 años.

314: Un paciente diabético de 69 años consulta por aparición brusca de dolor ocular derecho y visión doble. En la exploración hay ptosis derecha y parálisis de todos los movimientos de ese ojo, excepto la abducción. Las pupilas son normales así como la agudeza visual. El diagnóstico más probable es:

a) Aneurisma de arteria comunicante posterior. b) Oftalmitis fúngica diabética. c) Mononeuropatía diabética del ID par. d) Proceso expansivo del seno cavernoso.

315: Señale la afirmación correcta, entre las siguientes, acerca de la amaurosis fugaz:

a) Consiste en breves episodios de ceguera bilateral. b) Se asocia a dolor ocular.

Page 105: Cuestionario de Urología

c) Su causa es isquemia transitoria en el lóbulo occipital. d) Puede deberse a estenosis de carótida.

316: Hombre de 59 años, ex bebedor y gastrectomizado por hemorragia digestiva hace quince años. No sigue tratamiento alguno. Consulta por un cuadro insidioso de dificultad para caminar que empeora en la oscuridad. En la exploración se objetiva una ataxia de la marcha cayendo al suelo en la prueba de Romberg, respuestas plantares extensoras y conservación de la sensibilidad algésica estando abolidas la vibratoria y posicional. Señale, entre las siguientes, la afirmación correcta:

a) El diagnóstico es degeneración cerebelosa alcohólica. b) Se trata de una degeneración combinada sub aguda de la médula espina!. c) Hay que descartar una lesión centromedular. d) El diagnóstico es esclerosis múltiple forma primaria progresiva.

317: Una mujer de 25 años, indigente y con un embarazo no controlado de cuatro meses, es atendida en Urgencia por hiperemesis gravídica. Mientras recibe rehidratación con solución glucosada intravenosa, empieza a deteriorarse su nivel de conciencia, se queja de molestias visuales y es incapaz de mantenerse de pie. En la exploración destaca somnolencia, inatención, nistagmo de la mirada lateral y severa ataxia axial. Con mayor probabilidad, se trata de nna:

a) Preeclampsia, por lo que se debe detener .la sueroterapia para evitar subidas de tensi6n arteria1. b) Hipopotasemia por vómitos, debiéndose añadir cloruro potásico al suero glucosado. c) Encefalopatía de Wernicke, precisando detener la sueroterapia y administrar tiamina. d) Alcalosis metabólica por vómitos de repetición.

318: De los siguientes hallazgos de la esclerosis múltiple, señale el MENOS frecuente:

a) Atrofia óptica. b) Trastornos del esfínter vesical. c) Atrofias musculares. d) Síndrome vestibular.

319: Los familiares y educadores de un paciente de 10 años, sin antecedentes patológicos destacables, han observado que tiene momentos en que interrumpe sus actividades, no contesta si se le llama y realiza alguna sacudida muscular con los brazos. Presenta estos episodios varias veces al día. ¿Qué trazado, de los siguientes, espera encontrar en el EEG?:

a) Un foco de puntas en la región rolándica derecha. b) Un foco de ondas lentas temporal izquierdo. c) Descargas bilaterales de ondas lentas frontales. d) Descargas bilaterales de complejos punta-onda lenta.

320: Paciente de 25 años, convaleciente de un cuadro gripal de 10 días de evolución, que acude a Urgencias por debilidad generalizada. Es diagnosticado de síndrome de Guillain-Barré. ¿Qué dato clínico o resultado de prueba de laboratorio haría dudar de ese diagnóstico?:

a) Arreflexia universal. b) Oftalmoplejia. c) Atrofia muscular con fasciculaciones generalizadas. d) Ausencia de disociación albúmino-citológica en LCR.

321: Un paciente con hernia discal del interespacio L5-S1 y afectación de la raíz SI puede presentar los siguientes síntomas y signos, EXCEPTO:

a) Dolor en región lumbar, glútea y cara posterior de muslo y pierna. b) Déficit motor para la flexión dorsal del pie y el dedo gordo. c) Déficit de la sensibilidad al pinchazo en el borde lateral del pie.

322: Una mujer de 28 años sufre una cefalea intensa de instauración brusca. Pocos minutos después pierde la conciencia y cae al suelo, respondiendo sólo a estímulos dolorosos. Al llegar al hospital está estuporosa, persiste la cefalea y presenta vómitos y rigidez de nuca sin signos neurológicos focales. ¿Cuál de las siguientes afirmaciones es INCORRECTA?:

a) La causa más probable del cuadro es rotura de aneurisma intracraneal. b) Una causa menos probable podría ser la rotura de una malformación arteriovenosa. c) La pérdida transitoria de conciencia podría explicarse por un incremento brusco de la presión intracraneal por el sangrado. d) La punción lumbar para objetivar sangrado es la primera prueba a efectuar.

323: En relación con una crisis epiléptica que se presenta dentro de la primera hora tras un traumatismo craneoencefálico, señale la afirmación correcta entre las siguientes:

a) No suele tener trascendencia pronóstica en cuanto a la aparición posterior de crisis epilépticas. b) Indica que existe un alto riesgo de epilepsia postraumática tardía. c) Requiere iniciar de inmediato tratamiento antiepiléptico. d) Indica siempre que existe un hematoma intracraneal.

324: En relación con las fracturas de la base craneal, señale cuál de las siguientes afirmaciones es INCORRECTA:

a) Su localización más común es la región frontal y el peñasco. b) La Rx craneal las detecta con dificultad y la TAC craneal es más sensible. c) Deben sospecharse en pacientes que presentan hematoma periocular o en la región mastoidea. d) En ellas, es usual observar salida de líquido cefalorraquídeo por nariz o conducto auditivo externo.

325: En relación con la fractura lineal de la bóveda craneal, señale al afirmación INCORRECTA:

a) Su presencia en la Rx simple de cráneo es siempre indicación de TAC craneal urgente. b) Puede dar lugar a la formación demorada de un hematoma epidural. c) Su presencia conlleva indicación de ingreso para observación, aún cuando la TAC craneal sea normal. d) Cuando se combina con herida abierta del cuero cabelludo suprayacente, conlleva riesgo de infección intracraneal.

Page 106: Cuestionario de Urología

326: Señale la afirmación INCORRECTA en relación con la compresión medular:

a) La enfermedad tumoral más comúnmente responsable es la metástasis vertebral de carcinoma. b) De los tumores extramedulares intradurales los más frecuentes son el meningioma y el neurinoma. c) El grado de déficit neurológico en el momento de efectuar una descompresión quirúrgica se relaciona estrechamente con el pronóstico funcional final. d) El absceso extradural cursa habitualmente con fiebre y dolor local intenso, pero sólo rara vez produce paraplejía.

327: De las siguientes causas de hipercalcemia, ¿cuál es la más frecuente en pacientes hospitalizados?:

a) Hiperparatiroidismo primario. b) Enfermedad de Paget. c) Inmovilización. d) Neoplasia maligna.

328: Paciente de 55 años, diabético tipo 2 en tratamiento con hipoglucemiantes orales, que presenta TA 130/82 mmHg, creatinina plasmática: 0,8 mg/dL, hemoglobina glicosilada: 6% (normal <5,6%X excreción urinaria de albúmina en dos determinaciones de 24 horas: 180 y 220 mg/dA) ¿Qué actitud terapéutica de las siguientes es la indicada?:

a) Insistir en que efectúe una dieta adecuada. b) Descender los niveles de presión artttrial con diuréticos. c) Administrar un inhibidor de la enzima de conversión de la angiotensina. d) Iniciar tratamiento con insulina cristalina.

329: La ingestión de alguno de los siguientes nutrientes, en cantidades muy superiores a las recomendaciones dietéticas diarias, NO se ha asociado con enfermedad. Señálero:

a) Vitamina A. b) Energía (calorías aportadas por carbohidrato s, grasas o proteínas). c) Vitamina D. d) Vitamina B1 (tiamina).

330: Mujer de 45 años, con antecedentes de enfermedad de Graves-Basedow a los 37 años, tratada con 1131, quedando eutiroidea. Actualmente presenta exoftalmos leve, bocio difuso con un nódulo de unos 3 cm en lóbulo izquierdo, sólido en la ecografía. En la gammagraÍla la captación es uniforme con un área de hipocaptación a nivel del nódulo palpable. En la PAAF la citología sugiere carcinoma papilar. La conducta más aconsejable, entre las siguientes, es:

a) Observación periódica. b) Tiroidectornía total seguida de Il3l y L-T4. c) Hemitiroidectornía izquierda. d) Inyección de etanol en el nódulo tiroideo.

331: Paciente de 53 años, con historia de astenia, malestar general y poliuria, que muestra en un análisis una calcemia de 15 mg/dL, con cortisol, T4, TSH y PTH normales. Necesita tratamiento urgente mediante:

a) Vitamina D más diuréticos tiazídicos. b) Rehidratación, furosemida, calcitonina y bifosfonatos. c) Bifosfonatos de entrada. d) Calcitonina de entrada, seguida de bifosfonatos.

332: Paciente de 60 años, fumador, con historia de disfagia progresiva y adelgazamiento de 15 Kg en 6 meses. En la actualidad sólo tolera líquidos por vía oraal en estudio radiológico demuestra una lesión neoplásica en esófago. El paciente refiere intensa astenia. En la exploración clínica destaca una delgadez intensa con pérdida de panículo adiposo y masa muscular. Los estudios complementarios indican realizar esofagectomía. En relación con la nutrición de este enfermo, ¿cuál de las siguientes afirmaciones es correcta?:

a) El cuadro es sugerente de malnutrición proteico-calórica o marasmo. b) Es necesario instaurar inmediatamente una nutrición parenteral total. c) Será necesaria la nutrición parenteral total en el postoperatorio. d) La situación nutricional del paciente no influye en el pronóstico quirúrgico a corto plazo.

333: Hombre de 45 años, no fumador, sin historia familiar de cardiopatía isquémica. Peso 130 kg, altura 175 cm y colesterol de 200 mg/dL. Este paciente tiene mayor riesgo de padecer las siguientes enfermedades, EXCEPTO una. Señálela:

a) Diabetes mellitus. b) Cardiopatía isquémica. c) Hipertensión. d) Hipotiroidismo.

334: Señale cuál es la secuencia en que debe realizarse el tratamiento en el síndrome de Sheeham con panhipopituitarismo y afectación de GH, FSH, LH, TSH y ACTH:

a) L-tiroxina, cortisol, GH, hormonas sexuales. b) Cortisol, l-tiroxina, estrógenos-progestágenos. c) Hormonas sexuales, GR, l-tiroxina, cortisol. d) GR, l-tiroxina, estrógenos-progestágenos cortisol.

335: El valor biológico de las proteínas procedentes de las fuentes siguientes: legumbres, cereales, productos animales y verduras, puede ordenarse de mayor (valor biológico más alto) a menor (valor biológico más bajo), de la siguiente forma:

a) Proteínas animales> legumbres> cereales> verduras. b) Proteínas animales> cereales> legumbres> verduras. c) Proteínas animales> legumbres> verduras> cereales. d) Proteínas animales> verduras> cereales> legumbres.

336: ¿Qué hallazgo, de los siguientes, esperaría encontrar en la bioquímica sanguínea de un paciente con hipoparatiroidismo

Page 107: Cuestionario de Urología

secundario a cirugía de tiroides?:

a) Calcio bajo y fósforo elevado. b) Calcio y fósforo bajos. c) Calcio normal y fósforo elevado. d) Calcio alto y fósforo disminuido.

337: Mujer de 65 años con enfermedad de Graves, bocio difuso de pequeño tamaño con nódulo de 2 cm no funcionante asociado. ¿Cuál de los siguientes hechos aconseja indicar tratamiento quirúrgico, de entrada?:

a) La edad de la paciente. b) El tamaño del bocio. c) La toxicidad potencial de la medicación antitiroidea. d) La presencia de un nódulo no funcionante.

338: ¿Cuál de los siguientes tratamientos es el más adecuado en el carcinoma medular de tiroides sin metástasis a distancia?:

a) Tiroidectomía total más linfadenectomía central. b) Tiroidectomía subtotal más I131. c) Quimioterapia con adriamicina. d) Quimioterapia con ciclosfosfamida.

339: ¿Cuál de los siguientes datos o determinaciones pre o intraoperatorias indica con mayor probabilidad la presencia de malignidad en un insulinoma?:

a) Los hallazgos anatomopatológicos intraoperatorios. b) La presencia de marcadores tumorales en sangre. c) El grado de elevación de la insulina en sangre. d) La ocupación por el insulinoma de la vena mesentérica inferior.

340: La indicación del tratamiento quirúrgico en un hiperparatiroidismo primario asintomático podría basarse en todos los datos siguientes EXCEPTO en uno. Señálelo:

a) Reducción de aclaramiento de creatinina mayor del 30%. b) Presencia de litiasis renal asintomática diagnosticada radiológicamente. c) Calciuria de 24 horas normal. d) Calcemia mayor de 12 mg/dL.

341: ¿Cuál de los siguientes datos NO es considerado como un criterio de artritis reumatoide de la Asociación Americana de Reumatología?:

a) Rigidez matutina de > 1 hora de duración. b) Factor reumatoide positivo. c) Presencia de HLA-DR4. d) Nódulos reumatoides.

342: Un joven de 24 años refiere dolor en el glúteo derecho, irradiado por la cara posterior del muslo hasta la rodilla, de 10 días de evolución. Su médico le prescribe reposo, calor local y analgésicos. A los 15 días vuelve porque se le irradia el dolor a la otra nalga y, además, tiene dolor en un hombro y en la región dorsal baja acompañado de cierta rigidez matutina. El hemograma es normal, la radiografía de columna no revela alteraciones y en la de articulaciones sacroilíacas se aprecia un ligero borramiento del borde en el lado derecho. ¿Cuál de los siguientes diagnósticos es más probable?:

a) Espondilitis infecciosa. b) Artritis reumatoide. c) Hiperostosis esquelética idiopática difusa. d) Espondiloartritis anquilopoyética.

343: Hombre de 42 años que, un año antes, presentó lesiones eritematosas, descamativas y pruriginosas en superficie extensora de ambos codos. Seis meses después notó hinchazón y dolor en interfalángica del primer dedo de mano derecha y en interfalángicas (IF) de 2°, 3° Y 5° dedos de pie der echo. Ausencia de antecedentes de pleuritis, diarrea, lumbalgia, ciática, iritis y uretritis. El examen ÍIsico muestra las lesiones cutáneas referidas en codos y en el lado derecho del cuero cabelludo. Junto a los fenómenos inflamatorios articulares descritos, se comprueba deformidad en "salchicha" (dactilitis) en IF de 3° y 4° dedos de pie izquierdo. Las uñas presentan lesiones punteadas, ouicolisis y surcos horizontales palpables. Datos de laboratorio: ANA, factor reumatoide y serología de lúes negativos; hiperuricemia moderada y nivel elevado de IgA; PCR++, V. Sed. 30 mm en la hora. Rx de manos: pequeñas erosiones periarticulares en IF distales de 2° y 3° dedos de mano derecha y sugeren cia de osteolisis de fa-langes distales. Con más probabilidad el paciente tendrá:

a) Artritis reumatoide. b) Artritis psoriásica. c) Síndrome de Reiter. d) Artropatía amiloide.

344: Con el diagnóstico de fiebre reumática (reumatismo poliarticular agudo), a un niño de 14 años se le trata con 1.200.000 U de bencilpenicilina i.m. cada cuatro semanas y antiinflamatorios no esteroideos. A las ocho semanas del tratamiento está asintomático y los niveles de anticuerpos antiestreptolisina O (ASLO) siguen elevados. ¿Cuál de las siguientes afirmaciones respecto a este caso es correcta?:

a) El estreptococo faríngeo es resistente a la penicilina y se debe usar otro antibiótico. b) La dosis de penicilina no es la adecuada. c) Los niveles de ASLO tardan en normalizarse unos 6 meses. d) La enfermedad continúa en actividad y, por tanto, hay que usar corticosteroides.

345: Paciente de 70 años con antecedentes de episodios transitorios de dolor y tumefacción en rodilla derecha que, en su segundo día de postoperatorio por apendicitis aguda no complicada, desarrolla dolor y signos inflamatorios locales en ambas rodillas y tobillo derecho con temperatura elevada. Entre los siguientes, ¿cuál es el diagnóstico más probable?:

Page 108: Cuestionario de Urología

a) Artritis reumatoide de comienzo. b) Condrocalcinosis (pseudogota). c) Artritis séptica. d) Artritis de Lyme.

346: Indique cuál de los siguientes trastornos NO es causa de artropatía neuropática:

a) Diabetes mellitus. b) Tabes dorsal. c) Siringomielia. d) Hipotiroidismo.

347: Respecto a la poliarteritis nodosa (PAN), señale cuál de las siguientes afirmaciones es FALSA:

a) La lesión vascular es de distribución segmentaria y se localiza sobre todo en la bifurcación de los vasos. b) Riñón y corazón son los órganos más afectados. c) La afectación hepática es más frecuente en los casos asociados al virus B o C de la hepatitis. d) El 98% de los casos presenta anticuerpos C-ANCA positivos.

348: Ante un paciente de 54 años con condrocalcinosis, diabetes mellitus, alteración de la función hepática y una artritis simétrica afectando 2a y 3a articulaciones metacarpofalángicas, el primer diagnóstico que debe plantearse es:

a) Artritis reumatoide. b) Artrosis. c) Hemocromatosis. d) Artritis psoriásica.

349: La supervivencia en un paciente con diagnóstico de esclerodermia está especialmente determinada por la:

a) Gravedad de las lesiones cardíacas, pulmonares y renales. b) Severidad del fenómeno de Raynaud. c) Edad al diagnóstico. d) Afectación gastrointestinal.

350: Un albañil de 25 años es traído a Urgencias tras precipitarse desde un andamio a 6 metros de altura (caída en bipedestación). Tras exploración ÍIsica y radiológica simple detalladas, se le diagnostica un pequeño aplastamiento de la cuarta vértebra lumbar y fracturas conminutas bilaterales de ambas extremidades distales de la tibia. Tras dos horas de proceso dlagn6s6co inicia un cuadro de palidez, sudoración y frialdad cutánea, hipotensión y bradicardia. ¿Cuál de los siguientes diagnósticos debe sospecharse en primer lugar?:

a) Shock hipovolémico por hemorragia interna secundaria a desgarro de pedículo visceral. b) Shock hipovolémico secundario a sangrado de las fracturas. c) Shock hipovolémico por íleo paralítico secundario a la fractura vertebral. d) Shock neurogénico por dolor.

351: Una mujer de 24 años acude a Urgencias tras sufrir un accidente de esquí, describiendo que se le fueron separando las tablas y las extremidades inferiores hasta caerse. A la exploración no se aprecia derrame articular, duele al palpar la cara medial del cóndilo femoral medial y no se aprecian inestabilidades, pero duele al forzar el valgo. El diagnóstico más probables es lesión:

a) Meniscal del menisco medial. b) Meniscal del menisco lateral. c) Del ligamento cruzado anterior. d) Del ligamento colateral medial.

352: Un muchacho obeso de 14 años acude a la consulta por dolor en cara anterior de muslo y rodilla izquierdos, desde un partido de fútbol jugado 15 días antes. No presenta ningún signo de desarrollo puberalo Permanece con una actitud en flexión y rotación externa de la cadera y flexión de la rodilla, con limitación dolorosa a los intentos de movilización articular. El primer diagnóstico de sospecha debe ser:

a) Epifisiolisis femoral proximal. b) Artritis séptica de cadera. c) Artritis reumatoide juvenia) d) Bloqueo meniscal de la rodilla.

353: Paciente de 40 años con fractura del cuello del astrágalo, con desplazamiento posterior del cuerpo astragalino. Señale, de las complicaciones posibles que se relacionan, cuál es la más característica:

a) Pie cavo-valgo postraumático. b) Necrosis isquémica del cuerpo del astrágalo. c) Pie plano postraumático. d) Pie cavo-varo postraumático.

354: En un paciente de 25 años que sufre, en accidente de moto, una fractura abierta grado IIIB de tibia y peroné de la pierna izquierda, ¿cuál de los siguientes tratamientos es el más indicado?:

a) Reducción de la fractura, cierre de la herida y vendaje de yeso. b) Cierre de la herida y tracción transesquelética en calcáneo. c) Osteosíntesis con fijación externa. d) Osteosíntesis con placa y tomillos, tras cierre de la heri da.

355: Un joven de 22 años sufre un accidente de moto, cayendo al suelo con separación forzada de la cabeza y el hombro, lesionándose el miembro superior derecho. Señale cuál de los siguientes hechos es característico de este tipo de accidente:

a) Fractura de Colles. b) Fractura del 5° metacarpiano.

Page 109: Cuestionario de Urología

c) Esguince de codo. d) Lesión del plexo braquial.

356: Señale cuál de las siguientes características es propia del dolor que aparece como consecuencia de un esguince del tobillo que afecta al ligamento colateral externo:

a) Comienzo agudo y remisión progresiva. b) Es intenso al principio, sigue un período de latencia y después es continuo. c) Intensidad estable durante las cuatro primeras horas. d) Aumento progresivo de intensidad y sin interrupción.

357: El líquido del empiema pleural se caracteriza por:

a) Nivel hidroaéreo en el TAC torácico. b) Recuentos muy elevados de células mononucleares. c) pH inferior a 7.2. d) Valores de glucosa inferiores a 100 mg/dL.

358: Un trabajador sanitario tiene una prueba de la tuberculina que mide 0 mm. Al repetirla 10 días después, el diámetro de la induración mide 12 mm. ¿Cuál de las siguientes interpretaciones es más adecuada para estos resultados?:

a) Primoinfección tuberculosa entre las dos pruebas. b) Infección tuberculosa latente. c) Presencia de alergia. d) Tuberculosis activa hace años.

359: Un hombre de 45 años con leucemia mieloblástica aguda recibió su tercer ciclo de quimioterapia. A los 7 días desarrolló fiebre y un súbito empeoramiento del estado general. Presentaba una lesión nodular equimótica y dolorosa, con centro ulcerado y rodeada de eritema y edema, en la pierna izquierda. Tenía menos de 100 leucocitos/mm3, hemoglobina 7 gr/dL y 30.000 plaque-tas/mm3) ¿Qué germen es aislado con mayor frecuencia en un caso como éste?:

a) Aeromonas hydrophila. b) Klebsiella pneumoniae. c) Staphylococcus aureus. d) Pseudomonas aeruginosa.

360: Señale cuál de los siguientes gérmenes NO es un patógeno común en padentes neutropénicos:

a) Cryptococcus, b) Staphylococcus. c) Pseudomonas. d) Cándida.

361: Un hombre de 31 años que tenía contacto directo con perros, consultó por fiebre y cefalea de 4 días de evolución. Estaba febril y tenía un exantema máculo-papular en cara, tronco y extremidades, incluyendo palmas y plantas. Existía una lesión costrosa y negruzca entre las nalgas y adenopatías inguinales. ¿Cuál de los .siguientes gérmenes es el responsable?:

a) Borrelia burgdorferi. b) Rickettsia typhi. c) Treponemapallidum d) Ricketssia conorii.

362: Un veterinario de 40 años consultó por fiebre de 3 semanas de duración con dolor de espalda en los últimos días. Al final de la primera semana había tenido inflamación testicular que cedió con trimetoprimsulfametoxazoa) La exploración era normal, salvo por dolor a la percusión en la columna dorsolumbar. Tenía una hemoglobina de 12,2 g/dL, GGTP 320 u/L (normal 6-45) y fosfatasa alcalina 230 u/L (normal 41-117). La resonancia nuclear magnética mostraba epifisitis anterior de D12 y signos de espondilodiscitis D12-La) ¿Cuál debe ser la prueba diagnóstica siguiente en este caso?:

a) Biopsia para cultivo de la médula ósea. b) Punción aspiración vertebral. c) Hemocultivos y aglutinaciones para Brucella. d) Biopsia hepática.

363: Una joven de 18 años acude al hospital por fiebre y cefalea de varias horas de evolución. Los días previos había notado dolor de garganta y tos. Se observó tendencia al sueño, rigidez de nuca y petequias en conjuntivas y extremidades. El LCR era turbio y contenía 36.000 leucocitos/mm3, 200 mg/dL de proteínas y 20 mg/dL de glucosa (glucemia simultánea 120 mg/dL). El examen con Gram fue negativo. Señale cuál de las siguientes afirmaciones NO es correcta:

a) El diagnóstico más probable es meninguitis meningocócica. b) Los meningococos son siempre sensibles a la penicilina y, por tanto, la penicilina G es el tratamiento de elección. c) Como un significativo porcentaje de meningococos del grupo C son resistentes a la penicilina, la cefotaxima es un tratamiento más seguro. d) El empleo de dexametasona reduce el riesgo de secuelas neurosensoriales en niños con meninguitis bacteriana, pero su uso en un caso como éste es cuestionable.

364: ¿Cuál es la principal causa de epididimitis en varones heterosexuales activos menores de 35 años?:

a) Chlamydia trachomatis. b) Neisseria gonorrhoeae. c) Ureaplasma urealyticum. d) Escherichia coli.

365: Las manifestaciones clínicas del tétanos se producen por la:

a) Invasión tisular directa del sistema nervioso central. b) Producción de una exotoxina en la herida infectada.

Page 110: Cuestionario de Urología

c) Producción de citoquinas. d) Destrucción tisular marcada en la zona infectada con sobreinfección bacteriana polimicrobiana secundaria.

366: ¿Cuál de las siguientes pruebas es la más adecuada para evaluar la respuesta terapéutica en un paciente tratado de sífilis precoz?:

a) Realización de fondo oscuro sobre las lesiones que vayan apareciendo. b) Examen anual de líquido cefalorraquídeo (VDRL). c) Test de inmovilización del Treponema pallidum (TPI). d) Evaluación seriada del título del VDRL o RPR (pruebas no treponémicas).

367: Señale lo correcto en relación con la coriorretinitis por citomegalovirus:

a) Ocurre en cualquier estadio de la infección. b) Siempre cursa con viremia. c) Siempre afecta a la cámara anterior del ojo. d) Es progresiva y puede causar ceguera.

368: Acerca de la fiebre de origen desconocido, ¿cuál de las siguientes afirmaciones es FALSA?:

a) Clásicamente, se define como fiebre mayor de 38,3°C, en varias ocasiones, durante más de tres seman as, sin encontrarse un diagnóstico tras una semana de investigaciones en el hospital. b) Hoy en día se prefiere clasificarla como clásica, en neutropénico, nosocomial y asociada a infección por VIH. c) Es fundamental hacer una historia de los viajes realizados. d) En pacientes que la presentan durante más de seis meses, la causa más frecuente es infecciosa.

369: Actualmente, la indicación más habitual y fundamentada de la ribavirina es:

a) Hepatitis crónica B. b) Herpes genital recidivante resistente a aciclovir. c) Infección por virus sincitial respiratorio en niños pequeños, administrada en aerosol. d) Infección avanzada por VIH.

370: La candidiasis hepatoesplénica ocurre habitualmente en:

a) Pacientes con SIDA. b) Diabéticos tipo 1 con neuropatía avanzada. c) Pacientes con leucemia aguda y leucopenia prolongada en fase de resolución. d) Pacientes con cirugía pancreática mayor y con nutrición parenteral.

371: Los agentes causales más frecuentes de meningitis viral son:

a) Arbovirus. b) Herpesvirus. c) Virus de la coriomeningitis linfocitaria. d) Enterovirus.

372: Acerca de las hepatitis virales, ¿cuál de las afirmaciones siguientes es correcta?:

a) La hepatitis por virus de la hepatitis B es la más frecuente en España. b) El virus de Epstein-Barr no se incluye en el diagnóstico diferencial. c) La presencia de rash y poliartritis orienta el diagnóstico hacia hepatitis B. d) La hepatitis delta ocurre en pacientes infectados por virus de la hepatitis C.

373: La laparotomía para determinar la extensión en la enfermedad de Hodgkin está indicada fundamentalmente:

a) En todos los pacientes con estadios IA y IIA. b) Cuando se propone la radioterapia como único tratamiento. c) En la variedad histológica esclerosis nodular. d) Cuando se presume la afectación esplénica.

374: El diagnóstico de la trombocitemia esencial se basa fundamentalmente en:

a) La presencia de esplenomegalia. b) La cifra de plaquetas superior a 1.000.000 /mm3,. c) La exclusión de las restantes entidades mieloproliferativas crónicas. d) El aumento importante de los megacariocitos medulares.

375: En un paciente tratado durante años con vitamina B12 parenteral por anemia perniciosa, con buena respuesta, aparecen anemia microcítica moderada y progresiva. Señale cuál es la explicación más probable de esta evolución:

a) La dosis de vitamina B12 es inadecuada. b) Ha desarrollado anemia ferropénica por gastritis atrófica. c) El paciente tenía una talasemia minor "oculta" por la anemia perniciosa. d) Tiene un déficit asociado de ácido fólico.

376: Las anemias hemolíticas congénitas se clasifican en tres grandes grupos, dependiendo de dónde se encuentre la alteración: en la membrana, en las enzimas o en la hemoglobina. En cada una de las respuestas siguientes hay una anemia de cada grupo. De ellas, señale cuál contiene el tipo más frecuente de cada grupo en España:

a) Eliptocitosis, déficit de G6P-dehidrogenasa, beta talasemia minor. b) Xerocitosis, déficit de piruvato kinasa, alfa talasemia. c) Esferocitosis, déficit de triosa fosfato isomerasa, drepanocitosis. d) Esferocitosis, déficit de G6P-dehidrogenasa, talasemia minor.

377: Paciente de 28 años, sin antecedentes de haber recibido quimioterapia ni radioterapia previamente, con leucopenia de 1.300/µL, trombopenia de 25.000/µL y anemia de 7 g/dL de Hb, sin blastos en sangre periférica y con aspirado de médula ósea

Page 111: Cuestionario de Urología

muy hipocelular. Señale qué prueba de confirmación está indicada y cuál es el diagnóstico más probable:

378: Un paciente con prótesis valvular cardíaca metálica, que recibe habitualmente tratamiento anticoagulante oral, necesita ser sometido a cirugía programada por un problema abdominal. Ante el riesgo de sangrado durante la intervención, es necesario:

a) Disminuir la dosis de anticoagulante oral desde un día antes de la operación. b) Suspender el anticoagulante oral seis horas antes de la operación. c) Poner transfusión de plasma fresco inmediatamente antes de operar. d) Realizar la intervención previa sustitución del anticoagulante oral por heparina.

379: ¿Cuál de las siguientes combinaciones de datos, presentes en el momento del diagnóstico, se utiliza para establecer el estadio de Durie y Salmon para el mieloma múltiple?:

a) Hb, Ca sérico, Rx de huesos, tamaño del componente M, creatinina sérica. b) Estado general, cifra de plaquetas, creatinina sérica, beta2 microglubilina, porcentaje de células plasmáticas en médula ósea. c) Porcentaje de células plasmáticas en médula ósea, Hb, cifra de plaquetas, dolor óseo, velocidad de sedimentación globular. d) Estado general, tamaño del componente M, beta2 microglobulina, porcentaje de células plasmáticas en médula ósea.

380: Señale cuál de las siguientes respuestas resume las características fundamentales de la púrpura trombopénica idiopática aguda:

a) Edad adulta, curso crónico, no asociación con enfermedad viral. b) Edad pediátrica, alta tasa de anticuerpo s antiplaquetas en suero, recuperación rápida tras tratamiento con glucocorticoides. c) Cualquier edad, rápida respuesta al tratamiento, asocia ción con otras enfermedades autoinmunes. d) Edad pediátrica, recuperación rápida y completa, asociación con enfermedad viral.

381: Paciente de 65 años, que presenta un trastorno de la marcha junto con parestesias de ambas extremidades inferiores. La sensibilidad vibratoria está abolida. Existe una fuerte sospecha clínica de un déficit de cobalaminas, a pesar de que sus niveles plasmáticos son de 250 pg/mL (normal: 200-900 pg/mL). ¿Qué prueba realizaría para confirmar o excluir el diagnóstico?:

a) Repetir la cuantificación de cobalaminas séricas junto a la de folatos intracelulares. b) Cuantificar la homocisteína total plasmática. c) Realizar un test de Schilling d) Cuantificar el ácido metil-malónico y la homocisteína total plasmáticas.

382: Paciente de 35 años, diagnosticado de un artritis reumatoide de 8 años de evolución y tratado con AINES por vía oral. Presenta una anemia hipocroma, con niveles de ferritina sérica normales. Se quiere descartar una carencia de hierro. ¿Cuál sería la prueba diagnóstica definitiva?:

a) Realizar un estudio ferroquinético. b) Cuantificar la protoporfirina libre intraeritrocitaria. c) Estudiar por microscopía óptica los depósitos de hierro de la médula ósea. d) Medir niveles de G6P dehidrogenasa en hematíes.

383: ¿Cuál de los siguientes hechos NO sería de esperar en un paciente de 65 años con anemia y una gammapatía monoclonal IgM?:

a) Ausencia de lesiones líricas. b) Afectación de la médula ósea. c) Alteraciones visuales por hiperviscosidad. d) Fenómeno de Raynaud con la exposición al frío.

384: ¿Cuál de las siguientes situaciones NO es una indicación de diálisis en un paciente con insuficiencia renal aguda?:

a) Síntomas urémicos. b) Hiperpotasemia. c) Acidosis. d) Contracción del volumen extracelular.

385: De las siguientes afirmaciones, respecto a la enfermedad renal poliquística del adulto, ¿cuál es la correcta?:

a) Es más prevalente en mujeres, por el uso de anovulatorios. b) Se puede acompañar de quistes hepáticos. c) Sólo desarrolla hipertensión arterial si reciben antiinflamatorios. d) Rara vez evolucionan a insuficiencia renal crónica, excepto en presencia de hipertensión arterial.

386: Paciente de 65 años que acude a Urgencias por anuria absoluta de 24 horas de evolución, después de la administración de una sola dosis de un antiinflamatorio. ¿Qué exploración sería la más aconsejable, entre las siguientes, para el diagnóstico de la nefropatía?:

a) Determinación del antígeno específico prostático (PSA). b) Técnicas isotópicas para valorar la perfusión renal. c) Observar el ritmo de elevación de creatinina en plasma. d) Ecografía renal para valorar tamaño renal y aspecto del sistema excretor.

387: Un paciente de 48 años, con antecedentes de hepatitis e (anti HVC +), presenta edemas maleolares, proteinuria 3.5 g/día, creatinina sérica 1.6 mg/dL y microhematuria en el sedimento. ¿Qué proceso glomerular de los siguientes se encontrará con más probabilidad en la biopsia renal?:

Prueba Diagnóstico

a) Biopsia de médula ósea Aplasia medular

b) Estudio citoquímico Leucemia aguda

c) Test de Ham Aplasia medular

d) Estudio citogenético Linfoma con invasión de médula ósea

Page 112: Cuestionario de Urología

a) Cambios mínimos. b) Glomeruloesclerosis focal y esclerosante. c) Glomerulonefritis extracapilar. d) Glomerulonefritis membranoprofilerativa.

388: ¿De cuál de los siguientes cuadros es característica la aparición de cilindros hemáticos en el sedimento urinario?:

a) Necrosis tubular aguda. b) Lesión a cualquier nivel de las vías urinarias. c) Daño glomerular severo. d) Daño tubular.

389: Una mujer de 58 años presenta un cuadro de fiebre de cuatro semanas, llegándose al diagnóstico de toxoplasmosis. En las tres semanas previas, había recibido ampicilina oral a la dosis de 2 g/día, con lo que había descendido algo, sin desaparecer, la fiebre. Unos días antes del ingreso, vuelve a elevarse la fiebre, aparece eritema, oliguria de 500 a 700 mL/día, creatinina plasmática de 4.3 mg/dL y hematuria macroscópica con proteinuria de 2 g/24 h. En el sedimento, el 80 % de los hematíes están deformados y hay algunos eosinófilos. Los niveles de C3 y C4 en plasma eran normales. El cuadro revirtió al final, dejando una filtración glomerular del 75%. ¿Qué diagnóstico, de los siguientes, es el más probable?:

a) Nefritis intersticia1 aguda por hipersensibilidad. b) G1omerulonefritis aguda post-infecciosa. c) Necrosis tubular aguda. d) Glomerulonefritis membranosa.

390: ¿En cuál de los siguientes glomerulonefritis hay activación del complemento, por la vía alternativa?:

a) Aguda secundaria a endocarditis. b) Aguda postestreptocócica. c) Difusa lúpica. d) Por crioglobulinemia.

391: ¿Cuál de las siguientes situaciones NO es contraindicación de trasplante renal?:

a) Enfermedad coronaria muy avanzada. b) Hipertensión arterial que requiere 3 fármacos. c) Infección bacteriana activa. d) Anticuerpos preformados frente a antígenos del donante.

392: Señale la causa más frecuente de dilatación renal en la infancia:

a) Litiasis. b) Infecciones urinarias. c) Tumor de Wilms. d) Reflujo vésico-ureteral.

393: ¿Cuál de las siguientes circunstancias es una contraindicación absoluta para la litotricia extracorpórea con ondas de choque de la litiasis renal?:

a) Embarazo. b) Coagulopatía. c) Marcapasos cardíaco. d) Aneurisma aórtico abdominal)

394: Ante una ecografia renal en la que se observa una masa bien delimitada, que interrumpe el contorno renal, con ausencia de ecos en su interior y refuerzo posterior, ¿qué diagnóstico, entre los siguientes, es el más probable?:

a) Hidronefrosis. b) Carcinoma renal. c) Quiste renal simple d) Absceso renal

395: ¿Qué diagnóstico, entre los siguientes, es el más probaIiíeen un niño de 8 años con signos inequívocos de pubertad precoz y que, en la exploración, presenta una masa en el testículo derecho de 2 cm de diámetro?:

a) Tumor de células de Leydig. b) Seminoma. c) Tumor del saco vitelino. d) Teratoma.

396: Paciente con historia de cólicos nefríticos de repetición expulsivos, cuyo estudio metabólico muestra una hipercalciuria idiopática no influenciada por el ayuno ni la ingesta. De los siguientes, ¿cuál es el tratamiento adecuado?:

a) Fosfato de celulosa. b) Dieta hipoproteica y ortofosfatos. c) Alopurinol. d) Ingesta abundante de líquidos y tiazidas.

397: En un joven de 29 años sin circuncidar aparece una lesión prepucial de 1 cm de diámetro. Una biopsia revela la existencia de células escamosas que no sobrepasan la dermis. El tratamiento más adecuado, de los siguientes, es:

a) Circuncisión. b) Penectomía parcial. c) Irradiación externa. d) Coagulación por láser.

398: Un joven de 18 años presenta un cuadro clínico de instauración brusca consistente en fiebre, dolores articulares, erupción

Page 113: Cuestionario de Urología

aguda de pápulas, pústulas y nódulos que evolucionan a úlceras y costras, localizada en el tronco fundamentalmente, acompañado de leucococitosis. ¿Qué diagnóstico de los siguientes debe sospechar en primer lugar?:

a) Varicela. b) Exantema medicamentoso. c) Foliculitis aguda. d) Acnéfulminans.

399: Un paciente de 52 años, presenta por primera vez, una erupción de lesiones pruriginosas, habonosas, diseminadas por toda la superficie corporal, de evolución fugaz y una amplia placa edematosa en hemicara. ¿Cuál de los diagnósticos siguientes es el más probable?:

a) Urticaria aguda. b) Angioedema hereditario. c) Erisipela. d) Carbunco.

400: En dermatología, se denomina "Iupus pernio" a una forma de:

a) Lupus tuberculoso. b) Lupus eritematoso. c) Sarcoidosis. d) Pemiosis.

401: ¿Cuál de las siguientes afirmaciones, referentes a la leucoplasia vellosa oral que se asocia a la infección por VIH, es INCORRECTA?:

a) Suele ser asintomática. b) Existen células balonizadas en la histopatología. c) Es un marcador clínico de progresión de la infección por el VIH. d) El primer implicado en su patogenia parece ser la Cándida albicans.

402: Un cazador de 48 años comienza, una semana después de una cacería con perros, con fiebre alta, cefalea y una erupción maculopapulosa en tronco, miembros, palmas de las manos y plantas de los pies y algunas lesiones purpúricas. En el cuello tiene una úlcera necrótica con halo rojizo y una adenopatía regional dolorosa. ¿Qué tratamiento de los siguientes debe instaurarse para el proceso que más probablemente padece?:

a) Ninguno. b) Tetraciclinas. c) Antimoniales. d) Corticoides.

403: ¿Cuál de los siguientes cuadros NO es una manifestación cutánea de la diabetes mellitus?:

a) Eritema nodoso. b) Xantomatosis. c) Necrobiosis lipídica. d) Infecciones bacterianas en miembros inferiores.

404: ¿CuáI de los siguientes supuestos DESCARTARÍA como causa de una parálisis recurrencial unilateral?:

a) Aneurisma aórtico. b) Carcinoma de esófago. c) Absceso periarnigdalino. d) Cirugía de la glándula tiroides.

405: Hombre de 40 años que consulta por presentar cuadros de tumefacción dolorosa en la región submandibular derecha, en relación con la ingesta de alimentos, palpándose una concrección dura en el lado derecho del suelo de la boca. De los siguientes diagnósticos, ¿cuál le parece más probable?:

a) Sialadenosis submaxilar. b) Tumor de Warthin de glándula submaxilar. c) Ránula. d) Sialolitiasis submaxilar.

406: Con respecto a los tapones de cerumen es FALSO que:

a) Pueden ocasionar acúfenos. b) Pueden ocasionar hipoacusia de transmisión. c) Los conductos auditivos externos (CAE) estrechos los favorecen. d) El uso frecuente de bastoncitos de algodón evita su aparición.

407: Paciente de 55 años, trabajador de la madera, que acude a Urgencias por presentar tumefacción del ángnlo interno del ojo izquierdo desde hace dos días, refiriendo exofalmos izquierdo, obstrucción nasal y rinorrea espesa y ocasionalmente sanguinolenta por fosa nasal izquierda de dos meses de evolución. ¿Cuál sería su impresión diagnóstica?:

a) Adenocarcinoma etmoidal izquierdo. b) Glaucoma ocular izquierdo. c) Cuerpo extraño en fosa nasal izquierda. d) Atresia de coana izquierda.

408: Mujer de 42 años, que presenta otalgia derecha desde hace 14 días sin otra sintomatología. A la exploración, conducto auditivo exteruo y tímpano derechos sin hallazgos significativos. El diagnóstico más probable es:

a) Neuralgia del nervio trigémino. b) Síndrome disfunción-dolor de articulación témporomandibular.

Page 114: Cuestionario de Urología

c) Síndrome del agujero rasgado posterior. d) Neuralgia del nervio hipogloso.

409: La hipertrofia amigdalar de la infancia, aunque curse con una frecuencia de infecciones inferior a 4 episodios agudos anuales, puede ser causa de una de las siguientes manifestaciones:

a) Retraso del crecimiento ponderal. b) Retraso en la adquisición del lenguaje hablado. c) Disminución del rendimiento escolar d) Apnea del sueño.

410: Un hombre de 50 años acude a Urgencias por presentar dolor ocular y fotofobia intensa. En la exploración ocular se observa ojo rojo principalmente alrededor del limbo corneal, pupila en miosis y depósitos blanquecinos en endotelio. La tensión ocular es de 10 mmHg (normal <16). El diagnóstico más probable es:

a) Conjuntivitis infecciosa. b) Glaucoma agudo. c) Uveitis. d) Queratitis.

411: Ante un paciente que presenta pérdida brusca de la visión, acompañada de midriasis sin dolor ni enrojecimiento, debe pensarse en:

a) Una queratitis. b) Una obstrucción de la arteria central de la retina. c) Una iridociclitis. d) Un glaucoma agudo.

412: Señale qué grupo de fármacos tópicos, entre los siguientes, está formalmente CONTRAINDICADO en los enfermos con glaucoma:

a) Betabloqueantes no selectivos. b) Betabloqueantes selectivos β1 c) Adrenérgicos α2 agonistas. d) Parasimpáticolíticos.

413: Si una persona que presenta disminución de la agudeza visual, mejora al mirar a través de un agujero estenopeico, la causa más probable de sn déficit visual es:

a) Defecto de refracción. b) Ambliopía. c) Catarata. d) Coroiditis central serosa.

414: Ante un niño de 2 años cuyos padres refieren que en ocasiones desvía un ojo, la actitud más correcta es:

a) Esperar hasta los 5 años pira evaluación por el oftalmólogo. b) Remitirle inmediatamente al oftalmólogo. c) Esperar a que el niño pueda hablar y comunicarse para explorarlos objetivamente. d) Instaurar tratamiento con vitaminas.

415: Ante un síndrome de ClaudeBernard-Horner, en un paciente de 55 años, la primera prueba diagnóstica que debe solicitarse es:

a) TAC orbitario. b) Resonancia magnética orbitaria. c) Placa de tórax P-A y lateral.. d) Ecografía abdominal.

416: Si un paciente es tratado de un cuadro depresivo leve por su médico de Atención Primaria con inhibidores selectivos de la recaptación de serotonina (ISRS) a dosis terapéuticas (por ejemplo: 20 mg de fluoxetina) y, a la semana de iniciar el tratamiento, no obtiene la respuesta terapéutica deseada, ¿qué deberá bacer?:

a) Aumentar la dosis. b) Cambiar de antidepresivo por otro del mismo grupo terapéutico. c) Cambiar de antidepresivo por otro del grupo de los tricíclicos. d) Esperar, ya que la respuesta terapéutica tarda aproximadamente 2 semanas.

417: Los siguientes criterios son útiles en el diagnóstico de anorexia nerviosa EXCEPTO uno. Señálelo:

a) Rechazo a mantener un peso corporal de al menos el 85% del que correspondería a su altura y edad. b) Gran temor a ganár .peso, incluso teniendo un peso inferior al normal. c) Interés por vestir tallas de ropa más pequeñas de las que corresponderían a sus características antropométricas. d) Alteraciones en la percepción de la imagen corporal propia.

418: ¿Cuál de los siguientes fármacos se considera como terapéutica de elección para los trastornos bipolares, especialmente en el caso de los "cicladores rápidos"?:

a) Olanzapina. b) Sertralina. c) Inhibidores selectivos de la recaptación de serotonina (ISRS). d) Carbamacepina.

419: Un paciente de 40 años presenta, desde hace 2 meses, una tendencia que no puede resistir a realizar actos que le resultan desagradables. Se le ocurren pensamientos, percibe imágenes y siente impulsos reiterados y molestos que, aún reconociendo como absurdos, no puede tampoco evitar. Durante el último mes se siente triste y abatido, sin ganas de hacer nada.

Page 115: Cuestionario de Urología

Esporádicamente presenta episodios angustiosos de gran intensidad. ¿Cuál sería la hipótesis diagnóstica más probable, según la CJE-10?:

a) Trastorno mixto ansioso-depresivo. b) Trastorno obsesivo-compulsivo. c) Trastorno depresivo con síntomas obsesivos. d) Trastorno esquizo-afectivo.

420: ¿CuáI de los siguientes hechos NO se debe poner en práctica en el marco de una psicoterapia de apoyo a un paciente deprimido?:

a) Ofrecer comprensión, disponibilidad y competencia. b) Explicar el carácter temporal del trastorno. c) Estimular la toma de decisiones importantes. d) Destacar y valorar los logros alcanzados.

421: ¿CuáI de los siguientes trastornos presenta un mayor riesgo relativo para los parientes de primer grado de los probandos enfermos?:

a) Depresión mayor. b) Trastorno bipolar. c) Anorexia nerviosa. d) Trastorno por somatización.

422: A un joven de 19 años, sin antecedentes de interés, se le lleva a Urgencias en un estado de angustia extrema, sudoración, taquicardia, temblores, febrícula de 37,4°C y midri asis. La familia refiere que regresó así a casa tras una salida nocturna. Desconocen hábitos de consumo de tóxicos. El cuadro clínico orientaría el diagnóstico hacia:

a) Intoxicación por alucinógenos. b) Trastorno por angustia con agorafobia. c) Intoxicación por opiáceos. d) Intoxicación patológica por alcohol.

423: Se considera que hay gran riesgo de repetición de conducta suicida cuando se dan todas estas .circunstancias, EXCEPTO una. Señálela:

a) Ideación suicida frecuente y prolongada. b) Uso de método de alta letalidad. c) Viudedad o separación conyugal. d) Edad inferior a 25 años.

424: Un joven de 27 años llega a Urgencias con protrusión de lengua y espasmo de torsión de cuello. Un familiar que le acompaña sólo sabe decir que recientemente tuvo un breve ingreso psiquiátrico y le han puesto un tratamiento inyectable. En esta situación, el diagnóstico más probable es:

a) Tetania. b) Distonía aguda inducida por neurolépticos. c) Trastorno por ansiedad aguda. d) Corea de Huntington.

425: ¿Cuál de las siguientes afirmaciones es cierta para la llamada depresión reactiva?:

a) Debe tratase exclusivamente con métodos psicoterapéuricos. b) No debe tratarse con antidepresivos por el riesgo de producir un estado hipomaníaco. c) Suele presentar un menor riesgo de suicidio que la depresión endógena. d) Responde bien al tratamiento con sales de lirio.

426: ¿Cuál de los siguientes tratamientos es el más indicado para tratar a una mujer con recidiva de un disgerminoma?:

a) Gestágenos. b) Metrotexate. c) Taxol. d) Cirugía más radioterapia.

427: Una paciente de 16 años presenta amenorrea acompañada de profundas alteraciones del olfato. ¿Cuál de los siguientes diagnósticos es el más probable?:

a) Síndrome de amenorrea-galactorrea. b) Amenorrea de causa uterina. c) Síndrome de ovario poliquístico. d) Amenorrea por alteración hipotalámica.

428: Ante una paciente de 22 años, con caracteres sexuales secundarios muy poco desarrollados y amenorrea primaria, que además presenta hipertensión arterial e hipokaliemia, ¿en qué diagnóstico de los siguientes hay que pensar en primer lugar?:

a) Síndrome de Turner. b) Déficit de 17 alfa hidroxilasa. c) Síndrome de ovario refractario. d) Síndrome de déficit de receptores de estrógenos.

429: ¿Cuál de los siguientes tratamientos es el más adecuado en una mujer de 55 años, menopánsica desde hace uno por habérsele practicado una histerectomía con doble anexectomía, que padece un síndrome climatérico importante?:

a) Calcio oral y benzodiacepina. b) Estrógenos y progestágenos 19 norderivados. c) Estrógenos y veralipride.

Page 116: Cuestionario de Urología

d) Estrógenos exclusivamente.

430: ¿Cómo se encuentran los niveles de hormona folículoestimulante (FSH) en una mujer con menopausia fisiológica?:

a) Elevados, porque también lo están los andrógenos ováricoso b) Elevados, porque la retroacción negativa de las hormonas esteroides ováricas no existe. c) Bajos, porque el hipotálamo funciona inadecuadamente. d) Muy altos, porque la prolactina no inhibe el centro cíclico hipotalámico.

431: Una paciente secundigesta, con antecedente de parto vaginal anterior, ingresa en su 34ª semana de gestación por parto en curso. Mediante tacto vaginal se diagnostica dilatación de 6 cm y presentación pelviana (nalgas puras), variedad S.I.I.A., en II plano. Ecográficamente se estima que el peso fetal estará comprendido entre 2.100 y 2.300 g. Señale la conducta obstétrica correcta:

a) Permitir la evolución espontánea evitando la ayuda manual durante el período expulsivo. b) Dejar evolucionar hasta período expulsivo y practicar maniobras de ayuda manual para abreviar el período expulsivo tras la salida del ángulo inferior de la escápula. c) Permitir la evolución hasta período expulsivo y finalizar éste mediante aplicación de fórceps para evitar la distocia de cabeza última. d) Terminar el parto mediante cesárea.

432: En un registro cardiotocográfico intraparto aparecen dips 11 repetidos en relación con todas las contracciones, seguidos de una bradicardia fetal moderada. La paciente está recibiendo estimulación oxitócica i.v. y se registran 3 contracciones intensas cada 10 minutos. La exploración revela presentación cefálica (variedad O.I.I.A.) en II plano y una dilatación cervical de 6 cm. ¿Cuál es la conducta obstétrica adecuada en esta situación?:

a) Retirar la estimulación oxitócica. b) Proceder a la extracción fetal inmediata mediante cesárea. c) Realizar una microtoma de sangre fetal para determinar el pH. d) Administrar dosis altas de beta-miméticos.

433: Se hospitaliza a una gestante de 35 semanas por presentar presión arterial de 165/100 mmHg en repetidas tomas, edemas progresivos y proteinuria significativa. Se instaura tratamiento con hidralazina y sulfato de magnesio. Seis horas después, la paciente refiere cefalea intensa, epigastralgia y fotofobia. Se objetiva oliguria, presión arterial de 180/120 mmHg, aumento de los edemas, feto en presentación cefálica con cérvix formado y cerrado. La monitorización fetal no estresante, presenta una línea de base de 140 latidos/minuto, ritmo silente, ausencia de aceleraciones de la frecuencia cardíaca fetal y desaceleraciones tardías con cada contracción. ¿Cuál es la conducta correcta?:

a) Realizar un perfil biofísico fetal. b) Aumentar las dosis de antihipertensivos y anticonvulsivantes hasta controlar el cuadro y realizar una inducción del parto. c) Determinar la madurez pulmonar fetal mediante estudio del líquido arnniótico. d) Finalizar la gestación mediante cesárea.

434: Ante una mujer con un adenocarcinoma de endometrio en estadio II, ¿cuál de las siguientes opciones terapéuticas es la adecuada?:

a) Histerectomía simple. b) Histerectomía simple más doble anexectomía. c) Histerectomía radical tipo Wertheim-Meigs. d) Radioterapia y quimioterapia.

435: ¿Cuál de los siguientes postulados es FALSO?

a) La presencia de una cardiopatía congénita puede suponer un problema para una mujer embarazada. b) La cesárea disminuye la mortalidad de la mujer embarazada con estenosis pulmonar. c) Un buen método de control de la natalidad para la mujer con estenosis pulmonar es el de barrera. d) Con control médico adecuado, el parto vaginal es mejor en la mujer embarazada con estenosis pulmonar.

436: Una muchacha de 14 años, de tipo asténico, que juega de pivot en un equipo de baloncesto y calza un número 42, viene sufriendo de esguinces repetitivos en sus muñecas y tobillos. Hace algunos días sus familiares han descubierto que tiene la espalda "algo torcida" hacia un lado, y la escápula derecha algo más elevada que la izquierda. Últimamente nota que se fatiga mucho y palpitaciones que se incrementan excesivamente a los pocos minutos de estar jugando. También ha apreciado que está perdiendo agudeza visual. No refiere ningún otro antecedente. ¿Cuál de los siguientes es el diagnóstico más probable? :

a) Aracnodactilia. b) Gigantismo acromegálico. c) Acondroplasia. . d) Ostegénesis imperfecta

437: En un niño que padece una diarrea mucoliemorrágica, se comprueba la etiología por E. coli 0157 H7. ¿Cuál de las complicaciones que se enumeran está especialmente ligada a este germen?:

a) Síndrome hemolítico-urémico. b) Shock irreversible. c) Hemorragia subdural. d) Crisis febril aguda.

438: Señale cuál de las siguientes afirmaciones referentes al reflujo vesico-ureteral en el niño NO es correcta:

a) Se encuentra en el 30% de los niños con infección urinaria. b) Tiene tendencia a la "curación" espontánea. c) Tiene gran trascendencia como factor predisponente a la infección renal y pielonefritis crónica. d) Cuando se diagnostica antes de los 5 años la indicación quirúrgica es la norma.

439: La aparición de una metabemoglobinemia es el lactante pequeño puede estar en relación con la introducción precoz en la alimentación de:

Page 117: Cuestionario de Urología

a) Pollo. b) Espinacas. c) Gluten. d) Yema de huevo.

440: ¿Cuál de las siguientes medidas NO es eficaz para el tratamiento de la hipogalactia?:

a) Corregir los errores de técnica que hayan podido producirse. b) Administrar a la madre fármacos hiperprolactinemiantes. c) Aumentar la ingesta de líquidos en la dieta materna. d) Ofrecer el pecho al niño cada 4-6 horas.

441: Recién nacido de 24 días de vida, que desprendió el cordón umbilical a los 8 días y cuyo ombligo no ha cicatrizado. Presenta a su nivel una tumoración roja, prominente, circular y que segrega un contenido alcalino. La madre a veces nota en ella ruido de gases. El diagnóstico más probable será:

a) Persistencia del conducto onfalomesentérico. b) Fístula vésico-umbilical. c) Granuloma umbilical. d) Onfalocele.

442: Señale cuál de las siguientes circunstancias NO es contraindicación para administrar vacuna antipoliomielítica con virus vivos atenuados (Sabin):

a) Inmunodeficiencias congénitas (humorales y celulares). b) Niños con SillA. c) Prematuros hospitalizados. d) Niños alimentados con lactancia materna exclusiva.

443: Un niño de 18 meses, con retraso en el desarrollo estaturoponderal y polidipsia, presenta la siguiente analítica en sangre: Na 131 mEq/L; K 3,2 mEq/L; Cl 115 mEq/L; pH 7,24; CO3H 12 mEq/L; pCO2 28 mEq/L Y en orina: Na 29 mEq/L; K 38 mEq/L; pH 7. ¿Cuál, de los siguientes, es el diagnóstico más probable?:

a) Acidemia orgánica. b) Acidosis tubular distal. c) Síndrome de Fanconi. d) Síndrome de Bartter.

444: Un lactante de 6 meses, presenta una deshidratación del 10%, con la siguiente analítica en sangre: Na 159 mEq/L; K 4,5 mEq/L; Cl 116 mEq/L; pH 7,34; CO3H 20 mEq/L; pCO2 38 mEq/L Y en orina: Na 25 mEq/L; K 33 mEq/L; Osm 90 mOsm/L; pH d) ¿Cuál de los siguientes es el diagnóstico más probable?:

a) Acidosis tubular proximal. b) Acidosis tubular distal. c) Síndrome de Fanconi. d) Diabetes insípida.

445: Señale qué afirmación de las siguientes es INCORRECTA respecto al retraso del crecimiento de origen hipofisario:

a) Se acompaña frecuentemente de crisis hiperglucémicas. b) La edad talla suele ser menor que la edad ósea. c) En la disfunción neurosecretora de hormona de crecimiento, el pico de GH tras estimulación con clonidina puede ser normal. d) La pubertad está generalmente retrasada.

446: Señale cuál de las siguientes manifestaciones NO se encuentra en el síndrome hemolíticourémico del niño:

a) Trombocitopenia. b) Hipertensión arterial. c) Convulsiones. d) Insuficiencia hepática.

447: Señale cuál de las siguientes respuestas es INCORRECTA en relación con la poliglobulia del recién nacido:

a) El hematocrito medido en sangre capilar tiene que ser>66%. b) Se asocia con cianosis cutánea. c) Facilita las ictericias neonatales en la primera semana de vida. d) Puede ocasionar insuficiencia cardíaca.

448: Para determinar si la ingesta total de líquidos reduce el riesgo de padecer cáncer de vejiga urinaria, se estudió durante 10 años la frecuencia de este cáncer en un grupo de 47.999 hombres que no lo tenían al inicio del estudio. De todos ellos, se había obtenido información sobre el consumo de líquidos y se comparó la frecuencia de enfermedad según el volumen de líquidos con-sumidos, agrupado por quintiles. Entre los resultados, destacaba que la ingesta total de líquidos se asoció con el cáncer de vejiga, de forma que para el quintil superior (ingestión >2.531 mL de líquidos por día) se calculó un riesgo relativo de 0,51 (intervalos de confianza al 95 %: 0,32 a 0,80) al comparar con el quintil inferior (<1,290 mL por día). Señale la respuesta correcta:

a) Los autores han realizado un estudio de casos y controles. b) El valor de "p" obtenido a partir del estadístico de la comparación de las medidas de frecuencia de cáncer de vejiga entre los dos grupos de ingesta citados, debe ser mayor de 0.05. c) Se ha detectado un menor riesgo de padecer cáncer de vejiga en las personas que consumían mayor volumen de líquidos por día. d) El intervalo de confianza nos da una idea del grado de precisión estadística del estudio, de forma que cuanto más amplio, menos error aleatorio.

449: Se diseñó un ensayo clínico con asignación aleatoria en el que se probó si un determinado fármaco, respecto a un placebo, reducía la necesidad de intervención quirúrgica por hiperplasia benigna de próstata en pacientes con aumento del tamaño de la

Page 118: Cuestionario de Urología

próstata y síntomas urinarios moderados o graves. En el grupo de hombres tratados con el fármaco, se intervinieron un 5% y en el grupo tratado con placebo, un 10%. La reducción del riesgo con el fármaco fue del 50% (intervalos de confianza al 95%: 41 a 65%). Señale la respuesta correcta:

a) El número de personas que es necesario tratar con el fármaco para evitar una intervención quirúrgica es de 20. b) Por la amplitud de los intervalos de confianza se deduce que el estudio tiene una alta validez interna. c) La disminución absoluta del riesgo atribuible al fármaco es del 50%. d) Los resultados de este ensayo, en el caso de ser válidos, son generalizables a todos los pacientes con síntomas de obstrucción urinaria. .

450: Cuando en una investigación epidemiológica se trata de mejorar los instrumentos de medición, bien sea de la enfermedad, bien de la exposición, se está pretendiendo evitar:

a) Sesgos de la selección de sujetos. b) Errores aleatorios. c) Sesgo de pérdida de seguimiento. d) Sesgos de información.

451: Se diagnostica un cáncer de pulmón a un individuo asintomático al practicársele una Rx rutinaria de tórax y efectuar las oportunas comprobaciones. ¿Cuál de las siguientes afirmaciones es correcta?:

a) Probablemente el curso clínico será más prolongado que el que tienen los pacientes diagnosticados por presentar síntomas. b) Este diagnóstico precoz mejora sin ninguna duda el pronóstico. c) Si la afirmación anterior se comprueba en un gran número de pacientes similares al descrito, se puede afirmar la efectividad de la detección precoz. d) Los casos de cáncer detectados accidentalmente son .de crecimiento muy rápido.

452: Señale la afirmación correcta, de las siguientes, sobre la prevención y control de la hipertensión arterial (HTA):

a) El aumento de la actividad física es una medida de prevención primaria eficaz. b) Si en un hombre de 60 años se detecta por primera vez una TA de 160/95 mm Hg, debe iniciarse de inmediato el tratamiento farmaco1ógico. c) La restricción del consumo de potasio es una medida de prevención primaria eficaz. d) La HTA sólo es controlable mediante estrategias de prevención secundaria.

453: La probabilidad de padecer estenosis coronaria en hombres de más de 65 años, con angina de pecho de esfuerzo típica, es mayor del 90%. A un paciente de estas características se le practica un ECG de esfuerzo, que se informa como negativo. La sensibilidad es del 80% y su especificidad del 90%. ¿Cuál de las siguientes afirmaciones es correcta?:

a) Se puede descartar por completo estenosis coronaria porque la prueba es negativa. b) No se puede excluir la estenosis coronaria porque el valor predictivo positivo de la prueba es alto. c) No se puede excluir la estenosis coronaría porque el valor predictivo negativo de la prueba es bajo. d) Se podría descartar'e1 diagnóstico de estenosis coronaria si la especificidad de la prueba fuera de11O0%.

454: En un colegio mayor, 27 de los 100 alumnos que almorzaron en el comedor un día, desarrollan, unas horas después, un episodio de dolor cólico abdominal y diarrea, con duración e intensidad variable. Una de las siguientes actuaciones NO es adecuada para establecer el origen del brote. Señálela:

a) Hacer encuesta de los alimentos ingeridos a los 100 comensales. b) Hacer encuesta de los alimentos ingeridos sólo a los 27 afectos. c) Cultivar muestras de los restos de cada uno de los alimentos del menú. d) Practicar coprocultivos a los 27 afectos.

455: Un estudio para medir la prevalencia de un fenómeno relacionado con la salud en la población general es un:

a) Estudio de doble cohorte retrospectivo. b) Ensayo de campo. c) Estudio transversal. d) Ensayo de intervención poblacional.

456: ¿Cuál de los siguientes es un procedimiento para el control de factores de confusión en estudios epidemiológicos?:

a) Prueba de la chi cuadrado. b) Apareamiento ("matching"). c) Análisis de la varianza. d) Análisis por intención de tratar.

457: ¿Cuál de los siguientes es un criterio de causalidad en los estudios epidemiológicos?:

a) Significación estadística. b) Potencia estadística. c) Precisión del estimador de efecto. d) Fuerza de asociación.

458: En un ensayo clínico se comparan 3 tratamientos (p.e. placebo, tratamiento establecido y un tratamiento nuevo). La variable respuesta es continua (p.e. nivel de glucosa en sangre). Aceptando que la variable tiene una distribución normal, la prueba correcta para comparar la respuesta es:

a) La "t" de Student. b) La de Wilcoxon. c) El análisis de la varianza. d) La de Kruskal-Wallis.

459: La realización de cierta prueba para el diagnóstico de determinada patología arroja los siguientes datos:

Enfermedad

Page 119: Cuestionario de Urología

Señale la afirmación correcta:

a) La sensibilidad de la prueba es de a/(a+b). b) La prevalencia de la enfermedad es (a+b)/(a+b+c+d). c) El valor predictivo positivo es a/(a+c). d) El odds post-prueba es igual al odds preprueba multiplicado por (a/(a+c)) / (l-(d/b+d)).

460: Señale la afirmación correcta sobre la epidemiología y prevención del cáncer:

a) Para todos los grupos de edad, el cáncer es en España, la quinta causa de mortalidad. b) La prevención del cáncer de mama se basa casi exclusivamente en la prevención primaria. c) El hábito de fumar aumentar el riesgo de padecer cáncer de pulmón y de vejiga. d) El cáncer de pulmón es, desde 1995, el más frecuente en España en hombres yen mujeres.

461: Un ensayo clínico aleatorio evalúa la eficacia de un nuevo hipolipemiante en la prevención secundaria del infarto agudo de miocardio (IAM), comparándolo con placebo. Se observa que el riesgo relativo (RR) de IAM del grupo que ha recibido el tratamiento es 0,90 (intervalo de confianza del 95%: 0,75-1,05). ¿Cuál de las siguientes afirmaciones es cierta?:

a) Debe recomendarse la utilización del hipolipemiante, ya que reduce de forma significativa la incidencia de IAM. b) El tratamiento es más eficaz que el placebo en la prevención secundaria del IAM, ya que el RR es menor que 1. c) El resultado es estadísticamente significativo, ya que el intervalo de confianza excluye el valor RR=0. d) El resultado no es estadísticamente significativo.

462: Cuando en un contraste de hipótesis NO se rechaza la hipótesis nula. ¿Cuál de las siguientes afirmaciones es correcta?:

a) Se ha demostrado que la hipótesis nula es verdadera. b) Se ha demostrado que la hipótesis nula es falsa. c) Se ha demostrado que la hipótesis alternativa es verdadera. d) No se ha demostrado nada.

463: Una mujer de 17 años ha sido diagnosticada recientemente de lupus eritematoso sistémico grave y puesta en tratamiento con esteroides a dosis elevadas. La radiografia de tórax es normal y el Mantoux de 12x15 mm de induración. ¿Qué actitud tomaría?:

a) Efectuar quimioprofilaxis con isoniacida durante 6 meses. b) Esperar 2 años para realizar quimioprofilaxis. c) No utilizar quimioprofilaxis. d) Tratar con isoniacida más rifampicina.

464: ¿En cuál de las siguientes situaciones es más probable que podamos encontrar una reacción negativa a la tuberculina?:

a) Niño de 2 años, inmunocompetente, al que se vacunó con BCG al nacer. b) Mujer de 25 años que consulta por esterilidad y a quien se diagnostica tuberculosis genital. c) Hombre de 45 años con lesiones pulmonares cavitarias y baciloscopia y cultivo positivos para M. tuberculosis. d) Hombre de 70 años, tuberculoso antiguo, muy grave, con un patrón miliar en Rx de tórax y con hemocultivo positivo para M. tuberculosis.

465: Un varón de 76 años, fumador, sin otros antecedentes significativos, consulta por un cuadro de malestar general, disnea y dolores óseos generalizados. El cuadro se ha desarrollado de forma gradual en los últimos 2 meses. La exploración fisica no aporta datos significativos. Hemograma: leucocitos 6.500 (69% neutrófilos, 1% cayados, 22% linfocitos, 5% monocitos, 2% eosinófilos, 1% basófIlos), hemoglobina 8,2 g/dL, VCM 90 fl, plaquetas 115.000 VSG 120. En el autoanalizador, destaca una urea de 150 mg/dL (normal <60), creatinina 2.8 mg/dL (n<1.5), proteínas totales 8.5 g/dL (n 5.5 - 8), y una albúmina 2.3 g/dL (n 3.5 - 5.5). Las pruebas de función hepática son normales. El ionograma muestra Na 135 mEq/L, K 5.3 mEq/, Cl 101 mEq/L, Ca 10.8 mg/dL. En la orina elemental se observa una proteinuria de 4.0 g/L. La placa de tórax muestra hiperinsuflación pulmonar, aumento de los hilios pulmonares, que se interpreta como secundario a hipertensión pulmonar, y osteoporosis vertebral. ¿Cuál sería su propuesta?:

a) Solicitar un proteinograma y una serie ósea para completar el estudio. b) Solicitar una biopsia de arteria temporal e iniciar tratamiento con esteroides. c) Solicitar una citología de esputo y un estudio de sangre oculta en heces. d) Remitir al enfermo a un Servicio de Nefrología.

466: ¿Cuál de los siguientes motivos es de más peso para decidir tratar a un enfermo de una determinada enfermedad con un fármaco?:

a) Saber que el mecanismo de acción del fármaco es adecuado para la fisiopatología de la enfermedad. b) Leer que los expertos en el tema lo han empleado y comunican que los resultados son excelentes. c) Haberlo empleado en otros tres casos previos consecutivos, portadores de la misma enfermedad, con resultados excelentes. d) Leer un ensayo clínico en el que la probabilidad de obte ner un buen resultado sea mayor que con los tratamientos previos.

467: Para estudiar la utilidad de una nueva prueba diagnóstica para la enfermedad "E", se apli.ca a 100 enfermos con "E" y resulta positiva en 95 y negativa en 5 y, luego, a 100 sujetos sanos, sin "E", y resulta positiva en 3 y negativa en 97. ¿Qué afirmación de las siguientes es correcta?:

a) La sensibilidad de la prueba es 0.97, luego es muy buena para hacer diagnóstico precoz. b) La especificidad de la prueba es 0.95, luego es muy mala para confmnar diagnósticos de sospecha. c) Al no conocer la prevalencia de "E" en este medio, no se puede saber ni la sensibilidad ni la especificidad. d) La sensibilidad de la prueba es de 0.95 y su especificidad es de 0.97. Hay que comparar estas cifras con las de otras pruebas antes de introducir la nueva.

468: ¿Cuál de las siguientes sustancias puede provocar la formación de placas pleurales?:

Prueba Presente Ausente

Positiva A b

Negativa C d

Page 120: Cuestionario de Urología

a) Hierro. b) Estaño. c) Aluminio. d) Asbesto.

469: ¿Qué afirmación de las siguientes es correcta respecto a la vitamina D?:

a) Es transportada en el plasma por la garnmaglobulina. b) Aumenta la reabsorción tubular de calcio. c) Inhibe la absorción de fosforo en el intestino. d) Su déficit, aunque sea intenso, nunca produce síntomas del sistema nervioso central.

470: ¿Cuál de las siguientes circunstancias NO conduce a aumento del gasto cardíaco:

a) Disminución de la postcarga. b) Disminución de la frecuencia cardíaca. c) Aumento del estado inotrópico. d) Aumento de la precarga.

471: En condiciones fisiológicas, más de un 40% de la sangre de un adulto en reposo se encuentra en:

a) Pequeñas venas de la circulación sistémica. b) Grandes venas de la circulación mayor. c) El conjunto de la circulación pulmonar. d) Capilares y pequeñas arteriolas sistémicas.

472: En una espirometria, el volumen residual es igual a:

a) Volumen de cierre. b) Capacidad pulmonar total menos capacidad vital. c) Capacidad pulmonar total menos volumen de reserva inspiratorio menos volumen de reserva espiratorio. d) Capacidad residual funcional menos volumen corriente.

473: La Ley de Laplace explica razonablemente los fenómenos de la:

a) Contractilidad cardíaca. b) Circulación capilar. c) Postcarga cardíaca. d) Espiración.

474: ¿A cuál de las siguientes formas de transporte a través de la membrana pertenecen los canales iónicos?:

a) Transporte activo secundario. b) Difusión simple. c) Cotransporte. d) Difusión facilitada.

475: La secreción ácida gástrica es inhibida por la:

a) Gastrina. b) Enterogastrona. c) Acetilcolina. d) Distensión gástrica.

476: Cuando aumenta la secreción de jugo gástrico desde 0,5 mL/min hasta 3 mL/min, en sujetos jóvenes sanos, se observa que:

a) Aumenta la concentración de hidrogeniones y la de cloro. b) Aumenta la concentración de hidrogeniones y disminuye la de cloro. c) No se modifica la concentración de hidrogeniones ni la de cloro. d) Aumenta la concentración de sodio y la de potasio.

477: Señale cuál de los siguientes asertos es FALSO respecto de los antibióticos aminoglicósidos:

a) Son bactericidas y actúan inhibiendo la síntesis proteica. b) Su captación por los ribosomas bacterianos es un proceso activo que requiere consumo energético. c) La resistencia es debida generalmente a inactivación enzimática. d) Las bacterias resistentes a gentamicina son susceptibles a tobramicina.

478: El tratamiento de elección de la granulomatosis de Wegener es:

a) D-penicilamina. b) Esteroides a dosis altas. c) Ciclofosfamida. d) Azatioprina.

479: Paciente de 62 años, que ha sufrido un infarto de miocardio hace tres meses y que consulta por palpitaciones. En el estudio con monitorización eleco trocardiográfica (Holter) hay frecuentes sístoles prematuras ventriculares. ¿Cuál de las siguientes drogas anti-arrítmicas está demostrado que disminuye la potencial mortalidad en esta situación?:

a) Metropolol. b) Amiodarona. c) Lidocaina d) Ecainida

480: En un paciente diagnosticado de angina de pecho, se inicia tratamiento con mononitrato de isosorbide en presentación "retard", cuyo efecto dura 12 horas. Señale cuál deJas siguientes afirmaciones es correcta:

Page 121: Cuestionario de Urología

a) Debe administrarse una vez al día. b) Debe administrarse en caso de dolor. c) Debe administrarse dos veces al día. d) Debe administrarse cada 48 horas.

481: ¿Cuál de los signientes fármacos podría provocar una intoxicación digitálica al añadido al tratamiento de un paciente que recibe digoxina?:

a) Cloruro potásico. b) Tiroxina. c) Resincolestirarnina. d) Veraparnil

482: A pesar de una correcta protocolización, un paciente tratado con digoxina desarrolló una intoxicación digitálica. Los niveles plasmáticos de digoxina fueron 4 ng/mA) La función renal era normal y el t1/2 plasmática para la digoxina era de 1.6 días. ¿Cuántos días deberá suspenderse la administración de digoxina para alcanzar un nivel de 1 ng/mL?:

a) 1.6. b) 2.4. c) 3.2. d) 4.8.

483: Un paciente hipertenso consulta porque, tras iniciar tratamiento para su hipertensión, tiene tos pertinaz. ¿Cuál de las siguientes medicaciones cree que lo justificaría?:

a) Arnlodipino. b) Clortalidona. c) Clonidina. d) Enalapria.

484: Un síndrome muy grave, que aparece raramente y se caracteriza por la aparición de catatooía, inestabilidad del pulso, estupor e hipertermia maligna, puede aparecer tras la administración de:

a) Bromocriptina. b) Antidepresivos tricíclicos. c) Antitiroideos. d) Neurolépticos a dosis altas.

485: De entre las siguientes benzodiacepinas, señale la de acción más corta:

a) Diazepam. b) Clordiazepóxido. c) Lorazepam. d) Triazolam.

486: ¿Cuál de los siguientes fármacos constituye un tratamiento de fondo en el asma?:

a) Sulbutamol inhalado. b) Loratadina oral. c) Budesonida inhalada. d) Ipatropio inhalado.

487: La artritis reumatoide es más frecuente en familiares de personas que la padecen. Ello es debido a una:

a) Herencia autosómica dominante. b) Herencia autosómica recesiva. c) Asociación entre susceptibilidad a la enfermedad y de terminados alelos HLA de clase I. d) Asociación entre susceptibilidad a la enfermedad y de terminados alelos HLA de clase II.

488: Los anticuerpos antigliadina y antiendomisio son específicos de la enfermedad celíaca, especialmente si son de la clase:

a) IgG. b) IgM. c) IgA. d) IgD

489: En un episodio agudo de púrpura trombocitopénica idiopática con número muy bajo de plaquetas y riesgo grave de sangrado, el tratamiento de elección para elevar lo más rápidamente posible el número de plaquetas será:

a) Garnmaglobulina i.v. b) Corticosteroides a altas dosis por vía i.m. c) Corticosteroides a altas dosis por vía i.v. d) Inmunosupresores.

490: ¿Cuál de las siguientes determinaciones de anticuerpos es, de resultar positiva, más específica de lupus eritematoso sistémico?:

a) Antinucleares. b) Anti-RNP. c) Anti-SSA. d) Anti-Sm.

491: En un enfermo con un síndrome de Goodpasture que le ha conducido a una insuficiencia renal crónica:

a) No debe realizarse un trasplante renal, pues la enfermedad le dañará el riñón trasplantado. b) El trasplante puede realizarse si la producción de anticuerpos anti-membrana basal ha cesado.

Page 122: Cuestionario de Urología

c) Debe seguir siendo tratado indefinidamente con inmunosupresores para proteger su pulmón. d) Debe seguir siendo tratado indefinidamente con esteroides para proteger su pulmón.

492: ¿Cuál de las siguientes afirmaciones, relativas al tratamiento con gammaglobulina intravenosa, es correcta?:

a) Debe hacerse en sujetos con infecciones de repetición del aparato respiratorio. b) Debe administrarse a los sujetos que tengan unos niveles bajos de inmunoglobulinas en suero. c) Debe ser un tratamiento de mantenimiento en los enfermos con inmunodeficiencias humorales. d) Debe reservarse sólo a los enfermos con inmunodeficiencias humorales y, en éstos, emplearse sólo durante los períodos en que padezcan infecciones.

493: ¿En cuál de las siguientes enfermedades cutáneas aparecen auto-anticuerpos IgG contra la superficie de los queratinocitos?:

a) Pénfigo vulgar. b) Penfigoide. c) Dermatitis herpetiforme. d) Dermatitis atópica.

494: Señale qué afirmación, entre las siguientes, relativas a la anatomía quirúrgica de la glándula tiroides es correcta:

a) La arteria tiroidea inferior no debe cortarse nunca en las operaciones del tiroides. b) El nervio recurrente laríngeo inerva todos los músculos intrínsecos de la laringe. c) La posición de las glándulas paratiroides es muy constante. d) Las paratiroides tienen poco riesgo de lesionarse en la tiroidectomía subtota!.

495: Señale qué afirmación de las siguientes, relativas a los bloqueos nerviosos terapéuticos y sus bases anatómicas, es INCORRECTA:

a) El bloqueo del plexo cervical se hace inyectando a lo largo del borde posterior del esternocleidomastoideo. b) El bloqueo del frénico se hace inyectando a lo largo del borde anterior del esternocleidomastoideo. . c) La compresión del nervio frénico produce parálisis duradera reversible. d) El bloqueo supraclavicular del plexo braquial se hace inyectando por encima del punto medio de la clavícula.

496: Señale cuál de las siguientes estructuras NO está contenida en el oído medio:

a) Huesos martillo, yunque y estribo. b) Rama timpánica del V par craneal. c) Músculos estapedio y tensor del tímpano. d) Nervio cuerda del tímpano, rama del VII par.

497: Señale qué afirmación de las siguientes, relativas a la anatomía del pulmón, es correcta:

a) La anatomía normal de los bronquios no influye en la localización de las lesiones por aspiración. b) La división de los pulmones en segmentos no tiene ninguna aplicación en radiología. c) La división de los pulmones en segmentos no tiene ninguna aplicación en la práctica de la cirugía. d) Los bronquios plincipales acompañan a las arterias pulmonares en los hilios pulmonares.

498: Señale a cuál de las siguientes estructuras NO se extienden habitualmente las infecciones de las cavidades nasales:

a) Fosa craneal anterior. b) Fosa craneal media. c) Tejidos blandos de la retrofaringe. d) Oído medio.

499: Señale qué afirmación de las siguientes, relativas a la anatomía funcional de las costillas, es correcta:

a) Su punto más débil para fracturarse es el extremo próximo al esternón. b) La primera costilla es, generalmente, la más delgada de todas. c) La presencia de costillas cervicales puede producir síntomas neurológicas. d) La presencia de costillas cervicales nunca produce síntomas vasculares.

500: ¿Cuál de estos parámetros NO se utiliza en la clasificación de Child-Pugh para graduar la severidad de la hepatopatía crónica?:

a) Encefalopatía presente o previa. b) Ascitis presente o previa. c) Nivel de bililTubina sérica. d) Sangrado por varices presente o previo.

501: Si un paciente de 60 años, con antecedentes de tabaquismo, presenta una imagen de aspecto nodular visible en la radiografía a unos 3 cm de la carina traqueal en el pulmón derecho, la actitud más correcta sería:

a) Hacer una punción percutánea con aguja guiada por TAC. b) Hacer una broncofibroscopia. c) Prescribir tratamiento antiinflamatorio y hacer broncofibroscopia si la imagen no desaparece. d) Hacer broncofibroscopia sólo si tiene esputos hemoptoicoso d) Vigilar la evolución con radiografías cada seis meses, y hacer broncoscopia si crece el nódulo.

502: Paciente de 25 años, con historia de 3 meses de evolución de cansancio progresivo, adelgazamiento, diarrea episódica y febrícula. T.A. 100/60 mmHg, P.A. 98 Ipm, Mantoux +++. Rx tórax: infiltrado en segmento posterior de lóbulo superior derecho. Inicia tratamiento tuberculostático con 3 drogas (hidrazida, rifampicilla y etanbutol). Tras dos semanas de tratamiento, el paciente refiere aumento del cansancio, mayor frecuencia de las diarreas y, además, vómitos frecuentes. T.A. 90/60 en decúbito, 70/50 en bipedestación. P.A. 110 Ipm "mollis el parvus". Niega fiebre y síntomas respiratorios. ¿Cuál de las siguientes circunstancias es, con más probabilidad, la causa del cuadro actual?:

a) Error en el diagnóstico inicial. b) Generalización del foco pulmonar inicial.

Page 123: Cuestionario de Urología

c) Desarrollo de hepatitis tóxica por hidracidas. d) Presencia de insuficiencia suprarrenal desenmascarada por rifampicina.

503: ¿Cuál de las siguientes afirmaciones es cierta en relación con la hipertensión sistólica aislada?:

a) Se define como una presión arterial sistólica mayor o igual a 165 y diastólica menor de 95 mmHg. b) Comporta un riesgo cardiovascular menor que la hipertensión diastólica. c) Es el tipo de hipertensión más frecuente en la edad media de la vida. d) Se asocia frecuentemente a hipotensión ortostática.

504: Todas las enfermedades siguientes, EXCEPTO una, se han relacionado con un agente infeccioso. Señálela:

a) Angiodisplasia de colon. b) Sarcoma de Kaposi del inmunodeprimido. c) Linfoma MALT gástrico. d) Úlcera péptica.

505: Señale la frase más adecuada de las siguientes sobre el diagnóstico de la esclerosis múltiple:

a) Requiere un escáner cerebral. b) Requiere una resonancia magnética cerebral. c) Precisa realizar potenciales evocados visuales. d) El diagnóstico se puede realizar con la clínica.

506: Respecto al tratamiento del Lupus eritematoso sistémico señale, entre las siguientes, la respuesta INCORRECTA:

a) Se deben utilizar glucocorticoides a dosis altas y fármacos citotóxicos, en los casos de afectación orgánica severa potencialmente reversible. b) Los fármacos antipalúdicos son eficaces para el control de formas leves moderadas de la enfermedad, siendo re-comendable vigilar periódicamente la posible toxicidad retiniana. c) En períodos de inactividad de la enfermedad, es posible prescindir del tratamiento, aunque las remisiones completas son raras. d) En caso de insuficiencia renal terminal deben recibir tratamiento con diálisis, estando contraindicado el transplante renal de cadáver.

507: Ante una sospecha de enfermedad renal poliquística del adulto, el paso diagnóstico siguiente más razonable, entre los que se señalan, es:

a) Tomografía. b) Urografía intravenosa. c) Ecografía. d) Tomografía axial (TAC)

508: Señale qué afirmación, entre las siguientes, relativas a los cuerpos extraños en faringe es correcta:

a) Si son afilados, pueden lesionar el nervio laríngeo interno. b) En los niños, es excepcional que lleguen a estómago y se eliminen. c) No se suelen poder extraer por inspección directa con faringoscopio. d) La radiología sólo excepcionalmente es de utilidad en su manejo.

509: Una mujer de 73 años acude de urgencia por presentar pérdida de visióu aguda y severa en ojo izqnierdo, debido a una neuropatía óptica isquémica anterior no arterítica. Si iluminamos con una linterna su ojo derecho, ¿qué ocurrirá con las pupilas cuando se pase a iluminar el ojo izquierdo?:

a) La pupila derecha se dilatará y la izquierda no se modificará. b) Se producirá una miosis bilateral. c) No se producirán modificaciones porque el ojo izquierdo está prácticamente ciego. . d) Se dilatarán ambas pupilas simétricamente.

510: Un patrón en la manometría esofágica que muestre un aumento de la presión basal del esfínter esofágico inferjor (EEI) junto a una disminución o ausencia de su relajáción con la deglución es sugerente de:

a) Espasmo esofágico difuso. b) Acalasia. c) Esclerodermia con afectación esofágica. d) Enfermedad por reflujo gastroesofágico.

511: Tras realizar una endoscopia digestiva a un paciente de 51 años, se nos informa que el diagnóstico de sospecha es "esófago de Barrett". ¿Cuál es, entre las siguientes, la conducta más adecuada?:

a) El paciente debe ser remitido al Servicio de Cirugía para realizar una funduplicatura. b) La indicación correcta es la esofaguectornía. c) Lo más adecuado, para tener seguridad diagnóstica completa, es esperar al resultado de la biopsia antes de tomar decisiones. d) Debemos pautar tratamiento médico con Omeprazol de por vida y no aconsejar más revisiones.

512: De los siguientes, señale el que se considera factor etiológico de la gastritis tipo B (no autoinmune):

a) Hipersecreción de ácido. b) Hipersecreción de gastrina pancreática. c) Tratamiento con antinflamatorios no esteroideos. d) Infección por Helicobacter pylori.

513: Varón de 29 años, sexualmente activo (6 parejas en los últimos 18 meses), bebedor ocasional, no usa drogas ni fármacos, sólo paracetamol ocasionalmente. Consulta porque ha tenido un cuadro gripal durante el que se ha observado color amarillo en los ojos, orinas algo oscuras mientras tuvo fiebre y heces normales. Se comprueba leve ictericia escleral, resto de exploración normal.

Page 124: Cuestionario de Urología

Analítica general y hepática normal salvo bilirrubina total 3,1 mg/dl (directa 0,7 mg/dl). ¿Qué prueba tendrá entre las siguientes, probablemente mayor rendimiento diagnóstico?:

a) Test de detección de VIR. b) Test de ayuno. c) Biopsia hepática. d) Ultrasonografía hepática.

514: Paciente cirrótico de larga evolución que acude a urgencias por aumento del perímetro abdominal y dolor abdominal difuso. La paracentesis diagnóstica da salida a un líquido con 600 células/rnm3 con 80% de polimorfonucleares y 0,795 de proteínas/dI. ¿Cuál de estas medidas es la más adecuada en esta situación?:

a) Realizar una paracentesis evacuadora total con reposición del albúmina. b) Iniciar tratamiento con isoniacida, rifampicina, piracina mida y etambutol a las dosis habituales. c) Enviar una muestra de líquido ascítico al laboratorio de microbiología y no iniciar tratamiento hasta conocer el resultado. d) Iniciar de inmediato tratamiento con cefotaxima a dosis de 2 g. i.v. cada 6 u 8 horas.

515: Una mujer de 47 años que tomó anovulatorios durante 3 años, hace 15 años, no bebedora, desarrolló a lo largo de los últimos meses un cuadro de astenia, prurito ocasional y leve ictericia. El estudio muestra bilirrubinemia de 2,7 mgldl, AST 72 u.i., ALT 85 u.i., fosfatasa alcalina triple del límite superior de la normalidad y garnma glutamil transpeptidasa quíntuple. La ecografia abdominal es normal. Tras confirmar su diagnóstico de sospecha mediante el pertinente estudio, ¿cuál sería el tratamiento?:

a) Ácido ursodesoxicólico. b) Prednisona. c) Azatioprina. d) Papilotornía transendoscópica.

516: El tratamiento de la hepatitis C crónica se basa en la administración de interferón alfa, pero no todos los pacientes responden al tratamiento. Indique cuál de los siguientes patrones se asocia más estrechamente a una respuesta favorable:

a) Genotipo 3, viremia baja, ausencia de cirrosis. b) Genotipo 1b, viremia elevada, ausencia de cirrosis. c) Genotipo 2, viremia elevada, presencia de cirrosis. d) Genotipo lb, viremia baja, ausencia de cirrosis.

517: Paciente varón de 30 años afecto de una artritis reumatoide y una colestasis moderada asintomática de años de evolución, acude al hospital por una hemorragia digestiva por varices esofágicas. La analítica a su ingreso muestra únicamente una discreta colestasis, siendo la bilirrubina, transaminasas, pruebas de coagulación, alfafetoproteína, y antígeno carcinoembrionario normales. Como parte del estudio diagnóstico se realiza: ecografía abdominal (hígado nodular de bordes abollonados, vena porta dilatada permeable), biopsia hepática (ausencia de fibrosis o nódulos de regeneración, hepatocitos normales dispuestos en trabéculas de 2-3 células). Frente a este cuadro clínico el diagnóstico más probable es:

a) Cavemomatosis portal. b) Cirrosis hepática. c) Metástasis hepáticas de un carcinoma de colon. d) Hiperplasia nodular regenerativa.

518: En relación a un paciente diagnosticado de una úlcera duodenal no complicada mediante endoscopia, señalar la respuesta FALSA:

a) Se debe investigar si está infectado por Helicobacterpylori y en caso positivo tratar la infección. b) Si se trata la infección por Helicobacter pylori, una vez confirmada la erradicación es recomendable mantener un tratamiento con antisecretores gástrico s a dosis bajas para evitar una recidiva ulcerosa. c) Durante la endoscopia no es necesario biopsiar sus bordes para descartar malignidad. d) El riesgo de recidiva u1cerosa si se logra la erradicación de Helicobacter pylori es menor del 20%.

519: Un paciente con antecedentes de colitis ulcerosa estable, acude a Urgencias por fiebre, diarrea intensa con rectorragia y dolor abdominal. La rectosigmoidoscopia demuestra úlceras y exudado purulento y hemorrágico. El enema muestra afectación severa de todo el colon. Se inicia el tratamiento con esteroides y antibióticoso 48 horas después el paciente presenta hipotensión, taquicardia e intenso dolor abdominal. Se realiza placa de abdomen que muestra dilatación de 6 cm del colon. ¿Cuál sería la impresión diagnóstica y el tratamiento adecuado?:

a) Se trata de una colitis pseudomembranosa asociada. Medidas de soporte y tratamiento con Metronidazol o Vancomicina. b) Se trata de una perforación de colon. Tratamiento quirúrgico con colectomía urgente. c) Brote muy severo de colitis u1cerosa. Medidas de soporte, intensificación de tratamiento esteroideo y antibióticos de amplio espectro. d) Se trata de un megacolon tóxico. Tratamiento intensivo y colectomía si no mejora en 24 horas.

520: En algunos casos, resulta imposible distinguir entre colitis ulcerosa y enfermedad de Crohn. Todos los datos expuestos, EXCEPTO uno, son más característicos de enfermedad de Crohn que de colitis ulcerosa. Señálelo:

a) Mectación segmentaria. b) Presencia de granulomas. c) Sangrado rectal. d) Fístulas perianales.

521: Una paciente de 22 años con 3 brotes previos de colitis ulcerosa con buena respuesta a esteroides y 5-ASA; actualmente asintomática y en tratamiento con 5-ASA, le solicita consejo para quedarse embarazada. Indique al respecto, entre las siguientes, la respuesta correcta:

a) Debe desaconsejarse el embarazo, ya que produce exacerbaciones severas de la enfermedad. b) No se puede utilizar 5-ASA por sus efectos teratogénicoso. c) El embarazo produce un "efecto protector" sobre los brotes, observándose una disminución estadísticamente significativa de los mismos durante la gestación. d) La mitad de las pacientes embarazadas sufrirán un brote de colitis, aunque pueden utilizarse 5-ASA y esteroides con

Page 125: Cuestionario de Urología

seguridad.

522: ¿Cuál, entre los siguientes, es el tratamiento más adecuado de la obstrucción intestinal no quirúrgica en la fase terminal de la enfermedad?

a) Sonda nasogástrica, aspiración continua, sueroterapia intravenosa. b) Administración de vitaminas grupo B y laxantes. c) Alimentación Parenteral total. d) Administración por vía subcutánea de: morfina, buscapina y haloperidollas 24 horas.

523: Cuando en el curso de una colonoscopia en un paciente aparentemente sano se objetiva un pólipo, ¿qué es lo más adecuado?:

a) Biopsiar el pólipo. b) Tomar una citología exfoliativa. c) Resecarlo con asa de polipectomía. d) Citarlo al cabo de unos días para resecarlo.

524: Paciente de 60 años de edad, bebedor de 60 g. de alcohol al día, que consulta por padecer desde hace 3 días un dolor epigástrico que cede al ventosear, dolores articulares, 4-6 deposiciones diarreicas al día sin sangre visible, fiebre de 38° y aftas bucales y anemia microcítica. ¿Cuál, entre los siguientes, es el diagnóstico más adecuado?:

a) Pancreatitis crónica. b) Enfermedad de Crohn. c) Colitis seudomembranosa. d) Hepatitis alcohólica.

525: Señale cuál es la técnica quirúrgica indicada, en un paciente con adenocarcinoma ductal de páncreas, situado en la cabeza pancreática y potencialmente resecable:

a) Duodenopancreatectomía cefálica. b) Pancreatoyeyunostomía lateral. c) Resección córporo-caudal del páncreas. d) Gastro-yeyunostomía.

526: La anatomía patológica de un varón de 45 años, apendicectomizado, es informada como: Tumor carcinoide apendicular de 1 cm de diámetro, que llega hasta la submucosa, localizado a nivel de la punta del apéndice. ¿Cuál es la actitud a seguir ante este enfermo? :

a) Reoperarle y realizarle una hemicolectomía derecha. b) Revisiones periódicas. c) Tratamiento quimioterápico. d) Tratamiento radioterápico.

527: Respecto al carcinoma gástrico señala la FALSA:

a) Ha aumentado su incidencia global en los últimos años en los países occidentales. b) La anemia perniciosa es un factor predisponente. c) Después de gastrectomía subtotal por enfermedad benigna aumenta el riesgo de padecer cáncer gástrico. d) La colonización por Helicobacter pylori es un factor de riesgo.

528: ¿Cuál es la causa más frecuente de obstrucción del intestino delgado en un paciente adulto, sin intervenciones abdominales previas?

a) neo biliar. b) Cáncer de colon. c) Tumoración estenosante de intestino. d) Incarceración de intestino delgado en orificio hemiario.

529: Ante un paciente de 70 años con alteraciones del tránsito intestinal, pérdida de 5 kg. de peso en los dos últimos meses y rectorragias, la exploración de elección es:

a) Gastroscopia. b) TAC abdominal c) Prueba de sangre oculta en heces. d) Colonoscopia.

530: A un paciente de 70 años, colecistectomizado, con ictericia de 48 horas de evolución, Bilirrubina total de 8 mgldl y Bilirrubina directa de 6 mgldl, fosfatasa aIcalina 620 UI/L, fiebre de 39°C y leucocitosis mayor de 20000 con desv iación izda, se le realiza ecografía abdominal siendo informada como coledocolitiasis. El tratamiento inicial debe ser:

a) Reposición hidroelectrolítica y antibioterapia únicamente, posponiendo cualquier otro proceder a la desaparición de los síntomas y signos de la infección. b) Reposición hidroelectrolítica, antibioterapia y laparotomía urgente. c) Reposición hidroelectrolítica, antibioterapia y corticosteroides. d) Reposición hidroelectrolítica, antibioterapia y esfinterotomía + drenaje biliar mediante colangiografía retrógrada endoscópica.

531: En la pancreatitis aguda, en caso de duda diagnóstica por .su presentación clínica atípica, con una situación clínica de gravedad, donde el dolor abdominal no es característico o el aumento de amilasa o lipasa sérica ha constituido un hallazgo inesperado, ¿qué prueba diagnóstica es la indicada en primer lugar para confirmar o no este diagnóstico?:

a) Ecografía abdominal. b) Radiografías simples de abdomen, antero posterior y lateral. c) Tomografía axial computarizada abdominal. d) Colangiopacreatografía retrógrada endoscópica.

Page 126: Cuestionario de Urología

532: En un paciente diagnosticado de pancreatitis aguda, a los dos días de su evolución, ¿cuál de los siguientes datos carece de valor para pronosticar su carácter severo?:

a) Obesidad. b) Valores de arnilasa y/o lipasa sérica elevados más de 10 veces el lírnite normal. c) Tener unos valores de proteína C reactiva sérica superiores a 120 mg/dl. d) Presentar una puntuación APACHE II (Acute Physiology and Chronic Health Evaluation) superior a 8.

533: A un paciente con fibrosis pulmonar le ha sido trasplantado un pulmón hace 15 días. Desde hace 2 días presenta una infección pulmonar con cambios radiográficos. El neumólogo pensará que se trata de una infección por:

a) Citomegalovirus. b) Hongos. c) Bacterias. d) Protozoos.

534: Una paciente de 44 años con litiasis biliar, ingresa por urgencias a causa de un episodio de dolor abdominal alto, severo, irradiado a la espalda. Unos días después, se objetiva radiológicamente derrame pleural izquierdo; al punzarlo, resulta ser de aspecto serohemorrágico, con caracteres de exudado, con abundantes eosinófilos. ¿Qué estudio, de los siguientes, realizaría en primer lugar para establecer la causa del derrame? :

a) Biopsia pleural con aguja. b) TAC de tórax. c) Arnilasa sérica. d) Arnilasa en el líquido pleural

535: La causa de la disnea, en un paciente varón de 40 años que consulta por este síntoma, parece deberse a una neumonitis por hipersensibilidad. Apoyamos nuestro diagnóstico en los siguientes hechos EXCEPTO:

a) Exposición a antígenos potencialmente provocadores. b) Eosinofilia periférica y en esputo. c) Presencia de precipitinas séricas. d) Alteración de la difusión pulmonar.

536: Paciente de 24 años, fumador de 20 cigarrillos al día tratado con un trasplante aIogénico de médula ósea tras ser diagnosticado de leucemia aguda mieloblástica. La presencia de enfermedad de injerto contra el huésped obligó a un tratamiento con 30 mg de metilprednisolona al día y azatioprina 100 mg al día. Seis meses después del trasplante aparece tos no productiva y disnea de instauración rápidamente progresiva hasta hacerse de mínimos esfuerzos. En la auscultación respiratoria se evidencia una espiración alargada. En la radiografia de tórax existe una insuflación pulmonar bilateral. La espirometría evidencia una capacidad vital forzada (CVF) del 59% un volumen espiratoria máximo en el primer segundo (VEMS) del 36% y un cociente VEMS/FV C del 42 %. El lavado broncoalveolar muestra una neutrotilia del 62 %. ¿Cuál es del diagnóstico más probable?:

a) Enfisema centrolobulillar. b) Neumopatía intersticial por citostáticos. c) Bronquiolitis obliterante con neumonía organizada. d) Bronquiolitis obliterante.

537: ¿Cuál es la prevalencia de neumococos con sensibilidad disminuida a la penicilina en España?

a) 0-5% b) 90-100% c) 10-20% d) 30-40%

538: En la enfermedad pulmonar obstructiva incipiente originada en la pequeña vía aérea, la medida más sensible de las que recoge la espirometría es:

a) Capacidad vital forzada. b) Volumen espiratorio forzado por segundo. c) Flujo espiratorio forzado entre el 25 y el 75% de la capacidad vital. d) Relación entre volumen espiratorio forzado por segundo y capacidad vital forzada.

539: Paciente de 69 años, fumador de 30 cigarrillos/día desde hace 40 años, que a raíz de un episodio de hemoptisis se le practica una TAC torácica que demuestra la presencia de una masa perihiliar de 5 cm que provoca una atelectasia completa de lóbulo superior derecho y ausencia de adenopatías mediastínicas y de otras anomalías torácicas. La broncoscopia confirma la presencia de una masa de aspecto neoplásico en la entrada del lóbulo superior derecho situada a más de 2 cm de la carina principal cuyo diagnóstico histológico es de carcinoma escamoso. La espirometría forzada muestra una FVC de 4.200 (88%) y un VEMS de 2.400 (76%) con un cociente VEMS/FVC de 69%. ¿Cuál de las siguientes afirmaciones es correcta?:

a) Se trata de cáncer de pulmón en estadio III. b) El tratamiento de elección es la quimioterapia neoadyuvante y posterior cirugía de resección. c) La clasificación TNM que le corresponde es T2 NO MO. d) La presencia de una alteración ventilatoria obstructiva contraindica la resección quirúrgica del tumor.

540: Paciente de 50 años, diagnosticado de EPOC, en fase estable y que actualmente ha cesado en el hábito de fumar cigarrillos. Se le realiza una espirometría que muestra que el volumen espiratorio máximo en el primer segundo (VEMS) es del 60% del valor teórico. Un hemograma revela un hematocrito del 58 %. La gasometría arterial muestra una PaO2 de 75 mmHg. ¿Cuál de las siguientes exploraciones complementarias estaría indicada en primer lugar en este paciente?:

a) Prueba de provocación bronquial inespecífica con metacolina. b) Polisomnografía con registro de variables cardiorrespiratorias. c) Cateterismo pulmonar con catéter de Swan-Ganz. d) Determinación de las relaciones ventilación-perfusión pulmonares mediante técnica de eliminación de gases inertes múltiples.

541: En una radiografía postero-anterior de tórax practicada a un hombre de 44 años asintomático, se observa una masa redonda

Page 127: Cuestionario de Urología

de unos 3 cm de diámetro, de contornos nítidos en la parte medial del lóbulo inferior izquierdo que, aunque llega junto a la silueta cardíaca, tiene todo su contorno visible. ¿Cómo consideraría la citada masa?:

a) La masa es muy probablemente de origen pleural. b) Lo más probable es que se trate de derrame pleural en la CIsura. c) La masa es de localización posterior al corazón. d) La masa es muy probablemente de origen parietal.

542: Paciente de 56 años diagnosticado de carcinoma epidermoide de pulmón con metástasis óseas, hepáticas y pulmonares. Basalmente, su índice de Karnofsky es de 20 y está encarnado el 100% del día. Sigue tratamiento con cloruro mórfico 10 mg/4 h s.c., dexametasona 4 mg/8 h, s.c., haloperidol 2,5 mg/8 h s.c. y midazolan 7,5 mg s.c. por la noche. El paciente comienza con agitación psicomotriz progresiva. Se realiza analítica que evidencia calcemia de 13 mg/dl. No se produce mejoría tras administrar 2 dosis de 5 mg vía s.c. de haloperidoI. ¿Qué actuación propondría?:

a) Sedación con midazolan por vía subcutánea por agitación como evento terminal. b) Administración de clodronato para corregir hipercalcemia. c) Administración de calcitonina para corregir hipercalcemia. d) Continuar con la administración de 5 mg de haloperidol cada 4 h. Hasta alcanzar 30 mg.

543: Un varón de 70 años presenta, en una radiografía de tórax rutinaria, un nódulo pulmonar de 2,5 cm. De diámetro, no calcificado en posición posterior de LSD. ¿Qué actitud entre las siguientes es más correcta?:

a) No indicar más estudios ya que en un varón de 70 años es muy probable una antigua infección tuberculosa, que no precisa tratamiento. b) Indicar profilaxis con isoniacida debido a la elevada incidencia de la enfennedad tuberculosa. c) Indicar tratamiento con rifampicina, isoniacida y etambutol. d) Sospechar tumor maligno y solicitar un estudio citológico por punción.

544: Un hombre de 20 años consulta por dificultad respiratoria que se agudiza con los esfuerzos físicos la radiografía de tórax es anodina. El estudio gasométrico demuestra Hipercapnia y gradiente alveolo arterial de oxígeno normal. ¿Hacia cuál de las siguientes patologías orientará su diagnóstico?:

a) Enfennedad neuromuscular. b) Asma bronquial. c) Enfennedad intersticial pulmonar. d) Neumonitis por hipersensibilidad.

545: Una mujer de 50 años con artritis reumatoide, refiere tos seca y disnea progresiva. En la radiografía de tórax hay tenue afectación basal. La gasometría detecta Hipoxemia con pH y pCO2 normales. La espirometría demuestra reducción de la capacidad vital y del volumen residual. ¿Cuál es su sospecha diagnóstica?:

a) Embolismos pulmonares. b) Neumonía aspirativa. c) Broncoespasmo. d) Fibrosis pulmonar.

546: Un joven de 18 años, con antecedentes de alergia a pólenes y eczema, acude a Urgencias por dificultad respiratoria progresiva, con Broncoespasmo muy severo que obliga a actuación terapéutica inmediata. Señale de los siguientes datos observados, ¿cuál es el que MENOS nos indica la gravedad de la situación?:

a) Hipersinsuflación torácica. b) Sibilancias. c) Hipercapnia. d) Pulso paradójico.

547: Para la valoración de la fase evolutiva de una fibrosis pulmonar, señale cuál de las siguientes pruebas es la MENOS adecuada:

a) Radiografía de tórax. b) Scanner torácico de alta resolución. c) Garnmagrafía con Galio 67. d) Pruebas de función respiratoria.

548: Una mujer fumadora de 45 años, consulta por disnea progresiva. La gasometría detecta Hipoxemia corregible con oxígeno, hipocapnia y disminución de la capacidad de difusión del monóxido de carbono (Dlco). ¿Cuál de las siguientes patologías es la más probable?:

a) Proteinosis alveolar. b) Comunicación inter-ventricular. c) Hipertensión pulmonar. d) Neumonía obstructiva.

549: Una paciente de 40 años con antecedentes de asma hronquial acude al Servicio de Urgencias de un Hospital por presentar disnea de reposo de 24 horas de evolución; en la exploración física destaca Taquipnea a 32 respiraciones/minuto, sibilancias inspiratorias y espiraforjas; el hemograma y la radiografía det6rax son normales; en la gasometría arterial se observa un signo de gravedad que hace indicar el ingreso de la paciente en una Unidad de Cuidados Intensivos. ¿Cuál de entre los siguientes signos, le parece el más probable?:

a) Presión parcial de Oxígeno de 70 mmHg. b) Presión parcial de Dióxido de Carbono menor de 30 mmHg. c) Presión parcial de Oxígeno de 60 mmHg. d) Presión parcial de Dióxido de Carbono mayor de 40 mmHg.

550: Cuántos ganglios debe incluir una resección pulmonar por cáncer para poder afirmar que el estadio es N0:

a) Tres.

Page 128: Cuestionario de Urología

b) Uno mediastínico. c) Seis o más. d) Es indiferente el número. Sólo importa que sean negativos.

551: La taquicardia ventricular sostenida tiene especial mal pronóstico cuando aparece:

a) Sin cardiopatía. b) Sin síntomas hemodinámicos ni síncope. c) Tardíamente después de un infarto. d) Precozmente, después de un infarto, con mala fracción de eyección.

552: Un hombre de 48 años con trombosis de la vena femoral derecha, asociada con celulitis, que antes de empezar con la anticoagulación presenta dolor pleurítico de lado izquierdo, disnea y hemoptisis. ¿Cuál de las siguientes pruebas es la mejor para excluir el diagnóstico de embolismo pulmonar? :

a) Una garnmagrafía pulmonar de ventilación nonnal. b) Un electrocardiograma nonnal. c) Un gradiente P(A-a)02, igual o menor de 18 mmHg. d) Una garnmagrafía pulmonar de perfusión nonnal.

553: Mujer de 32 años que consulta por haher comenzado una hora antes con palpitaciones. Se realiza un ECG que muestra taquicardia regular de QRS estrecho a 180 l.p.m. Al aplicar masaje en el seno carotídeo se produce una disminución repentina de la frecuencia ventricular causada por la terminación de la taquicardia. ¿Qué tipo de arritmia padece esta paciente, con más probabilidad?:

a) Taquicardia sinusal. b) Fibrilación auricular. c) Taquicardia ventricular. d) Taquicardia por reentrada de nodo AV.

554: Señale cuál de los siguientes NO es considerado criterio Mayor de Framingham para el diagnós6co de Insuficiencia cardíaca:

a) Cardiomegalia. b) Reflujo Hepatoyugular. c) Disnea de esfuerzo. d) Galope por tercer tono.

555: Entre los efectos adversos frecuentes de los diuréticos se encuentran los siguientes, EXCEPTO uno:

a) Ginecomastia. b) Calambres musculares. c) Dislipemia secundaria. d) Anemia hemolítica.

556: Un paciente de 61 años presenta un dolor anginoso típico de 35 minutos de duración, siendo atendido en un Centro Extrahospitalario. TA 110/60. Pulso arterial 86 I.p.m., no signos de insuficiencia cardíaca. Todas las siguientes medidas pueden recomendarse EXCEPTO una:

a) Canalizar una vía periférica. b) Tratar el dolor con cloruro mórfico. c) Poner un antiarrítmico parental. d) Dar una aspirina oral.

557: Paciente de 60 años, que acude a urgencias con dolor retroesternal de inicio súbito e intenso, rasgante y sensación de muerte, que irradia a la espalda, que presenta hipertensión arterial y ECG normal. Sospecha en primer lugar:

a) Hernia de hiato con reflujo importante y esperaría al resultado de la gastroscopia. b) Infarto agudo de ventrículo derecho y realizaría ECG incluyendo derivaciones V3R y V4R. c) Disección de aorta y realizaría un ecocardiograma transesofágico y un TAC. d) Embolismo pulmonar izquierdo por irradiación y le daría heparina.

558: ¿Cuál de las siguientes formas de cardiopatía isquémica suele responder al tratamiento con diuréticos?:

a) Angina de Prinzmetal. b) Angina de decúbito. c) Angina de reciente comienzo. d) Angina posinfarto.

559: El objetivo terapéutico a alcanzar en un paciente en el que se inicia tratamiento con acenocumarol por haber presentado un segundo episodio de embolismo pulmonar es prolongar el tiempo de protrombina hasta alcanzar un INR (International Normalized Ratio) entre:

a) 1-2 b) 1,5-2 c) 2-3 d) 3-4

560: A una mujer de 53 años, asintomática, se le realiza una radiografía de tórax por haber sido diagnosticado su marido de tuberculosis pulmonar y tener Mantoux de 17 rom. La radiografia muestra cardiomegalia con dilatación de la arteria pulmonar y sus ramas y aumento de la trama vascular. En la auscultación cardíaca se encuentra un soplo sistólico eyectivo pulmonar con desdoblamiento amplio y fijo del segundo tono. El ECG muestra desviación del eje a la derecha con patrón rSr en precordiales derechas. Indique entre los 'siguientes, el diagnóstico más probable:

a) Comunicación interauricular. b) Estenosis mitral. c) Hipertensión pulmonar primaria.

Page 129: Cuestionario de Urología

d) Pericarditis tuberculosa.

561: Cuál de los siguientes, NO se considera un criterio mayor (criterios de Duke), en el diagnóstico de Endocarditis infecciosa:

a) Ecocardiograma positivo para Endocarditis. b) Nuevo soplo de regurgitación valvular. c) Hemocultivos positivos. d) Fiebre.

562: Hombre de 74 años, con historia de un mes de evolución con debilidad, fiebre y dolor en región lumbar. Dos años antes se le había implantado una válvula aórtica biológica por una estenosis aórtica calcificada. A la exploraciónfisica está algo confuso con T" de 37,9, TA de 110/80, pulso arterial de 88 I.p.m. y auscultación cardíaca con soplo sistólico III/IV en foco aórtico sin componente diastólico. El hemograma muestra 11300 leucocitos con 87% de PMN, Hb de 10,1 gr/dl, Hto de 32% y VSG de 55 mmlh. Los hemocultivos fueron positivos para Enterococcus faecalis y la ecocardiografia transesofágica mostró una verruga en la válvula aórtica. Tras 14 días de tratamiento con ampicilina y gentamicina el paciente continuaba con fiebre y el ECG mostró un PR de 0,26 segundos. ¿Qué actitud, de las siguientes, recomendaría? :

a) Sustituir la ampicilina por una cefalosporina de 3ª generación. b) Sustituir la gentamicina por Vancomicina. c) Añadir un antihistamínico por sospecha de fiebre medicamentosa. d) Realizar una nueva ecocardiografía transesofágica para descartar infección perivalvular.

563: Mujer de 81 años que consulta por disnea de 2 días de evolución. La exploración física y la RX de tórax muestra signos de insuficiencia cardíaca. El día previo a comenzar con los síntomas había tenido un episodio de dolor centrotorácico de 4 horas de duración. ¿La determinación de cuál de los siguientes marcadores cardíacos séricos sería más útil para confirmar el diagnóstico de infarto de miocardio? :

a) CPK. b) CPK-MB. c) Troponina T. d) LDH.

564: Mujer de 78 años diagnosticada de cardiopatía hipertensiva con función sistólica conservada, que en los últimos 2 años ha tenido 3 episodios de fibrilación paroxística cardiovertidos eléctricamente. Durante este tiempo ha recibido diversos tratamientos que incluían algunos de los siguientes fármacos: propafenona, amiodarona, digoxina, dialtiazem y captopril. Actualmente consulta por un cuadro de 2 meses de evolución de debilidad general y apatía, añadiéndose en la última semana disnea progresiva hasta ser de pequeños esfuerzos. El ECG muestra fibrilación auricular con frecuencia ventricular a 130 l.p.m., la RX de tórax cardiomegalia con signos de congestión pulmonar y el estudio de función tiroidea una T4 libre elevada con una TSH indetectable. ¿Cuál de los fármacos utilizados puede ser el responsable del cuadro que, actualmente, presenta la paciente? :

a) Propafenona. b) Amiodarona. c) Digoxina. d) Diltiazem.

565: Mujer de 76 años con historia de insuficiencia cardíaca por cardiopatía hipertensiva en fibrilación auricular crónica que seguía tratamiento con enalapril, digoxina, furosemida y acenocumarol. Consulta por presentar en la última semana náuseas e incremento de la disnea. La exploración muestra T A de 130/80 mmHg, pulso arterial de 116 I.p.m. rítmico; en la auscultación pulmonar se oyen crepitantes en las bases y en la auscultación cardíaca refuerzo del segundo tono. El ECG muestra una taquicardia rítmica de QRS estrecho a 116 Ipm. ¿Qué actitud entre las siguientes es la más adecuada?

a) Suspender anticoagulantes orales. b) Realizar monitorización de Holter. c) Solicitar niveles de digoxina. d) Asociar propanolol.

566: Un paciente refiere disnea de moderados esfuerzos y se le ausculta un primer tono fuerte, chasquido de apertura y soplo diastólico con refuerzo presistólico y en el ECG presenta ondas P con signos de crecimiento de la aurícula izquierda. El diagnóstico de presunción es:

a) Doble sesión mitral en ritmo sinusal. b) Estenosis mitral en fibrilación auricular, probablemente severa. c) Mixoma de auríctila izquierda d) Estenosis mitral en ritmo sinusal.

567: Señale la respuesta correcta en relación con los aneurismas de la aorta ascendente:

a) Sólo está indicada la cirugía cuando su tamaño sea igual o supere los 8 cm. de diámetro. b) La presencia de una insuficiencia valvular aórtica por alteración de la posición de las valvas por el crecimiento del aneurisma es incluso más grave que el propio crecimiento del aneurisma y causa de indicación quirúrgica. c) A veces se presenta una insuficiencia valvular aórtica secundaria al crecimiento aneurismático, pero suele ser poco importante y no es causa de indicación quirúrgica, d) Lo más importante además del tamaño de aneurisma, para hacer la indicación de cirugía es el contenido trombótico demostrado por un TAc.

568: Un paciente de cincuenta años, con buen estado general, nos refiere que es aficionado a correr, pero que lo ha dejado porque desde hace un mes y medio nota opresión precordial cuando lleva corridos uno o dos kilómetros. ¿Cuál es, entre las siguientes, la actitud correcta? :

a) Solicitar una prueba de esfuerzo y si se confirma la existencia de isquemia coronaria, indicar un tratamiento médico adecuado. b) Se trata de un caso leve de isquemia coronaria, dado que se presenta después de un considerable esfuerzo y por ello indicaremos tratamiento médico con betabloqueantes y/o inhibidores del calcio. c) Por tratarse de un caso de angina estable no creemos que está indicada la cirugía, por ello indicaremos tratamiento con aspirina,

Page 130: Cuestionario de Urología

d) Solicitar prueba de esfuerzo y si se confirma la existencia de isquemia coronaria, indicaremos la realización de una coronariografía, tras lo cual se realizará, si fuera preciso, un procedimiento revascularizador.

569: Señale, entre las siguientes, la indicación correcta respecto a una paciente joven con estenosis (mitral) e insuficiencia mitral ambas severas y sintomáticas en clase funcional de II/IV desde hace 1 año, que no mejora con tratamiento médico, y con una válvula sin afectación del aparato subvalvular ni calcio en las valvas:

a) Valvuloplastia mitral con balón porque la anatomía es favorable, b) Recambio valvular por una prótesis. c) Comisurotomía mitral aislada bien sea cerrada o abierta. d) Esperar a que su clase funcional sea IV.

570: Paciente de 65 años, intervenido de fractura de cadera derecha de 15 días antes, que consulta por dolor e hinchazón en miembro inferior derecho. A la exploración destaca aumento de la temperatura local con edema hasta la raíz del miembro. ¿Cuál de las siguientes considera que es la actitud a seguir?:

a) Realizar una garnmagrafía pulmonar de ventilaciónperfusión. Si fuera negativa indicar tratamiento con heparina subcutánea a dosis profiláctica. b) Indicar la colocación de un filtro de la cava inferior dada la contraindicación de establecer un tratamiento anticoagulante por la proximidad de la cirugía. c) Solicitar una radiografía de la cadera intervenida. d) Solicitar una ecografía-doppler color para confirmar el diagnóstico de trombosis venosa profunda, e iniciar tratamiento con heparina de bajo peso molecular a dosis terapéuticas.

571: Todas las siguientes son características de vértigo periférico, EXCEPTO una, señálela:

a) Comienzo brusco. b) Nistagmo unidireccional. c) Náuseas y vómitos acompañantes. d) Pérdida de audición en todos los casos aunque sea subclínica (demóstrada por audiometría).

572: Un paciente de 65 años consulta por dificultad para iuiciar la micción, incontinencia e impotencia refiriendo que durante años, ha presentado temblor, bradicinesia, rigidez, anhidrosis y síncopes posturales de repetición. El diagnóstico más probable, entre los siguientes, es:

a) Parálisis supranuclear progresiva. b) Síndrome de Parkinson. c) Forma autosómica de síndrome de Landry-Guillain Barré. d) Síndrome de Shy-Drager.

573: En relación con la miasteuia gravis, señale cuál de las afirmaciones siguientes es verdadera:

a) Es más frecuente en varones. b) Los músculos distales son los que se afectan con mayor frecuencia en las fases iniciales de la enfermedad. c) La estimulación eléctrica repetitiva a frecuencias altas es siempre diagnóstica. d) El tratamiento de elección de la miastenia generalizada en pacientes jóvenes es la timectomía.

574: La enfermedad neurológica (afectación del sistema nervioso central o periférico) con mayor incidencia, entre los siguientes, es:

a) Herpes zoster. b) ACV o ictus. c) Demencia. d) Jaqueca.

575: Señale cuál de estos fármacos está CONTRAINDICADO en los pacientes con enfermedad de Parkinson muy ancianos:

a) Levodopa + benseracida. b) Levodopa + carbidopa. c) Levodopa retard d) Anticolinérgicos.

576: Ante una mujer de 80 años con cefalea hemicraneal derecha incapacitante y claudicación al masticar. El primer estudio complementario que hay que solicitar, entre los siguientes, es:

a) Hemograma, VSG y bioquímica sérica rutinaria. b) Tomografía computada de cráneo. c) Resonancia magnética de cráneo. d) Rx de la articulación témporo-mandibular.

577: Una joven de 18 años con historia de anorexia y depresión leve en los últimos meses acude a un servicio de urgencias por cefalea y fiebre de cuatro días de evolución, por lo que fue tratada con un antibiótico que no recuerda. El examen neutológico evidenció leve rigidez de nuca, el escáner craneal fue normal. La punción lumbar mostró un LCR con 550 leucocitos por mm3, proteinorraquia de 350 mg/dl y glucorraquia de 8 mg/dl. Señale el diagnóstico MENOS probable, entre los siguientes:

a) Meningitis tuberculosa. b) Meningitis por neumococo. c) Meningitis por meningococo "decapitada". d) Meningitis viral

578: La encefalopatía anóxica-isquémica puede provocar secuelas neurológicas permanentes. De las que se citau, señale la MENOS frecuente:

a) Coma o estupor persistente. b) Demencia. c) Coreoatetosis.

Page 131: Cuestionario de Urología

d) Crisis convulsivas.

579: Paciente de 58 años operado 3 meses antes de aneurisma de la arteria comunicante anterior manifestado por hemorragia subaracnoidea. El postoperatorio inmediato fue favorable pero en el último mes el paciente desarrolla dificultad progresiva para la marcha, apatía, torpeza mental e incontinencia urinaria. ¿Cuál de las siguientes complicaciones parece más probablemente la responsable del deterioro neurológico tardío?:

a) Hidrocefalia comunicante. b) Vasoespasmo de las arterias cerebrales anteriores. c) Hiponatremia por secreción inadecuada de hormona antidiurética. d) Resangrado por oclusión incompleta del saco aneurismático.

580: Paciente de 35 años de edad presenta cuadro de dolor lumbar irradiado a lo largo de la pierna izquierda, acompañado de parestesias en 4° y 5° dedo del pie izquierdo. La e xploración neurológica puso de manifiesto dificultad para caminar de puntillas, reflejo aquíleo abolido y un signo de Lassegue positivo a 30° en pierna izquierda. Mediante resonancia magnética se objetivó la presencia de una hernia discal. ¿En qué nivel encontraríamos la imagen de la hernia discal?:

a) L2-L3 b) L3-L4 c) L5-S1 d) D12-Ll

581: En los craneofaringiomas es característica la presencia de:

a) Hemorragias intratumorales. b) Quistes y calcificaciones. c) Zonas de necrosis. d) Áreas sarcomatosas.

582: Una de las siguientes afirmaciones, en relación con la neuralgia esencial del trigémino es INCORRECTA:

a) Incide preferentemente en la sexta y séptima décadas de la vida. b) La exploración neurológica del paciente es habitualmente normal. c) Los paroxismos dolorosos suelen despertar al paciente por la noche. d) La localización del dolor en el territorio de la primera rama trigeminal es menor frecuente que en la segunda y tercera.

583: Un hombre de 74 años acude a Urgencias por presentar desde una hora antes dolor retroesternal intenso y diaforesis. Cuatro años antes había sido diagnosticado de arteritis temporal. Fue tratado con prednisona durante 18 meses. A la exploración, la temperatura: es de 36,6°C, la frecuencia cardíaca d e 120 I.p.m., la frecuencia respiratoria de 28/m, y la TA de 150/90. No hay ingurgitación yugular ni crepitantes pulmonares; la auscultación cardíaca no añade nada. La radiografia de tórax y el electrocardiograma son normales. El diagnóstico más probable entre los siguientes es:

a) Infarto de miocardio. b) Pericarditis aguda. c) Esofagitis aguda. d) Disección aórtica.

584: ¿Cuál de las siguientes afirmaciones es INCORRECTA en relación con la espondilodiscitis infecciosa?:

a) La radiología simple es poco útil para el diagnóstico precoz de esta entidad. b) La espondilodiscitis brucelar es a menudo una manifestación precoz de la infección por brucelas. c) El comienzo de la raquialgia puede ser agudo, con manifestaciones sistémicas, o larvado. d) El tratamiento antibiótico inicial de la espondilodiscitis infecciosa por gétmenes piógenos debe ser Parenteral a dosis máximas durante al menos 4-6 semanas.

585: Una paciente de 66 años acude a su consulta por presentar dolor intenso y tumefacción en su rodilla derecha desde el día anterior, confirmándose la presencia de un derrame sinovial a la exploración. Una radiografia de la articulación podrá aportar datos útiles para el diagnóstico solamente si el paciente sufre:

a) Una artritis séptica. b) Una espondiloartropatía. c) Una artritis por pirofosfato cálcico. d) Un hemartros.

586: En relación con la artritis gonocócica ¿Cuál de las siguientes afirmaciones es FALSA?:

a) La infección gonocócica diseminada suele cursar con artritis, tenosinovitis y dermatitis. b) La mayoría de los pacientes con enfermedad gonocócica diseminada tienen síntomas genitourinarios simultáneamente. c) Las deficiencias de las proteínas de la secuencia final del complemento (C5-C9) confieren un riesgo elevado de presentar artritis gonocócica. d) En pacientes con artritis gonocócica la tasa de aislamien to en cultivo de Neisseria gonorrheae es mayor en localizaciones genitourinarias que en el líquido sinovial.

587: Un paciente de 39 años con insuficiencia renal crónica en tratamiento con hemodiálisis desde hace 12 años, consulta por presentar poliartritis simétrica de hombros, carpos y rodillas y síndrome de túnel carpiano bilateral confirmado en estudio neurofisiológico. El recuento celular del líquido sino vial muestra 100 células/mmc) ¿Cuál de las siguientes considera que es la causa más probable de su artritis?:

a) Artritis por depósito de pirofosfato cálcico. b) Amiloidosis por depósito de beta 2 microglobulina. c) Gota úrica poliarticular. d) Osteodistrofia renal.

588: ¿Cuál de las siguientes afirmaciones es FALSA en relación con la artrosis o enfermedad articular degenerativa?:

a) Los hallazgos radiológicos característicos incluyen pinzamiento del espacio articular, esclerosis subcondral, osteofitos y en

Page 132: Cuestionario de Urología

estadios más evolucionados quistes subcondrales y luxaciones articulares. b) La hipertrofia ósea de las articulaciones interfalángicas distales se conoce con el nombre de nódulo s de Heberden. c) El líquido sinovial de las articulaciones afectas de artrosis muestra una viscosidad reducida y una celularidad escasa. d) La rigidez que aparece típicamente tras períodos de inactividad es habitualmente de duración inferior a 15-30 minutos.

589: Una paciente de 42 años desde hace 1año viene notando entumecimiento y cambios de color en las manos generalmente inducidos por el frío, habiendo notado primero palidez, más intensa en los dedos, seguidos de cianosis. Además ha notado marcado engrosamiento de la piel que ocurrió inicialmente en las manos, para luego extenderse a brazos, cara, y últimamente en cara anterior del tórax. Unos anticuerpos antinucleares son positivos y los anticuerpos antitopoisomerasa I también lo son. Indique cuál de las siguientes manifestaciones NO corresponde a esta enfermedad:

a) Hipertensión pulmonar. b) Esofagitis. c) Poliartritis. d) Hipercalcemia.

590: Indique cuál de las siguientes proposiciones le parece CIERTA en relación con un episodio agudo de gota:

a) Se acompaña siempre de hiperuricemia. b) El tratamiento requiere alopurinol. c) Las bursas no se inflaman. d) Los diuréticos Tiazídicos inducen hiperuricemia y gota.

591: Atiende a un paciente de 37 años por una artritis en rodilla y tobillo izquierdos, muñeca derecha e interfalangianas distales del 20 y 3er dedo de la mano derecha, presentando además tumefacción evidente del 20 dedo del pie izquierdo que es doloroso, sobre todo a la extensión. En la exploración aprecia psoriasis en el cuero cabelludo. ¿Qué prueba de las siguientes es necesaria para establecer el diagnóstico de artritis psoriásica?:

a) Proteína C Reactiva. b) Factor reumatoide. c) Biopsia sinovial. d) No precisa más datos para el diagnóstico.

592: Paciente varón de 22 años de edad con hemartros sin restos de gotas de grasa en la rodilla tras un traumatismo jugando al fútbol ¿Cuál es la lesión más frecuente que hay que sospechar?:

a) Rotura de ligamento cruzado anterior. b) Rotura de la arteria poplítea. c) Fractura del cuello del peroné. d) Rotura de menisco.

593: Ante una fractura patológica de fémur secundaria a una metástasis ósea de cáncer de mama en una mujer de 70 años. ¿Qué actitud terapéutica seguiría?:

a) Tracción transesquelética. b) Quimioterapia aislada. c) Radioterapia y tratamiento hormonal. d) Tratamiento quirúrgico y tratamiento oncológico específico.

594: Hombre de 25 años de edad presenta, tras caída casual sobre la mano derecha en hiperextensión de la muñeca, dolor selectivo a nivel de la tabaquera anatómica de la muñeca. ¿Qué tipo de lesión hay que sospechar?:

a) Fractura de Colles. b) Fractura de estiloides cubital. c) Luxación perilunar del carpo. d) Fractura de escafoides de la muñeca.

595: Durante una sustitución veraniega de Pediatría una madre de 41 años nos consulta que a su hija primogénita de 9 meses cada vez resulta más difícil colocarie el pañal. El parto fue por cesárea y la niña ha seguido controles rutinarios del recién nacido sano. Al explorarla apreciamos una marcada dificultad para la separación de los muslos del bebé y una cierta resistencia a la movilidad activa de las caderas, así como asimetría de los pliegues inguinales y glúteos. Nuestra actitud diagnóstica deberá ser:

a) Radiografía de caderas. b) Ecografía de caderas. c) Exploración neurológica exhaustiva. d) Punción lumbar y análisis de líquido cefalorraquídeo.

596: Un trabajador de la construcción es traído al Servicio de Urgencias tras caerle unas piezas de encofrado con bordes afilados metálicos sobre la muñeca derecha. A la exploración presenta una herida incisocontusa en la cara volar de la muñeca en su mitad cubital, sangrado profuso contínuo, imposibilidad para la flexión aislada del cuarto dedo, e imposibilidad para la abducción y adducción de los dedos segundo a quinto sobre el plano de la mesa, e hipoestesia de cara volar de cuarto dedo, siendo el resto de la exploración neurovascular y tendinosa normales. Las lesiones más probables serán:

a) Sección de arteria cubital, nervio mediano y flexor superficial de cuarto dedo. b) Sección de plexo venoso superficial, nervio mediano y flexor superficial de cuarto dedo. c) Sección de plexo venoso superficial, nervio cubital y flexor superficial de cuarto dedo. d) Sección de plexo venoso superficial, nervio cubital y flexor profundo de cuarto dedo.

597: A la Urgencia es traído un hombre de 80 años atropellado. Respira adecuadamente y no presenta sintomatología torácica ni heridas sangrantes, está consciente y orientado y sus TAC craneal y abdominal son normales. Con radiología simple se le diagnostica de fractura conminuta bilateral de ambas mesetas tibiales, fractura subtrocantérea de fémur derecho y fractura de ra-mas ilio-isquiopubianas izquierdas. Tras esta evaluación diagnóstica, que se prolonga durante 2 horas, inicia un cuadro de frialdad, sudoración, palidez, hipotensión y taquicardia. La actitud más correcta será:

a) Perfusión inmediata de cristaloides y expansores del plasma seguida de cirugía inmediata del fémur. b) Inmovilización de fracturas mediante férulas y tracciones.

Page 133: Cuestionario de Urología

c) Reposición de volemia y estabilización inmediata de las fracturas. d) Laparotomía exploradora, valoración de lesiones urinarias y cirugía de las fracturas.

598: Una enfermera de 45 años de edad acude a nuestra consulta por un dolor muy intenso en el hombro izquierdo, no irradiado, de 15 días de evolución, que le impide dormir. La movilidad activa del hombro es completa, y está conservada, y es dolorosa la palpación de la región anterior del manguito de los rotadores y de la cabeza humeral. La radiografía simple es rigurosamente normal. Nuestra actitud deberá ser:

a) Infiltración local de corticoide y anestésico local. b) AINEs, reposo en cabestrillo, aplicación de calor local. c) Resonancia magnética. d) Ecografía.

599: La causa más frecuente de infección secundaria del sistema nervioso central en los pacientes con Síndrome de Inmunodeficiencia Adquirida (SIDA) es:

a) Criptococosis meníngea. b) Toxoplasmosis cerebral. c) Meningitis bacteriana. d) Leucoencefalopatía multifocal progresiva.

600: En la infección por el virus de la inmunodeficiencia humana, las células diana, linfocitos CD4+, van disminuyendo con la evolución de la enfermedad, llevando a la aparición de enfermedades oportunistas. Una de ellas muy frecuente en este grupo de pacientes, es la neumonía por Pnenmocystis cariuii. ¿Por debajo de qué cifra de CD4+ se aconseja la prordaxis contra ésta?:

a) Con cualquier cifra de CD4+. b) 500/mm3 c) 350/mm3 d) 200/mm3

601: Entre los siguientes parámetros analíticos, en un paciente con infección por el virus de la inmunódeficiencia humana (VIH), ¿cuál es el que mejor predice la evolución de la enfermedad hacia el Síndrome de Inmunodeficiencia Humana (SIDA)?:

a) Niveles de ARN del VIH. b) Niveles de antígeno del VIH. c) Niveles de linfocitos CD4+. d) Niveles de linfocitos CD8+.

602: Señale la respuesta correcta en cuanto a la tuberculosis en pacientes con infección por VIH.

a) En nuestro medio se asocia siempre a depleción importante de linfocitos CD4+ (<200/mm3). b) Se asocia a Mantoux positivo en la mayoría de los casos. c) La forma extrapulmonar más frecuente es la tuberculosis pleural. d) La respuesta al tratamiento suele ser similar a la de los pacientes sin infección VIH.

603: Un paciente de 67 años presenta cefalea, confusión y fiebre de 38°C, desde hace 2 semanas. En la e xploración se aprecia rigidez de nuca y parálisis del VI par craneal del lado izquierdo. El TAC craneal muestra un aumento de densidad en cisternas basales. El LCR presenta 300 células (80% linfocitos), glucosa 20 mg/dL, proteínas 200 mg/dL. La tinción de gram no muestra gérmenes y el Ziehl es negativo. ¿Cuál es su diagnóstico?:

a) Meningitis meningocócica. b) Meningitis neumocócica. c) Meningitis tuberculosa. d) Meningoencefalitis herpética.

604: Una enfermera de neonatología de 52 años nos consulta porque a su marido le acaban de diagnosticar de tuberculosis pulmonar activa. Tres meses antes de la exposición, esta mujer se realizó test tuberculíruco (PPD) que fue negativo en 2 determinaciones, con una semana de intervalo. Refiere buena salud y la placa de tórax que se realizó tras la exposición, es normal. Se repite el test tuberculínico (PPD) a las 4 semanas de la exposición, apreciándose una induración de 8 mm. De las siguientes ¿cuál es la medida más adecuada para esta mujer?:

a) Repetir el test tuberculínico (PPD) en un mes para comprobar si ha aumentado la induración a más de 10 mm. En este momento no requiere tratamiento y puede seguir trabajando. b) Repetir la placa de tórax ahora. Si negativa, realizar periódicamente placas de tórax, especialmente si desarrolla síntomas pulmonares. Puede continuar en su trabajo a menos que desarrolle síntomas pulmonares. c) Comenzar inmediatamente tratamiento con isoniacida durante un último de 6 meses. Puede incorporarse al trabajo en 15 días. d) Repetir la placa de tórax ahora y si es negativa, comenzar profilaxis con isoniacida durante al menos 6 meses e in-corporarse a su trabajo.

605: Una mujer de 25 años acude a su consulta por tos .con expectoración amarillenta, desde hace varias semanas; durante los últimos 3 días el esputo contiene "hilillos" de sangre. Desde el inicio del cuadro ha presentado malestar general, astenia, sudoración nocturna y sensación febril. Refiere ingesta ocasional de alcohol y fuma un paquete de cigarrillos al día, pero niega consumo de drogas por vía intravenosa. A la exploración estádelgada y pálida, la temperatura es de 37,5°C, 90 latidos/minuto, 14 respiraciones/minuto; en el tórax se aprecia matidez a la percusión, roncus y respiración bronquial en el campo pulmonar superior derecho; siendo normal el resto de la exploración. El diagnóstico más probable es:

a) Neumonía neumocócica. b) Embolismo pulmonar. c) Neumonía por Legionella. d) Tuberculosis.

606: Un estudiante de 20 años acude a urgencias con una historia de cefalea progresiva, somnolencia, náuseas y vómitos. A la exploración, tiene una temperatura de 39°C y está e stuporoso. El paciente no puede cooperar con la exploración y no se visualiza adecuadamente el fondo de ojo. Tiene rigidez de nuca y parálisis del VI par craneal izquierdo. El resto de la exploración ÍIsica es

Page 134: Cuestionario de Urología

normal. Tras extraer 2 hemocultivos, ¿cuál es la decisión inmediata más adecuada?:

a) TAC craneal y punción lumbar (si no contraindicada por los resultados de dicho TAC), seguido de la administración intravenosa de cefotaxima y ampicilina. b) Punción lumbar y pruebas de laboratorio, seguido de la administración intravenosa de ampicilina. c) Administración intravenosa de cefotaxima o cefriaxona, seguido de TAC craneal y punción lumbar (si no contraindicada por los resultados de dicho TAC). d) Admitir para observación después de realizar TAC craneal y punción lumbar (si no contraindicada por los resultados de dicho T AC) difIriendo el tratamiento antimicrobiano hasta tener los resultados del análisis del LeR.

607: Un niño de 14 años previamente sano, presenta cefalea y desorientación progresiva de 24 horas de evolución. Una hermana de 12 años de edad, había sido tratada 2 años antes de meningitis meningocócica. El examen ÍIsico revela una temperatura de 39,5°C, tiene rigidez de nuca y está sonmoliento. En las pruebas de laboratorio destacan 18.000 leucocitos/µL, con un 70% de neutr6filos y 12 % de cayados. La creatinina, glucosa, electrolitos y bioquímica hepática son normales. El LCR muestra 380 leucocitos/µL, con 98 % de neutróf'dos, glucosa de 19 mg/dL y proteínas de 165 mg/dL. En la tinción de Gram del LCR se observan diplococos gram negativos y en el cultivo se aísla Neisseria meningitidis. El paciente responde adecuadamente al tratamiento con penicilina. ¿Qué exploración diagnóstica consideraría más indicada en este momento?:

a) Niveles de inmunoglobulinas. b) Properdina sérica. c) Nivel de complemento hemolítico total. d) Estudios de la función de los neutrófilos.

608: Señalar los agentes etiológicos más frecuentes de la meningitis aguda puruIenta en adultos inmunocompetentes:

a) Staphylococcus aureus y Bacilos gram negativos. b) Neisseria meningitis, Hemophilus influenzae y Streptococcus pneumoniae. c) Streptococcus pneumoniae y Neisseria meningitidis. d) Bacilos Gram negativos, Streptococcus agalactiae y Listeria monocytogenes.

609: La mayoría de las meningitis producidas tras un traumatismo craneal abierto o relacionados con infecciones parameníngeas del área otorrinolaringológica están producidas por:

a) Hemophilus influenzae. b) Neisseria meningitidis. c) Streptococcus pneumoniae. d) Bacilos Gram negativos.

610: Una paciente de 25 años con una historia de contactos sexuales con distintas parejas en los últimos meses, consulta por fiebre elevada, cefalea, adenopatías inguinales y ulcerosas dolorosas en vulva. ¿Cuál es el diagnóstico más adecuado?:

a) Infección por Citomegalovirus. b) Primoinfección por Herpes Simple tipo 2. c) Vaginitis candidiástica. d) Enfermedad pélvica inflamatoria por Chlamydia.

611: Una paciente de 44 años, sometida a mastectomÍa izquierda con vaciamiento ganglionar, presenta.8 meses después de la intervención un cuadro febril con celulitis extensa en brazo izquierdo, que se resuelve con tratamiento antibiótico. Seis meses más tarde, acude de nuevo por un cuadro similar. ¿Cuál es el agente etiológico más probable?:

a) Staphylococcus epidennidis. b) Streptococcus agalactiae. c) Corynebacterium haemolyticum. d) Streptococcus pyogenes.

612: Un colega te pide consejo acerca de 3 pacientes, de la núsma fanúlia, que llevan varios días con dolor en hipocondrio derecho, diarrea y fiebre. En los análisis rutinarios se detecta una eosinof'ilia del 40% (2.500 eosinófilos/µL). ¿Qué pregunta te parece más relevante en la Anamnesis, de cara a diagnosticar el patógeno responsable?:

a) Ingesta de lácteos sin higienizar. b) Ingesta de pastelería o comida sin refrigerar. c) Ingesta de berros. d) Contacto con perros.

613: Señale de entre las siguientes una característica cierta de la infección por Vibrio cholerae (Cólera):

a) Provoca afectación prioritaria de intestino grueso. b) Es característica la ausencia de leucocito s en heces. c) Se asocia a la destrucción de células de la mucosa intestinal por efecto de cito toxinas. d) La cuantía del inócu10 y la acidez gástrica son factores de poca importancia en la evolución de la infección.

614: Un varón de 45 años se presenta en su consulta por diarrea severa de curso ondulante desde hace 3 meses. Refiere 3 a 10 deposiciones acuosas al día. Ha perdido 7,5 kg de peso en este tiempo. Hace 6 meses presentó una meningitis criptocócica. En la actualidad rechaza hacer tratamiento antirretroviral y la cifra de linfocitos CD4+ es de 26/µL. Se han analizado 6 muestras de heces, no encontrándose evidencia de patógenos bacterianos, huevos, parásitos o Clostridium difficile. La diarrea no mejoró con Loperamida. ¿Cuál debería ser el procedimiento siguiente para encontrar una causa tratable a su diarrea?:

a) TAC abdominal. b) Colonoscopia con biopsia mucosa. c) Ecografía abdominal. d) Gammagrafía con galio.

615: Se encuentra con un paciente en buen estado general y con adenopatías múltiples. La biopsia es diagnóstica de un linfoma folicular. Cuál de las siguientes afirmaciones es correcta respecto a estos pacientes:

a) No es demorable el inicio de tratamiento.

Page 135: Cuestionario de Urología

b) La mayoría de los pacientes tienen síntomas B. c) En ocasiones se transforman a una forma histológica más grave. d) No es frecuente la afectación ósea.

616: En la deficiencia de vitamina B12. ¿Cuál de las siguientes afirmaciones es correcta?:

a) La anemia se instaura rápidamente. b) Los síntomas neurológicos pueden aparecer sin anemia. c) Los reticulocitos están elevados. d) Puede aparecer ataxia (signo de Romberg) por afectación del cerebelo.

617: ¿Cuál de las siguientes afirmaciones es correcta? La absorción de hierro en un individuo normal:

a) Es mayor si la dieta es rica en hierro inorgánico. b) Es mayor si la dieita es rica en hierro hémico. c) Se inhibe por el ácido ascórbico de los cítricos. d) Aumenta con los fitatos de las verduras.

618: Es característico de una anemia megaloblástica por déficit de folatos todo lo indicado, EXCEPTO:

a) Macrocitosis en sangre periférica. b) Aumento de los neutrófilos poli-segmentados. c) Elevación de la LDH. d) Elevación del ácido metil-malónicos en plasma.

619: En un paciente con 60 años, buen estado general, exploración física normal y un hemograma con leucocitos 70.000/mm3 (90% linfocitos), sin anemia ni trombopenia, el diagnóstico más probable y tratamiento más adecuado, son:

a) Diagnóstico: Leucemia linfática crónica. Tratamiento: no precisa. b) Diagnóstico: Leucemia linfática crónica estadio A. Tratamiento: no precisa. c) Diagnóstico: Linfoma leucemizado. Tratamiento: Poliquimioterapia. d) Diagnóstico: Leucemia prolinfocítica. Tratamiento: Fludarabina.

620: Se encuentra usted ante un paciente que le envían con la sospecha de padecer, por la historia clínica y la exploración fisica, especialmente las lesiones cutáneas, una mastocitosis sistémica. ¿Qué prueba debe realizar para confirmar el diagnóstico?:

a) La búsqueda del signo de Darier (producción de habones y eritema tras la manipulación digital de una lesión cutánea). b) Una gastroscopia diagnóstica de ulcus gastroduodenal. c) Repetición seriada de los niveles de histamina sanguínea. d) Biopsia de médula ósea.

621: El ácido holo-trans-retinoico (ATRA) es una terapéutica especialmente eficaz en una de las siguientes variedades de leucemia:

a) Leucemia aguda linfoblástica. b) Leucemia aguda promielocítica. c) Leucemia mieloide crónica. d) Leucemia linfática crónica.

622: ¿Cuál de las siguientes tinciones histoquímicas es determinante para identificar una de las variedades de síndrome mielodisplásico?:

a) Peroxidasa. b) Perls (hierro). c) PAS. d) Fosfatasas alcalinas granulocíticas.

623: La causa más frecuente de hipercoagulabilidad hereditaria es:

a) Déficit de proteína C. b) Déficit de proteína S. c) Déficit de antitrombina III. d) Factor V de Leiden.

624: La infiltración linfomatosa de la sangre periférica es especialmente frecuente en:

a) Linfoma de Burkitt. b) Linfoma de células grandes difuso. c) Linfoma linfocítico bien diferenciado difuso. d) Linfoma de Hodgkin.

625: Señale cuál de las siguientes afirmaciones es correcta, en relación con las crisis hemolíticas por déficit de glucosa-6-fosfato deshidrogenasa eritrocitaria:

a) Son menos frecuentes en las mujeres. b) Son independientes de la ingestión de habas verdes. c) Son independientes de la intoxicación por bolas de naftalina antipo1illas. d) Son independientes de la ingestión de antipalúdicos.

626: Señale cuál de los siguientes hechos NO está presente en el síndrome de la neoplasia endocrina múltiple familiar tipo 1 (MEN-I):

a) Hiperplasia paratiroidea de las cuatro glándulas. b) Diátesis ulcerogénica por hipergastrinemia. c) Tumores pancreáticos productores de polipéptido intestinal vasoactivo (VIP). d) Carcinoma medular tifoideo.

Page 136: Cuestionario de Urología

627: Un paciente de 37 años ha sido sometido a una tiroidectomía total por un tumor maligno tiroideo multifocal, productor de calcitonina. ¿Qué investigación genética puede detectar la afección en sus familiares?:

a) Expresión excesiva del oncogén HER-2/neu (c-erb B2). b) Mutación en el protooncogén RET. c) Amplificación del protooncogén N-myc. d) Mutación en el gen BRCA-I.

628: Mujer de 52 años que padece crisis renoureterales de repetición. En el estudio de la causa se detecta hipercalcemia y niveles plasmáticos de hormona paratiroidea molécula intacta elevados. Con vistas al tratamiento quirúrgico de dicha causa, ¿qué método de localización de la lesión es obligado realizar preoperatoriamente?:

a) Arteriografía selectiva. b) Resonancia magnética nuclear. c) Ninguno. d) Ecografía.

629: La determinación de tiroglobulina sérica tiene su mayor utilidad en el seguimiento de pacientes:

a) Tratados con tiroidectomía total por cáncer diferenciado de tiroides. b) Tratados con hemitiroidectomía por cáncer diferenciado de tiroides. c) Tratados con tiroidectomía total por cáncer anaplásico de tiroides. d) Con tiroides de Riedel.

630: Si un hombre obeso de 50 años presenta valores repetidos de glucemia en ayunas entre 110-125 mg/dl. ¿Cuál es, entre las siguientes, la actitud más correcta?:

a) Establecer el diagnóstico de Diabetes Mellitus e iniciar tratamiento dietético. b) Realizar curva de glucemia con sobrecarga oral para precisar el diagnóstico. c) Recomendar bajar peso sin hacer más averiguaciones. d) Comenzar tratamiento con dosis bajas de Metforrnina.

631: ¿Cuál de los siguientes fármacos está CONTRAINDICADO durante el embarazo de la mujer diabética?:

a) Derivado de hierro por vía oral. b) Insulina de acción rápida. c) Insulina de acción intermedia. d) Sulfonilureas.

632: Ante un diabético tipo I (InsulinDependiente) que empieza a presentar valores elevados y repetidos de microalbuminuria (por encima de 30 microgramos/minuto o mg/24 horas). ¿Cuál de las siguientes actitudes NO es correcta?:

a) Mejorar significativamente su control metabólico. b) Si es hipertenso controlar más exigentemente sus cifras tensionales. c) Añadir a su tratamiento habitual dosis bajas de corticoides, por ejemplo Prednisona 5 mg/día. d) Valorar periódicamente su función renal: creatinina y aclaramiento de creatinina.

633: ¿Cuál de los siguientes datos analíticos de la sangre es característico tanto de la insuficiencia adrenal aguda de origen adrenal como de la insuficiencia adrenal aguda de origen hipotálamo-hipofisario?:

a) Elevación de la ACTII. b) Hipopotasemia. c) Hiperglucemia. d) Hiponatremia.

634: En hipertensos con sospecha clínica de Feocromocitoma, ¿cuál de los siguientes grupos de hipotensores está CONTRAINDICADO en el tratamiento inicial del paciente?:

a) Antagonistas del calcio. b) Betabloqueantes. c) Diuréticos. d) Alfabloqueantes.

635: Una paciente con aspecto cushingoide presenta niveles mínimos, indetectables, de adrenocorticotropina (ACTH), cortisolemia y cortisollibre urinario elevados, sin descenso de éstos últimos tras la administración de 0,5 mg/6 h ni de 2 mg/6 h de dexametasona. Entre los siguientes, ¿cuál es el diagnóstico más probable?:

a) Adenoma hipofisario secretar de ACTII. b) Bloqueo adrenal congénito. c) Adenoma adrenal. d) Tumor secretar de hormona liberadora de corticotropina (CRH). .

636: Un hombre de 45 años, previamente sano, es diagnosticado de neumonía del lóbulo inferior derecho. A la exploración física el paciente está consciente y orientado, T A 120/80 en supino que sube a 130/90 tras la bipedestación. Buen estado de hidratación. Diuresis de 24 horas: 1 litro, con balance negativo. Presenta la siguiente analítica sanguínea: Hemoglobina 12 g/di. Creatinina 0,6 mg/dl. Calcio 8,5 mg/dl. Ácido úrico 2,2 mg/dl. Sodio 127 mEqn. Potasio 3,4 mEqn. Proteínas totales 5,8 g/di. La actitud a seguir ante la hiponatremia de este paciente es:

a) Restricción hídrica. b) Tratamiento con furosemida. c) Tratamiento nutricional intensivo. d) Solución salina hipertónica intravenosa.

637: Un hombre de 40 años refiere astenia e impotencia coeundi. En la analítica, presenta una prolactina sanguínea en niveles diez veces por encima de los límites normales superiores. En la resonancia magnética se evidencia un tumor hipofisario. Los niveles de LH, FSH y testosterona más probables serán:

Page 137: Cuestionario de Urología

a) Niveles elevados de LH y FSH, niveles bajos de testosterana. b) Niveles bajos de LH, FSH y testosterona. c) Niveles bajos de LH y FSH, normales de testosterona. d) Niveles elevados de LH, FSH y testosterona.

638: Son sugestivos de estenosis de la arteria renal los siguientes hallazgos en un sujeto joven, EXCEPTO:

a) Alcalosis metabólica. b) Hiperpotasemia. c) Presencia de síntomas de insuficiencia vascular en otros órganos. d) Soplos abdominales.

639: ¿Cuál es la Glomerulonefritis que con mayor frecuencia se asocia a la existencia de reflujo vesicoureteral? :

a) Nefropatía IgA. b) Glomerulonefritis Membranosa. c) Glomerulonefritis Membrano-Proliferativa. d) Hialinosis segmentaria y focal.

640: Paciente de 45 años que acude a la urgencia del hospital por dolor en fosa lumbar derecha irradiado a genitales, de carácter cólico, de dos días de evolución. Posteriormente desarrolló fiebre alta con escalofríos, detectándose en el examen de la sangre una leucocitosis de 20.000 por mm3, con desviación a la izquierda, y en el sedimento de la orina, microhematuria, piuria y bacteriuria, ¿qué exploración complementaria, de entre las siguientes, sería la más adecuada como primera opción, en este caso?:

a) Ecografía abdominal. b) Urolografía retrógrada. c) Tomografía renal. d) TAC abdominal.

641: De entre los siguientes fármacos ¿cuál es el que produce hiponatremia con más frecuencia?:

a) Furosemida. b) Digital. c) Prednisona. d) Tiazidas.

642: Paciente de 65 años, diagnosticada de insuficiencia cardíaca, que sigue tratamiento habitual con Inhibidores de la Enzima Convertidora de la Angiotensina (IECAS), Espironolactona y Furosemida. En una analítica de rutina presenta unas cifras de potasio sérico de 5,8 mEqlL, ¿qué variaciones deberemos realizar en su tratamiento?:

a) Elevar las dosis de Furosemida. b) Añadir resinas de intercambio iónico. c) Añadir Digitálicos. d) Suspender los IECAS y la Espironolactona.

643: Hombre de 75 años que acude al Hospital por oliguria de varios días de evolución y dolor en hipogastrio, donde se palpa una masa. En el examen de la sangre se detecta una creatinina de 3 mg/dL y un potasio de 5,8 mEq/L. ¿Qué.maniobra, de entre las siguientes, considera la más recomendable en este caso?:

a) Hemodiálisis. b) Sondaje urinario. c) Urografía intravenosa d) TAC abdominal

644: La asociación de Insuficiencia Renal Aguda, fiebre, artralgias, exantema y eosinofilia sugiere preferentemente:

a) Granulomatosis alérgica (Churg Strauss). b) Nefritis intersticial aguda. c) Angeitis leucocitoclástica. d) Granulomatosis de Wegener.

645: Un niño de 6 años presenta edema generalizado y proteinnria de 8 gramos al día, sin hematuria, hipertensión ni disminución de la función renal. La actitud más adecuada en este caso sería:

a) Confirmar la existencia de proteinuria. b) Precisar la selectividad de la proteinuria. c) Realizar biopsia renal. d) Administrar esteroides.

646: La enfermedad renal denominada Glomerulonefritis membranosa (GNM) cursa habitualmente con un Síndrome Nefrótico (SN) de larga evolución. Se han descrito diferentes causas etiológicas inductoras de GNM y se acepta que de poder controlar la causa o eliminar el agente, la enfermedad renal podría remitir. De las abajo enumeradas, todas, EXCEPTO una, pueden ser inductoras de Glomerulonefritis Membranosa. Señálela:

a) Virus B de la hepatitis. b) Tumores sólidos del colon. c) Dermatitis herpetiforme. d) Picaduras por determinados insectos (abejas, avispas).

647: Una paciente de 35 años de edad con aspiración nasogástrica por estenosis pilórica presenta un potasio sérico de 3 mEq/L (valores normales 3,5-5 mEq/L) al tercer día de su ingreso. Señale cuál de estas afirmaciones es la correcta en este contexto:

a) La causa más importante de la hipopotasemia es por la aspiración nasogástrica. b) Las pérdidas renales de potasio son mínimas como mecanismo compensatorio renal ante las pérdidas digestivas.

Page 138: Cuestionario de Urología

c) Se requeriría la administración de volumen y cloruro sódico para su colTección. d) Rara vez se acompaña de alcalosis metabólica.

648: ¿Qué marcador o conjunto de marcadores, entre los siguientes, es de utilidad clínica en el manejo del cáncer testicular?:

a) Antígeno carcino-embrionario y alta fetoproteína. b) Gonadotropina coriónica (subunidad Beta). c) Antígeno específico prostático y antígeno carcinoembrionario. d) Alfa-fetoproteína y carnitina.

649: ¿Cuál es el mejor método de estadificación local en el cáncer vesical?:

a) Ecografía. b) Scanner. c) Resonancia Magnética Nuclear. d) Resección trasuretral.

650: Un hombre de 71 años presenta un cuadro constituido por polaquiuria, tenesmo vesical, y dolor suprapúbico de repetición, que no cede con tratamiento antibiótico. La analítica de orina sólo muestra una hematuria macroscópica con pequeños coágulos. ¿En qué proceso etiológico pensaría en primer lugar?:

a) Cistitis por E. Coli. b) Carcinoma de células transicionales. c) Cistitis intersticial. d) Cistitis gonocócica.

651: Una paciente de 52 años consulta por una historia de varios meses de evolución de síndrome irritativo vesical y dolor suprapúbico. La exploración fisica no revela datos de interés, los cultivos de orina son repetidamente negativos, así como las citologías urinarias. En la exploración cistoscópica se objetivan petequias submucosas a la distensión vesical y las biopsias han sido inflamatorias. ¿Cuál es el diagnóstico de presunción?:

a) Carcinoma in situ. b) Litiasis yuxtameatal. c) Carcinoma infiltrante. d) Cistitis intersticial.

652: Las ampollas del pénfigo vulgar se caracterizan por:

a) Aparecer sobre piel aparentemente sana. b) Transformarse rápidamente en úlceras profundas. c) Ser tensas y resistentes a la rotura. d) Ser intensamente pruriginosas.

653: La isotretinoína es un fármaco que se emplea por vía oral para tratar el acné en algunos pacientes. ¿Cuál de las siguientes respuestas es FALSA?:

a) Puede elevar los niveles séricos de colesterol y triglicéridos. b) Produce casi siempre una importante sequedad de piel y mucosa. c) Es teratógeno y por ello es obligatorio que las mujeres que lo tomen realicen una anticoncepción durante al menos 2 años después de finalizado el tratamiento. d) Puede alterar el proceso de cicatrización de las heridas y facilitar la aparición de queloides.

654: Un hombre de 65 años presentar un herpes zóster en la cara. ¿Cuándo debe pedirse consulta con un oftalmólogo?:

a) Presenta lesiones en la punta y lado de la nariz. b) El nervio facial y el auditivo están afectados. c) Los nervios supraorbitales están afectados. d) El paciente presenta un síndrome de Ramsay-Hunt.

655: ¿Cuál de las siguientes manifestaciones cutáneas suele acompañar a la atopia?:

a) Piel seca. b) Congestión facial. c) Onicodistrofia. d) Pemiosis.

656: En una paciente afecta de lupus eritematoso cutáneo subagudo, el tipo de autoanticuerpos circulantes que encontraremos con mayor frecuencia será:

a) Anti-Ribonucleoproteína. b) Anti-Sm. c) Anti-Ro. d) Anti-histona.

657: Los siguientes factores pueden provocar un brote de psoriasis, EXCEPTO:

a) Infecciones. b) Factores hormonales. c) Trauma. d) Medicamentos.

658: El tratamiento de la retinopatía diabética se efectúa con:

a) Fármacos anticolinérgicos. b) Fotocoagulación con láser. c) Inyección de insulina subconjuntival. d) Antiinflamatorios no esteroideos.

Page 139: Cuestionario de Urología

659: ¿Cuál de las siguientes afirmaciones realizadas sobre un tipo concreto de migraña es FALSA?:

a) La migraña oftalmopléjica es típica de niños antes de los 10 años de edad. b) La migraña complicada se asocia característicamente a uveítis e hipertensión ocular. c) El aura visual de la migraña clásica tiene una progresión típicamente centrífuga. d) El único síntoma visual característico de la migraña común es la fotofobia.

660: ¿Cuál de los siguientes enunciados es INCORRECTO en la hipermetropía?:

a) El ojo hipermétrope es típicamente grande, no sólo en su diámetro anteroposterior sino en todos los demás. b) El ojo está predispuesto al glaucoma de ángulo cerrado. c) Es el error de.refracción en el que los rayos paralelos de luz son llevados a un foco por detrás de la retina. d) Se puede asociar estrabismo divergente.

661: La motilidad de los párpados depende de:

a) III y VII par craneal. b) II par craneal. c) VI par craneal. d) III y VI par craneal.

662: Señale qué forma de glaucoma, entre las siguientes, es la más frecuente en la práctica clínica:

a) La secundaria a diabetes. b) La primaria de ángulo estrecho. c) La congénita. d) La primaria de ángulo abierto.

663: Ante un cuadro de fotopsia persistente en el ojo derecho localizada a la altura de la punta de la nariz y visión de puntos móviles en forma de lluvia, la localización más probable del desgarro retiniano será:

a) Temporal superior. b) Nasal superior. c) Nasal inferior. d) Temporal inferior. d) En cualquier cuadrante.

664: La inclusión del epitelio queratinizante en el oído medio es característico de:

a) Otitis tuberculosa. b) Perforación timpánica. c) Colesteatoma. d) Otitis serosa.

665: Cuando hablamos de una Ránula nos referimos a:

a) Quiste sebáceo en la mucosa yugal. b) Quiste perialveolar de los incisivos. c) Quiste dermoide retroorbitario. d) Quiste de retención por obliteración de conductos excretores sublinguales.

666: El Síndrome de Ramsay-Hunt se caracteriza por parálisis facial que acompaña a:

a) Otitis media colesteatomatosa. b) Parálisis iatrogénica. c) Parálisis por traumatismo craneal. d) Herpes zóster.

667: Las pruebas de Rinne negativo es propio de las sorderas de:

a) Percepción. b) Transmisión. c) Tumores del acústico. d) Cofosis.

668: Si durante una intervención quirúrgica de tiroides seccionamos el nervio recurrente derecho de un paciente, ¿cuál será, entre las siguientes la alteración clínica más probable?:

a) Disnea y posición media de la cuerda vocal derecha. b) Disfonía y posición paramediana de la cuerda vocal derecha. c) Voz bitonal y posición intermedia de la cuerda vocal izquierda. d) Disnea intensa con posición paramediana bilateral.

669: El fibroangioma nasofaríngeo se caracteriza por:

a) Epístaxis en la pubertad masculina. b) Epístaxis en la pubertad femenina. c) Epístaxis en el adulto (30-40 años). d) Rinorrea en la edad juvenil del varón.

670: ¿Cuál de las siguientes medicaciones reduce el aclaramiento renal y aumenta las concentraciones de litio hasta niveles potencialmente peligrosos?:

a) Ibuprofeno. b) Furosemida. c) Tetraciclina. d) Teofilina.

Page 140: Cuestionario de Urología

671: ¿Cuál de los siguientes datos es una característica principal de la encefalopatía de Wernicke?:

a) Inicio agudo. b) Asociación a crisis comiciales alcohólicas. c) Asociación a arnnesia retrógrada. d) Tratamiento eficaz con ácido fólico.

672: Una mujer de 25 años es enviada al psiquiatra por un cirujano plástico, debido a que afirma constantemente que tiene hinchada una parte de la cara. El cirujano no ha podido comprobar en ningún momento esta afirmación. El diagnóstico más probable es de:

a) Simulación. b) Bulimia nerviosa. c) Trastorno de conversión. d) Trastorno dismórfico corporal.

673: ¿Cuáles de los siguientes problemas psiquiátricos ve aumentada su frecuencia con la edad?:

a) Depresión. b) Trastornos psicóticos. c) Trastornos fóbicos. d) Suicidio.

674: Los criterios diagnósticos para el trastorno por estrés postraumático incluyen todos los siguientes EXCEPTO:

a) La re-experimentación del episodio. b) El aumento del grado de alerta. c) La evitación de estímulos. d) El estado de ánimo deprimido.

675: Atendemos a una chica de 16 años que nos cuenta que lleva un tiempo con menos apetito y durmiendo menos ya que le cuesta poder dormirse. Es una buena estudiante, pero en la última evaluación le han quedado cuatro asignaturas y le cuesta concentrarse en los estudios. Los dos últimos fines de semana no ha salido con sus amigas porque no le apetecía y se siente irritada con su familia, aunque no entiende por qué. Tiene dolores de cabeza y a veces le viene la idea de la muerte a la imagina-ción aunque piensa que no lo haría por sus sentimientos religiosos. Nunca le había sucedido algo parecido. El diagnóstico sería:

a) Trastorno depresivo mayor. b) Trastorno ciclotímico. c) Trastorno distímico. d) Episodio depresivo mayor.

676: ¿Qué grupo de Psicofármacos estarían CONTRAINDICADOS en el tratamiento de un Trastorno por Déficit de Atención?:

a) Antidepresivos. b) Barbitúricos. c) Anticonvulsivantes. d) Estimulantes.

677: En el trastorno paranoide de la personalidad es FALSO que:

a) Exista un exceso de suspicacia. b) Se tengan ideas delirantes de perjuicio. c) Se tienda a ser rencoroso. d) Se confíe poco en los demás.

678: Consultan unos padres porque su hijo, de cinco años, lleva varias noches despertándose agitado como si hubiera soñado algo que le angustia. Cuando acuden a su lado por la noche, el niño les mira y dice palabras que no tienen ningún significado. Al cabo de un rato vuelve a dormirse y por la mañana no recuerda nada de lo ocurrido. El diagnóstico sería:

a) Pesadillas. b) Terrores nocturnos. c) Foco Epiléptico. d) Disomnia.

679: Acude a su centro una mujer primigesta de 35 años por sensación de dinámica regular. Se trata de una gestación gemelar, monocorial, biamniótica en la semana 36 + 4 cuyo curso ha sido normal; la exploración revela un cervix borrado, con 5 cm de dilatación, blando y centrado; primer gemelo en presentación cefálica, segundo en transversa, bolsa íntegra y amnioscopia negativa. ¿Cuál es la actitud correcta de cara al parto?:

a) Realizar una cesárea urgente, ya que se trata de una gestación biarnniótica. b) Dejar evolucionar el parto por vía vaginal. c) Dejar evolucionar el parto vaginal del primer gemelo y realizar una cesárea no electiva del segundo. d) fuiciar la administración de tocolíticos, puesto que se trata de una gestación de 36 semanas y es previsible que los fetos sean muy inmaduros.

680: Cuando se administra Sulfato de Magnesio para el tratamiento de la preeclampsiaeclampsia y aparecen signos de sobredosificación ¿quéantídoto se debe emplear?:

a) Carbonato sódico. b) Sulfato feIToso. c) Nitroprusiato. d) Gluconato cálcico.

681: Desde 1960 se observa una preocupante elevación de la tasa de cesáreas. Este incremento se asocia entre otras, pero de forma preferente a una de las indicaciones expuestas a continuación:

Page 141: Cuestionario de Urología

a) Placenta Previa. b) Prolapso de cordón. c) Cesárea anterior (Iterativa). d) Embarazo gemelar. d) Fiebre intraparto.

682: Tercigesta de 38 años, con partos anteriores normales. Gestante de 36 semanas, ha comenzado con sangrado de color rojo brillante, escaso e indoloro. ¿Cuál de los signientes métodos es el más eficaz para confirmar o descartar la sospecha de placenta previa?:

a) Placentografía indirecta con replección vesical. b) Arnniografía. c) Gammagrafía isotópica. d) Angiografía. d) Ecografía transabdominal o transvaginal.

683: En relación con el riesgo de muerte fetal, uno de los siguientes es un signo ominoso en el estudio mediante Doppler, señálelo:

a) Flujo diastólico invertido en la arteria umbilical. b) Redistribución cerebral. c) Flujo diastólico creciente en la aorta abdominal. d) Escotadura postsistólica en las venas arcuatas.

684: Entre las siguientes, ¿qué medida es la más fiable para calcular la edad gestacional durante el primer trimestre?:

a) Longitud craneocaudal. b) Longitud del fémur. c) El diámetro biparietal. d) El área cefálica.

685: Son factores de riesgo de cáncer de mama todos los siguientes, EXCEPTO:

a) Historia familiar de cáncer de mama. b) Tabaquismo. c) Radiaciones ionizantes. d) Menopausia tardía.

686: En una paciente sometida a conización por sospecha de neoplasia intraepitelial cervical escarnosa de alto grado (CIN III), la Anatomía Patológica nos informa de zonas de invasión del estroma con una profundidad de 6 mm. ¿Cómo denominaríamos a este caso?:

a) Carcinoma in situ. b) Estadio Ial de carcinoma de cuello. c) Estadio Ia2 de carcinoma de cuello. d) Estadio Ib de carcinoma de cuello.

687: ¿En cuál de las siguientes circunstancias NO existe riesgo aumentado para utilizar contraceptivos orales?:

a) Paciente fumadora de más de 35 años. b) Antecedentes de Hepatitis A. c) Antecedentes de Trombosis venosa profunda. d) Antecedentes de Colostasis intrahepática.

688: Paciente nuligesta de 23 años diagnosticada de tumoración ovárica ecográficamente sospechosa de malignidad. Se informa de la posibilidad de realizar cirugía conservadora en caso de cumplirse ciertos requisitos indispensables. ¿Cuál de los siguientes supuestos desaconsejaría dicha actitud quirúrgica?:

a) Ca. Epitelial infiltrante bien diferenciado. b) Biopsia de Epiplon negativa. c) Tumor encapsulado. d) Ascitis.

689: Los Koilocitos son células patognomónicas de infección por:

a) Virus herpes simple 1 b) Virus herpes simple 2 c) Papiloma virus humano. d) Gonococia.

690: La forma más frecuente de hiperplasia suprarrenal congénita es la deficiencia de:

a) 17-hidroxilasa. b) 2l-hidroxilasa. c) ll-hidroxilasa. d) 3-beta-hidroxiesteroide.

691: El agente más frecuente de sepsis de comienzo precoz en el recién nacido es:

a) Virus Herpes Simple. b) Estreptococo del grupo B. c) Cándida albicans. d) Stafilococus aureus.

692: Un niño de 7 meses presenta fiebre alta desde tres días antes, acompañada de hiperemia faríngea. La fiebre cede al cuarto día de la enfermedad, momento en que aparece un exantema morbiliforme, que desaparece en un plazo de 3 días. El diagnóstico

Page 142: Cuestionario de Urología

más probable es:

a) Sarampión. b) Rubéola. c) Reacción medicamentos a a antitérmicos. d) Infección por herpes virus humano 6.

693: Un médico, que no dispone de tablas de crecimiento y desarrollo, necesita valorar el peso de un niño de 3 años. ¿Qué fórmula entre las siguientes, debería utilizar para calcular el peso medio de un niño normal de esa edad?:

694: ¿Cuál de las siguientes afirmaciones es verdadera en relación con la apendicitis aguda en el niño?:

a) Suele comenzar con dolor en fosa ilíaca derecha. b) La ausencia de fiebre y la existencia de menos de 10.000 leucocitos hacen bastante improbable el diagnóstico de apendicitis aguda. c) El enema opaco es una exploración de gran ayuda para establecer el diagnóstico en el niño. d) En el lactante la perforación intestinal es muy rara.

695: Señale cuál de los siguientes NO es factor de riesgo para contraer una nasofaringitis o catarro común:

a) Asistencia a guardería. b) Aspiración pasiva del humo del tabaco. c) Bajo nivel económico familiar. d) Exposición a baja temperatura ambiental.

696: En los niños que asisten a guarderías han sido comunicados contagios de las siguientes enfermedades infecciosas, EXCEPTO de:

a) Otitis media aguda. b) Hepatitis A. c) Infección por VIH. d) Varicela.

697: ¿Qué vacuna desaconsejaría en un niño en situación de inmunodeficiencia clínica significativa?:

a) DTP (Difteria, Tétanos, Tos ferina). b) Hepatitis B. c) Haemophilus Injluenzae B. d) Triple vírica (Sarampión, Rubéola, Parotiditis).

698: Señale cuál de los siguientes agentes patógenos es el principal causante de otitis media aguda en el niño:

a) Haemophilus Injluenzae no tipable. b) Pseudomona aeruginosa. c) Micoplasma pneumoniae. d) Streptococcus pneumoniae.

699: Un niño capaz de correr, subir escaleras solo sin alternar los pies, hacer una torre de seis cubos y elaborar frases de tres palabras, presenta un desarrollo psicomotor propia de la siguiente edad:

a) 15 meses. b) 18 meses. c) 24 meses d) 36 meses.

700: ¿En cuál de las siguientes enfermedades metabólicas NO hay hiperamoniemia?:

a) Acidemia propiónica. b) Déficit de Ornitin Transcarbamilasa. c) Hiperglicinemia no cetósica. d) Fenilcetonuria.

701: La enfermedad de Hirschsprung o Megacolon Congénito:

a) Puede estar asociada a una mutación del protoogen RET del cromo soma 10, con una penetrancia incompleta en la mayoría de los casos. b) Se asocia a una ausencia de células gangionares en el plexo de Meissner y de Auerbach del intestino. c) Se asocia a una ausencia de fibras colinérgicas y adrenérgicas en el segmento intestinal del afectado. d) No se asocia a un retraso en la emisión del meconio en el período neonatal.

702: La ecograffa prenatal realizada a una mujer gestante de 29 semanas, muestra a un feto con una ectasia pieloureteral izquierda importante:

a) Debe de provocarse el parto urgentemente para el tratamiento precoz de una malformación del tracto urinario. b) Debe realizarse una punción percutánea del riñón fetal afectado, para colocar un catéter que facilite el drenaje urinario nefroarnniótico. c) Debe de mantenerse la gestación a término y realizar un tratamiento quirúrgico sobre el riñón izquierdo, en las primeras 24 horas de vida.

a) Edad en años x 8 x 1/2 =

b) Edad en años x 80 x 1/2 =

c) (Edad en años x 2) + 8 =

d) Edad en meses x 10 x 1/2 =

Page 143: Cuestionario de Urología

d) Debe de mantenerse la gestación a término y realizar al nacimiento, una ecografía abdominal, una urografía intravenosa y una uretrocistografía miccional, para el diagnóstico diferencial de la anomalía.

703: La hernia inguinal del recién nacido:

a) Se debe a una debilidad de la pared posterior del conducto inguinal. b) Es más frecuente en el sexo femenino. c) Se debe a un déficit transitorio de la secrección de Gonadotrofinas entre la l0a y l2a semana de gestación. d) Se produce por la persistencia del conducto peritoneo vaginal (Processus vaginalis) durante la vida postnatal.

704: Señalar la afirmación FALSA con respecto a Legionella:

a) Su hábitat natural es el agua. b) Es un cocobacilo Gram negativo nutricionalmente exigente y anaerobio. c) El diagnóstico de legionelosis puede establecerse mediante la detección de antígeno de Legionella en orina. d) Son activos frente a esta bacteria los macrólidos y algunas quinolonas.

705: Cuál de los microorganismos siguientes NO produce enteritis por mecanismo invasor:

a) Shigella dysenteriae. b) Campylobacter jejuni. c) Escherichia coli enteroinvasor. d) Bacillus cereus.

706: Cuál de los siguientes microorganismos NO produce síndrome mononucle6sico:

a) Virus de la inmunodeficiencia humana (VIH). b) Toxoplasma gondii. c) Virus de Epstein-Barr. d) Listeria monocytogenes.

707: Durante el primer encuentro con un antígeno se produce una respuesta inmune llamada primaria, mientras que en contactos sucesivos con el mismo antígeno se produce una respuesta secundaria. Respecto a este fenómeno se realizan a continuación varias afirmaciones de las que una es FALSA. Señálela:

a) La respuesta primaria está mediada por células nativas o vírgenes y la respuesta secundaria, lo está por células memoria. b) La respuesta secundaria se desencadena de una manera más rápida que la respuesta primaria. c) La tirosina fosfatasa CD45, que regula la activación celular se expresa en la superficie de las células T memoria en su versión CD45RA. d) En la respuesta secundaria se produce un mayor número de linfocitos y las células B producen mayores niveles de anticuerpos con una mayor afinidad por el antígeno.

708: En relación a la respuesta inmune, señale cuál es la respuesta verdadera:

a) Los antígenos que de manera más frecuente desencadenan una respuesta inmune son los timoindependientes, y entre ellos los polisacáridos de las paredes bacterianas. b) Un antígeno puede ser procesado por las células dendríticas y por los linfocitos B, y ser presentado a los linfocitos T CD4 en el contexto de moléculas del complejo mayor de histocompatibilidad de clase I. c) Un antígeno puede ser procesado por los macrófagos y presentado a los linfocitos T CD8 en el contexto de moléculas del complejo mayor de histocompatibilidad de clase II. d) El linfocito T CD4 reconoce antígenos en el contexto de moléculas del complejo mayor de histocompatibilidad del clase II y como resultado expresa moléculas coestimuladoras como CD 154 (CD40 ligando).

709: Indique cuál de las siguientes afirmaciones es INCORRECTA, en relación con las inmunoglobulinas:

a) Están formadas por dos cadenas ligeras idénticas y dos cadenas pesadas idénticas. b) Disponen de dominios variables e hipervariables para ligar el antígeno. c) La zona constante de las cadenas ligeras define la clase y subclase del anticuerpo. d) La zona constante de las cadenas pesadas es capaz de unirse al complemento.

710: ¿Cuál de las siguientes afirmaciones, relativas a la inmnoidad innata, es INCORRECTA?:

a) Son componentes de la misma el complemento, las proteínas de fase aguda y las citocinas. b) Su componente celular incluye células fagocíticas, células liberadoras de mediadores inflamatorios, células natural Killer, y linfocitos T. c) Algunos de los tipos celulares implicados en la respuesta innata actúan también como células presentadoras de antígeno. d) La respuesta inmune innata se repite sin cambios independientemente del número de veces que se encuentre el antígeno.

711: Indique cuál de los siguientes afluentes venosos NO pertenece al sistema de la vena porta:

a) Vena mesentérica superior. b) Vena gastroepiploica inferior. c) Vena mesentérica inferior. d) Vena hemorroidal inferior.

712: Indique cuál de los siguientes elementos NO se sitúa en el mediastino posterior:

a) Aorta torácica descendente. b) Vena ácigos. c) Nervios esplácnicos. d) Vena cava superior.

713: Indique la afirmación FALSA con relación al pedículo hepático:

a) El pedículo hepático está fomado por la arteria hepática, vena porta, colédoco y nervios hepáticos. b) La vena porta asciende entre las hojas del epiplon gastrohepático. c) La arteria hepática propia y sus ramas siguen un curso variable en el hilio hepático.

Page 144: Cuestionario de Urología

d) El colédoco se sitúa entre las hojas del epiplon gastrocólico en su trayecto hacia el duodeno.

714: Señale cuál de las siguientes afirmaciones sobre la topografia encefálica NO es correcta:

a) La cápsula extrema se sitúa entre el claustrum o antemuro y la corteza del lóbulo de la ínsula. b) El núcleo tálamo óptico se sitúa lateralmente o externo al núcleo caudado. c) La cápsula externa se sitúa por fuera del núcleo putámen. d) El tálamo óptico forma parte del suelo del ventriculo lateral.

715: Señale la afirmación FALSA con relación a la vascularización cardíaca:

a) El borde agudo del corazón está vascularizado por la arteria coronaria derecha. b) La arteria interventricular anterior o descendente anterior es rama de la arteria coronaría derecha. c) Las arterias coronarías son ramas de la aorta torácica ascendente. d) El tabique interventricular o septo está irrigado por las arterias coronarías derecha e izquierda.

716: La secreción del ácido en la célula parietal gástrica se lleva a cabo por una A TPasa especffica que intercambia hidrogeniones (H+) del citosol por:

a) Cl. b) HCO3 c) Na+ d) K+

717: ¿Cuál de los siguientes factores contribuye a que durante noa isquemia coronaria grave se produzca vasoconstricción?:

a) Aumento de K+ extravascular. b) Liberación de adenosina. c) Producción de radical superóxido por la xantina oxidasa. d) Liberación de óxido nítrico.

718: ¿Cuál de los siguientes NO se considera un criterio de identificación de una sustancia como neurotransmisor?:

a) La neurona presináptica debe contener los mecanismos bioquímicos necesarios para su síntesis. b) Es necesario identificar el RNA mensajero específico para dicha sustancia en la neurona presináptica. c) Debe existir un mecanismo bioquímico capaz de finalizar l a acción de la sustancia. d) La estimulación del terminal presináptico debe liberar la sustancia.

719: En el cerebro, las principales agrupaciones de cuerpos celulares que contienen serotonina están situados en una de las siguientes zonas:

a) Núcleos arcuatos y periventriculares del hipotálamo. b) Núcleo de la banda diagonal de Broca. c) Sustancia negra del mesencéfalo. d) Núcleos del rafe del tronco encefálico.

720: ¿Qué tipo de proteína se deposita en la amiloidosis asociada a hemodiálisis?:

a) AA. b) AL. c) β2 microglobulina. d) Transtiretina mulada.

721: ¿Cuál de los tipos histológicos de cáncer de mama presenta las siguientes características: Buena delimitación, grupos celulares sincitiales, células grandes con macronucleolo, abundantes mitosis y moderado a denso infiltrado inflamatorio linfoplasmacitario circundante?:

a) Carcinoma ductal convencional. b) Carcinoma lobulillar "in situ". c) Carcinoma mucinoso. d) Carcinoma medular.

722: Mujer de 25 años con adenopatías cervicales y mediastínicas. Se obtiene una biopsia de una de las adenopatías cervicales. La descripción histopatológica de la muestra fijada en formol e incluida en parafina, refiere la alteración de la arquitectura ganglionar por la presencia de una fibrosis birrefringente con luz polarizada, que forma nódulos y con células que presentan un espacio claro circundante con núcleos grandes junto a algunas células grandes de núcleo bilobulado y nucléolos eosinófilos en espejo. El resto corresponde a una población linfoide reactiva con eosinófilos. Dicha descripción corresponde a:

a) Linfoma no Hodgkin de alto grado. b) Linfoma no Hodgkin de bajo grado. c) Sarcoidosis. d) Enfennedad de Hodgkin tipo esclerosis nodular.

723: El hallazgo morfológico que permite establecer con certeza el diagnóstico histológico de hepatitis crónica es:

a) Aparición de plasmáticas en el infiltrado. b) Presencia de folículos linfoides. c) Pérdida de conductos biliares. d) Fibrosis portal.

724: La substitución de una célula epitelial adulta, por otra célula adulta, de un tipo diferente, es frecuente el fenómeno inicial en el desarrollo de un carcinoma. A este proceso se le denomina:

a) Metaplasia. b) Anaplasia. c) Displasia. d) Distrofia.

Page 145: Cuestionario de Urología

725: Indique cuál de las siguientes lesiones pigmentarias, constituye con mayor frecuencia un precursor potencial del melanoma cutáneo.

a) Nevus congénito. b) Nevus azul. c) Nevus de células fusifonnes yepitelioides. d) Nevus displásico.

726: ¿Cuál de los siguientes fármacos inmunosupresores utilizados en el trasplante renal para prevenir el rechazo es nefrotóxico?:

a) Azatioprina. b) Micofenolato Mofetil. c) Glucocorticoides. d) Ciclosporina.

727: ¿Cuál de los siguientes fármacos utilizables en el tratamiento de la úlcera péptica, debe administrarse con precaución, por su potencial abortivo en las mujeres fértiles?:

a) Hidróxido de Aluminio. b) Ranitidina. c) Trisilicato de magnesio. d) Misoprostol.

728: Una mujer que está tomando anticonceptivos orales comienza un tratamiento de Rifampicina que durará algunos meses. ¿De entre las siguientes consideraciones cuál le parece adecuada? :

a) Que suspenda los anticonceptivos orales porque aumen tan los niveles de Rifampicina favoreciendo su toxicidad. b) Que tome precauciones adicionales para evitar un embarazo ya que la Rifampicina disminuye el efecto de los an-ticonceptivos al aumentar su metabolismo. c) Que tome precauciones adicionales para evitar un embarazo ya que la Rifampicina disminuye el efecto de los an-ticonceptivos al reducir su absorción. d) Que la Rifampicina aumenta la toxicidad de los anticonceptivos orales y por tanto tiene ahora un riesgo aumentado de enfennedad tromboembólica.

729: ¿Cuál de los siguientes cambios farmacocinéticos tiene lugar en el paciente anciano?:

a) Disminución del volumen de distribución de los fármacos liposolubles. b) Disminución de la biodisponibilidad de los fánnacos con alta tasa de extracción hepática. c) Disminución de la unión de los fánnacos a la albúmina. d) Disminución de la velocidad de las reacciones de fase II a nivel hepático.

730: ¿En qué tipo de fármacos NO suele tener utilidad clínica la determinación de sus concentraciones séricas? :

a) Los que tienen un margen terapéutico estrecho. b) En los que existe una buena relación entre la concentración plasmática y el efecto. c) En los que la relación entre la dosis y la concentración sérica es predecible. d) En aquellos con efectos tóxicos iniciales difíciles de evaluar.

731: ¿Cuál de los siguientes medicamentos se elimina mejor por hemodiálisis?:

a) La heparina no fraccionada que tiene un alto peso molecular, se une extensamente a proteínas y tiene un volumen de distribución pequeño. b) El diazepan que es liposoluble, se une ampliamente a proteínas y se excreta en pequeña cantidad por riñón. c) El propranolol que tiene una alta unión a proteínas plasmáticas, un elevado volumen de distribución y un peso molecular bajo. d) La amoxicilina que tiene una baja unión a proteínas plasmáticas, un volumen de distribución pequeño y se excreta fundamentalmente por el riñón.

732: La escalera analgésica de la Organización Mundial de la Salud es un método para el ajuste racional de la analgesia. De las siguientes afirmaciones sobre dicha escalera, señale la correcta:

a) Consta de varios escalones dependiendo de los fármacos utilizados, con un máximo de seis escalones. b) Permite la utilización conjunta de opiáceos débiles (codeína) y potentes (morfina) en dolores de intensidad severa. c) Si se administra conjuntamente ibuprofeno, dexametasona y amitriptilina, se considera que se está utilizando el primer escalón de la escalera analgésica. d) No se permite la utilización de fármaco s adyuvantes de! dolor en el tercer escalón analgésico.

733: Con respecto a los Sistemas de Información en Salud Pública es FALSO que:

a) Los registros hospitalarios de cáncer permiten conocer la incidencia, la población atendida de un hospital y el nivel de efectividad de su tratamiento en términos de supervivencia. b) El CMBD (conjunto mínimo básico de datos) es una fuente de datos individuales sobre la salud de las personas generado por las actuaciones administrativas de los servicios sanitarios. c) Las encuestas de salud son herramientas de la medición del nivel de salud y de utilización de los servicios sanitarios de una comunidad. d) En España se realiza el padrón habitualmente aquellos años que terminan en 1 y en 6 y su acceso, con algunas limitaciones, es público.

734: Con respecto a la comparación de mortalidad entre distintas poblaciones, es FALSO que:

a) La Razón de mortalidad proporcional estandarizada es útil cuando no se conoce la población expuesta. b) Un valor de RME (razón de mortalidad estandarizada) igual a 1 significa que la frecuencia de la enfermedad es igual en la población a estudio y en la población de referencia. c) La razón de mortalidad proporcional permite comparar el riesgo de una determinada causa respecto del total de causas. d) Cuando las tasas de una determinada enfermedad en las dos poblaciones que comparamos son iguales se puede estimar el riesgo relativo (RR) en un estudio de mortalidad proporcional.

Page 146: Cuestionario de Urología

735: Con relación a las características generales de las vacunas, es FALSO que:

a) El término de inmunización es más amplio que el de vacunación y hace referencia al proceso de inducción o provisión de inmunidad artificial. b) La inmunización pasiva consiste en la administración al individuo sano susceptible de anticuerpos producidos en otro huésped con el objeto de proporcionarle una protección inmediata aunque temporal. c) La clasificación sanitaria de las vacunas se basa en los objetivos epidemiológicos que se pretenden alcanzar con la aplicación de las vacunaciones a la población. d) La vacuna antitetánica es una vacuna sistemática que tiene como objetivo proteger a la comunidad y no una protección individual.

736: Después de informar de su situación a un enfermo de cáncer incurable, es inexcusable:

a) Ofrecer garantías de continuidad en los cuidados a él y a sus familiares. b) Llamar por teléfono a los familiares para que estén preparados cuando el paciente llegue a casa. c) Pautarle durante los primeros días algún psicofármaco para atenuar e! impacto recibido. d) Sugerirle que inmediatamente "arregle los papeles".

737: Cuando un enfermo nos pregunta que cuánto tiempo le queda de vida, procuraremos:

a) Ser lo más exactos posibles, de acuerdo con la bibliografía existente hasta el momento. b) Aunque lo sepamos a ciencia cierta, sólo se lo diremos al familiar de referencia, nunca al enfermo. c) Decirle una cifra menor de la que esperamos para que se lleve una agradable sorpresa al comprobar que vive más tiempo del esperado. d) Ser ambiguos en la respuesta y hablar de "expectativas limitadas" en vez de cifras exactas. .

738: ¿Por qué es importante estandarizar las tasas de mortalidad por edad?:

a) Para ofrecer una información más detallada. b) Para conseguir tasas específicas por grupos de edad. c) Para obtener una cifra real de mortalidad. d) Para poder comparar tasas de diferentes lugares eliminando la confusión que genera el factor edad.

739: La incidencia acumulada de una enfermedad es:

a) La proporción de la población que padece la enfennedad en un momento dado. b) La proporción de sanos que contraen la enfennedad a lo largo de cierto período. c) La tasa de incidencia de la enfennedad. d) Una razón entre la odds de exposición de los casos y la odds de exposición de los controles.

740: ¿Cuál de las siguientes afirmaciones es INCORRECTA para considerar que un estudio científico sobre pruebas diagnósticas aporta una buena evidencia?:

a) Se trata de una comparación ciega e independiente entre la prueba a estudio y el estándar seleccionado. b) Se ha reducido el espectro de enfennedad para poder valorar a pacientes en situación de alto riesgo. c) Se aplicó el patrón de referencia independiente del resultado de la prueba diagnóstica. d) El tamaño muestral fue calculado para los índices de sensibilidad y especificidad.

741: ¿Cuál de las siguientes afirmaciones es verdadera respecto a los factores de confusión en un estudio observacional?:

a) Se asocian a una posible causa y constituyen un factor de riesgo para el efecto estudiado. b) Son equivalentes a los sesgos de infonnación no diferencial. c) Se corrigen aumentando el tamaño de la muestra. d) Se evitan con un buen entrenamiento de los investigadores que participan en el estudio.

742: En 1980, 89.538 enfermeras de EE.UU. entre 34 y 59 años y sin historia previa de cáncer, cumplimentaron un cuestionario dietético validado previamente y diseñado para medir el consumo individual de grasa total, grasa saturada, ácido linoleico y colesterol, así como otros nutrientes. Las enfermeras se clasificaron entonces en cinco grupos de igual tamaño según los niveles de su ingesta de grasa. Se comparó la aparición de nuevos casos de cáncer de mama entre los grupos. El diseño del estudio es:

a) Ecológico o de base poblacional. b) De prevalencia. c) Ensayo clínico. d) Cohortes.

743: Se evalúa la validez de criterio de una nueva prueba de hibridación para la detección de la infección por virus del papiloma humano (VPH) en el cuello uterino. El estándar es la hibridación por transferencia de Southern. En la mnestra de 400 sujetos se obtienen resultados positivos en 70 de las 140 enfermas y en 26 de las sanas. ¿Cuál es la sensibilidad de la prueba?:

a) 0,17. b) 0,3d) c) 0,18. d) 0,50.

744: En un ensayo controlado, indique cuál de los siguientes factores NO condiciona el tamaño de la muestra del estudio:

a) El error aleatorio de primera especie o de tipo I (α) que se acepta en su diseño. b) El error aleatorio de segunda especie o de tipo II (β) que se acepta en su diseño. c) El poder estadístico del estudio. d) El nivel de enmascaramiento de los tratamientos en estudio.

745: Si se quiere realizar un ensayo clínico de diseño cruzado, el estudio NO podrá ser al mismo tiempo:

a) De fase II. b) Con asignación aleatoria. c) De grupos paralelos.

Page 147: Cuestionario de Urología

d) De tamaño de muestra predeterminado.

746: De los siguientes diseños de ensayos clíuicos, indique aquel en el que el tamaño de la muestra no está predeterminado, sino que depende de los resultados que se van obteniendo a lo largo de la experiencia:

a) Ensayo cruzado. b) Ensayo de diseño factorial. c) Ensayo de diseño secuencial. d) Ensayo de grupos paralelos.

747: Suponga que en la lectura de un ensayo clíuico en que se han incluido 50 pacientes por grupo, observa usted que los resultados se refieren a 47 pacientes en un grupo y 42 en el otro. Con respecto a los pacientes que faltan, el autor comuuica que fueron retirados por uo seguir el tratamiento y que no hay diferencias estadísticamente significativas entre grupos en cuanto al porcentaje de pérdidas de pacientes. ¿Cómo cree que podría afectar este hecho a los resultados del ensayo?:

a) De ninguna forma, ya que no se pueden estudiar los efectos de los tratamientos en pacientes que no los reciben. b) La validez del estudio podría ser dudosa, ya que la pérdida de pacientes tal vez esté relacionada con los efectos de los tratamientos. c) De ninguna forma si en el diseño del estudio se habían previsto las pérdidas y, por tanto, se incluyó un número de pacientes mayor para compensarlas. d) Aumentando la probabilidad de un resultado negativo falso al reducirse la muestra, pero sin que este hecho aumente las posibilidades de sesgos.

748: Suponga que en la lectura de un ensayo clínico cuya variable principal de respuesta es la mortalidad a cinco años, observa usted que los pacientes de un grupo tienen una media de edad al inicio del tratamiento de 64 años y los del otro de 52 años, no siendo estadísticamente significativas estas diferencias. ¿Cómo cree que podría afectar este hecho a los resultados del ensayo?:

a) De ninguna fonna, ya que las diferencias iniciales en la edad se deben al azar. b) Invalida el estudio, ya que los grupos no tienen inicial mente el mismo pronóstico. c) Invalida el estudio, ya que significa que la asignación al azar de los pacientes a los grupos (aleatorización) no ha producido grupos inicialmente homogéneos y, por tanto, ha sido incorrecta. d) Haría necesario utilizar alguna técnica estadística que permita comparar la mortalidad entre grupos teniendo en cuenta las posibles diferencias entre ellos en las características basales.

749: En el análisis principal de un ensayo clínico sobre la prevención de la ffipocalcemia neonatal mediante la administración de vitamina D durante la gestación a la madre, se encuentra que los hijos de madres suplementadas tienen mayor calcemia. Pero cuando, en un análisis secundario, se estudian por separado los niños alimentados artificialmente y los que reciben lactancia natural, sólo hay diferencias significativas en el primer subgrupo. ¿Cuál de estas interpretaciones del hecho es correcta?:

a) No es correcto estudiar la interacción entre la intervención y un factor mediante contrastes de hipótesis independientes dentro de los subgrupos fonnados en función de los valores de dicho factor. Por tanto, este estudio no aclara si el efecto de la vitamina D depende del tipo de lactancia. La vitamina D sólo es eficaz en caso de lactancia artificial. b) La vitamina D sólo es eficaz en caso de lactancia artificial. c) Es dudosa la eficacia de la vitamina D, ya que los resultados del estudio carecen de consistencia interna. d) En caso de lactancia natural la vitamina D es también eficaz, pero su eficacia no se detecta ya que al formar subgrupos se reduce el número de pacientes estudiados.

750: Si la intervención A es más eficiente que la intervención B, qniere decir que:

a) A es más barata que B. b) A es más efectiva que B. c) B tiene más efectos adversos. d) Una unidad de resultado conseguida con A es menos costosa que con B.

751: Un estudio de evaluación económica, donde se comparan la urografia intravenosa, la ecografía pieloureteral y el TAC helicoidal, en el que tanto los costes que se originan como los beneficios que se obtienen se valoran en unidades monetarias, se define como:

a) Estudio de coste-beneficio. b) Estudio de coste-utilidad. c) Estudio de coste-efectividad. d) Estudio de coste-eficacia.

752: Medir la utilidad requiere:

a) Conocer los costes intangibles. b) Ajustar los costes y consecuencias en el tiempo. c) Ajustar el tiempo de supervivencia por la calidad de vida. d) Controlar las variables de asignación de costes.

753: La razón fundamental para realizar evaluación económica en salud es que:

a) La medicina es cara. b) Los recursos son escasos. c) La inflación en sanidad es mayor que la inflación media. d) Hay que ahorrar.

754: La diferencia de incidencia entre expuestos y no expuestos a un determinado factor es una medida de:

a) De frecuencia del factor. b) De riesgo asociado a dicho factor. c) De incidencia del factor. d) De impacto del factor.

755: Si un estudio permite hacer inferencias correctas sobre el efecto de un factor de riesgo en los participantes del estudio, diremos que tienen una buena:

Page 148: Cuestionario de Urología

a) Sensibilidad. b) Especificidad. c) Valores predictores. d) Validez interna.

756: ¿Cuál de las siguientes situaciones corresponde a la fase terminal de la enfermedad?:

a) Cáncer de mama con metástasis ósea ganglionares, hepáticas y pulmonares que sólo ha recibido honnonoterapia. b) Cáncer de pulmón intervenido hace un año con recaída local y metástasis hepáticas tratadas con radioterapia y dos líneas de quimioterapia sin respuesta. c) Paciente con cirrosis hepática y episodios de encefalopatía intermitentes en el tiempo. d) Paciente con demencia senil y neumonía aguda que produce insuficiencia respiratoria.

757: La diarrea por déficit de lactasa es:

a) Una manifestación del síndrome de Dumping. b) Una diarrea de tipo secretor. c) Una manifestación del síndrome del intestino corto. d) Una diarrea de tipo osmótico.

758: Señale la respuesta FALSA:

a) Omeprazol es componente habitual de la Triple Terapia de erradicación de Helicobacter pylori. b) Los antagonistas de receptores H2 son útiles en el tratamiento de la úlcera gástrica. c) Sucrafalto se administra tras las comidas, ya que se inactiva por acción del ácido clorhídrico. d) Misoprostol está indicado en el tratamiento de pacientes de úlcera péptica que precisan tomar antiinflamatorios no esteroideos de forma prolongada.

759: Veinticuatro horas tras la dilatación endoscópica de esófago por estenosis, un paciente presenta dolor en el costado derecho, que le impide la respiración profunda, acompañado de fiebre. El estudio radiológico muestra un derrame pleural basal derecho. ¿Qué tipo de resultados, entre los siguientes, espera encontrar en el estudio del líquido pleural?:

a) Abundancia de polinucleares y pH entre 7.20 y 7.40. b) PH inferior a 7 y niveles altos de amilasa. c) PH inferior a 7 y niveles altos de colesterol y triglicéridos. d) Abundantes polinucleares y niveles bajos de glucosa.

760: El pronóstico de un paciente ingresado de urgencia por un episodio de ICC es desfavorable cuando existen todos los factores que a continuación se indican, salvo uno. Señálelo:

a) Fracción de eyección deprimida (<25%). b) Concentraciones séricas bajas de Na y/o K. c) Identificación de factores precipitantes reconocibles. d) Incapacidad para caminar en llano 3 minutos tras la recuperación de la situación aguda.

761: ¿Cuál de las siguientes afirmaciones NO es correcta en relación con la fiebre mediterránea familiar?:

a) La afectación articular puede ocurrir de forma aguda o crónica con afectación predominante de grandes articulaciones. b) La manifestación cutánea más frecuente es el eritema erisipeloide casi siempre localizado en la región anterior de las piernas, sobre tobillos o dorso de pies. c) La amiloidosis sistémica, tipo AA, es una complicación grave de la enfermedad que se manifiesta por nefropatía de aparición precoz y evolución rápida a insuficiencia renal. d) Las primeras manifestaciones de la enfermedad ocurren raramente en personas menores de 20 años.

762: Ante un paciente donante de sangre en el que se obtienen un resultado positivo en la prueba de inmunoabsorción ligada a enzimas (ELISA) para el VIH, la actitud más correcta a tomar es:

a) El resultado positivo en este caso es improbable por lo que se considerará un falso positivo. b) Iniciar cuanto antes un tratamiento antirretroviral. c) Repetir la prueba de ELISA. d) Realizar cultivo de virus a partir del plasma del paciente.

763: En el seguimiento del estado nutricional de un paciente hospitalizado, lo más importante es:

a) Medir semanalmente la cifra de hemoglobina. b) Medir semanalmente los niveles de vitamina Bl2 y ácido fólico. c) Medir semanalmente el índice creatinina-altura. d) Pesar diariamente al paciente.

764: ¿Qué parámetro, entre los siguientes, es el más adecuado para diagnosticar, en un caso de insuficiencia renal aguda, el origen prerrenal de la misma?:

a) Las cifras de creatinina sérica. b) Las cifras de creatinina urinaria. c) Concentración de potasio urinario mayor de 20 mEq/L. d) Concentración de sodio urinario menor de 10 mEq/L.

765: El aumento de la cantidad de vello en las zonas donde es habitual su presencia en la anatomía femenina se denomina:

a) Hirsutismo. b) Virilización. c) Hipertricosis. d) Androgenismo femenino.

766: ¿Qué actuación posee MENOR utilidad en el tratamiento de la acalasia de esófago?:

Page 149: Cuestionario de Urología

a) Dilatación neumática. b) Cardiomiotomía quirúrgica. c) Inyección intraesfinteriana de toxina botulínica. d) Tratamiento endoscópico con Argon.

767: Hombre de 45 años de edad diagnosticado de úlcera péptica duodenal a los 25 años, y que presenta reagudizaciones estacionales que trata con Ranitidinao Coincidiendo con una fase aguda se realiza endoscopia oral que demuestra una úlcera bulbar en cara posterior, de 1 cm. de diámetro. La prueba de ureasa antral es positiva. ¿Cuál, entre los siguientes, le parece el tratamiento más conveniente?:

a)Clavulánico + Ciprofloxacino + Bismuto Coloidal durante 7 días. b) Ciprofloxacino + Bismuto Coloidal + Pantoprazol durante 10 días. c) Amoxicilina + Bismuto Coloidal + Omeprazol durante 20 días. d) Metronidazol + Claritromicina + Pantoprazol durante 7 días.

768: Uno de los siguientes datos clínicos NO es sugerente del síndrome de colon irritable:

a) Presencia de moco en las heces. b) Dolor recurrente en hipogastrio. c) Alternancia de diarrea/estreñimiento. d) Diarrea nocturna.

769: Paciente de 38 años, con fenómeno de Raynaud y esclerodactilia, presenta diarrea de heces pastosas, en número de 2-3 deposiciones/día, pérdida de 7 kg de peso, anemia con volumen corpuscular medio de 112 fl, vitamina B12 en sangre, 70 pg/ml (normal, 200-900 pg/ml), ácido fólico sérico, 18 ng/ml (normal, 6-20 ng/ml), grasas en heces, 13 g/día. La prueba con mayor sensibilidad, específica y sencilla para el diagnóstico del síndrome digestivo que padece este enfermo es:

a) Anticuerpos antiendomisio tipo IgA. b) Prueba del aliento con 14C-D-xilosa. c) Determinación de la lactasa en la mucosa intestinal. d) Prueba del aclaramiento de la α1-antitripsina en heces.

770: En enero de 2004 acude a nuestra consulta una mujer de 62 años, con una estenosis mitral en fibrilación auricular, en tratamiento con Digoxina y Acenocumarol, entre cuyos antecedentes destaca una histerectomía simple realizada en 1995 por metropatía hemorrágica. En el curso de la intervención fue preciso aplicarle una transfusión de sangre. Ahora viene por presentar aumento progresivo del perímetro abdominal sin quebrantamiento del estado general. Entre los datos de la exploración ÍIsica comprobamos un abdomen distendido, no tenso, con oleada. El borde iuferior hepático se palpa a 3 cm. del reborde costal. En decúbito supino, la percusión a nivel del mesogastrio es mate, y en los flancos es timpánica, signos que no se modifican al adoptar la paciente el decúbito lateral. No se aprecian edemas en miembros inferiores. ¿Cuál entre los siguientes, le parece el diagnóstico más verosímil?:

a) Quiste simple de ovario. b) Cirrosis hepática por virus C. c) Infiltración peritoneal por siembra carcinomatosa. d) Insuficiencia ventricular derecha.

771: Un paciente de 78 años de edad, previamente sano, que vive en una Residencia de Ancianos bastante masificada y con insuficientes recursos higiénicos, padece un cuadro diarreico desde hace 6 semanas. Refiere molestias abdominales tipo retortijón, febrícula ocasional y 4-6 deposiciones diarias, alguna de ellas nocturna, con mucosidad y, en ocasiones, con hebras de sangre. Entre los diagnósticos que se enumeran a continuación seleccione el que le parece MENOS probable:

a) Cáncer de colon. b) Infección por Clostridium Difficile. c) Enfermedad de Crohn. d) Salmonelosis.

772: Una mujer de 55 años diagnosticada de proctocolitis ulcerosa (con pancolitis) hace 17 años. En la última Colonoscopia realizada de revisión se observa: desaparición de las haustras, con pérdida del patrón vascular, pseudopólipos dispersos y a nivel de sigma un área de disminución de la luz con estenosis que se biopsia. El estudio histológico muestra displasia severa de alto grado. Indique cuál de las siguientes respuestas es la actitud más adecuada:

a) Tratamiento con corticoides y valorar respuesta a los 3 meses. b) Tratamiento con inmunosupresores e Infliximab y repetir biopsia al mes. c) Tratamietno con inmunosupresores y realizar hemicolectomía izquierda. d) Pan-proctocolectomía con ileo-anastomosis con reservorio.

773: Mujer de 58 años, sin hábitos tóxicos ni antecedentes patológicos de interés, que desde hace dos meses refiere astenia franca y ningún síntoma digestivo. Toma ibuprofeno de forma ocasional por dolores lumbares inespecíficos. La analítica revela Hb 9 g/dL, Hematrocrito 29%, VCM 79 fl y Ferritina 14 µg/dl. ¿Cuál de las entidades que a continuación se mencionan NO debe in-cluirse en el diagnóstico diferencial de este caso?:

a) Lesiones agudas o crónicas de la mucosa gastrointestinal secundarias al consumo de AINE. b) Cáncer colorrectal. c) Angiodisplasia de colon. d) Gastritis atrófica.

774: Solo una de las siguientes caracterÍsticas es propia del síndrome de Budd-Chiari:

a) La mayor parte de los casos son de origen congénito. b) No suele complicarse con hemorragia por rotura de varices. c) Su frecuencia es elevada en sujetos anticoagulados condicumarínicos. d) Se acompaña de ascitis con alto contenido en proteínas (>3 g/dI).

775: Ante un individuo de 25 años, asintomático y con elevación de la concentración sérica de bilirrubina no conjugada (inferior a 5 mgldl), ausencia de hemólisis y normalidad de la función hepática, ¿cuál es el diagnóstico más probable?:

Page 150: Cuestionario de Urología

a) Hepatitis aguda por el virus de la hepatitis A. b) Anemia falciforme. c) Síndrome de Gilbert. d) Coledocolitiasis.

776: ¿Cuál de las siguientes respuestas NO es cierta en la Hepatopatía alcohólica?:

a) La a-glutamiltranspeptidasa sérica es un marcador biológico, no exclusivo de consumo de etanol. b) La relación GOT:GTP es generalmente <1. c) En la fase de cirrosis, existe un aumento en la incidencia de Hepatocarcinoma. d) Progresa más rápidamente en las mujeres.

777: Paciente de 45 años con antecedentes etílicos cuyo padre falleció por enfermedad hepática no alcohólica. Consulta por dolores articulares, encontrándose en la exploración: hepatomegalia, pérdida del vello corporal y atrofia testicular. Entre los datos analíticos destaca: glucosa basal 180 mgs/dl, GOT y GTP 3 veces por encima de los valores normales de referencia; HbsAg negativo; Anti-HVC negativo; Fe sérico 210 mcgr/dl (normal: 105+40). ¿Qué determinación confirmaría probablemente el diagnóstico?:

a) Anticuerpos anti LKM. b) La tasa de Uroporfirina en orina. c) Los niveles séricos de ferritina. d) La alfa-feto proteína en plasma.

778: Una mujer de 45 años ha presentado varios episodios de pancreatitis aguda documentada radiológicamente con tomografía computarizada. No ingiere alcohol, ni está sometida a medicación alguna, no refiere antecedentes familiares de enfermedad pancreática y las cüras de colesterol y triglicéridos siempre han sido normales. En el último ingreso, exceptuando la hiperamilasemia y una leve alteración de la bioquímica hepática, el resto de los parámetros analíticos fueron normales. La ecografía abdominal fue, como en las ocasiones previas, normal. ¿Cuál, de las que se expresan a continuación, cree que es la etiología más probable de la que es la etiología más probable de la pancreatitis aguda en esta paciente?:

a) Infecciones virales. b) Vasculitis. c) Insuficiencia renal. d) Microlitiasis vesicular.

779: Un hombre de 45 años acude a Urgencias porque lleva 6 horas con dolor abdominal continuo con exacerbaciones localizado en epigastrio e hipocondrio derecho, junto con náuseas y dos episodios de vómitos alimentariobiliosos. No es bebedor habitual. En los últimos meses ha tenido molestias similares pero más ligeras y transitorias, que no póne en relación con ningún factor desencadenante. Exploración: obesidad, dolor a la palpación profunda bajo reborde costal derecho, signo de Murphy negativo. Ruidos intestinales disminuidos. Analítica: AST (GTP) 183 u.i./I y amilasemia 390 u.i./I, bilirrubina Total de 2,4 mgr., resto normal. Ecografía: colelitiasis múltiple sin signos de colecistitis, colédoco dilatado (14 mm de diámetro) basta su porción distal donde hay una imagen de 1,5 mm, bien delimitada que deja sombra acústica. ¿Cuál sería su actitud?:

a) Tratamiento conservador hasta que se resuelva la pancreatitis. b) Laparotomía con colecistectomía y exploración del colédoco. c) Colangiopancreatografía retrógrada endoscópica (CPRE) con papilotomía para tratar de extraer el cálculo y posteriormente colecistectomía programada. d) Ácido ursodesoxicólico a dosis de 300 mg/8 horas por vía oral.

780: ¿Cuál de las siguientes afirmaciones es verdadera con respecto a la diverticulitis perforada localizada?:

a) Suele asentar en el ciego. b) La edad de presentación más frecuente es entre la segunda y la quinta década de la vida. c) La prueba diagnóstica más adecuada es la Tomografía Axial Computerizada de abdomen. d) El tratamiento de elección es la colectomía subtotal.

781: Hombre de 70 años de edad con antecedentes de demencia senil, estreñimiento crónico y abuso de laxantes que presenta náuseas y dolor abdominal. A la exploraci6n el abdomen está distendido y doloroso a la palpación, sobre todo en el flanco izquierdo, con percusión timpánica y ruidos "metálicos". En la radiografía de abdomen se aprecia gran distensión del colon, que se incurva produciendo una imagen en "grano de café". El diagnóstico más probable en este paciente es:

a) Síndrome de Ogilvie. b) Diverticulitis aguda. c) Vólvulo de ciego. d) Vólvulo de sigma.

782: ¿Cuál de las siguientes afirmaciones respecto a la valoración del dolor abdominal agudo en el anciano es cierta?:

a) Comparando con los jóvenes, en los pacientes ancianos son menos frecuentes problemas como la apendicitis o colecistitis. b) Para una misma causa de abdomen agudo, los ancianos tienen el mismo pronóstico que los jóvenes. c) Comparado con los jóvenes, los ancianos tienen menos dolor y sensibilidad a la palpación abdominal pero es más probable que tengan fiebre y leucocitosis. d) En los ancianos, la forma atípica de presentación de la enfermedad y el retraso en el diagnóstico conducen a una mayor mortalidad que en los jóvenes.

783: Paciente de 55 años de edad que fue intervenido de un tumo de colon sigmoide. Se trataba de un adenocarcinoma de 3 cm. que invadía la pared del colon sobrepasando la muscular; también presentaba un ganglio epicólico metastatizado (Asher-Coller B2). ¿Cuál de las opciones terapéuticas que a continuación se mencionan es la más eficaz en este caso?:

a) Bastaría con el tratamiento quirúrgico realizado. b) Radioterapia coadyuvante. c) Realizar quimioterapia postoperatoria (S-Fluorouracilo y Ácido fólico). d) Asociar S-Fluorouracilo y Radioterapia.

784: En un paciente de 66 años, diabético no insulinodependiente, al que se le ha realizado una endoscopia, se ha detectado una

Page 151: Cuestionario de Urología

lesión extensa en el cuerpo gástrico. La biopsia viene informada como adenocarcinoma de tipo intestinal ¿Cuál de las siguientes afirmaciones es más correcta?:

a) Unos valores de CEA, CA 19.9 Y CA 72.4 en el rango de la normalidad nos excluyen la presencia de enfermedad diseminada. b) Es obligatorio realizar un tránsito gastrointestinal para verificar la extensión. c) El siguiente paso es la realización de ecografía y TAC abdominal pélvico. d) Solicitaremos una ecografía endoscópica para descartar la presencia de metástasis hepáticas y pancreáticas.

785: En la pancreatitis crónica que exige tratamiento quirúrgico por la intensidad del dolor y el deterioro general hay un parámetro decisivo a la hora de elegir entre una técnica derivativa o una técnica de resección glandular. ¿Cuál es?:

a) Diámetro del conducto de Wirsung y del colédoco. b) Tamaño global del páncreas. c) Antigüedad de la enfermedad. d) Existencia de diabetés.

786: En relación a los factores etiológicos y manifestaciones clínicas del carcinoma hepatocelular las siguientes afirmaciones son correctas EXCEPTO:

a) Asienta sobre una hepatopatía crónica en el 90% de los casos y se ha descrito la implantación tumoral sobre cirrosis de cualquier etiología. b) Es más frecuente en el sexo masculino y edad superior a 50 años. c) En menos del 10% de los casos la primera manifestación es un hemoperitoneo secundario a la rotura del tumor a cavidad peritoneal. d) La determinación de niveles de alfa-fetoproteina se utiliza como screening en la población para el diagnóstico precoz de tumores de pequeño tamaño.

787: Una mujer de 44 años acudió al área de Urgencias de un hospital por disnea y palpitaciones. La exploración física muestra ausencia de ondas "a" del pulso venoso. La auscultación cardiaca es típica de la estenosis mitral. ¿Cuál de las siguientes respuestas es obligadamente FALSA en la exploración de esta paciente?:

a) El primer tono cardíaco se oye fuerte. b) La intensidad del pulso carotideo, es variable. c) Puede auscultarse un chasquido de apertura, inmediatamente antes del soplo mesodiastólico. d) El soplo diastólico finaliza en una acentuación presistólica.

788: ¿Cuál de las siguientes combinaciones forma el trípode en el que asienta el tratamiento farmacológico de la mayoría de pacientes con insuficiencia cardiaca y disfunción ventricular?:

a) Betabloqueantes, calcioantagonistas e inhibidores de la enzima conversiva de la angiotensina (IECA). b) IECA, diuréticos y calcioantagonistas. c) Diuréticos, betabloqueantes e IECA. d) Betabloqueantes, antagonistas de los receptores de la angiotensina (ARA-U) e IECA.

789: ¿Cuál de los siguientes sistemas neurohormonales que están activados en pacientes con Insuficiencia Cardiaca, NO aumenta las resistencias vasculares sistémicas?:

a) Renina angiotensina. b) Péptidos Natriuréticos. c) Hormona antidiurética. d) Actividad Adrenérgica.

790: Un paciente de 80 años acude al médico tras haber presentado un sincope brusco mientras subía un tramo de escaleras. La exploración física muestra un soplo sistólico eyectivo 3 sobre 6, y en el electrocardiograma se observa un ritmo sinusal normal y signos de hipertrofia del ventrículo izquierdo. ¿Qué exploración diagnóstica solicitaría en primer lugar?:

a) Un test en tabla basculante. b) Un Holter de 24 horas. c) Un ecocardiograma-Doppler. d) Un estudio electrofisiológico.

791: ¿Cuál de estas afirmaciones es FALSA en relación con la miocardiopatía hipertrófica?:

a) Los sujetos jóvenes con esta enfermedad y antecedentes familiares de muerte súbita son candidatos a la implantación de un desfibrilador automático. b) La fibrilación auricular es frecuente en esta enfermedad. c) El tratamiento de elección de los pacientes con miocardiopatía hipertrófica obstructiva en ritmo sinusal e insuficiencia cardíaca es digoxina por vía oral. d) La fibrilación auricular es en estos pacientes un factor precipitante de insuficiencia cardiaca.

792: Una mujer de 72 años acude a Urgencias con un dolor torácico sugestivo de isquemia miocárdica de 4 horas de evolución. En el ECG se observa un descenso del segmento ST de 2 mm. en V2-V6. ¿Cuál de las siguientes opciones terapéuticas NO es adecuada?:

a) Enoxaparina. b) Clopidogrel. c) Ácido acetilsalicílico. d) Activador tisular del plasminógeno (t-PA).

793: Nos avisa la enfermera porque al tomar la tensión arterial a un hombre de 47 años, que acudía al ambulatorio por las recetas de su madre, presentaba cifras de 160/100 y a los 15 minutos 164/98. El paciente se encuentra bien, en su historia el último registro es de un catarro hace cuatro años, y no viene reflejado nada llamativo en sus antecedentes personales. ¿Cuál sería la actitud más adecuada?:

a) Administrar nifedipino sublingual y actuar en función de la respuesta.

Page 152: Cuestionario de Urología

b) Administrar una tiazida y programar para estudio de hipertensión arterial. c) Programar al menos dos citas para realizar despistaje de hipertensión arterial. d) Enviar al servicio de Nefrología para el estudio de su hipertensión arterial.

794: En relación con los factores de riesgo de ateroesclerosis, ¿cuál de las siguientes afirmaciones es la correcta?

a) Los niveles disminuidos de colesterol HDL son factor de riesgo de ateroesclerosis dependiendo del nivel de colesterol total. b) Se ha demostrado de forma definitiva que la terapia sustitutiva con estrógeno s en la mujer postmenopaúsica disminuye el riesgo cardiovascular. c) La hiperfibrinogenemia se considera factor de riesgo. d) La diabetes mellitus es factor de riesgo de ateroesclerosis por sus efectos en el árbol microvascular.

795: ¿Cuál es entre los siguientes el diagnóstico más probable de un hombre de 38 años con claudicación intermitente al caminar y con Fenómeno de Raynaud en las manos?:

a) Síndrome antifosfolípido. b) Esclerodermia. c) Poliarteritis nodosa. d) Tromboangeítis obliterante.

796: Mujer de 30 años con antecedentes de un aborto espontáneo, que acude a Urgencias por una Trombosis Venoso Profunda limitada a la pantorrilla derecha sin factor desencadenante. ¿Cuál de las siguientes afirmaciones NO es correcta?:

a) Esta indicada la realización de un estudio de hipercoagulabilidad. b) Estará indicada la utilización de medias elásticas tras el control del episodio agudo. c) La duración del tratamiento anticoagulante no debe ser menor a 3 meses. d) Debe realizarse, siempre que sea posible, una gammagrafía pulmonar.

797: Hombre de 65 años con disnea progresiva y cansancio que acude a la consulta porque desde hace 3 meses presenta disnea de pequeños esfuerzos y ortopnea. A la exploración se detecta un soplo pansistólico en foco mitral y por ecocardiografia se comprueba la existencia de una insuficiencia mitral degenerativa con prolapso del velo posterior por rotura de cuerdas tendinosas. La fracción de eyección ventricular izquierda era 40 % Y el estudio hemodinámico demostró que las arterias coronarias no presentaban lesiones significativas. Indique el tratamiento electivo en este caso clínico:

a) Tratamiento médico hasta que se detecte que la fracción de eyección ventricular izquierda sea menor de 30%. b) Reparación de la válvula mitral mediante resección del segmento del velo posterior afectado por la rotura de las cuerdas y anuloplastia mitral. c) Reparación de las cuerdas rotas. d) Sustitución de la válvula mitral por bioprótesis.

798: Hombre de 55 años con hipertensión arterial severa mal controlada. Acude por dolor interescapular intenso con tensión arterial 200/110 rnrnHg. Se realiza TAC torácico en el que se aprecia disección aórtica aislada a nivel de aorta torácica descendente desde la arteria subclavia. Se confirma mediante ecocardiograma transesofágico un desgarro intimal 2 cm. distal a la subclavia, con imagen de disección aórtica desde el desgarro hasta unos 5 cm. por debajo. ¿Cuál es la actitud terapéutica más adecuada?:

a) Control estricto de la tensión arterial con labetalol endovenoso. b) Intervención quirúrgica emergente de sustitución de aorta descendente. c) Control estricto de la tensión arterial con hidralacina endovenosa. d) Intervención quirúrgica programada en breve plazo de reparación mediante parche de la zona de desgarro.

799: En el post-operatorio inmediato, un politraumatizado está cianótico y muy hipotenso, auscultándose además muchas sibilancias. La medición de la presión venosa y de la presión capilar o de enclavamiento pulmonar están muy elevadas. ¿Qué medida terapéutica NO necesitaría en absoluto?:

a) Oxigenoterapia. b) Noradranalina. c) Reposición de la volemia. d) Diuréticos.

800: ¿Cuál de las siguientes respuestas, que se refieren a la exploración del aparato respiratorio, es cierta?:

a) Las sibilancias son audibles, especialmente en la fase inspiratoria. b) El indicador clínico más fiable del signo “cianosis central” es su presencia en las partes acras de las extremidades. c) La percusión de una zona de Neumotorax tiene un tono mate. d) La auscultación de respiración bronquial precisa que exista permeabilidad bronquial.

801: Enfermo con disnea, Pa02 59 mmHg respirando aire y 65 mmHg con O2 al 40% (Fi02 0,4). La PaCO2 es 40 mmHg. ¿Cuál de los signientes diagnósticos es MENOS probable?:

a) Fibrosis Pulmonar Idiopática. b) Neumonía por Klebsiella. c) Edema pulmonar cardiógeno. d) Atelectasia (colapso).

802: ¿Cuándo está indicada la medición de volúmenes pulmonares (capacidad pulmonar total y volumen residual)?:

a) En todos los pacientes con obstrucción crónica al flujo aéreo la primera vez que se les hace estudio funcional. b) En los pacientes en los que se sospecha restricción. c) En pacientes con patología neuromuscular. d) En los sujetos fumadores para detectar alteraciones no observables en la espirometría convencional.

803: Los siguientes fármacos pueden asociarse con la inducción de un ataque agudo de asma EXCEPTO uno de ellos que es, en general, bien tolerado. Señálelo:

a) Aspirina.

Page 153: Cuestionario de Urología

b) Acetaminofen. c) Ácido mefenámico. d) Naproxeno.

804: Un paciente de 65 años de edad y con enfermedad pulmonar obstructiva crónica (EPOC) presenta aumento progresivo de su disnea a lo largo de varios días con aumento de tos y expectoración verdosa. El médico que le atiende considera que hay muy baja probabilidad clínica de que exista una tromboembolia pulmonar añadida (TEP), pero no obstante realiza un test de dímero-D por técnica ELISA que es negativo. ¿Cuál de las siguientes afirmaciones es correcta?:

a) Debe realizarse arteriograffa pulmonar para descartar el TEP. b) Debe iniciarse inmediatamente tratamiento anticoagulante con heparina. c) El test de dímero - D por ELISA carece de sensibilidad en el diagnóstico de TEP. d) Se puede descartar embolia pulmonar en este caso, dada la baja probabilidad clínica junto con la negatividad del dímero - D por ELISA.

805: Una campesina de 30 años no fumadora y sin antecedentes de haber inhalado otras substancias nocivas presenta un patrón intersticial difuso con aumento de volumen pulmonar en la radiografía de tórax y un pequeño derrame pleural. El diagnóstico más probable es:

a) Neumonitis por hipersensibilidad. b) Sarcoidosis. c) Histiocitosis X. d) Linfangioleiomiomatosis.

806: Paciente de 65 años, fumador de 25 paquetes/año, de profesión ingeniero, sin antecedentes de interés. Presenta disnea de esfuerzo progresiva y tos seca desde hace 1 año. A la exploración presenta acropaqnias y estertores crepitantes, bilaterales y persistentes. La radiografía de tórax muestra imágenes retículonodulillares basales y simétricas, con reducción de los campos pulmonares. La exploración funcional pulmonar únicamente presenta: capacidad de difusión (Dlco), 43% referencia; SaO2 94% en reposo, y 72%, al esfuerzo. ¿Cuál es el diagnóstico más probable?:

a) Enfermedad pulmonar obstructiva crónica. b) Bronquiolitis obliterativa con neumonía organizada. c) Neumonía eosinófila crónica. d) Fibrosis pulmonar idiopática.

807: En una radiografía de tórax la presencia de calcificación en cáscara de huevo en los ganglios de hilio es un signo bastante típico de:

a) Berilosis. b) Silicosis. c) Asbestosis. d) Talcosis.

808: En relación con la pleuritis tuberculosa, indique la afirmación FALSA:

a) Es la causa más común de exudado pleural en muchos lugares del mundo. b) Se considera una reacción de hipersensibilidad a la proteína tuberculosa. c) En su exudado predominan los nautrófilos, primero, y los linfocitos después. d) Responde mal a los tuberculostáticos, aunque en ocasiones se resuelve espontáneamente.

809: Un paciente de 70 años con historia de iusuficiencia cardiaca consulta por disnea. La radiografía de tórax demuestra derrame pleural. ¿En cuál de las siguientes situaciones estaría indicado realizarle una toracocentesis diagnóstica?:

a) El derrame es bilateral. b) El paciente tiene disnea de reposo. c) El paciente tiene dolor torácico unilateral que empeora con la inspiración profunda. d) El derrame ocupa más de 1/3 del hemitórax.

810: Paciente de 42 años que acude a la consulta por disnea progresiva. La radiografía de tórax muestra una opacidad completa de hemitórax derecho. Tráquea y estructuras mediastínicas están en la línea media. ¿Cuál es el diagnóstico más probable?:

a) Derrame pleural masivo. b) Carcinoma de pulmón. c) Hemotórax. d) Cuerpo extraño.

811: El trasplante pulmonar es en la actualidad una terapéutica válida para pacientes en insuficiencia respiratoria terminal en la que se han agotado todos los tratamientos médico-quirúrgicos. Entre las siguientes, indique la enfermedad que más frecuentemente se beneficia de este proceder terapéutico:

a) Carcinoma broncogénico. b) Fibrosis quística. c) Distres respiratorio del adulto. d) Hemoptisis masiva.

812: Ante un paciente de 70 años con un carcinoma epidermoide pulmonar en el lóbulo superior derecho e imágenes sugestivas de adenopatías paratraqueales derechas de 1 centímetro en una tomografía axial computorizada. ¿Cuál de las siguientes exploraciones será la de primera elección para establecer específicamente la afectación tumoral de dichas adenopatías?:

a) Una tomografía por emisión de positrones. b) Una resonancia magnética. c) Una mediastinoscopia. d) Biopsia transbronquial.

813: La displasia cráneo-cervical es una malformación heterogénea de la base del cráneo, que va desde una mera impresión basilar a una acusada deformidad que incluye platibasia, convexobasia, acortamiento del clivus y aplanamiento de la fosa

Page 154: Cuestionario de Urología

posterior. ¿Cuál es el mecanismo patogénico de esta displasia?:

a) Trastorno en el desarrollo del cráneo membranoso. b) Trastorno en el desarrollo del cráneo encondral. c) Falta de fusión cráneo-vertebral. d) Inmadurez del cerebelo.

814: Hombre de 85 años de edad con antecedentes de hemorragia cerebral hace 2 años. Ingresa por cuadro agudo de hemiparesia derecha y sonmolencia. En el TAC urgente se objetiva un gran hematoma intracerebral lobar frontoparietal izquierdo. El paciente no es hipertenso. ¿Cuál, entre las siguientes, es la etiología más probable de la hemorragia del paciente?:

a) Metástasis. b) Aneurisma. c) Traumatismo. d) Angiopatía amiloide.

815: Los infartos lacunares suponen alrededor del 20% de todos los accidentes cerebrovasculares. ¿Cuál de los siguientes enunciados le parece FALSO en relación con la localización de dichos infartos lacunares?:

a) La hemiparesia motora pura se produce por un infarto en el brazo posterior de la cápsula blanca interna. b) Ictus sensitivo puro por un infarto de la porción ventrolateral del tálamo. c) La hemiparesia atáxica, por infarto en el cerebelo. d) La disartria y mano torpe, por infarto en la protuberancia.

816: Un hombre de 28 años, acude a consulta refiriendo desde hace 10 días un cuadro de alteración de la sensibilidad de hemicuerpo que incluye la cara. Tiene como antecedentes haber padecido una visión borrosa por el ojo izquierdo hace 1 año, que recuperó por completo en 1 mes. En la exploración actual se objetiva una hemihipoestesia izquierda con signo de Babinski de ese lado. ¿Qué prueba diagnóstica es la más apropiada para conocer la etiología más frecuente de este proceso?:

a) TAC cerebral con contraste. b) Estudio rutinario del LCR. c) Estudios serológicos de virus. d) Resonancia magnética cerebral.

817: Señale cuál es la indicación de tratamiento con interferón beta en pacientes con esclerosis múltiple en la actualidad:

a) Tratamiento sintomático de los brotes. b) Prevención de los brotes en pacientes con formas clínicas recurrentes-remitentes. c) Tratamiento de la discapacidad de las formas primarias progresivas. d) Tratamiento para las neuritis ópticas.

818: Un paciente de 60 años refiere que desde hace años le tié1nblan las manos al sostener la cuchara, el vaso o el boÍígrafo, sobre todo si está nervioso o fatigado, y estos síntomas mejoran con pequeñas cantidades de, vino. Su padre, ya fallecido, había presentado temblor en las manos y la cabeza. La exploración neurológica sólo muestra temblor de actitud simétrico en ambas ma-nos. Este cuadro clínico es probablemente consecuencia de:

a) Un hipo tiroidismo familiar. b) Una enfermedad de Parkinson incipiente. c) Síntomas de deprivación etílica. d) Un temblor esencial.

819: ¿Cuál de los siguientes hallazgos es muy frecuente en la enfermedad de Parkinson idiopática?:

a) Demencia al inicio. b) Mioclonías. c) Blefaroespasmo. d) Pérdida de movimientos asociados en la marcha.

820: Todas las siguientes pruebas complementarias son recomendadas en la rutina diagnóstica de la demencia, MENOS una ¿cuál es ésta?:

a) Electrolitos séricos. b) Serología luética. c) TAC cerebral. d) Genotipo ApoE.

821: El síndrome de las piernas inquietas es un trastorno crónico frecuente. En relación al mismo, indique la respuesta correcta entre las que a continuación se enumeran:

a) Con frecuencia se asocia a una deficiencia de hierro. b) Es un trastorno de origen psicogénico en la mayoría de los casos. c) Es una manifestación de la polimeuropatía sensitiva de fibra pequeña. d) La existencia de movimientos periódicos en las piernas durante el sueño es condición necesaria para el diagnóstico.

822: En relación con la epilepsia, es FALSO que:

a) En la esclerosis mesial del lóbulo temporal suele haber antecedentes de crisis febriles. b) En la fase tónica de una crisis generalizada tonicoclónica hay cianosis y midriasis. c) Las crisis febriles suelen aparecer entre los 3 meses y los cinco años de edad. d) Las crisis de ausencia típica se relacionan con patología del lóbulo temporal.

823: ¿Cuál entre las siguientes, es la prueba más específica para el diagnóstico de miastenia grave?:

a) Test del cloruro de edrofonio. b) Electromiograma con estimulación repetitiva. c) Electromiograma de fibra muscular aislada.

Page 155: Cuestionario de Urología

d) Determinación de anticuerpos anti-receptor de acetilcolinao

824: Un hombre de 30 años presenta episodios de cefalea periocular derecha, que le despiertan por la noche, muy intensos, de unos 30 minutos de duración. Le hacen levantarse de la cama. ¿Cuál sería su sospecha diagnóstica?:

a) Migraña común. b) Neuralgia del trigémino. c) Cefalea en racimos. d) Sospecharía un tumor cerebral o una hipertensión intracraneal.

825: El tumor más frecuente a nivel del ángulo pontocerebeloso es el:

a) Glioma de tronco cerebral. b) Neurinoma del V par. c) Colesteatoma del poro acústico. d) Neurinoma del acústico.

826: Una mujer de 30 años refiere un cuadro de mialgias, fiebre, palpitaciones, nerviosismo y dolor en el cuello. La velocidad de sedimentación globular está elevada, los niveles séricos de trüodotironina (T3) y tiroxina (T4) están elevados y los de tirotropina (TSH) están bajos. ¿Cuál sería el tratamiento más adecuado para esta paciente?:

a) Corticosteroides y antitiroideos. b) Antiinflamatorios no esteroideos y antitiroideos. c) Sólo antitiroideos. d) Antiinflamatorios no esteroideos y betabloqueantes.

827: Respecto al tratamiento de la enfermedad de Graves, ¿cuál de las siguientes afirmaciones es FALSA?:

a) Tras el tratamiento con metimazol puede producirse la remisión de la enfermedad. b) Es correcto añadir tiroxina al tratamiento con antitiroideos para prevenir el hipotiroidismo. c) El riesgo de recurrencia del hipertiroidismo es mayor tras el tratamiento con yodo radiactivo que con antitiroideos. d) El tratamiento de elección de la enfermedad de Graves durante el embarazo es el propiltiouracilo.

828: En relación a las masas suprarrenales asintomáticas. ¿Cuál de los siguientes estudios diagnósticos considera que es el MENOS útil?:

a) Medición de catecolaminas y metanefrinas en orina de 24 horas. b) Punción aspiración con aguja fina para diagnóstico diferencial de tumores suprarrenales primarios benignos y malignos. c) Medición de sodio y potasio en orina de 24 horas. d) Test de supresión con 1 mg de dexametasona para corti sol.

829: Una mujer de 41 años con obesidad troncular, hipertensión e intolerancia a la glucosa, presenta una excreción urinaria de cortisollibre elevada, inadecuada supresión del cortisol sérico a dosis elevadas de dexametasona y ACTH sérico no detectable. La prueba de localización de esta lesión más adecuada es:

a) RMN de hipófosis. b) TAC torácico. c) TAC abdominal. d) Ecografía suprarrenal.

830: La hiperplasia suprarrenal congénita agrupa a un conjunto de déficits enzimáticos que ocurren en la generación de los glucocorticoides. ¿Cuál es el más frecuente?:

a) La hidroxilación de C2l b) La hidroxilación de C17α c) Lahidroxilación de C11β d) La hidroxilación de C18

831: Un paciente con un síndrome polidipsicopoliúrico presenta los siguientes resultados del test de la sed: Osmolaridad urinaria 700 mOsm/kg y tras la administración de Vasopresina 710 mOsm/kg. Indique el diagnóstico más probable:

a) Diabetes insípida verdadera. b) Polidipsia primaria. c) Insensibilidad de los osmoreceptores. d) Diabetes insípida nefrogénica.

832: La metformina es un fármaco muy útil en el tratamiento de la Diabetes Mellitus. Si Ud. lo utiliza debe conocer cuál de las siguientes afirmaciones NO es cierta:

a) Se indica especialmente en los diabéticos tipo II obesos. b) Puede producir molestias gastrointestinales. c) El riesgo de hipoglucemias secundarias es excepcional. d) Lo puede utilizar en pacientes con hepatopatías activas.

833: ¿Cuál de las siguientes características NO corresponde con la diabetes Mellitus tipo I (insulindependiente)?:

a) Suele diagnosticarse en personas menores de 30 años. b) Precisa de tratamiento con insulina desde el inicio para sobrevivir. c) Se desencadena por un proceso autoinmune con insulinitis que destruye los islotes pancreáticos productores de insulina. d) Patogénicamente se produce una resistencia a la insulina en los receptores del hígado, músculo y adipocitos.

834: De acuerdo a los criterios diagnósticos de la American Diabetes Association (2002) ¿ante quéresultado diagnosticaría una diabetes mellitus en ausencia de una descompensación aguda metabólica?:

a) Una determinación al azar de glicemia venosa de 156 mg/dl, asociada a clínica de poliuria, polidipsia y pérdida de peso. b) Glicemia venosa de 128 mg/dl, tras dos horas del test de tolerancia oral a la glucosa. c) Dos determinaciones en días diferentes de glicemia venosa en ayunas de 130 mg/dl y 135 mg/dl, respectivamente, sin

Page 156: Cuestionario de Urología

clínica hiperglucémica. d) Hallazgo de una única glicemia venosa al azar de 210 mg/dl, sin clínica de poliuria, polidipsia y pérdida de peso.

835: A un paciente con Porfiria cutánea tarda, le debemos informar de los hechos que a continuación se enumeran, SALVO uno. Señale éste:

a) Debe evitar la exposición solar en la playa. b) Se debe a un defecto enzimático, la URO descarboxilasa hepática. c) Los traumatismos pueden producirle ampollas. d) La afectación neurológica afecta a los músculos proximales.

836: ¿Cuál es el tratamiento de elección de la hipercalciuria idiopática?:

a) Furosemida. b) Restricción de calcio en la dieta. c) Hidroclorotiazida. d) Calcitonina.

837: En el reconocimiento médico a un hombre de 18 años, de 180 cm. de altura y 92 kg. de peso, se descubre una distribución ginoide de la grasa, ausencia de vello facial y corporal, ginecomastia y un tamaño testicular de 1,5 cm. En las pruebas complementarias se confirma una elevación de la LH y la FSH y un azoospernia. ¿Cuál sería la conducta a seguir?:

a) Esperar a que cumpla 21 años y repetir el estudio. b) Iniciar sin más pruebas un tratamiento con testosterona. c) Se debería hacer un cariotipo. d) Determinar la concentración de cloro en el sudor.

838: Acude a consulta un hombre de 67 años de edad, fumador de 48 paquetes/año, con diabetes mellitus de reciente diagnóstico. Índice de Masa Corporal 32 Kg/m2, TA 148/92, Glicemia basal 98 mg/dl; Glucosuria negativa, microalbuminuria negativa; colesterol total 274 mg/dl; LDL 190 mg/dl; HDL 30 mg/dl. Su médico de familia realiza una intervención en los hábitos de vida (tabaco, alimentación y ejercicio) e interviene farmacológicamente con mefformina, simvastatina y enalapril. Pasados 6 meses, ¿cuál de las siguientes situaciones reflejaría un buen control del paciente, con un riesgo coronario menor?:

a) El paciente no fuma. IMC 26.8, TA 129/78; HbAlc 6.8; colesterol total 198; LDL 98; HDL 46. b) El paciente no fuma. IMC 30, TA 140/90, HbAlc 7, colesterol total 230; LDL 140; HDL 45. c) El paciente fuma. IMC 25, TA 124/74, HbAlc 5,4, colesterol total 190; LDL 90; HDL 46. d) El paciente no fuma. IMC 25, TA 138/88, HbAlc 7,2, colesterol total 190; LDL 90; HDL 46.

839: En relación con el metabolismo de las lipoproteínas sólo una de las siguientes afirmaciones es cierta. Indique cuál:

a) La forma esterificada del colesterol es soluble en medio acuoso (anfipático) y por ello recubre la superficie de las lipoproteínas. b) Las lipoproteínas VLDL contienen fundamentalmente colesterol. c) La deficiencia familiar de lipoproteinlipasa se caracteriza por unos niveles muy elevados de triglicéridos y plasma lechoso. d) En la hipercolesterolemia poligénica es típica la existencia de xantomas.

840: En el caso de una paciente de 45 años de edad con historia de 6 meses de evolución, consistente en artritis simétrica que incluye muñecas, rigidez matutina de 2 horas y una radiología simple con erosiones en carpo. ¿Cuál de los siguientes hallazgos inmunológicos NO se correlaciona con el cuadro clínico descrito?:

a) Aumento en el compartimento sinovial de citoquinas de origen macrofágico. b) Presencia en suero de inmunoglobulinas con efecto antiinmunoglobulinas G. c) Presencia en compartimento sinovial de auto-anticuerpos con reactividad contra la inmunoglobulina G. d) Presencia en la superficie de sus linfocitos B de HLA-DR6.

841: Una mujer de 60 años acude por la aparición aguda de inflamación y dolor en su rodilla derecha. No refería antecedente traumático o una historia previa de artritis. La exploración física demostró la presencia de derrame articular y aumento de temperatura en su rodilla derecha. El factor reumatoide fue negativo y el ácido úrico sérico era de 3,2 mg/dl. El diagnóstico más probable se establecería por:

a) La respuesta terapéutica a esteroides intraarticulares. b) Niveles elevados de calcio sérico.' c) Un recuento de células blancas en líquido sinovial de 500/mm3 con baja viscosidad. d) La presencia de una fina línea de calcificación en la radiografía de la rodilla afectada.

842: Hombre de 30 años, que preseuta un cuadro clínico de 10 meses de evolución consistente en dolor lumbar continuo, que le despierta por la noche, y que se acompaña de rigidez matutina de tres horas de duración. ¿Cuál sería el tratamiento de primera elección?:

a) Diazepan oral. b) Dexametasona intramuscular. c) Indometacina oral. d) Matamizol intramuscular.

843: Paciente de 72 años, noruega, residente en la Costa del Sol, acude a Urgencias por uu episodio brusco y auto limitado de pérdida de visión 2 horas antes. Desde la semana previa aquejaba cefalea. En el último mes, había perdido peso en el curso de un proceso caracterizado por febrícula, artralgias, asteuia, auorexia, cervicobraquialgia bilateral y mialgias en región glútea y muslos. En el examen fisico destacaban una temperatura. de 37,8° C y palidez cutaneomucosa; el resto de la expl oración, incluyendo examen oftalmológico y neurológico, fue normal. Se objetivaron: Hb 9,8 gr/dl, valor hematocrito 29%, VCM 87, leucos 9800/mm3, plaquetas 470.000/mm3 y VSG 72 mm a la 1ª hora. ¿Cuál de las siguientes actitudes es la correcta?:

a) Iniciar tratamiento con 60 mg/día de prednisona y proseguir estudio. b) Transfundir dos unidades de concentrado de hematíes. c) Iniciar tratamiento con bolos IV de ciclofostamida. d) Ingresar, iniciar antibioterapia empírica, tras extracción de hemocultivos y proseguir estudios.

Page 157: Cuestionario de Urología

844: En el tratamiento del síndrome antifosfolipídico es cierto qué:

a) La presencia de anticuerpo s antifosfolípido en una embarazada sin antecedente de trombosis o abortos es una indicación para iniciar el tratamiento. b) La anticoagulación se realiza en la actualidad con hepafina de bajo peso molecular, ya que con este procedimiento no se requieren controles. c) La anticuagulación manteniendo un INR alto (-3) es el tratamiento de elección en pacientes que ya han tenido trombosis. d) La anticoagulación no es efectiva si no va acompañada de tratamiento inmunosupresor.

845: Hombre de 35 años que presenta desde hace 1 mes fiebre en agujas, artralgias y artritis y exantema asalmonado vespertino. La exploración física puso de manifiesto artritis en pequeñas y grandes articulaciones. Se palpaba una hepatomegalia de 2 cm. lisa y un polo de bazo. La velocidad de sedimentación estaba aumentada: 140 mm en la la hora. En el hemograma se evidenció una leucocitosis de 23.000/mm3 con 80% de neutrófIlos. La ferritinemia plasmática era de 10.000 ng/ml (N = 15-90). Los hemocultivos fueron negativos. El ecocardiograma transtorácico fue normal. La radiografia de tórax fue normal. ¿Cuál es su diagnóstico?:

a) Endocarditis aguda. b) Lupus eritematoso sistémico. c) Enfermedad de Still del adulto. d) Sepsis por Staphylococcus Aureus

846: En relación con las vasculitis sistémicas, señale cuál de las siguieutes afirmaciones es FALSA:

a) La poliarteritis nodosa (PAN) clásica cursa con frecuencia con glomerulonefritis y capilaritis pulmonar. b) La presencia de anticuerpo s anti-citoplasma de los neutrófilos con patrón perinuclear es mucho más frecuente en la poliarteritis microscópica que en la PAN clásica. c) El tratamiento más eficaz para la granulomatosis de Wegener consiste en la administración conjunta de ciclo fosfamida y glucocorticoides. d) La presencia de asma bronquial grave y eosinofilia periférica son características de la granulomatosis alérgica de Churg-Strauss.

847: ¿Cuál de las siguientes afirmaciones es FALSA con respecto a la esteuosis del canal espinal lumbar?:

a) Se da con más frecuencia en varones. b) Es frecuente en ancianos. c) A la exploración, los pacientes presentan positividad para las maniobras de Lassegue y Bragard. d) Sus síntomas consisten en dolor, parestesias y sensación de debilidad en los miembros inferiores y se desencadenan con la marcha.

848: Indique cuál de los siguientes fármacos inhibe la osteogénesis fracturaria:

a) Paracetamol. b) Indometacina. c) Colchicina. d) Calcitonina.

849: Durante el proceso de rehabiUtadón, tras una fractura conminuta de la cabeza del radio, tratada quirúrgicamente mediante exéresis total de la misma, el paciente experimenta dolor en la muñeca de la extremidad afectada. ¿Cuál es la causa más probable de este dolor?:

a) Lesión yatrogénica del nervio interóseo posterior. b) Disfunción de la articulación radiocubital distal. c) Fractura de la estiloides cubital. d) Distrofia simpático-refleja de muñeca.

850: Paciente de 27 años de edad, jugador de balonmano, con un cuarto episodio de luxación anterior del hombro derecho tras sufrir un traumatismo deportivo. Señale la conducta habitual a seguir:

a) Reducción e inmovilización de la luxación durante tres semanas. b) Reducción de la luxación y movilización precoz de la articulación. c) Rehabilitación específica de la musculatura del hombro tras la reducción de la luxación. d) Reconstrucción quirúrgica.

851: Enfermo de 65 años de edad con dolor invalidante en ambas rodillas secundario a artritis reumatoide, que no mejora con esteroides y metotrexato, ¿cuál es el tratamiento ortopédico correcto?:

a) Implantación de una prótesis total de rodilla en cada una de las articulaciones afectadas. b) Osteotornía varizante, dada la edad del paciente, y si fracasa, implantación de una prótesis total pasado el tiempo adecuado. c) Sinovectornía mediante cirugía artroscópica. d) Osteotornía valguizante, ya que es la deformidad de la rodilla más frecuente en este tipo de pacientes.

852: Seleccionar la relación FALSA sobre la edad de incidencia de las siguientes lesiones óseas:

a) Metástasis de neuroblastoma - menores de 3 años. b) Osteocondroma - niños de 5-10 años. c) Mieloma múltiple - mayores de 50 años. d) Quiste óseo esencial- mayores de 50 años.

853: Un niño de 7 años consulta por dolor en la muñeca derecha tras haberse caído de la bicicleta 2 días antes. A la exploración presenta leve tumefacción e intenso dolor en la extremidad distal del radio, sin deformidad alguna. Radiológicamente se aprecia en la proyección lateral una línea de fractura que va desde la cortical metafisaria dorsal hasta la línea articular radiocarpiana. El diagnóstico será:

a) Fractura de Smith. b) Fractura de Colles.

Page 158: Cuestionario de Urología

c) Fractura en tallo verde de metáfisis distal del radio. d) Epifisiolisis distal de radio.

854: Una enfermera de 55 años presenta dolor intenso en columna torácica sin trauma o esfuerzo violento previo. Al practicar radiografías se observan aplastamientos a nivel T5 y T9. ¿Cuál será la causa más frecuente de esta lesión?:

a) Tuberculosis vertebral. b) Metástasis. c) Enfermedad de Pagel. d) Infección vertebral.

855: Un paciente que presenta en el sedimento de orina microhematuria, proteinuria y cilindros hemáticos, ¿cuál de los siguientes cuadros patológicos padece?:

a) Lesión glomerular. b) Lesión túbulo-intersticial. c) Obstrucción de la vía urinaria. d) Neoplasia renal.

856: Un paciente asintomático, hipertenso de 65 años en tratamiento farmacológico, acude a su médico que le encuentra en la analítica los siguientes parámetros: Hb 14,1 g/dI, VCM 88 fl, Urea 75 mg/dl, Creatinina 1,4 mg/dl, Sodio sérico 128 mEq/l, Potasio sérico 2,8 mEqn/l, Cloro 89 mEq/l. Lo más probable sería:

a) Que tenga una anomalía en la absorción de cloro, tipo Bartter. b) Que la hipertensión sea secundaria a una tubolopatía perdedora de potasio. c) Que en el tratamiento que reciba exista un diurético de asa. d) Que en el tratamiento se incluya un inhibidor de la enzima convertidora de la angiotensina.

857: En la poliquistosis renal del adulto, ¿cuál de las siguientes afirmaciones es FALSA?:

a) Es una enfermedad hereditaria, autosómica dominante. b) Habitualmente se detecta en la primera infancia con ecografía. c) Es causa de deterioro progresivo de la función renal. d) Frecuentemente cursa con hipertensión arterial.

858: La policía encuentra en la calle, inconsciente e inmóvil, a altas horas de la madrugada a un indigente que presenta múltiples hematomas y fetor etílico. En el hospital se le detecta urea de 200 mg/dl, creatinina de 6 mg/dl, ácido úrico de 10 mg/dl y CPK de 1500 U/l. El diagnóstico probable es:

a) Necrosis tubular aguda alcohólica. b) Fracaso renal agudo por urato. c) Necrosis tubular aguda por hemólisis. d) Fracaso renal agudo por rabdomiolisis.

859: Paciente de 70 años, que hace 2 semanas fue sometido a una coronariografía, acude al hospital por aparición de lesiones purpúricas palpables en miembros inferiores, elevación de la creatinina sérica a 3 mg/dl, proteinuria de 19/24h, hipocomplementemia y microhematuria y leucocituria en el sedimento urinario. ¿Cuál es el diagnóstico más probable?:

a) Glomerulonefritis aguda rápidamente progresiva. b) Glomerulonefritis aguda postestreptocócica. c) Síndrome hemolítico-urémico. d) Enfermedad atero-embólica.

860: Paciente de 82 años de edad que refiere síndrome constitucional de tres semanas de evolución, con astenia, anorexia y pérdida de peso con oligoanuria progresiva en las veinticuatro horas previas al ingreso hospitalario. No signos de hiperhidratación. Creatinina plasmática 6 mg/dl. Proteinuria 1 gr/24h. Sedimento: micohematuria. Determinación de ANCA positivo, patrón p-ANCA anti MPO. ECO renal que muestra riñón derecho pequeño y riñón izquierdo de tamaño normal. ¿Cuál cree que es el procedimiento más adecuado y prioritario?:

a) Iniciar tratamiento sustitutivo con diálisis. b) Proceder a practicar biopsia renal. c) Iniciar tratamiento con pulsos intrevenosos de Metilprednisolona y Ciclofosfamida oral. d) Plantear plasmaféresis.

861: En relación con el tratamiento de los pacientes diabéticos tipo 2 con nefropatía, sólo una de las siguientes respuestas es correcta. Señálela:

a) La metformina se puede utilizar sin riesgo en diabéticos con insuficiencia renal de intensidad moderada (creatinina sérica 1,3-1,7 mg/dl). b) Los bloqueantes de los canales de calcio, tipo dishidropiridina (por ejemplo nifedipino) disminuyen el grado de proteinuria y detienen la progresión de la insuficiencia renal. c) Los diuréticos del asa pueden aumentar la proteinuria por lo que no deben ser utilizados en la nefropatía diabética con proteinuria en rango nefrótico. d) Los inhibidores del enzima conversor de la angiotensina (IECA) frenan la evolución de la nefropatía diabética tanto por su efecto hipotensor como por su efecto reductor de la proteinuria.

862: Una paciente de 65 años de edad en tratamiento con antiinflamatorios no esteroideos durante tres semanas por una artropatía degenerativa, presenta un cuadro de fiebre y exantema cutáneo. En la analítica de sangre presenta eosinof'dia y una creatinina de 2 mg/dl y en el sedimento urinario hematuria, piuria y proteinuria de 1 g/día. ¿Cuál es la actitud que se debe seguir en este caso?:

a) Suspender el tratamiento con antiinflamatorios. b) Añadir glucocorticoides. c) Administrar sueroterapia. d) Administrar diuréticos del asa.

Page 159: Cuestionario de Urología

863: Paciente trasplantado renal de 2 meses de evolución que acude al servicio de urgencias por síndrome febril de 3 días de evolución bien tolerado y acompañado de epigastralgias. En la analítica practicada destaca una moderada leucopenia (2400/mm3) con una leve elevación en la cifra de transaminasas (ALT 75 U/l; AST 89 U/l). ¿Cuál sería el primer diagnóstico de sospecha?:

a) Tuberculosis pulmonar. b) Infección por Helicobacter pilorii. c) Infección por Pneumocistis carinii. d) Infección por Citomegalovirus

864: Un hombre de 29 años con antecedentes de dolor tipo cólico en fosa renal izquierda que cedió con tratamiento analgésico, se le practica una urografía intravenosa apreciándose defecto de replección radiotransparente de 6 x 7 mm. en tercio distal de uréter izquierdo. El pH de la orina fue de 5,5; asimismo se observan cristales de urato, 9-12 hematíes por campo y escasa leucocituria. ¿Cuál sería el tratamiento más apropiado?:

a) Alopurinol vía oral. b) Ureteroscopia con extracción del cálculo. c) Nefrolitotol1Úa endoscópica percutánea. d) Alcalinización de la orina por vía oral.

865: Mujer de 63 años que es diagnosticada de carcinoma de células escamosas del trígono vesical, con invasión de la capa muscular. ¿Cuál sería su actitud terapéutica en este caso?:

a) Radioterapia externa con 7000 rads. b) Quimioterapia adyuvante seguida de Cistectomía radical. c) Resección transuretral seguida de inmunoterapia intravesical (BCG). d) Cistectomía radical con extirpación de cara anterior de vagina.

866: Hombre de 77 años, que refiere clínica de prostatismo de años de evolución, que presenta elevación del PSA (Antígeno Prostático Específico) (89 ng/ml) y dolor en columna lumbar desde hace 2 meses. Al tacto rectal la próstata está aumentada de tamaño, de consistencia dura en ambos lóbulos, superficie nodular y límites mal definidos. Tras realizarle una ecografía transrectal cou biopsias prostáticas ecodirigidas, es diagnosticado de un adenocarcinoma de próstata pobremente diferenciado, que afecta a ambos lóbulos y que infiltra las vesículas seminales. La gammagrafía ósea confirma la presencia de metástasis en columna lumbar. ¿Qué tratamiento de los siguientes, aconsejaría en primer lugar?:

a) Prostatectomía radical. b) Quimioterapia intensiva. c) Hormonoterapia. d) Radioterapia pelviana externa.

867: ¿A cuál de los siguientes factores NO se asocia la Pielonefritis Aguda por Pseudomonas Aeuruginosa?:

a) Embarazo. b) Sonda urinaria. c) Nefrolitiasis. d) Manipulación urológica.

868: Mujer de 32 años de edad, embarazadade 11 semanas, sin antecedentes personales de interés salvo alergia a las penicilinas, acude a su médico de familia con el objeto de recoger los resultados de la analítica del primer trimestre, en el que se evidencia una bacteriucia, estando la paciente asintomática. Una vez comprobada la bacteriuria, ¿qué actuación terapéutica y de control debería ser aconsejada en este caso?:

a) Buena hidratación y vigilancia de síntomas urinarios o fiebre. b) Cefalexina 500 mg/6h durante 3-7 días y cultivo urinario a la semana de haber finalizado el tratamiento. c) Cotrimoxazol 800/160mg/12h durante 3-7 días y cultivo urinario mensual hasta el final de la gestación. d) Nitrofurantoína 100 mg/6h durante 3-7 días y cultivo urinario mensual hasta el final de la gestación.

869: ¿En cuál de las siguientes circunstancias es EXCEPCIONAL la existencia de trombocitosis reactiva?:

a) Hemorragias. b) Noeplasias epiteliales. c) Anemia refractaria con exceso de blastos en transformación. d) Anemia ferropénica.

870: Paciente, mujer de 50 años de edad, previamente diagnosticada de Lupus Eritematoso Sistémico. Acude por presentar astenia progresiva y disnea de moderados esfuerzos. La exploración demuestra ictericia conjuntival y esplenomegalia a 4 cm. del reborde costal. En analítica destaca: valor Hematocrito: 24%, Hemoglobina: 8 gr/dl, Reticulocitos así como el índice de producción reticulocitario aumentados y en el frotis se observa policromatofilia, anisopoiquilocitosis y esferocitosis. Elevación de la bilirrubina de predominio indirecto y haptoglobina muy disminuida. ¿Qué prueba analítica solicitaría para orientar y completar el diagnóstico de la anemia de esta paciente?:

a) Punción y biopsia de médula ósea. b) Test indirecto con suero de antiglobulina humana (Prueba de Coombs). c) Test de autohemólisis. d) Dosificación de Vitamina Bl2 y Ácido Fólico en suero.

871: Paciente de 38 años que consulta pos astenia y orinas oscuras. En la exploración se objetiva ictericia y la biología muestra Hb: 6 g/dI; leucocitos 3.109/l, plaquetas 86.109/l, aumento de recitulocitos y LDH con haptoglobina baja. Los hematíes carecían de proteínas de membrana CD55 / CD59 y un estudio molecular puso de mauifiesto alteraciones del gen PIG. ¿Cuál le parece el diagnóstico más correcto?:

a) Anemia hemolítica por anticuerpos calientes. b) Anemia hemolítica por anticuerpos fríos. c) B- Talasemia intermedia. d) Hemoglobinuria Paroxística Nocturna.

872: Señale cuál de las siguientes anemias macrocíticas NO muestra rasgos megaloblásticos en la médula ósea:

Page 160: Cuestionario de Urología

a) Anemia de la enfermedad de Biermer. b) Infestación por Diphyiobotrium iatum. c) Anemia del hipotiroidismo. d) Anemia postgastrectomía.

873: Los pacientes con trombocitopenia autoinmune presentan:

a) Una trombopenia de origen central. b) Un tiempo de Ivy (hemorragia) normal. c) Un trastorno asociado de la agregación plaquetaria en más del 50% de los casos. d) Una médula ósea con aumento de megacariocitos.

874: ¿Cuáles son las alteraciones genéticas primarias más frecuentes en neoplasias hematológicas?:

a) Mutaciones puntuales en genes supresores de tumores. b) Delecciones de genes supresores de tumores. c) Mutaciones puntuales en proto-oncogenes. d) Translocaciones cromosómicas con activación de protooncogenes asociados.

875: ¿Cuál de las siguientes afirmaciones sobre la enfermedad de Hodgkin es cierta?:

a) El prurito forma parte de los síntomas B. b) Un paciente con afectación pulmonar, sin ganglios mediastínicos o hiliares, es un estadio III. c) La forma histológica más frecuente en nuestro medio es la celularidad mixta. d) La variedad depleción linfocítica es la de peor pronóstico.

876: Un hombre de 60 años presenta molestias en hipocondrio izquierdo desde hace 5 meses. El hemograma muestra 50 x 109 leucocitos/l con neutrofilia, basofilia, eosinofilia y presencia de formas inmaduras mieloides, hemoglobina 14 g/dl Y 450 x 109 plaquetas/l. En la exploración física destaca una esplenomegalia palpable a 4 cm del reborde costal. Indique cuál sería su actitud inicial:

a) Realizar TAC abdominal en busca de adenopatías para estadiaje. b) Esplenectonúa diagnóstica y terapéutica. c) Realizar estudio citogenético y molecular para establecer el diagnóstico. d) Iniciar quimioterapia intensiva de forma urgente.

877: ¿Cuál es la principal causa de morbilidad y mortalidad en pacientes diagnosticados de Mieloma Múltiple?:

a) Insuficiencia renal. b) Infecciones bacterianas. c) Hemorragias. d) Hiperca1cemia.

878: Un paciente con enfermedad de Hodgkin que recibió múltiples líneas de quimioterapia y un autotrasplante de médula ósea, del que se recuperó adecuadamente que permanece en remisión, presenta 3 años más tarde la siguiente analítica: Hb 80 g/l, leucocitos 1,2 x 109/l y plaquetas 30 x 109/l. El diagnóstico más probable, entre los siguientes, es:

a) Hemoglobinuria Paroxística Nocturna. b) Síndrome mielodisplásico secundario. c) Hepatopatía crónica con hiperesplenismo. d) Aplasia medular.

879: Señale, entre.las siguientes, cuál es la consecuencia clínica principal que origiua la alteración genética conocida como Protrombina 20210:

a) Tendencia frecuente a hemorragias cutáneas mucosas. b) Agregación plaquetaria y trombopenia. c) Resistencia a las heparinas convencionales pero no a las de bajo peso molecular. d) Tendencia a desarrollo de patología trombótica venosa (trombofilia).

880: En la interpretación de los resultados de los hemocultivos practicados a un paciente con fiebre. ¿Cuál de los siguientes datos nos haría pensar que no estamos ante un caso de contaminación?:

a) Aislamiento de bacterias que normalmente colonizan la piel. b) Aislamiento de cocos gram positivos. c) Aislamiento de bacterias difteroides. d) Aislamiento del mismo microorganismo en 3 hemocultivos con la misma sensibilidad.

881: ¿Qué es el dengue?:

a) Una enfermedad causada por un proxvirus. b) Una enfermedad limitada a los países del centro de África. c) Una enfermedad vírica que puede producir una fiebrehemorrágica. d) Una zoonosis que afecta al hombre ocasionalmente.

882: Un joven de 16 años realiza un viaje de fin de curso por Europa. Al mes de regreso comienza con malestar general, odinofagia y fiebre; en la exploración destaca hipertrofia amigdalar con exudado blanquecino, adenopatías occipitales, laterocervicales dolorosas; en el hemograma se observa leucocitosis de 15000/mm3 con un 70% de linfocitos, alguno de ellos atípico. Ante la sospecha diagnóstica se debe realizar:

a) Biopsia ganglionar. b) Biopsia de médula ósea. c) Tratamiento con Penicilina. d) Serología para virus de Epstein Barr.

883: Un chico de 13 años viene con su madre al final de nuestra consulta de "viernes tarde", debido a que al salir del colegio tenía

Page 161: Cuestionario de Urología

fiebre (38,5° C) y ha vomitado algo de comida. "Segu ro que es de la garganta, su hermano estuvo igual hace 5 días", comenta la madre. En la historia del Centro de Salud no se observa ningún antecedente de interés. El paciente se queja de dolor de cabeza, al tragar saliva y de dolor abdominal, no tiene tos, ni rinorrea, ni ronquera. La exploración es normal excepto la presencia de varias adenopatías cervicales anteriores bilaterales de más de un cm. de diámetro dolorosas, exudado blanco-grisáceo en pared posterior de la faringe y amígdalas grandes y eritematosas. ¿Cuál sería la actitud más adecuada?:

a) Instaurar tratamiento con una penicilina a dosis y tiempo adecuado por una posible faringitis estreptocócica. b) Hacer una toma de exudado para cultivo, prescribir analgésicos y antipiréticos, en espera del resultado previsto a los 6-7 días. c) Instaurar tratamiento con unmacrólido a dosis y tiempo adecuado, evitando las penicilinas por el riesgo de reacción ante una posible mononucleosis. d) Administrar moxifIoxacino para aseguramos de la eficacia antibiótica.

884: Un hombre de 45 años acudió al área de Urgencias de un hospital por fiebre elevada y exantema mácnlo-papnloso generalizado, incluyendo palmas y plantas. El paciente vive en el campo con perros frecuentemente parasitados por garrapatas. Señale la enfermedad a la que se refiere, el germen causante y el tratamiento adecuado:

a) Fiebre botonosa, Ticckettsia Conori: Doxiciclina. b) Kala-azar, Leishmaniae Donovani: Antimoniales. c) Dengue, Arenaviridae Aegypti: Tratamiento sintomáticico. d) Fiebre Q, Coxiella Burnetti, Doxiciclina.

885: Paciente de 64 años, fumador, que acude a urgencias por un cuadro de 48 h. de evolución de fiebre y tos con expectoración mucopurulenta. La radiografía de tórax muestra una condensación alveolar en lóbulo inferior derecho y un pequeño infiltrado en ellóbulo inferior izquierdo. La gasometría arterial muestra un pH de 7,39, una pO2 de 54 mmHg y una pCO2 de 29 mmHg. ¿Cuál de las siguientes opciones terapéuticas le parece más adecuada?:

a) Claritromicina 500 mg IV /12h. b) Iprofloxacina 200 mg IV/12h. c) Amoxicilina-Ácido clavulánico 1g IV/8h. d) Ceftriaxona 2g IV/24h + Claritromicina 500 mg IV/12h.

886: Un paciente consulta por diarrea sanguinolenta y fiebre de 39° C de más de una semana de duración . En el coprocultivo se aísla Campylobacter Jejuni. ¿Cuál de los siguientes antibióticos es el recomendado como de primera elección?:

a) Amoxicilina. b) Ciprofloxacino. c) Cefotaxima. d) Eritromicina.

887: La malaria es una enfermedad parasitaria erradicada en nuestro país, pero en los últimos años estamos asistiendo a un resurgir de casos debidos a la inmigración y a los viajes a países tropicales. En relación a la malaria, cuál de las siguientes afirmaciones es FALSA:

a) La gravedad de la infección está en relación con el grado de parasitemia. b) La infección pro plasmodium faciparum es la más grave. c) La malaria cerebral es una manifestación típica de plasmodium vivax. d) La infección se ha descrito en adictos a drogas por vía parenteral que comparten jeringuillas.

888: Un enfermero de Urgencias le consulta porque ha tenido un accidente en el que ha recibido un pinchazo profundo, sin guantes, con una aguja gruesa visiblemente manchada de sangre, de un paciente adicto a drogas por vía parenteral. Tras interrogar al paciente, declara que comparte habitualmente jeringuillas intravenosas y que nunca se ha realizado una serología para el VIH. ¿Cuál de las siguientes es la actitud más correcta?:

a) Esperar al día siguiente a que esté el resultado de la serología de VIH. b) Iniciar inmediatamente tratamiento con tres antiretrovirales. c) Iniciar inmediatamente tratamiento con AZT. d) Tranquilizar al enfermo debido a bajo riesgo de transmisión del VIH y reevaluar en un mes.

889: EI tratamiento antirretroviral en un paciente VIH+ con carga viral de 575.000/mm3 y una cifra de linfocitos de CD4 de 450/mm3:

a) Es obligado. b) Sólo estaría indicado si la carga viral fuera superior a 1 millón de copias/ml. c) En ningún caso se prescribiría si los CD4+ son >200/ml. d) Está indicado si el paciente 10 desea.

890: En la neumonía por Pneumocistis carinii en los pacientes con infección por VIH ¿cuál de las siguientes respuestas es la verdadera?:

a) El diagnóstico definitivo se realiza por cultivo del esputo en medios específicos. b) El uso de glucocorticoides está contraindicado. c) La Pentamidina intravenosa es el tratamiento alternativo de elección en las formas graves. d) El riesgo de padecerla es independiente de la cifra de linfocito s CD4+.

891: Mujer de 33 años de edad, ecuatoriana, acude a su médico de familia para la lectura de Mantoux, realizado en el contexto de un estudio de contactos. Una prima suya que vive en su casa y duerme en la misma habitación (junto con otras 7 personas), ha sido diagnosticada de una tuberculosis pulmonar bacilífera (mas de 50 bacilos por campo). Su médico aprecia una induración de 7 mm. en la lectura del PPD. ¿Cuál de las siguientes afirmaciones es la correcta?:

a) Se trata de una infección tuberculosa y hay que iniciar tratamiento quimioprofiláctico de inmediato. b) Se trata de una Tuberculosis y hay que comenzar con tratamiento antituberculoso. c) Se trata de una infección tuberculosa y hay que descartar enfermedad tuberculosa, previo a comenzar el tratamiento quimioprofiláctico. d) El PPD no es significativo, ya que al tratarse de una persona inmigrante de una zona de alta prevalencia de TBC debería

Page 162: Cuestionario de Urología

tener una induración mayor de 15 mm.

892: ¿En cuál de las siguientes localizaciones de la enfermedad tuberculosa está indicado el tratamiento coadyuvante con glucocorticoides para mejorar la supervivencia?:

a) Pulmonar. b) Meníngea. c) Ganglionar. d) Genitourinaria.

893: En relación a la sffilis, señale la afirmación correcta:

a) La lúes secundaria cursa excepcionalmente con manifestaciones cutáneas. b) El control del tratamiento se puede realizar valorando los títulos de positividad de las pruebas treponémicas. c) Durante los períodos de latencia de la enfermedad se negativizan las pruebas no treponémicas. d) El chancro sifilítico es indurado, no doloroso y muy rico en bacterias.

894: La radioterapia es un componente del tratamiento curativo de los siguientes tumores, EXCEPTO en:

a) Cáncer de mama. b) Linfoma de Hodgkin. c) Cáncer de próstata. d) Melanoma.

895: La combinación de quimioterapia y radioterapia en un cáncer de pulmón no microcítico estadio III B ha mejorado la supervivencia media, comparada con radioterapia exclusiva de:

a) 10 a 14 meses. b) 6 a 28 meses. c) 20 a 24 meses. d) 15 a 60 meses.

896: Mujer de 47 años de edad que acude sola a la consulta de su médico de familia. Se encuentra \muy nerviosa, agitada y llorando. Presenta hematoma periobitario derecho y equimosis en brazos, tronco y abdomen. Tras una escucha activa, recogimiento emocional y una exploración minuciosa, el médico averigua que se trata de un caso de maltrato por parte de su marido alcohólico. ¿Cuál de las siguientes medidas deberá realizar inexorablemente?:

a) Obligar a la paciente a que ponga una denuncia en la comisaría más cercana. b) Hacer que la próxima visita la haga conjunta con su marido. c) Tranquilizar a la paciente y decide que lo más seguro es que no se vuelva a repetir. d) Tramitar el parte de lesiones por vía judicial.

897: En un paciente con dolor oncológico de intensidad leve-moderada, ¿cuál de las siguientes asociaciones de fármacos le parece la más adecuada?:

a) Ibuprofeno + Diclofenaco. b) Bupremofima + Celecoxib. c) Paracetamol + Codeina. d) Ácido acetil salicílico + naproxeno.

898: Anciano de 95 años de edad con pluripatología senil diagnosticado de insuficiencia cardiaca congestiva en fase terminal por lo que ha precisado de múltiples ingresos hospitalarios, habiéndose demostrado en uno de ellos, mediante ecocardiografia, una Fracción de Eyección ventricular inferior al 20 %. Tras una semana de ingreso hospitalario con tratamiento adecuado con oxígeno, vasodilatadores, diuréticos y fármacos inotrópicos positivos parenterales, el paciente se encuentra en situación de fracaso multiorgánico, y presenta una disnea muy intensa secundaria a un edema agudo de pulmón. ¿Cuál sería la conducta más adecuada a seguir?:

a) Balón de contrapulsación - aórtico. b) Cateterismo cardíaco con angiografía coronaria y ventriuclografía izquierda. c) Catéter de Swan - Ganz para valorar mejor el tratamiento diurético. d) Morfina intravenosa.

899: Las queratosis actínicas son:

a) Lesiones anodinas que se desarrollan por el envejecimiento de los queratinocitos. b) Displasias epidérmicas incipientes que pueden evolucionar a un carcinoma espinocelular invasor. c) Equivalentes a las pecas o a los léntigos solares. d) Más frecuentes en la espalda de los hombres de edad avanzada que han trabajado al aire libre durante muchos años.

900: ¿Cuál de los siguientes procesos determina una alopecia cicatrizal definitiva?:

a) Psoriasis. b) Liquen. c) Eritema exudativo multiforme minoro d) Pseudotiña amiantácea.

901: Enferma de 70 años que consulta refiriendo la aparición desde hace dos años de lesiones maculosas violáceas y asintomáticas en dorso de antebrazos que desaparecen espontáneamente en dos o tres semanas, de forma irregular y tamaño variable entre uno y cinco cm. de diámetro. Las lesiones continúan apareciendo desde entonces sin periodicidad fija. El diagnóstico más probable es:

a) Vasculitis leucocitoclástica. b) Eczema xerodémiico. c) Angiosarcoma de Kaposi. d) Púrpura senil.

Page 163: Cuestionario de Urología

902: Un niño de 6 años acude a consulta por un cuadro de febrícula de 3 días de evolución, con dolor a la deglución. Los datos más relevantes de la exploración física son lesiones erosivas en el paladar y vesículas intraepidérmicas no agrupadas en palmas y plantas. Entre los siguientes diagnósticos, ¿cuál es el más probable?:

a) Eritema multiforme. b) Rickettsiosis. c) Síndrome de Steven-Jonhson. d) Enfermedad de pie, mano, boca.

903: ¿Cuál es la dermatosis intensamente pruriginosa que se asocia a aparición de vesículas agrupadas y a enteropatía por sensibilidad al gluten y en la que se observan depósitos cutáneos de IgA?:

a) La dermatitis seborreica. b) El pénfigo foliáceo. c) La enfermedad de Hailey-Hailey. d) La dermatitis herpetiforme.

904: Debemos sospechar un retinoblastoma en un niño que presenta los siguientes síntomas:

a) Dolor, fotofobia y lagrimeo. b) Estrabismo y leucocoria. c) Lagrimeo, fotofobia y aumento del diámetro comeal. d) Fotofobia y quemosis conjuntival.

905: Un paciente de 58 años de edad que se trata con insulina desde hace 12 años, acude a Urgencias por haber notado una repentina disminución de visión en ojo derecho. La agudeza visual es de contar dedos a 50 cm. No tiene dolor y el segmento anterior del ojo es normal. ¿Cuál es la causa más probable de esta pérdida de visión?:

a) Catarata. b) Uveítis anterior. c) Presbicia. d) Hemorragia vítrea.

906: Hombre de 32 años, sin antecedentes patológicos de interés, que acude a la consulta por visión borrosa en su ojo derecho de 3 días de evolución. La agudeza visual es de 0,5, el examen del polo anterior no muestra alteraciones y al examen de fondo de ojo se observa cicatriz coriorretiniana y células en la cavidad vítrea. El paciente relata dos episodios oculares similares no diag-nosticados. ¿Qué exploraciones complementarias solicitaría para establecer el diagnóstico?:

a) Test de Mantoux y tinción de Ziehl-Neelsen en esputo. b) Serología toxoplásmica. c) Radiografía de tórax. d) Serología luética.

907: ¿Cuál de los siguientes hallazgos exploratorios es el que menos nos ayuda para hacer el diagnóstico de glaucoma Crónico Simple?:

a) Excavación papilar aumentada (>0,5). b) Asimetría en la excavación de ambas papilas. c) Alteraciones en el campo visual. d) Agudeza visual disminuida (<0,5).

908: Uno de los siguientes signos o síntomas NO esperaría encontrar en una Parálisis oculosimpática o Síndrome de Horner:

a) Ptosis. b) Midriasis. c) Disminución de la sudoración ipsolateral. d) Ausencia de dilatación de la pupila tras instilación de cocaína tópica.

909: Las cadenas ganglionares recurrenciales están particularmente afectadas en los cánceres de:

a) Glotis. b) Vestfbulo laringeo. c) Seno piriforme. d) Amfgdala palatina.

910: En el cáncer de laringe una disfonía continuada (de más de 30 días) la lesión más lógica sería:

a) Tumor borde libre de epiglotis. b) Tumor de vestfbulo laringeo. c) Tumor glótico. d) Tumor subglótico.

911: Chica de 17 años de edad que acude a Urgencias por presentar fiebre, disfagia y trismus con inflamación en fosa amigdalina derecha y paladar blando con úvula desplazada alIado contralateral. Este cuadro se encuentra localizado en:

a) Espacio maseterino. b) Entre el constrictor superior de la faringe y mucosa amigdalar. c) Espacio retrofaringeo. d) Entre el constrictor superior y vaina de grandes vasos.

912: Muchacho de 14 años, que consulta por obstrucción nasal, hidrorrinorrea, anosmia y cefalea fronto-orbitaria de meses de evolución. Como antecedentes de interés destaca la presencia de un retraso constitucional del crecimiento y bronquitis asmática. En la rinoscopia anterior se observan, en ambas fosas nasales, múltiples masas semitransparentes, blandas y móviles. Señale la respuesta INCORRECTA:

a) Estaria indicado el tratamiento con corticoides inhalados durante largos periodos.

Page 164: Cuestionario de Urología

b) Las exacerbaciones agudas deberian ser tratadas con antibióticos, y antiinflamatorio.s no esteroideos. c) De cara a completar el diagnóstico seria necesario la realización de un test del sudor. d) Está indicada la realización de un TAC.

913: Mujer de 34 años, madre de 3 hijos, que acude a su consulta presentando una hipoacusia derecha, con Rinne negativo derecho y positivo izquierdo y Weber con lateralización aliado derecho. La otoscopia es normal en ambos oídos. Su sospecha inicial será:

a) Otitis serosa. b) Timpanoesclerosis. c) Hipoacusia súbita. d) Otosclerosis.

914: Lo que básicamente diferencia una bulimia nerviosa de una anorexia nerviosa es:

a) La pérdida de control sobre la comida. b) La preocupación excesiva por la figura y el peso. c) El ejercicio excesivo. d) La autoinducción del vómito.

915: Todas las siguientes son características asociadas a mal pronóstico en la esquizofrenia EXCEPTO una, señálela:

a) Existencia de un factor estresante precipitante. b) Inicio progresivo. c) Antecedentes familiares de esquizofrenia. d) Soltero.

916: ¿Cuál de las signientes opciones relacionadas con el consumo de sustancias es INCORRECTA?:

a) La intoxicación producida por sustancias se define como un síndrome reversible que se caracteriza fundamentalmente por la aparición de cambios psicológicos o comportamentales desadaptativos y fisiológicos. b) La abstinencia se define como un síndrome específico de una sustancia debido al cese o reducción de su consumo, que causa un malestar clínicamente significativo y/o un deterioro de la actividad laboral y social. c) El abuso o consumo perjudicial se define como un patrón desadaptativo de consumo que no llega a cumplir los criterios diagnósticos de la dependencia. d) El concepto de tolerancia a una sustancia hace referencia al hecho de que con el consumo continuado se precisan cada vez dosis menores para producir el mismo efecto.

917: Una paciente de 62 años refiere que un famoso cantante le manifiesta desde hace años su amor, a través de insinuaciones o gestos en sus intervenciones públicas. Ha tratado, sin recibir respuesta, de comunicarse con él, mediante llamadas telefónicas, cartas o incluso yendo a su domicilio, por lo que fue denunciada. No sufre alucinaciones y su capacidad de juicio, fuera del tema citado, es totalmente adecuada. ¿Cuál sería la primera sospecha de diagnóstico?:

a) Depresión psicótica no congruente con el humor. b) Trastomode ideas delirantes persistentes. c) Demencia incipiente. d) Disfunción sexual hipererótica.

918: De entre los siguientes síntomas, señale cuál sería más IMPROBABLE encontrar en un episodio maníaco:

a) Aumento de las necesidades del sueño. b) Distraibilidad. c) Verborrea. d) Grandiosidad.

919: ¿Cuáles son los actos obsesivos más frecuentes en el trastorno obsesivo-compulsivo?:

a) Comprobaciones y rituales de limpieza. b) Recuentos mentales. c) Evitar pisar las cruces en las baldosas. d) Jaculatorias repetitivas de conjura.

920: La anhedonia total se relaciona fundamentalmente con:

a) Manía. b) Depresión. c) Fobias sociales. d) Ataques de pánico.

921: En relación con el autismo, ¿cuál de las siguientes afirmaciones NO es correcta?:

a) Es frecuente el retraso en el desarrollo intelectual. b) Hay una interacción social anómala y restringida. c) Más frecuente en sexo masculino. d) Son frecuentes las conductas desafiantes y provocativas.

922: Un hombre de 73 años con Enfermedad Pulmonar Obstructiva Crónica está ingresado en una planta de Medicina Interna tras ser atendido en Urgencias por una Insuficiencia Respiratoria Global, secundaria a una Infección Respiratoria. Durante su segunda noche en el hospital, presenta agitación, desorientación temporal y espacial, falsos reconocimientos, insomnio y agresividad verbal y física hacia el personal cuidador. El paciente se arranca la mascarilla de oxígeno y las vías de perfusión. Es portador de una prótesis de cadera derecha. La enfermera de turno le avisa a Vd., que es el médico de guardia. ¿Cuál de los siguientes comportamientos asistenciales es correcto en el contexto clínico descrito?:

a) Invitar al paciente a firmar el Alta Voluntaria, previa información de los riesgos derivados del no tratamiento de su condición patológica. b) No iniciar ningún procedimiento diagnóstico ni terapéutico al tratarse de un problema psiquiátrico.

Page 165: Cuestionario de Urología

c) Proceder a la sujeción mecánica del paciente para posibilitar su sedación mediante la administración de cloracepato dipotásico por vía LM. d) Proceder a la sujeción mecánica del paciente para posibilitar su sedación mediante la administración de haloperidol por vía IV. y la realizaci6n de los procedimientos diagnósticos y terapéuticos que se estimen indicados.

923: ¿Cuál es el método de detección (screening) de la Diabetes Gestacional que se recomienda en todas las embarazadas?:

a) Determinación de glucosa en orina con tiras reactivas. b) Determinación de glucemia en ayunas. c) Determinación de glucemia después de una sobrecarga oral de 50 gramos de glucosa. d) Test de tolerancia oral a la glucosa con sobrecarga de 100 gramos (curva de glucemia).

924: ¿Cuál de los siguientes marcadores de cromosomapatías del primer trimestre tiene más valor?:

a) Gonadotropina coriónica. b) La alfafetoproteína. c) La PAPP-A. d) La sonoluscencia nucal.

925: ¿Cuál de los siguientes procedimientos es el más seguro para diagnosticar el sufrimiento fetal intraparto?:

a) Los registros cardiotocográficos. b) La auscultación fetal. c) La ecografía doppler color. d) La microtoma fetal.

926: ¿Cuál de las siguientes respuestas NO constituye una distocia mecánica en el período expulsivo de la presentación podálica?:

a) Procidencia o descenso de un solo pie. b) Distocia de hombros por elevación de un brazo. c) Rotación de la cabeza fetal a occipito-sacra. d) Expulsivo lento de la cabeza fetal.

927: Una primigesta de 37 semanas de edad gestacional ingresa por rotura prematura de membranas de 24 horas de evolución. No hay trabajo de parto ni signos de infección amniótica. El test de Bishop (de maduración cervical) es de 6 puntos. El feto está en cefálica y no hay signos de sufrimiento fetal. ¿Cuál es la conducta obstétrica indicada?:

a) Cesárea. b) Inducción del parto con oxitocina. c) Administrar antibióticos y corticoides e inducir el parto 48 horas después. d) Administrar antibióticos y esperar el comienzo espontáneo del parto.

928: ¿Cuál de los siguientes métodos diagnósticos es el esencial para el seguimiento y control de la paciente que ha sido sometida a evacuación de una mola hidatiforme por vía vaginal?:

a) Radiografía de tórax. b) Ecografía con sonda vaginal. c) Dosificaciones de β-HCG en suero. d) Determinaciones de CA125 en suero.

929: En relación con el Tratamiento Hormonal sustitutivo combinado con estrógenos y gestágenos de forma prolongada, las siguientes afirmaciones son ciertas, EXCEPTO una. Señale ésta:

a) Disminuye la descalcificación propia de la menopausia. b) Mejora el trofismo vaginal. c) Aumenta el riesgo de Cáncer de Mama. d) Aumenta el riesgo de Cáncer de Endometrio.

930: Una mujer de 21 años presenta una amenorrea secundaria. Los niveles plasmáticos degonadotropinas (LH y FSH) son inferiores a 10 mUIImI. Los niveles de Prolactina y de Hormona Tireoestimulante (TSH) son normales. La paciente no menstrúa tras la administración de progestágeno, pero sí lo hace al administrar un estrógeno junto con un progestágeno. Cuál de los diagnósticos que a continuación se relacionan es el más correcto?:

a) Síndrome de ovario poliquístico. b) Fallo ovárico autoinmune. c) Tumor hipotalárnico o hipofisario. d) Disgenesia gonadal.

931: En una mujer de 38 años, nuligesta y con deseos de tener hijos, la citología ginecológica informa de una lesión escamosa de alto grado. 'La colposcopia muestra una lesión acetoblanca extensa del ectocérvix que se introduce por el canal endocervical. Las biopsias de esa lesión informan de la presencia de un carcinoma in situ, pero en una de ellas hay un foco de carcinoma epidermoi-de que invade el estroma cervical en 2 mm. de profundidad. ¿Cuál de las siguientes opciones terapéuticas es In más correcta?:

a) Histerectomía total, salpingo-ooferectomía y linfadenectomía pélvica. b) Radioterapia abdominopélvica. c) Braquiterapia (radioterapia intracavitaria). d) Conización cervical y control posterior.

932: Los análogos de las gonadotropinas se utilizan para las siguientes indicaciones, MENOS una:

a) Tratamiento de la menopausia. b) Disminución del tamaño de los miomas. c) Recuperación de la anemia antes del tratamiento quirúrgico de los miomas. d) Disminución del tamaño de los endometriomas.

Page 166: Cuestionario de Urología

933: ¿Cuál de los siguientes hechos es cierto en el cáncer de ovario?:

a) Es más frecuente en las multíparas. b) Es más frecuente en las mujeres que han tomado varios años anticonceptivos orales. c) Suelen dar precozmente metrorragias. d) Se implanta fácilmente por la serosa peritoneal.

934: De las que a continuación se relacionan, son factores de peor pronóstico en un cáncer de mama los siguientes, EXCEPTO uno. Señálelo:

a) Tamaño tumoral superior a 3 cm. b) Afectación axilar. c) Sobreexpresión de Her2. d) Receptores hormonales positivos.

935: ¿Cuál es el gen más frecuentemente implicado en el síndrome de cáncer de mama hereditario?:

a) K-RAS b) HER-2 ó ERBB2 c) ATM d) BRCA1

936: En relación con el cáncer de mama en estadios localizados, ¿cuál de las siguientes afirmaciones es FALSA?:

a) La cirugía conservadora con radioterapia radical no compromete la supervivencia. b) La radioterapia post mastectomía está indicada si existen factores de alto riesgo locoregional (metástasis ganglionares, afectación margen de resección, etc.). c) La quimioterapia está contraindicada en ausencia de metástasis ganglionares. d) El componente intraductal extenso es un factor de riesgo para la recidiva local.

937: La posición en decúbito prono durante el sueño en los lactantes se ha asociado con:

a) Una mayor ganancia de peso. b) Mayor riesgo de neumonía por aspiración. c) Disminución del tránsito intestinal. d) Mayor riesgo de muerte súbita.

938: Una niña de 10 años de edad acude para valoración de un posible hipocrecimiento. Su talla actual se encuentra en el percentil l0 de la población general. Su talla genética está situada en el percentil 15 de la población de referencia. Su desarrollo sexual corresponde a una pubarquia 1 y a una telarquia 1 de Tanner. Su edad ósea es de 9 años. La velocidad de crecimiento del último año ha sido de 5 centímetros/año. ¿Qué situación cree usted que presenta la niña?:

a) Un crecimiento normal. b) Un hipocrecimiento por deficiencia de la hormona del crecimiento. c) Un hipocrecimiento por una enfermedad celíaca. d) Un hipocrecimiento por un síndrome de Turner.

939: Niña de cuatro años, asintomática, con antecedentes de ingreso neonatal durante dos meses por prematuridad. Presenta un buen estado general y de desarrollo ponderoestatural, tiene pulsos arteriales aumentados y se le ausculta un soplo continuo en región subclavicular izquierda. ¿Cuál es, de los siguientes, el diagnóstico más probable?:

a) Comunicación interventricular. b) Tetralogía de Fallot. c) Conducto arterioso persistente. d) Comunicación interauricular.

940: Lactante de 21 días de vida que presenta, desde hace 4 días, vómitos postpandriales que cada vez son más intensos y frecuentes. Señale cuál de las siguientes afirmaciones es cierta en relación a la enfermedad del niño:

a) Esta enfermedad es más frecuente en niños. b) Suele cursar con acidosis metabólica. c) El tratamiento es médico y la recidiva frecuente. d) La enfermedad está producida por una bacteria.

941: En la enfermedad de Hirschsprung una de las siguientes afirmaciones es FALSA:

a) Es una anomalía congénita. b) Predomina en el sexo masculino. c) Se caracteriza por alteraciones en las células ganglionares del plexo intramural. d) La distensión de la ampolla rectal ocasiona relajación del esfínter anal interno.

942: La malabsorción puede acompañar a todos los cuadros que se exponen EXCEPTO uno. Señale éste:

a) Fibrosis quística. b) Páncreas divisum. c) Déficit severo de IgA. d) Pancreatitis crónica.

943: Todo lo que sigue acerca del síndrome nefrótico en la infancia, es cierto EXCEPTO:

a) Colesterol sérico elevado. b) El 85% experimenta cambios mínimos de la enfermedad. c) Reabsorción reducida de sodio por el riñón. d) Triglicéridos sérico s elevados.

944: Un niño de cuatro años de edad muestra un importante retraso de crecimiento, lesiones de raquitismo resistentes al tratamiento con dosis habituales de vitamina D y poliuria. ¿Cuál de las siguientes asociaciones considera que permite el

Page 167: Cuestionario de Urología

diagnóstico de síndrome de Fanconi?:

a) Glucosuria + hiperarninoaciduria + alcalosis + hiperfosforemia. b) Glucosuria + hipoglucemia + acidosis metabólica + hipofosforemia. c) Glucosuria + hiperarninoaciduria + acidosis metabólica + hipofosforemia. d) Glucosuria + hiperarninoaciduria + alcalosis metabólica + hipofosforemia.

945: En la enfermedad de Kawasaki, todas las afirmaciones son correctas MENOS una. Señálela:

a) El diagnóstico se realiza habitualmente mediante biopsia ganglionar. b) Se acompaña de picos febriles elevados de al menos 5 días de evolución. c) La base patogénica de la enfermedad es una vasculitis. d) Las complicaciones más graves se relacionan con la afectación coronaria.

946: La anemia ferropénica del niño se caracteriza por los siguientes hechos EXCEPTO uno:

a) Es la causa más frecuente de anemia nutricional de la infancia. b) La lactancia materna prolongada evita su aparición. c) Su desarrollo está favorecido por las infecciones repetidas. d) El diagnóstico se basa en la determinación de la ferritina sérica.

947: Una niña de 10 años con atresia biliar, tratada con la técnica de Kasai cuando era lactante, presenta ahora torpeza progresiva, disminución de los reflejos tendinosos profundos y ataxia. El diagnóstico más probable es:

a) Encefalopatía hepática. b) Deficiencias de vitamina A. c) Encefalitis. d) Deficiencia de vitamina E.

948: Niño de 6 años que desde los 9 meses de vida ha tenido infecciones bacterianas de repetición (dos neumonías, otitis supuradas, sinusitis). En la analítica tenía una IgG de 103 mg/dl, IgA<6mg/dl, e IgM 25 mg/dl, linfocitos totales 2010/mm3, un número normal de linfocitos T y ausencia de linfocitos B. ¿Cuál de los siguientes es el diagnóstico correcto?:

a) Enfermedad granulomatosa crónica. b) Síndrome Variable Común de Inmunodeficiencia. c) Inmunodeficiencia combinada severa. d) Agarnmaglobulinemia ligada al sexo.

949: Un recién nacido presenta una deformidad rígida de ambos pies con sus plantas enfrentadas, los bordes laterales orientados hacia distal y los bordes mediales hacia craneal, y ambos antepiés más próximos a la línea media y más distales que los retropiés. ¿Cuál es la denominación más adecuada?:

a) Pies planos. b) Pies cavos. c) Pies valgos. d) Pies zambos.

950: El objetivo principal de contraste de hipótesis estadístico, aplicado a ciencias de la salud es:

a) Confurnar la hipótesis alternativa y la hipótesis nula. b) Distinguir entre 'las diferencias debidas al azar y las diferencias clínicas. c) Aumentar el valor predicitivo del estudio para tener certeza en las conclusiones. d) Establecer el error alfa o error tipo I.

951: Para contrastar la hipótesis, en un estudio de casos y controles, de que la media de colesterol (medio en mgldl) es diferente de los pacientes que presentan accidentes cerebrovasculares (100 casos), que en los pacientes que no lo presentan (100 controles). ¿Qué prueba estadística es la más apropiada para realizar este contraste?:

a) Correlación de Speaiman. b) Prueba de la t. c) Prueba de chi cuadrado. d) Correlación de Pearson.

952: Se está realizando un estudio para conocer el efecto de un nuevo analgésico. La variable principal de respuesta es una escala de dolor con los siguientes valores: 1 (no dolor), 2 (dolor leve), 3 (dolor moderado) y 4 (dolor intenso). ¿De qué tipo de variable se trata?:

a) Variable dicot6mica. b) Variable continua. c) Variable discreta. d) Variable ordinal.

953: En un estudio de casos y controles se estudia la relación entre cáncer de cavidad bucal (efecto) y consumo de bebidas alcohólicas (exposición). Como definiría el fenómeno que puede producir el hecho que el hábito de fumar se asocia a la aparición de cáncer bucal y también a un mayor consumo de bebidas alcohólicas?:

a) Sesgo de información. b) Sesgo por factor de confusión. c) Un error aleatorio. d) Sesgo de observación.

954: Identificamos en la práctica clínica habitual una población definida de pacientes con artritis reumatoide que no han respondido a un tratamiento con metotrexate, y que según criterio clínico reciben tratamiento o bien con etanercept o bien con infliximab. Se-guimos la evolución de la respuesta clínica de la enfermedad a largo plazo con el objetivo de realizar una comparación entre ambos grupos. En este caso estamos planteando un:

Page 168: Cuestionario de Urología

a) Estudio de cohortes. b) Estudio coste-efectividad. c) Estudio caso-control. d) Ensayo clínico pragmático o naturolístico.

955: ¿Cuál de los siguientes estudios es aquel en el cual los participantes reciben todas las opciones en estudio en diferentes momentos a lo largo de la experiencia?:

a) Ensayo de diseño cruzado. b) Ensayo de grupos paralelos. c) Ensayo de diseño factorial. d) Ensayo clínico enmascarado.

956: Señale la respuesta correcta. Un estudio en el que se seleccionan dos grupos de sujetos, uno formado por enfermos y el otro por personas libres de la enfermedad de interés, en el que se compara la exposición a posibles factores de riesgo en el pasado es un:

a) Estudio retrospectivo de doble cohorte. b) Ensayo de campo. c) Estudio retrospectivo de una serie de casos. d) Estudio de casos y controles.

957: En un estudio de seguimiento a largo plazo de pacientes con fibrilación auricular crónica en la práctica clínica habitual se ha documentado un riesgo relativo de padecer una hemorragia intracraneal para anticoagulantes orales (en comparación con antiagregantes plaquetarios) de 3 (intervalos de confianza del 95% 1,5-6). ¿Cuál de las siguientes afirmaciones es cierta?:

a) El diseño corresponde a un estudio de casos y controles. b) Los intervalos de confianza del riesgo relativo estimado confirman que las diferencias no alcanzan la significación estadística. c) El riesgo relativo estimado significa que la incidencia de hemorragia intracraneal es un 3% superior en el grupo tratado con anticoagulantes orales. d) El riesgo relativo estimado significa que la incidencia de hemorragia intracraneal entre los pacientes tratados con anticoagulantes orales ha sido tres veces superior a la de los tratados con antiagregantes plaquetarios.

958: En un estudio de seguimiento a largo plazo de pacientes con hipertensión arterial en la práctica clínica habitual, comparando los pacientes tratados con fármacos del grupo terapéutico A con fármacos del grupo terapéutico B, se ha documentado un riesgo de padecer un infarto de miocardio mayor para los pacientes tratados con fármacos del grupo terapéutico A (riesgo relativo de 2 con intervalos de confianza del 95% 1,4-3,1). ¿Cuál de las siguientes afirmaciones es cierta?:

a) El diseño corresponde a un ensayo clínico. b) Los intervalos de confianza del riesgo relativo estimado confirman que las diferencias alcanzan la significación estadística. c) El riesgo relativo estimado significa que la incidencia de infarto de miocardio oscila entre el 1,4 y el 3,1 %. d) El riesgo relativo estimado significa que existe una alta proporción de pacientes tratados con fármacos del grupo A presentaron un infarto de miocardio.

959: El riesgo de presentar un accidente vascular cerebral (A Ve) en una población de hipertensos sin tratamiento es del 6% a los 5 años de seguimiento. En una reunión se nos afirma que tratando a dicha población con el nuevo fármaco A conseguiremos evitar un A VC por cada 20 pacientes tratados. A cuanto debería reducir la incidencia del A VC el fármaco A para que dicho comentario fuera cierto:

a) 5%. b) 4%. c) 3%. d) 1%.

960: Ha realizado usted un estudio en el que ha seguido durante 2 años 1000 pacientes que han consumido AINEs (antiiflamatorios no esteroideos) y 2000 controles. En los consumidores de AINEs ha encontrado 50 pacientes con hemorragia digestiva alta, mientras que en los controles únicamente 10. Asumiendo que estas diferencias no se deben a sesgos ni a factores de confusión, ¿podría indicar cuál de las siguientes afirmaciones es correcta?:

a) El riesgo relativo de presentar hemorragia digestiva por el consumo de AlNEs es 5 y el riesgo atribuible en los expuestos 0,90 b) El riesgo relativo de presentar hemorragia digestiva por el consumo de AINEs es 10 y el riesgo atribuible en los expuestos 0,045 c) El riesgo relativo de presentar hemorragia digestiva por el consumo de AINEs es 10 y el riesgo atribuible en los expuestos 0,90 d) El riesgo relativo de presentar hemorragia digestiva por el consumo de AlNEs es 15 y el riesgo atribuible en los expuestos 0,045

961: Queremos investigar la relación entre hemorragia digestiva e ingesta de medicamentos mediante un estudio epidemiológico de tipo caso-control. En cuanto a la metodología, ¿cuál sería la respuesta correcta?:

a) Definiremos los casos como los pacientes que presentan hemorragia digestiva atribuida a medicamentos. b) Definiremos los controles como pacientes que presentan hemorragia digestiva no atribuida a medicamentos. c) La estimación del riesgo de hemorragia producida por los medicamentos será una razón de incidencias. d) Es fundamental la realización de una anarnnesis farmacológica idéntica en los casos y en los controles.

962: En el diseño de un ensayo clínico cuyo objetivo es demostrar que una nueva estrategia terapéutica no es peor que otra existente (estudio de noinferioridad) son correctas todas las afirmaciones siguientes EXCEPTO una. Indique cuál:

a) Debe establecerse a priori la diferencia entre tratamientos que aceptaríamos como clínicamente irrelevante (delta o límite de no inferioridad). b) El estudio debe diseñarse de forma que fuese capaz de detectar diferencias relevantes entre los tratamientos evaluados, si éstas existiesen.

Page 169: Cuestionario de Urología

c) Para el cálculo del límite de no-inferioridad debe estimarse con la mayor precisión posible la magnitud del efecto placebo, si existiese, en la población diana. d) El análisis por intención de tratar es el único que se considera válido en los estudios de no-inferioridad.

963: En un eusayo clínico cruzado de dos períodos ¿cuál es la consecuencia de que el efecto residual de ambas intervenciones no sea igual?:

a) Que hay que recurrir a técnicas estadísticas multivariantes. b) Ninguna, precisamente en esta circunstancias es cuando este tipo de ensayo plantea mayores ventajas. c) Que el efecto del fármaco usado en segundo lugar depende del efecto del fármaco, administrado en primer lugar. d) Que ya no es posible utilizar a cada sujeto como su propio control, con lo que el tamaño muestral puede hacerse insuficiente.

964: Si en un ensayo clínico se usa placebo ¿cuál es el proceder correcto?:

a) Por respeto a los principios bioéticos comunicárselo a los sujetos que vayan a recibirlo. b) No comunicárselo a los sujetos ya que en caso contrario se pierde el efecto placebo. c) Comunicárselo a los sujetos una vez concluido el estudio. d) Comunicárselo a los sujetos en el momento de solicitar su consentimiento para ser incluidos en el estudio.

965: El Sistema Español de Farmacovigilancia, a través de su programa de notificación espontánea de reacciones adversas, constituye un sistema de vigilancia epidemiológica de las reacciones adversas a medicamentos en nuestro país. Indique cuál de los siguientes aspectos NO es propio de este programa:

a) El sistema recoge las notificaciones enviadas voluntariamente por los profesionales sanitarios. b) El programa colabora con el Programa Internacional de Farmacovigilancia de la Organización Mundial de la Salud. c) Permite comparar el perfil de notificaciones de nuestro país con el de otros países. d) Permite cuantificar el riesgo absoluto de los medicamentos una vez comercializados.

966: Señale cuál de los signientes objetivos NO es propio de los Estudios de Utilización de Medicamentos:

a) Obtener información cuantitativa sobre uso real de los medicamentos. b) Evaluar si la prescripción, de medicamentos se corresponde con los usos establecidos/adecuados de los fármacos. c) Evaluar la eficacia terapéutica de un fármaco en comparación con la utilización de un placebo. d) Evaluación de los factores que determinan la prescripción de los fárrnacos.

967: Los años de vida ganados ajustados por calidad (A V AC) se usan, particularmente, en los estudios de:

a) Coste-Utilidad. b) Coste-Efectividad. c) Coste-Beneficio. d) Coste-Eficacia

968: Todas las siguientes son actividades de prevención primaria, EXCEPTO:

a) Quimoprevención. b) Mamografía. c) Inmunizaciones. d) Quimoprofilaxis.

969: ¿Cuál de los siguientes factores NO está relacionado con una mayor utilización de los servicios sanitarios públicos?:

a) Presencia de enfermedades crónicas. b) Mayor tamaño familiar. c) Déficit de apoyo social. d) Nivel socioeconómico bajo.

970: Indique cuál de las siguientes afirmaciones es FALSA en relación con las actividades de prevención:

a) La prevención primaria tiene como objetivo disminuir la incidencia de la enfermedad. b) La prevención secundaria tiene como objetivo disminuir la prevalencia de la enfermedad. c) El valor predictivo positivo de una prueba de cribado aumenta al disminuir la prevalencia de la enfermedad. d) El valor ptedictivo positivo de una prueba de cribado aumenta al aumentar la especificidad de la prueba.

971: Está usted evaluando una prueba diagnóstica y finalmente se encuentra la siguiente tabla:

A partir de ella y redondeando decimales, señale cuál de las siguientes afirmaciones es verdadera:

a) El valor predictivo positivo es del 56% y el valor predictivo negativo del 13%. b) El valor predictivo positivo es del 44% y el valor predictivo negativo del 98%. c) El valor predictivo positivo es del 80% y el valor predictivo negativo del 90%. d) El valor predictivo positivo es del 80% y el valor predictivo negativo del 11 %.

972: Está usted comparando el diagnóstico clínico de úlcera gastroduodenal y su hallazgo en la autopsia en una serie de 10.000 pacientes. Al comenzar a analizar los datos se construye la siguiente tabla:

Enfermedad

SI NO

Prueba

Positiva 80 100

Negativa 20 800

Autopsia

Úlcera No úlcera

Diagnóstico clínico

Page 170: Cuestionario de Urología

De las siguientes cifras, ¿cuál es la más cercana a la sensibilidad del diagnóstico clínico de la úlcera?:

a) 3%. b) 43%. c) 87%. d) 98%.

973: La Educación para la Salud de la población, de forma general constituye una estrategia de:

a) Promoción de la salud. b) Protección de la salud. c) Prevención de la enfermedad. d) Potenciación de la salud.

974: El número de ingresos de hospitalización potenciales, en las camas asignadas a un servicio médico, en un período de tiempo, es:

a) Inversamente proporcional al índice de ocupación de camas. b) Directamente proporcional al índice de complejidad de la casuística de hospitalización. c) Inversamente proporcional a la frecuentación de urgencias. d) Inversamente proporcional a la estancia media.

975: ¿Cuál de los siguientes enunciados forma parte del concepto defiuitorio de lo que se entiende por "Cirugía mayor ambulatoria"?:

a) Son procedimientos quirúrgicos programados. b) Sólo se consideran los procesos con anestesia local o loco-regional. c) Es equivalente a cirugía de corta estancia. d) Incluye las intervenciones de gran simplicidad técnica con anestesia local, sin que precisen vigilancia especial.

976: La pauta habitual de 3 dosis de V acuna anti Hepatitis B a los O, 1 y 6 meses asegura una respuesta de anticuerpos protectores que se mide a través de:

a) Anti HBc. b) Anti HBe. c) Anti HBs. d) Anti HC.

977: En un paciente con hiperlipemia, la asociación de lovastatina y gemfibrozilo, presenta un elevado riesgo de:

a) Agranulocitosis. b) Rabdomiolisis. c) Fibrosis pulmonar. d) "Torsade des points".

978: Uno de los siguientes grupos de antibióticos antibacterianos actúa inhibiendo la síntesis de proteínas.

a) Quinolonas. b) Aminoglucósidos. c) Isoniacida. d) Cicloserina.

979: ¿Cuál de los siguientes psicofármacos ha demostrado claramente eficacia en el tratamiento de la adición al tabaco?:

a) Clorpromacina. b) Fluoxetina. c) Bupropión. d) Flunitrazepan.

980: ¿Cuál de las siguientes modificaciones farmacocinéticas es de esperar en el embarazo?:

a) Disminución de la absorción intramuscular en los dos primeros trimestres. b) Disminución del flujo sanguíneo renal. c) Disminución de absorción de los fármacos liposolubles. d) Aumento de la fracción libre de los fármacos ácidos débiles.

981: ¿Cuál de los siguientes fármacos puede producir MENOS efecto en enfermos con hepatopatía severa que en sujetos sanos normales?:

a) La difenilhidantoína. b) La lidocaína. c) La tetraciclina. d) La imiprarnina.

982: Una de las siguientes propiedades referidas a los virus RNA es cierta:

a) La mayoría de ellos se multiplican en el núcleo. b) La estructura del genoma no es determinante del mecanismo de transcripción y replicación. c) No es necesario que codifiquen RNA polimerasas RNA dependientes. d) El genoma RNA (+) de los retrovirus se convierte en DNA, que es integrado en la cromatina del huésped y transcrito como un gen celular.

983: ¿Cuál, entre los siguientes, es un virus persistente que puede permanecer' en latencia y reactivarse?:

Úlcera 130 20

No úlcera 170 9680

Page 171: Cuestionario de Urología

a) El virus de la hepatítis A. b) El virus respiratorio sincitial. c) El citomegalovirus. d) El virus de la poliomielitis.

984: Hombre de 20 años con clínica de infección urinaria. En el urocultivo se aísla una cepa con fenotipo sensible de Proteus Mirabilis. Señale el antimicrobiano MENOS apropiado para tratar esta infección:

a) Trimetoprim-sulfametoxazol. b) Cefuroxima. c) Ciprofloxacino. d) Nitrofurantoina.

985: La defensa frente a Mycobacterium Tuberculosis depende esencialmente y en último extremo de:

a) Los anticuerpos de la clase Ig G. b) El interferón alfa. c) Los leucocitos polinucleares basófilos. d) Los macrófagos activados por el interferón garnma.

986: Uno de los siguientes microorganismo NO tiene su hábitat natural en la boca ni en la nasofaringe. Señálelo:

a) Estreptococo del grupo "Viridans". b) Haemophilus Injluenzae. c) Moraxella Catarrhalis. d) Escherichia Coli.

987: ¿Cuál es el gen diana más frecuentemente alterado en neoplasias humanas?:

a) p53 (TP53). b) pl6 (INK4a). c) Retinoblastoma. d) K-Ras.

988: El término diferenciación aplicado al tejido neoplásico define:

a) El grado de similitud de las células neoplásicas desde el punto de vista morfológico y funcional con las células normales de las que derivan. b) La aparición dentro de un tumor de elementos neoplásicos que no están presentes en el órgano o en el tejido en el cual se originan. c) La presencia de un grado extremo de anaplasia. d) La presencia de áreas tumorales de morfología diferente en cambios adyacentes del tumor (por ejemplo adenocarcinoma y carcinoma epidermoide).

989: Los pacientes que desarrollan rechazo crónico del injerto tras trasplante renal, hepático o cardiaco, presentan como lesión común a todos ellos:

a) Infiltración inflamatoria intersticial de carácter mixto. b) Necrosis fibrinoide de la pared vascular. c) Proliferación fibrosa endointimal arterial estenosante. d) Angiogénesis difusa del injerto.

990: La malignización de un pólipo del aparato digestivo viene determinada por la invasión de las células cancerosas en la:

a) Serosa. b) Mucosa. c) Muscular. d) Muscular de la mucosa.

991: ¿Cuál de los siguientes procesos linfoproliferativos corresponde a un linfoma de linfocitos T?:

a) Linfoma folicular. b) Linfoma de células del manto. c) Micosis fungoides. d) Linfoma linfoplasmocitario.

992: Junto con las fibras motoras que configuran el nervio Motor Ocular Común, se encuentran fibras del Sistema Nervioso Autónomo de naturaleza parasimpática. ¿En qué ganglio terminan las fibras preganglionares parasimpáticas?:

a) Ótico. b) Pterigo-palatino. c) De Gasser. d) Ciliar.

993: En la articulación escápulo-bumeral, el llamado manguito de los rotadores está formado por cuatro músculos. De las secuencias citadas ¿cuál es la verdadera?:

a) Supraespinoso; Cabeza corta del Bíceps braquial; Infraespinoso; Deltoides. b) Supraespinoso; Infraespinoso; Redondo menor; SubescaPillar. c) Deltoides; Supraespinoso; Córaco-Braquial; Subescapular. d) Infraespinoso; Cabeza larga del Bíceps braquial; Deltoides; Subescapular.

994: El conducto biliar deriva del:

a) Ectodermo. b) Endodermo.

Page 172: Cuestionario de Urología

c) Mesodermo. d) Mesénquima.

995: En la anatomía de la primera vértebra cervical, uno de los siguientes componentes NO entra en su constitución:

a) Cuerpo. b) Tubérculo anterior. c) Arco posterior. d) Tubérculo posterior.

996: Uno de los siguientes NO es un derivado de la cresta neural:

a) Neuronas de Purkinje del cerebelo. b) Neuronas de los ganglio s raquídeos (DRG). c) Melanocitos de la piel. d) La médula adrenal.

997: La inmunogenicidad de las vacunas con antígenos polisacáridos puede incrementarse conjugándose con:

a) Lipopolisacáridos. b) DNA. c) RNA. d) Proteínas.

998: Los parásitos, protozoos y helmintos, provocan una notable variedad de respuestas inmunes. Señale la respuesta FALSA:

a) Las infecciones helmínticas generan títulos de IgE superiores a los inducidos por otros agentes infecciosos. b) De forma específica los helmintos estimulan los linfocitos T CD4+ facilitadores, que producen ILA e IL-5. c) Los protozoos intracelulares activan con frecuencia células B. d) Ciertos parásitos como Schistosoma Mansonii producen huevos capaces de inducir la formación de granulomas en el hígado y otros órganos.

999: ¿Cuál de las siguientes respuestas es FALSA en elación a los receptores para el antígeno?:

a) Los receptores de los linfocitos T reconocen fragmentos de antígenos presentados por moléculas de inmunoglobulinas. b) Los receptores de los linfocito s B reconocen antígenos en su forma nativa. c) Los receptores de los linfocito s T tienen dos cadenas variables. d) Los receptores de los linfocitos B tienen cadenas variables que constituyen inmunoglobulinas.

1000: Todas las aseveraciones siguientes sobre las moléculas de histocompatibilidad (moléculas HLA) son ciertas MENOS una. Señálela:

a) Son polimórficas. b) Las moléculas de clase I presentan péptidos a los linfocitos T CD4+ y las de clase II a los linfocitos T CD8+. c) Constan de dos cadenas polipeptídicas. d) Su mision es la presentación de péptidos a los que se unen en el interior de las células.

1001: ¿Cuál es la base genética molecular de la mayoría de síndromes de cáncer familiar?:

a) Mutación dominante en línea germinal de un protooncogen. b) Mutación no dominante en línea germinal de un protooncogen y mutación dominante somática del segundo alelo. c) Mutación en línea germinal de un alelo de un gen supresor de tumores e inactivación somática del segundo alelo. d) Mutación en línea germinal de los dos alelos de un gen supresor de tumores.

1002: En relación con el metabolismo del hierro, señale la afirmación correcta:

a) La absorción de hierro tiene lugar en el íleon. b) La asimilación de hierro mejora con una dieta vegetariana. c) La mayor parte del hierro sérico se localiza en la ferritinao d) La absorción de hierro medicamentoso empeora con losalimentos.

1003: ¿Cuál es la consecuencia del aumento de la frecuencia de descarga de los barorreceptores del seno carotídeo?:

a) Activación del centro vasoconstrictor del bulbo. b) Inhibición del centro parasimpático vagal. c) Vasoconstricción arteriolar. d) Disminución de la frecuencia cardiaca.

1004: ¿Cual de las siguientes afirmaciones con respecto a la contracción del músculo liso y del músculo esquelético es cierta?:

a) El músculo liso no tiene filamentos de actina y miosina. b) Al contraerse las células del músculo liso se acortan menos que las del músculo esquelético. c) El ciclo de unión y liberación de la miosina a la actina es de menor duración en el músculo liso que en el esquelético. d) Para mantener la misma tensión de contracción, el músculo liso requiere menos energía que el músculo esquelético.

1005: ¿Cuál de los siguientes fenómenos es el responsable de la fase 0 (fase rápida) de despolarización de un miocardiocito ventricular?:

a) Aumento de conductancia para el sodio (gNa). b) Aumento de conductancia para el potasio (gK). c) Aumento de conductancia para el cloruro (gCI). d) Aumento de conductancia para el calcio (gCa).

1006: En relación con las metástasis hepáticas procedentes de carcinoma colo-rectal, señale, de las siguientes afirmaciones, cuál es la más correcta:

a) La cirugía está contraindicada cuando se aprecia más de 1 lesión metastásica.

Page 173: Cuestionario de Urología

b) La afectación microscópia del margen de resección no influye en el pronóstico siempre que se realice quimioterapia postoperatoria. c) La supervivencia a los 5 años después de un tratamiento quirúrgico radical es del 30-50%. d) Se ha demostrado que la crioterapia consigue resultados mejores a los de la resección.

1007: Un hombre de 47 años, fumador de 50 paquetes/año, cartero en activo, obeso (IMC 31 Kg/m2), diabético desde hace 10 años, controlado con insulina, hipertenso (toma nifedipino 30 mg/día), con insuficiencia renal crónica (creatinina habitual 2 mg/dl) y con hipercolesterolemia (240 mg/dl) acude al médico de cabecera por disnea desde hace una semana que le obliga a pararse cuando camina en llano (previamente lo hacía sin dificultad); no ha tenido fiebre ni dolor torácico. Cree que estáalgo acatarrado, pues por las noches tose sin expectorar. La tensión arterial es 150/90. No tiene edemas. Tiene algunos roncus dispersos, crepitantes en bases y se oye un posible tercer tono, aunque está algo taquicárdico (108 Ipm) para valorar bien este dato. Los análisis son normales salvo la creatinina de 2,5 mg/dl. Una radiografia de tórax muestra afectación aIveolointersticial bilateral ba-sal, sin cardiomegalia. ¿Cuál sería la aproximación diagnóstica más correcta en este momento?:

a) Una espirometría demostrará casi con seguridad un patrón obstructivo, por lo que se debe comenzar tratamiento broncodilatador. b) La fibrosis pulmonar idopática requiere una biopsia pulmonar para su diagnóstico. Se le debe recomendar una broncoscopia o biopsia transbronquial. c) Una ergometría ambulante sería recomendable para descartar una isquemia silente, muy frecuente en diabéticos, pero no es preciso si el electrocardiograma basal no muestra ondas Q ni alteraciones de la repolarización. d) Está indicado el ingreso y descartar lo antes posible una cardiopatía isquémica, iniciar tratamiento diurético parenteral y vasodilatador (inhibidores de la angiotensina convertasa), posiblemente añadir un betabloqueante y controlar los factores de riesgo cardiovascular.

1008: Una mujer de 36 años presenta desde hace 2 meses parestesias en la mano derecha de predominio nocturno sobre todo en la mitad radial de la palma de la mano, tres primeros dedos y mitad radial del cuarto dedo. La intensidad ha ido en aumento apareciendo dolor irradiado hacia el antebrazo que incluso llega a despertarla. ¿Qué estudio y con qué finalidad considera que debe realizarse?:

a) Una Resonancia Magnética Cervical para diagnosticar una hernia discal C5-C6 b) Un estudio Doppler del miembro superior derecho para diagnosticar una estenosis de la arteria radial. c) Un electromiograma y velocidades de conducción para diagnosticar un síndrome de túnel del carpo derecha. d) Una Resonancia Magnética del Cráneo para diagnosticar una Esclerosis Múltiple.

1009: En relación al déficit aislado de inmuglobulina A, señale la respuesta correcta:

a) Debe ser tratada con inmunoglobulinas durante los episodios de infección respiratoria. b) Favorece la aparición de púrpura de Schonlein-Henoch. c) Puede provocar reacciones postransfusionales. d) Disminuye la incidencia de enfermedades autoinmunes.

1010: ¿Qué entenderemos por "riñón de mieloma"? :

a) Cualquier fracaso renal en un paciente portador de un mieloma. b) La infiltración renal por el mieloma. c) La precipitación de cadenas ligeras en los túbulos renales. d) El depósito de proteínas de Bence-Jones en el glomérulo.

1011: En un hemograma rutinario de un paciente no fumador de 65 años, se obtienen los siguientes parámetros: concentración de hemoglobina 19 g/dl, valor hematocrito 55%. La masa eritrocitaria medida por dilución de hematíes marcados con Cr51 y los niveles de eritropoyetina sérica están elevados. La saturación arterial de oxígeno es del 95 %. ¿Cuál, entre los siguientes, es el diagnóstico más probable?:

a) Enfermedad pulmonar obstructiva crónica. b) Policitemia vera. c) Policitemia espúrea. d) Carcinoma renal.

1012: Los ensayos clínicos pragmáticos se reconocen por una de las siguientes características:

a) El efecto del tratamiento se evalúa en las condiciones más experimentales posibles. b) Los pacientes que se incluyen en estos ensayos clínicos son muy homogéneos. c) Las pautas de tratamiento son fijas para todos los sujetos. d) En estos ensayos se intenta que los sujetos que se incluyen sean muy parecidos a los pacientes que se ve en la práctica clínica.

1013: El tratamiento empírico de la meningitis purulenta a germen desconocido en una persona anciana (>75 años de edad), es:

a) Penicilina 2xl06 UU/4h i.v. b) Ceftriaxona 6g/24h./i.v. c) Cefotaxima 2g/4h/i.v. + Vancomicina 1g/12h/i.v. d) Ceftriaxona 2g i.v./24 h + Ampicilina 2g/4h/i.v.

1014: Si hacemos fijar la vista en un punto a un paciente con vértigo y nistagmo de origen periférico, observaremos que el nistagmo:

a) No se modifica. b) Disminuye o desaparece. c) Aumenta. d) Cambia de dirección.

1015: El marcador morfológico de la progresión de una hepatitis crónica es:

a) La hepatitis de la interfase. b) La esteatosis. c) La densidad del infiltrado inflamatorio portal.

Page 174: Cuestionario de Urología

d) La necrosis lobulillar.

PARTE 3 de 3 (736 cuestiones)

1. En un paciente con transaminasas elevadas y un cociente GOT/GPT superior a dos, ¿cuál sería el diagnóstico más probable?

a. Hepatitis vírica aguda. b. Hepatitis tóxica. c. Hepatitis alcohólica d. Hepatitis granulomatosa

2. El reflujo gastroesofágico puede relacionarse con cualquiera de las entidades siguientes EXCEPTO una. Señálela:

a. Fibrosis pulmonar. b. Broncoespasmo. c. Neumonía recurrente. d. Sinusitis.

3. Cuál de las siguientes situaciones es la indicación más adecuada para solicitar una amoniemia?:

a. Un escolar asintomático que dice haber ingerido amoniaco dos horas antes. b. Un cirrótico conocido con ascitis y edemas, que ingresa por fiebre, aunque con sensorio normal. c. Un cirrótico conocido, que ingresa por hemorragia digestiva alta, que se choca poco después. d. Un paciente sin historia previa de hepatopatía que ingresa en coma, sin causa clara aparente.

4. Un paciente ingresa con el cuadro clínico de pancreatitis aguda severa. La ecografía y TAC abdominal demuestran la existencia de coledocolitiasis y gran necrosis pancreática. ¿Cuál de estas opciones terapéuticas es la más conveniente?:

a. Ácido ursodeoxicólico. b. Litotricia por ultrasonidos. c. Colangiopancreatografía retrógrada con papilotomía endoscópica. d. Metronidazol intravenoso.

5. De los siguientes fármacos todos, EXCEPTO uno, se utilizan en el tratamiento de la enfermedad de Crohn. Señálelo:

a. Prednisona. b. Azatioprina. c. Mesalazina. d. Cloroquina.

6. Varón de 60 años, bebedor de más de 100 g/día de alcohol desde los 20 años, con cuadros recurrentes de dolor posprandial intenso en zona abdominal derecho, desde hace 10 años. Durante estos episodios, la determinación de enzimas pancreáticas en sangre son normales. La TAC abdominal muestra microcalcificaciones en la porción cefálica del páncreas sin otros hallazgos. La pancreatografía retrógrada endoscópica muestra estenosis del conducto de Wirsung a nivel cefalocorporal. El enfermo ha dejado el alcohol hace 3 años y está en tratamiento con reemplazamiento enzimático, alcalinos y 60 mg de sulfato mórfico al día. ¿Cuál de los propuestos sería el tratamiento más adecuado?:

a. Aumentar el doble el tratamiento analgésico. b. Disminuir la ingesta de grasas y proteínas. c. Aumentar la dosis de enzimas pancreáticas, suspendiendo la alcalinización del medio intestinal. d. Duodenopancreatectomía cefálica con preservación pilórica.

7. ¿Cuál de los siguientes tumores malignos del hígado es el más frecuente en los países desarrollados?:

a. Colangiocarcinoma. b. Hepatocarcinoma. c. Metástasis de cáncer de estómago. d. Metástasis de cáncer de cólon.

8. La asociación de fiebre en picos, dolor en hipocondrio derecho e ictericia, conocida como tríada de Charcot, es muy sugerente de:

a. Colangitis aguda. b. Colangitis esclerosante primaria. c. Colecistitis aguda. d. Pancreatitis aguda.

9. Un ejecutivo de una gran empresa, de 40 años y sin antecedentes de interés, acude al Servicio de Urgencias por presentar un dolor brusco y de gran intensidad, localizado en el epigastrio, que ha comenzado una hora antes y no cede. A la exploración, el paciente está pálido, sudoroso, hipotenso, con respiración superficial: el abdomen contracturado es muy doloroso a la palpación superficial, mostrando desaparición de la matidez hepática a la percusión. ¿Cuál de los siguientes es el diagnóstico más probable?:

a. Apendicitis aguda. b. Pancreatitis aguda. c. Perforación gástrica. d. Colecistitis aguda.

10. Un hombre de 20 años acude a Urgencias por la mañana por dolor en fosa ilíaca derecha (FID). Refiere que ha tenido dolor periumbilical el día anterior y que ha vomitado dos veces por la noche. En el examen físico está afebril y el abdomen presenta dolor a la palpación en FID: los sonidos intestinales son normales. La analítica de sangre muestra 10.000 leucocitos/mm3 y la orina 5-10 leucocitos por campo. Señale lo más apropiado, respecto a la solicitud de consulta quirúrgica:

a. Debe ordenarse inmediatamente. b. Debe solicitarse si desarrolla fiebre. c. Debe solicitarse si aparece sensibilidad de rebote.

Page 175: Cuestionario de Urología

d. Debe solicitarse si los leucocitos aumenta a 15.000 /mm3.

11. Ante un paciente que presenta dolor abdominal de tipo cólico, vómitos, distensión abdominal e incapacidad para evacuar gases y heces, hay que pensar como primer diagnóstico en:

a. Pancreatitis aguda. b. Obstrucción intestinaa. c. Colecistitis aguda. d. Apendicitis aguda.

12. Ante un paciente con el diagnóstico clínico de sospecha de abdomen agudo por perforación de úlcera péptica, ¿cuál es la prueba diagnóstica que suele confirmar el diagnóstico en un 60-80% de los casos y a la que es necesario recurrir en primer lugar?:

a. TAC de tórax y abdomen con ingesta de gastrografín. b. Ecografía del piso abdominal superior en bipedestación y/o decúbito supino. c. RMN de tórax y abdomen con ingesta de material de contraste. d. Radiografía antero-posterior simple de tórax y abdomen en bipedestación.

13. ¿Cuál es el procedimiento diagnóstico de elección en un paciente con sospecha de colelitiasis?:

a. Colecistografía oral. b. Radiografía simple de abdomen. c. Colangiografía intravenosa. d. Ecografía abdominal.

14. Ante un caso clínico con el diagnóstico de sospecha de obstrucción intestinal, ¿cuál es la exploración complementaria a la que hay que recurrir para confirmar el diagnóstico?:

a. TAC de abdomen en decúbito lateral o supino. b. Ecografía abdominal en bipedestación y/o decúbito supino. c. RMN de abdomen en decúbito lateral o supino. d. Radiografía simple de abdomen en bipedestación.

15. El tratamiento de la oclusión intestinal mecánica completa es:

a. Cirugía urgente en todos los casos. b. Colocación de sonda de Miller-Abbott. c. Cirugía sólo si se demuestra que existe estrangulación. d. Cirugía urgente o diferida según el tipo de oclusión (simple o con estrangulación).

16. De entre las siguientes afirmaciones sobre el carcinoma gástrico precoz, señale cuál es FALSA:

a. Operado, tiene buen pronóstico, con supervivencia hasta del 95% de los casos a los cinco años. b. No sobrepasa la submucosa. c. No tiene capacidad para producir metástasis ganglionares linfáticas. d. Existe una clara correlación entre la profundidad de la invasión del tumor y la tasa de supervivencia.

17. Uno de los siguientes mecanismos patogénicos NO interviene en la peritonitis bacteriana primitiva. Señálelo:

a. Contigüidad desde una infección pulmonar o urinaria. b. Diseminación hematógena desde un foco extraabdominal. c. Vía canalicular ascendente a través del aparato genital femenino. d. Traumatismo abdominal abierto

18. ¿En cuál de las siguientes poliposis es más frecuente la aparición de un cáncer de cólon?:

a. Síndrome de Peutz-Jeghers. b. Poliposis cólica familiar. c. Síndrome de Cronkhite-Canadá. d. Poliposis juvenil.

19. Un paciente adulto que ha sufrido un accidente laboral acude al Servicio de Urgencias por presentar quemaduras de primer grado en todo el miembro superior izquierdo, de segundo grado en la cara anterior del tronco y de tercer grado en la cara anterior del miembro inferior izquierdo. El médico de guardia, para valorar el pronóstico vital del paciente y establecer el tratamiento general más adecuado, evalúa la superficie corporal quemada aplicando la "regla de los 9" o de WaUace. ¿Cuál sería el resultado?:

a. 9%. b. 18%. c. 27%. d. 36%.

20. Ante una mujer de 20 años, que presenta una tumoración de 2 cm de diámetro en el cuadrante ínfero-externo de la mama derecha, indolora, de consistencia firme, superficie lisa, forma ovoidea, móvil y bien delimitada del parénquima vecino, sin antecedentes de derrame por el pezón, sin "piel de naranja" ni retracción del pezón, ¿cuál sería su diagnóstico de presunción?:

a. Abceso. b. Carcinoma. c. Ectasia de los conductos mamarios. d. Fibroadenoma.

21. De las siguientes definiciones sólo una corresponde a la hernia incoercible. Señálela:

a. Su contenido no puede introducirse en la cavidad abdominal. b. Su contenido se introduce espontáneamente en la cavidad abdominal cuando el paciente adopta una posición de decúbito supino. c. Su contenido reaparece inmediatamente después de ser reducido a la cavidad abdominal.

Page 176: Cuestionario de Urología

d. Aparece un cierto tiempo después de haber sido tratada quirúrgicamente.

22. Ante una gasometría arterial con pH 7.25, pCO2 58 mmHg, pO2 60 mmHg Y bicarbonato 16 mEq/L, usted pensaría en:

a. Diabetes en descompensación cetósica. b. Sobredosificación de salicilatos. c. Sobredosificación de benzodiacepinas. d. Sepsis con mala perfusión periférica.

23. Señale cuál de los siguientes resultados del estudio de un líquido pleural es incompatible con empiema:

a. Proteínas>3 g/dL. b. Glucosa> 40 mg/dL c. LDH>600 U/dL. d. pH>7.20.

24. ¿Cuál de las siguientes es la mejor forma de localizar un derrame pleuralloculado?:

a. Radiografía de tórax en espiración forzada. b. Radiografía de tórax en decúbito ipsilateral. c. Radiografía de tórax en decúbito contralateraa. d. Ecografía.

25. Varón de 20 años, sin datos previos de interés, que consulta por cuadro febril y dolor pleurítico intenso. En la radiografia de tórax se objetiva derrame pleural unilateral cuyo análisis muestra caracterísitcas de exudado linfocitario. ¿Cuál, de los que se relacionan, es el diagnóstico más probable?:

a. Neumonía por Legionella. b. Tuberculosis pleural. c. Embolia de pulmón. d. Neumonía por citomegalovirus.

26. ¿Cuál es el tratamiento de elección de la neumonía por Legionella?:

a. Ciprofloxacino. b. Amoxicilina-clavulánico. c. Eritromicina. d. Tetraciclinas.

27. Uno de los hechos siguientes NO es propio de la neumonía eosinofílica:

a. Curso indolente pero progresivo de tos y disnea. b. Fiebre recurrente, tos y disnea. c. Síntomas asmáticos. d. Distribución central de los infiltrados alveolares.

28. Varón de 40 años, fumador y con una lesión periférica de 2 cm en pulmón derecho, que por biopsia transtorácica muestra ser un carcinoma epidermoide. Pruebas funcionales respiratorias normales. Investigación de extensión negativa para metástasis. La TAC torácica muestra un aparente ganglio paratraqueal derecho de 1,5 cm. ¿Cuál debería ser el paso siguiente?:

a. Ordenar revisión periódica que incluya TAC torácica. b. Practicar mediastinoscopia. c. Realizar biopsia de médula ósea. d. Iniciar radioterapia

29. Todas las entidades siguientes pueden ser causa del llamado síndrome de hemorragia alveolar, EXCEPTO:

a. Enfermedad por enticuerpos anti-membrana basaa. b. Lupus eritematoso sistémico. c. Granuloma eosinófilo. d. Hemosiderosis pulmonar idiopática.

30. ¿Cuál es el factor más importante en el producción de hipertensión arterial pulmonar?:

a. Enfisema. b. Policitemia c. Obstrucción bronquial. d. Hipoxemia.

31. Un paciente de 22 años, deportista y sin hábitos tóxicos, acude a Urgencias. Refiere dolor brusco en hemitórax izquierdo, con ligera disnea y tos, que se calma con el reposo. Presenta también ortopnea. Nada más llegar, se agudizan sus síntomas y presenta una disnea muy intensa con colapso circulatorio y bradicardia. ¿Cuál es el diagnóstico más probable entre los siguientes?:

a. Tromboembolismo pulmonar. b. Neumotórax a tensión. c. Derrame pleural derecho masivo. d. Infarto agudo de miocardio.

32. Señale cuál de estas situaciones suele cursar con hipoxemia con hipercapnia:

a. Metástasis pulmonares. b. Crisis de miastenia. c. Neumonía lobar. d. Tromboembolismo pulmonar.

33. El aumento de vibraciones vocales y la presencia de broncofonía en la auscultación pulmonar es sugerente de uno de los

Page 177: Cuestionario de Urología

siguientes cuadros. Señálelo:

a. Neumotórax. b. Neumonía lobar. c. Broncospasmo. d. Derrame pleural.

34. Si un enfermo de 50 años tiene una segunda neumonía en el lóbulo inferior derecho, tres meses después de la primera, con ligera pérdida de volumen, debe sospecharse:

a. Neumonía por Legionella Pneumophila. b. Neumonía por Neumomococo resistente a la antibioterapia previa. c. Obstrucción endobronquial. d. Embolismo pulmonar asociado.

35. ¿Cuál de los siguientes datos es el único que se correlaciona significativamente con la tasa de supervivencia, al año de alta, de enfermos con enfermedad pulmonar obstructiva crónica (EPOC) 'que requirieron ventilación mecánica?:

a. Presencia o ausencia de "cor pulmonale". b. Nivel de actividad física previo al episodio. c. Nivel de hipercarbia. d. Presencia de insuficiencia ventricular izquierda.

36. Un paciente de 56 años, fumador de 20 cigarrillos al día desde los 13 años de edad, presenta desde hace un mes expectoración hemoptoica escasa. La radiografía de tórax simple muestra una imagen de masa paratraqueal derecha de unos 10 cm de diámetro. ¿Qué estudio de los propuestos considera prioritario en este paciente?:

a. Broncoscopia. b. Baciloscopia de esputo y tinción de Zieha. c. Gammagrafía pulmonar con Galio. d. Ecografía abdominal.

37. Si un paciente varón de 60 años presenta un síndrome de vena cava superior, observándose una masa paratraqueal derecha en la Rx de tórax simple, la causa más probable será:

a. Tumor germinal. b. Tumor neurogénico benigno. c. Tumor neurogénico maligno. d. Carcinoma broncogénico.

38. En el empiema agudo tabicado, una opción terapéutica previa a la toracotomía puede ser:

a. Toracocentesis repetidas. b. Drenaje endotorácico e instilación de fermentos fibrinolíticos intrapleurales. c. Instilación de tetraciclina intrapleural. d. Toracoplastia.

39. La hipoxemia que aparece en la embolia de pulmón masiva se debe fundamentalmente a:

a. Alteración de la difusión secundaria a la disminución del tiempo de paso por el lecho vascular pulmonar. b. Disminución de la PO2 en sangre venosa secundaria a la insuficiencia cardíaca. c. Aumento del espacio muerto ventilatorio en las áreas mal perfundidas. d. Aumento de la mezcla venosa secundaria al mantenimiento de la perfusión en área mal ventilada.

40. Referente a la Tetralogía de Fallot, señale lo que NO sea correcto:

a. Representa el 10% aproximadamente de todas las cardiopatías congénitas. b. Se asocia a malformaciones de la circulación coronaría. c. El acabalgamiento de la aorta es debido a la dextroposición del gran vaso. d. En el 75% de los enfermos la obstrucción se localiza a nivel valvular.

41. La existencia de pulso paradójico puede detectarse en las siguientes situaciones clínicas, EXCEPTO:

a. Taponamiento cardíaco. b. Estenosis aórtica. c. EPOC. d. Tromboembolismo pulmonar.

42. ¿En cuál de los siguientes procesos NO es característica la presencia de soplo sistólico?:

a. Estenosis aórtica. b. Prolapso mitral. c. Mixoma auricular. d. Rotura de músculo papilar.

43. Uno de los siguientes datos clínicos NO es característico del infarto de ventriculo derecho. Señálelo:

a. Asociación con infarto inferior. b. Hipotensión arterial. c. Ingurgitación yugular. d. Edema agudo de pulmón.

44. Señale cuál de las siguientes afirmaciones es FALSA respecto a la prueba de esfuerzo:

a. Su sensibilidad en pacientes con lesión de un vaso es 40-84% b. Su sensibilidad en pacientes con enfermedad de dos vasos es 63-90%. c. Su sensibilidad en pacientes con enfermedad de tres vasos es 79-100>'%.

Page 178: Cuestionario de Urología

d. Su especificidad es del 30-40% en pacientes neuróticos, sin coronariopatía.

45. ¿En cuál de las siguientes situaciones NO se encontrará nunca una onda "a" gigante en el pulso yugular?:

a. Estenosis pulmonar. b. Bloqueo auriculoventricular completo. c. Fibrilación auricular. d. Taquicardia ventricular.

46. De los siguientes enunciados sobre el dolor precordial, uno es FALSO. Señálelo:

a. En casos dudosos, la prueba de esfuerzo puede ser de ayuda diagnóstica. b. La angina nocturna que ocurre durante las primeras horas del sueño, parece obedecer a insuficiencia cardíaca izquierda. c. El dolor del infarto es semejante al de la angina, pero más intenso y no guarda relación con el esfuerzo. d. El dolor de la pericarditis aguda se origina en el pericardio visceral.

47. Un dolor torácico anterior, opresivo, que afecta al borde superior del trapecio, que varia con la respiración en un sujeto fumador joven, es sugerente de:

a. Disección aórtica. b. Infarto de miocardio. c. Embolismo pulmonar. d. Pericarditis aguda.

48. La hibernación miocárdica es un concepto nuevo de la cardiopatía isquémica que significa:

a. La respuesta del miocardio isquémico al descenso de la temperatura. b. La disfunción ventricular resultante de la isquemia aguda. c. La pobre contractilidad del miocardio resultante de isquemia crónica. d. La multiplicidad de infartos que dan lugar a insuficiencia cardíaca.

49. Señale cuál de las siguientes cardiopatías presenta un cuadro clínico tan semejante al de una miocardiopatía restrictiva, que el diagnóstico diferencial puede requerir una biopsia endomiocárdica:

a. Estenosis aórtica. b. Miocardiopatía dilatada. c. Pericarditis constrictiva. d. Mixoma auricular izquierdo.

50. Señale la afirmación correcta en relación con el ritmo idioventricular acelerado en el infarto agudo de miocardio:

a. En general, es una arritmia benigna que no precisa de tratamiento específico. b. Con frecuencia, desemboca en fibrilación ventricular. c. Indica insuficiencia ventricular izquierda y mejora con la administración de diuréticos. d. Requiere la colocación de un marcapaso temporal.

51. Señale de los propuestos, el procedimiento terapéutico de elección para un paciente con síndrome de preexcitación (Wolff-Parkinson-White) y taquicardias recurrentes no controladas con fánnacos antiarrítmicos es:

a. Implantación de un marcapasos endocárdico definitivo en modo VVI. b. Ablación de la vía accesoria mediante catéter. c. Cirugía con circulación extracorpórea para excisión de la vía anómala. d. Ablación por catéter del nodo auóculo-ventricular.

52. ¿Cuál de las siguientes técnicas es la más útil para el diagnóstico de derrame pericárdico?:

a. Radiografía simple de tórax. b. Cateterismo cardíaco. c. Ecocardiografía. d. Electrocardiograma.

53. ¿Cuál de los siguientes signos clínicos es el que indica peor pronóstico (en términos de supervivencia) en la estenosis valvular aórtica grave?:

a. Aparición de disnea de esfuerzo como síntoma aislado. b. Calcificación de la válvula aórtica visible en fluoroscopia. c. Angina de pecho en presencia de arterias coronarias normales. d. Insuficiencia cardíaca derecha por hipertensión pulmonar.

54. ¿Cuál de las siguientes cardiopatías congénitas se acompaña de cianosis central y amnento del flujo arterial pulmonar?:

a. Comunicación interauricular. b. Drenaje venoso anómalo total. c. Estenosis pulmonar. d. Tetralogía de Fallot.

55. Un paciente de 50 años, fumador e hipertenso, acude a un Servicio de Urgencia por haberle aparecido, dos horas antes, estando en reposo, un dolor retroesternal intenso, irradiado al cuello. A la exploración, el paciente está sudoroso, mal perfundido, con una T A de 120/80 y una frecuencia cardíaca de 120 lpm. El resto de la exploración no ofrece hallazgos relevantes. El ECG muestra un ritmo sinusal, sin alteraciones en la repolarización. La determinación de CPK es de 400 U/mL (normal hasta 160), con una fracción MB de 3% (normal: inferior a 3.7%). En la primera hora de evolución, el paciente desarrolló un cuadro de afasia y hemiparesia derecha. ¿Cuál seria su planteamiento inicial?:

a. Intentaría el traslado inmediato a una Unidad Coronaria, con vistas a tratamiento fibronolítico o revascularización precoz. b. Solicitaría una T AC craneal para valorar la indicación de anticoagulación. c. Solicitaría una TAC craneal urgente para un eventual drenaje de hematoma. d. Solicitaría una TAC torácica o un ecocardiograma transesofágico urgente para descartar patología aórtica.

Page 179: Cuestionario de Urología

56. Un paciente de 54 años presenta disnea de medianos esfuerzos y dolor retroesternal con el ejercicio. El ecocardiograma detecta un orificio aórtico con área valvular de 0,70 cm2 y un gradiente transaórtico de 90 mmHg. ¿Cuál de las que a continuación se relacionan, sería la conducta a seguir?:

a. Realizar prueba de esfuerzo. b. Reemplazamiento valvular inmediato. c. Estudio hemodinárnico y coronariografía previos al reemplazamiento valvular. d. Tratamiento médico y evaluación ecocardiográfica dentro de 6 meses.

57. Un paciente de 45 años, con angina de esfuerzo que no se controla con propranolol y nitratos y enfermedad severa de un vaso, es un candidato a:

a. Angioplastia con balón. b. Puenteo coronario con vena safena. c. Puenteo coronario con arteria mamaria. d. Vigilacia periódica.

58. Indicar la pauta a seguir con un paciente de 65 años con estenosis mitral asintomática y una válvula mitral de 1,2 cm2:

a. Comisurotomía con balón. b. Reemplazamiento valvular mitra!. c. Comisurotomía quirúrgica. d. Vigilancia periódica.

59. Señale cuál de las siguientes afirmaciones en relación con la cirugía de revascularización coronaria es la correcta:

a. Algunos injertos de vena safena se oc1uyen precozmente, pero ya no lo hacen después del primer año. b. Los implantes de arteria mamaría interna se oc1uyen con más frecuencia que los injertos de vena safena. c. En pacientes con obstrucción de la descendente anterior, la supervivencia es mayor con implante de arteria mamaria interna que con injerto de vena safena. d. La revascularización coronaría con cualquier tipo de injerto, previene por completo la aparición de infarto de miocardio.

60. El lugar más frecuente donde se produce rotura de la aorta en un paciente que ha sufrido un traumatismo por deceleración es:

a. La aorta abdominal dista!. b. El itsmo aórtico. c. La aorta ascendente. d. El cayado aórtico.

61. ¿Cuál de las siguientes es la causa más frecuente de isquemia arterial aguda?:

a. Trombosis de un aneurisma. b. Traumatismo arteria!. c. Compresión extrínseca. d. Embolia arterial.

62. Paciente de 80 años con lesiones necróticas recuperables en pie derecho, dolor de reposo que le impide el sueño, hipertenso, cardiópata y con enfermedad pulmonar obstructiva crónica. Presenta obstrucción completa de arterias ilíaca primitiva y externa derecha, con revascularización en arteria femoral común derecha. ¿Qué tratamiento sería el de elección?:

a. Puenteo aorto-femoral derecho. b. Puenteo femoro-femoral cruzado. c. Endarterectomía iliofemoral derecha. d. Puenteo axilo-bifemoral.

63. En un paciente en quien se sospecha siringomelia, ¿cuál de las siguientes pruebas debe realizarse en primer lugar?:

a. Mielografía. b. Mielo-TAc. c. PET. (Tomografía por emisión de positrones). d. RMN medular.

64. ¿Cuál es la causa de crisis epilépticas más frecuente entre los 30 y 50 años de edad?:

a. Traumatismos. b. Malformaciones arteriovenosas. c. Trastornos metabólicos. d. Tumores cerebrales.

65. Un paciente de 45 años, diabético no insulín dependiente, en tratamiento con una sulfonilurea y sospecha de hábito etílico, acude a Urgencias por clara disminución del nivel de conciencia. En la exploración, está taquicárdico y sudoroso, sin focalidad neurológica. La glucemia es de 45 mg/dl. Con el diagnóstico de hipoglucemia inducida por antidiabéticos orales, se inicia tratamiento con glucosa hiperosmolar i.v., con lo que su situación mejora y vuelve a un nivel de conciencia normal. Sin embargo, a las pocas horas, el paciente refiere diplopia, el nivel de conciencia vuelve a deteriorarse y aparece un cuadro confusional. La glucemia en ese momento es de 220 mg/dL y los electrolitos son normales. ¿Qué actitud tomaría usted?:

a. Iniciaría tratamiento insulínico para corregir un coma hiperglucémico. b. Sospecharía una acidosis láctica e iniciaría tratamiento con bicarbonato 1M. c. Iniciaría inmediatamente una infusión de tiamina i.v. d. Solicitaría una TAC craneal por sospecha de hematoma subdural.

66. Un paciente de 40 años sufre, desde hace dos, episodios ocasionales de desconexión del medio de minutos de duración, durante los cuales parece hacer movimientos de masticación. Tras ellos, está confuso durante varios minutos. ¿Cuál, de los propuestos, es el diagnóstico más probable?:

a. Crisis tónica.

Page 180: Cuestionario de Urología

b. Crisis atónica. c. Crisis mioc1ónica. d. Crisis parcial compleja.

67. Un paciente de 65 años, con antecedentes de HT A e hipercolesterolemia, sufre un accidente isquémico transitorio en territorio carotídeo derecho. La valoración clínica y el ECG no muestran evidencia de cardiopatía. Se realiza arteriografía cerebral que muestra estenosis de la arteria carótida interna derecha del 30%. ¿Qué medida terapéutica estaría indicada en este paciente?:

a. Anticoagulación. b. Cirugía carotídea. c. Angioplastia carotídea. d. Antiagregantes plaquetarios.

68. Un paciente de 60 años con antecedentes de cáncer de pulmón, presenta una crisis epiléptica. Se realiza RMN cerebral, que muestra una lesión única sugerente de metástasis. No hay evidencia de metástasis extracerebrales. ¿Cuál de las siguientes afirmaciones es FALSA respecto al tratamiento del paciente?:

a. Debe administrarse medicación antiepiléptica. b. Los corticoides son útiles para disminuir el edema vasogénico. c. Si la lesión es accesible puede estar indicada la cirugía. d. La radioterapia craneal no está indicada.

69. La corea de Huntington (corea crónica progresiva hereditaria) puede diagnosticarse en muchos casos:

a. Con examen oftalmalógico de lámpara de hendidura. b. Mediante análisis de bandas oligoclonales en LCR. c. Mediante estudio de RMN craneal. d. Con la historia clínica, antecedentes familiares y signos clínicos.

70. Un paciente de 72 años, desarrolla un cuadro progresivo de parestesias en MMII, con dificultad para la marcha, de dos semanas de evolución. La exploración muestra marcha atáxica, signo de Babinsky bilateral y ausencia de reflejo aquíleo bilateral. La RMN medular ha sido normal y el hemograma muestra aumento de VCM de los hematíes. ¿Qué prueba diagnóstica indicaría?:

a. Mielografía. b. Biopsia muscular. c. Biopsia de nervio periférico. d. Niveles de BI2 y test de Schilling.

71. Un paciente de 63 años desarrolla un cuadro de síndrome de "cola de caballo" de aparición progresiva. En la Rx de tórax presenta una masa pulmonar parahiliar. Las RMN medular y craneal no muestran alteraciones. ¿Qué procedimiento sería el más indicado a continuación?:

a. Biopsia de nervio periférico. b. Mielografía. c. RMN médula un mes después. d. Punción lumbar.

72. Una paciente epiléptica de 35 años queda embarazada. Ha presentado la última crisis convulsiva generalizada hace 8 meses; sigue tratamiento con carbamazepina desde entonces. ¿Cuál sería la actitud más correcta entre las siguientes?:

a. Retirar el tratamiento por riesgo de malformaciones fetales graves. b. Ajustar la dosis a la mínima eficaz y mantener monoterapia. c. Añadir otro fármaco por el riesgo elevado de presentar crisis durante el embarazo. d. Aconsejar la interrupción voluntaria del embaraw.

73. Señale en cuál de los procesos siguientes, NO existe afectación de neurona motora:

a. Esclerosis lateral amiotrófica. b. Distrofia muscular de Duchenne c. Atrofia muscular espinal. d. Paraplejia espástica familiar.

74. Un niño de 7 años presente un turno! quístico, con calcificaciones, en la región selar. ¿Cuál de los siguientes diagnósticos es más probable?:

a. Craneofaringioma. b. Epindirnoma. c. Meningioma. d. Astrocitoma.

75. Una mujer de 60 años con antecedentes de jaqueca e HTA en tratamiento, acude al Servicio de Urgencias por haber sufrido una cefalea intensa instaurada de manera brusca tras haber realizado un esfuerzo físico. En la exploración se aprecia fotofobia, rigidez de nuca y una tensión arterial elevada, siendo normal el resto de la exploración neurológica. El diagnóstico más probables es:

a. Hematoma cerebral intraparenquimatoso hipertensivo. b. Tumor cerebral. c. Crisis de jaqueca. d. Rotura de aneurisma sacular intracraneal.

76. En relación con la hemorragia subaracnoidea aneurismática, señale cuál de las afirmaciones siguientes es INCORRECTA:

a. Conlleva una mortalidad global que alcanza el 45% de los casos. b. Los aneurismas saculares se localizan más frecuentemente en las arterias cerebral media y basilar. c. La complicación más temible durante el período agudo posthemorrágico es el resangrado del aneurisma. d. La administración de bloqueantes de la entrada de calcio tipo nirnodipino en la fase aguda, disminuye la incidencia de

Page 181: Cuestionario de Urología

isquemia cerebral secundaria.

77. Ante un paciente con debilidad muscular, hiperreninemia, hiperaldosteronismo, hipokalemia y alcalosis, sin hipertensión ni edemas, y con secreción urinaria de prostaglandina elevada, el diagnóstico más probable es:

a. Déficit hereditaria de 11-beta-hidroxi-esteroide-deshidrogenasa. b. Déficit de ll-beta-hidroxilasa. c. Síndrome de Barttet. d. Déficit de 21-hidroxilasa.

78. Paciente de 30 años que acude a consulta por presentar desde hace varios meses un cuadro inespecífico de astenia y malestar general. El paciente carece de antecedentes familiares o personales de interés, si bien refiere que se encuentra cada día más bronceado, a pesar de no tomar el sol. La T A es 70/40 mmHg. ¿ Cuál de las siguientes pruebas le permitiría descartar o confirmar su diagnóstico de sospecha?:

a. Detenninaciones de iones (Na y K) en suero. b. Detenninación de los niveles de cinc. c. Prueba de estimulación con ACTH ovino o humano. d. Determinación de los niveles de cortisol y su proteína transportadora a lo largo del día.

79. Mujer de 45 años, diabética conocida, que consulta por haber notado "un bulto en el cuello" hallado de forma casual. La paciente no presenta sintomatología clínica relevante, excepto la palpación de un nódulo de aproximadamente 3 cm de diámetro en el lóbulo tiroideo izquierdo. No hay historia personal o familiar de patología tiroidea. El estudio bioquímico y hematológico es normal, con buen control glucémico. Los niveles de hormonas tiroideas son normales. Se le realiza una gammagrafia tiroidea con 1131, observándose que el nódulo no capta (nódulo frío). ¿Qué debe hacer a continuación?:

a. Realizar una ecografía tiroidea. b. Repetir la garnmagrafía con TC99. c. Comenzar tratamiento con tiroxina. d. Biopsiar el nódulo mediante punción-aspiración con aguja fina.

80. En el tratamiento del hipotiroidismo primario, el objetivo es:

a. Ajustar la dosis de tiroxina al bienestar subjetivo del paciente. b. Normalizar los niveles de tiroxina y triyodotironina. c. Normalizar los niveles de TSH. d. Mantener una TSH elevada para mantener estimulado el tiroides.

81. ¿Cuál de los siguientes cuadros NO cursa con hiponatremia?:

a. Síndrome de secreción inadecuada de ADH. b. Hipotiroidismo. c. Síndrome de Cushirig. d. Utilización de diuréticos.

82. ¿Cuál es la forma clínica más prevalente de la neuropatía diabética?:

a. Polineuropatía distal simétrica sensitivomotora. b. Neuropatía autonómica. c. Neuropatía motora proximal. d. Radiculopatía.

83. Un paciente de 32 años, sin historia familiar de hipertensión, consulta por episodios de sofoco y cefaleas. La TA es de 180/114 mmHg. La glucemia basal es de 140 mg/dl. El resto de los análisis rutinarios, incluido ionograma, son normales. Los niveles de catecolaminas en orina de 24 horas están tres veces por encima del límite alto de lo normal. Ante este cuadro clínico, señale qué prueba sería más útil hacer a continuación para completar el estudio:

a. TAC abdominal para localizar la lesión responsable. b. Una gammagrafía con 1131-iodometil-19 norcolesterol, para ver el tamaño de la lesión. c. Tratamiento de prueba con espironolactona y suplementos de potasio. d. No hacer ninguna, pues con los datos disponibles ya podemos iniciar el tratamiento médico.

84. En un paciente se detecta de forma casual, al realizar una ecografía abdominal por presentar litiasis ureteral sintomática, una tumoración sólida en glándula suprarrenal izquierda de 9 cm de diámetro. ¿Cuál es la actitud a adoptar más correcta?:

a. Suprarenolectomía izquierda. b. Observación, con estudios de imagen periódicos. c. Estudio funcional de la masa para detectar hiperfunción. d. Estudio cito lógico de la masa por punción percutánea.

85. Mujer de 53 años, obesa, con antecedentes psiquiátricos y sin datos de cirugía previa. Consulta por episodios de disminución del nivel de conciencia con sudoración, que mejoran con la ingesta. Una prueba de hipoglucemia de ayuno resulta positiva. ¿Cuál de las siguientes afirmaciones es correcta?:

a. Debe someterse a dieta hipocalórica, sin reducir el aporte proteico y con suplementos vitamínicos. b. La TAC, la angiografía selectiva y el muestreo venoso portal transhepático, ayudan al diagnóstico. c. Debe realizarse, en primer lugar, un estudio de vaciamiento gástrico. d. No está indicado un diagnóstico de localización, pues generalmente tiene una lesión maligna avanzada.

86. Una mujer de 35 años consulta por presentar un nódulo de 2 cm de diámetro en región cervical anterior, que se moviliza con la deglución. No se palpan adenopatías laterocervicales. Ecográficamente es sólido y la punción-aspiración con aguja fina es informada como "proliferación folicular". ¿ Cuál es el tratamiento a aplicar?:

a. Supresión con hormona tiroidea. b. Tiroidectomía total con linfadenectomía funcional profiláctica. c. Tiroidectornía subtotal bilateral.

Page 182: Cuestionario de Urología

d. Hemitiroidectornía del lado en que se palpe el nódulo.

87. Un paciente presenta un nódulo en región cervical anterior dependiente de la glándula tiroides. ¿Cuál de las siguientes características de la exploración física es el indicador más fiable de malignidad?:

a. Presencia de adenopatías cervicales homolaterales. b. Irregularidad. c. Fijación a estructuras adyacentes. d. Gran tamaño.

88. Mujer de 30 años, con historia de 15 años de rinitis alérgica y de 10 años de asma extrínseco. Un estudio médico de 5 años atrás, demostró eosinofilia periférica notable, mantenida desde entonces. En el momento actual, refiere dolor abdominal, presenta un rash cutáneo y signos clinicos de mononeuritis múltiples, así como infiltrados pulmonares en la radiología simple de tórax. Con más probabilidad esta paciente tendrá:

a. Granulomatosis de Wegener. b. Lupus eritematoso sistémico. c. Síndrome de Churg-Strauss. d. Esclerodermia.

89. Enfermo de 27 años, con antecedentes de episodios recurrentes de dolor ocular, fotofobia y lagrimeo, que desarrolla dolor insidioso y progresivo en regi6n lumbar con exacerbación nocturna en cama. Datos relevantes de la exploración física: test de Schober positivo y soplo de regurgitación aórtica grado Il-IlINI. Rx de tórax: Sugerente de retracción fibrosa apical derecha. Con más probabilidad el paciente tendrá:

a. Enfermedad de Whipple. b. Síndrome de Reiter. c. Reumatismo poli articular agudo (fiebre reumática). d. Espondilitis anquilosante.

90. Una mujer de 32 años, consulta por un episodio de hemiparesia izquierda sugerente de ictus. Entre sus antecedentes refiere un hábito tabáquico, no se ha documentado hipertensión ni hiperglucemia, ha tenido un episodio previo de amaurosis fugax y dos episodios de tromboflebitis en extremidades inferiores. No refiere antecedentes quirúrgicos ni ingesta de medicación. Ha tenido tres abortos espontáneos. El hemograma y el estudio de coagulación son normales. La función hepática y renal y los electrolitos son normales. El colesterol total es de 260 mgldL (normal <240) y los triglicéridos de 160 mgldL (normal <150). La TAC en la fase aguda no aporta datos significativos. ¿Cuál sería su planteamiento?:

a. Se trata de una endocarditis infecciosa a partir de una tromboflebitis séptica. Iniciaría tratamiento antibiótico empírico en espera de los cultivos. b. El cuadro corresponde a un accidente aterotrombótico en una paciente con una hiperlipemia familiar. El origen más probable es la carótida. Solicitaría un estudio de troncos supra-aórticos. Iniciaría tratamiento hipolipemiante. c. Se trata de un embolismo de origen cardíaco en una paciente con una valvulopatía silente. Solicitaría un ecocardiograma urgente. d. El primer diagnóstico sería el de síndrome antifosfolípido. Solicitaría una determinación de anticuerpos anticardiolipina.

91. ¿En cuál de los siguientes cuadros intestinales NO aparece la artritis como una manifestación extraintestinal?:

a. Colitis ulcerativa. b. Enfermedad de Crohn. c. Enfermedad celíaca. d. Puenteo intestinal por obesidad.

92. ¿Cuál de las siguientes articulaciones se afecta con más frecuencia en la enfermedad por dep6s1to de cristales de pirofosfato cálcico?:

a. El tobi!lo. b. El codo. c. La muñeca. d. La rodilla.

93. ¿Cuál es la forma clínica más frecuente de artritis psoriásica?:

a. La forma asimétrica oligoarticular. b. La forma simétrica similar a la artritis reumatoide. c. La forma espondilítica. d. La forma con predominio de afectación de las falanges distales.

94. ¿Cuál de los siguientes fármacos se ha demostrado que es efectivo porque disminuye el número de brotes en la fiebre mediterránea familiar?:

a. Los estrógenos. b. Los corticosteroides. c. La fenilbutazona. d. La co1chicina.

95. ¿Cuál de las siguientes manifestaciones clínicas se encuentra, con MENOS frecuencia, en los pacientes con lupus eritematoso sistémico?:

a. Artralgias. b. Fotosensibilidad. c. Leucopenia (<4000/mm3) d. Hipertensión pulmonar.

96. Mujer de 25 años, que presenta en el estudio radiológico una tumoración multiloculada en la zona epifisaria del extremo distal de su fémur izquierdo. ¿Cuál de los siguientes diagnósticos es el más probable?:

Page 183: Cuestionario de Urología

a. Quiste óseo esencial. b. Sarcoma osteogénico. c. Mieloma múltiple. d. Tumor de células gigantes.

97. Ante un paciente de 23 años que presenta, tras un traumatismo banal, una deformidad del hombro derecho "en charretera", ¿cuál de los siguientes diagnósticos es el más probable?:

a. Luxación acromioc1avicular. b. Fractura proximal de húmero. c. Luxación anterior recidivante de hombro. d. Lesión paratumoral de húmero (quiste óseo aneurismático).

98. Mujer de 60 años que, sin patología previa, sufre caída casual, presentando fractura subcapital de cuello femoral izquierdo tipo 1 de Garden. ¿Cuál de los siguientes tratamientos es el más adecuado?:

a. Prótesis total de cadera. b. Yeso pelvipédico. c. Osteosíntesis con tornillos. d. Carga inmediata.

99. Ante una mujer de 72 años con grave ,afectación artrósica de la rodilla derecha, una de las siguientes etiologías contraindica la colocación de una prótesis de rodilla. Señálela:

a. Artrosis primaria. b. Genu valgo. c. Artrosis postraumática. d. Artropatía neuropática de Charcot.

100. Un paciente de 55 años, sin otros antecedentes de enfermedad grave, padece dolores en el tercio proximal de su pierna derecha desde hace un año, que ceden mal a los analgésicos comunes. A la inspección, se observa una discreta tumoración a dicho nivea. Realizada una exploración radiológica convencional, se aprecia una imagen en el tercio proximal de la diáfisis tibial, con patrón permeativo, junto con lisis de una cortical y focos de calcificación centraa. No existen alteraciones de labora-toriosignificativas. ¿Cuál sería su diagnóstico de sospecha?:

a. Osteoma osteoide. b. Osteosarcoma. c. Defecto fibroso corticaa. d. Condrosarcoma.

101. Un paciente de 20 años, jugando al fútbol, sufre una torsión en su rodilla al clavar los tacos de la bota en el césped y girar bruscamente el tronco. La rodilla es dolorosa en el tercio posterior de la interlínea articular medial, duele al forzar la flexión, no presenta inestabilidades ligamentosas y se aprecia un derrame articular escaso y seroso. El diagnóstico más probable es lesión:

a. Meniscal del menisco mediaa. b. Meniscal del menisco lateraa. c. Del ligamento cruzado anterior. d. Del ligamento cruzado posterior.

102. Un paciente de 24 años, trasladado desde 400 Km, hace 48 horas, a nuestro hospital, diagnosticado de fractura bilateral de fémures y tibias, ingresado con inmovilizaciones provisionales hasta ser intervenido, inicia bruscamente un cuadro de disnea intensa, petequias generalizadas, estupor y obnubilación. Debemos sospechar:

a. Neumonía nosocomial con sepsis generalizada secundaria. b. Coagulacion intravascular diseminada por coagulopatía de consumo c. Tromboembolismo pulmonar masivo. d. Síndrome de embolia grasa.

103. Un paciente de 51 años acude a consulta por dolor lumbar intenso irradiado por toda la cara posterior del miembro inferior derecho hasta el dedo gordo, de 48 horas de evolución, con signo de Lassegue positivo, sin déficit motor, sin alteración de reflejos ni de esfinteres, con parestesias ocasionales localizadas en la cara anterior del muslo. Nuestra actitud debe ser:

a. Instaurar tratamiento con reposo, analgésico s y/o antiinflamatorios y vigilar evolución. b. Solicitar resonancia magnética para confirmar hernia discal y valoración quirúrgica. c. Enviarlo a un Servicio de Urgencias para valoración por traumatología y/o neurocirugía. d. Solicitar electromiograma para valoración de alteraciones nerviosas periféricas.

104. Un varón de 38 años, natural de Barcelona, consultó por fiehre de 45 días de evolución y pérdida de peso. Había recibido distintos antibióticos sin éxito. Dos meses antes había estado en el norte de Marruecos. Tenía mal estado general, un soplo sistólico UIVI en punta, hepatomegalia de 7 cm y esplenomegalia de 12 cm. leucocitos 2100/mm3, hemoglobina 9 g/dL Y plaquetas 34000/mmc. Fosfatasa alcalina 340 UIL (normal 40-117), TGP 345 UIL (normal 5-43). ¿Cuál es el diagnóstico más probahle?:

a. Endocarditis infecciosa. b. Leucemia de "células peludas". c. Ehrlichiosis. d. Leishmaniasis visceral.

105. Un homhre de 35 años, heroinómano activo, consultó por fiebre de 5 días de evolución y dolor lumbar. La exploración mostraba un paciente caquéctico, con múltiples lesiones de venopunción, abscesos cutáneos y dolor al levantar el muslo izquierdo. El Gram de un absceso, mostraba cocos Gram positivos en "racimos". Una TAC reveló un absceso del psoas. ¿Cuál es la conducta más adecuada?:

a. Tratamiento con cloxacilina y drenaje mediante aspiración. b. Tratamiento con vancomicina y gentamicina y aspiración.

Page 184: Cuestionario de Urología

c. Tratamiento con cloxacilina y rifampicina que hará innecesario el drenaje. d. Tratamiento oral con ciprofloxacina y rifampicina.

106. Un paciente de 32 años, con relaciones heterosexuales no selectivas, desarrolló disuria y exudación uretraa. Dos días después notó dolor e inflamación en el testículo derecho. Tenía una secreción mucoide en cantidad moderada y tumefacción testicular muy dolorosa. El examen con Gram del exudado mostraba polimorfonucleares pero no se vieron bacterias. ¿Cuál es la actitud más correcta?:

a. Tratamiento con ceftriaxona 2 gr por vía intramuscular. b. Tratamiento con ceftriaxona y azitromicina. c. Investigar la posibilidad de brucelosis y esperar. d. Investigar Chlamydia trachomatis y tratar con doxicilina.

107. ¿Cuál de los siguientes hallazgos es MENOS frecuente en la endocarditis infecciosa asociado a la drogadicción intravenosa?:

a. Afectación de la válvula tricúspide. b. Afectación de la válvula mitral. c. Afectación de la válvula aórtica. d. Afectación de la válvula pulmonar.

108. Uno de los siguientes fármacos NO es útil en el tratamiento de la infección por el virus de la inmunodeficiencia humana:

a. Lamivudina. b. Zidovudina. c. Didanosina. d. Citarabina.

109. ¿En cuál de las siguientes situaciones NO está indicado tratar una infección urinaria asintomática?:

a. En mujeres embarazadas. b. En pacientes neutropénicos. c. En mujeres de edad avanzada. d. En lactantes.

110. La articulación que se afecta con mayor frecuencia en la artritis infecciosa es:

a. Caderas. b. Articulaciones sacroilíacas. c. Tobillos. d. Rodillas.

111. Una mujer de 34 años presenta fiebre (38.5°C), acompañada de escalofríos, odinofagia y tos persistente con escasa expectoración de 4 días de evolución. En la radiografía de tórax se evidencia infiltrados segmentarios en lóbulo inferior derecho. Como único antecedente epidemiológico de interés refiere que su hijo de 6 años presenta tos sin otros síntomas desde 10 días antes. El agente responsable más probablemente implicado es:

a. Mycoplasma pneumoniae. b. Streptococcus pneumoniae. c. Virus parainfluenzae 3 d. Virus sincitial respiratorio.

112. ¿Cuál de los siguientes antibióticos NO es útil en el tratamiento de infecciones por anaerobios?:

a. C10ranfenicol b. Metronidazoa. c. Clindamicina. d. Ofloxacino.

113. Un paciente de 27 años consulta por fiebre, astenia, mialgias y malestar generaa. En la explora-ción destaca leve enrojecimiento faríngeo, microadenopatías cervicales, esplenomegalla de 2 cm y hepatomegalia de 4 cm levemente dolorosa. ¿Qué microorganismo es el más probable?:

a. Chlamydia psittaci. b. Coxiella burnetii. c. Citomegalovirus. d. Escherichia coli.

114. Señale cuál de las siguientes asociaciones, infección-oportunista, en pacientes VIH-profilaxis es FALSA:

a. Pneumocystis carinii - Cotrimoxazol. b. Citomegalovirus - Acic1ovir. c. Mycobacterium tuberculosis - Isoniacida. d. Toxoplasma gondii - Pirimetamina + Sulfadiacina.

115. Un adulto sin antecedentes de interés, .acude a su médico de cabecera con un episodio compatible con bronquitis aguda. ¿Cuál es la etiología microbiológica más probable?:

a. Pseudomonas .aeruginosa. b. Mycoplasma pneumoniae. c. Viral. d. Branhamella catarrhalis.

116. ¿Cuál de los siguientes datos NO es característico de la leucemia linfática crónica de células B?:

a. 60% de linfocitos en médula ósea. b. Hipogarnmaglobulinemia. c. Más de un 60% de las células de la médula ósea y sangre periférica son CD20 y CD5 positivas.

Page 185: Cuestionario de Urología

d. Expresión de cadenas ligeras kappa y lambda en los linfocitos.

117. Mujer de 64 años, diagnosticada de anemia megaloblástica hace 5 y tratada correctamente hasta hace 3 años, fecha en que se le practica gastrectomía total por tumor carcinoide de estómago. Desde entonces ha estado sin tratamiento. En la actualidad consulta por astenia y se objetiva palidez. Los parámetros hemicitométricos son: hematíes 3.130.000/µL/ Hb 9.6 g/dL, Hto. 28.4%, VCM 90.7 µ3 , plaquetas 130.000/µL, leucocitos 4.200/µL con fórmula normal y ferritina por debajo de lo normal. Se le realiza una endoscopia digestiva alta, que es normal. El diagnóstico más probable es:

a. Anemia megaloblástica por déficit de vitamina BI2 b. Anemia megaloblástica por déficit de ácido fólico. c. Déficit combinado de hierro y vitamina BI2 d. Déficit de hierro.

118. ¿Cuál de los siguientes datos NO es de utilidad diagnóstica en la leucemia mieloide crónica?:

a. Basofilia. b. Leucocitosis c. Esplenomegalia. d. Cromosoma Ph en los fibroblastos.

119. Los fenómenos tromboembólicos en un estado de hipercoagulabilidad primario, suelen acompañarse de los siguientes hallazgos, EXCEPTO:

a. Trombosis recurrente. b. Localización inusual. c. Historia familiar positiva. d. Neoplasia oculta.

120. Un paciente presenta trombocitopenia y el estudio de médula ósea revela una cifra de megacariocitos normal o elevada. ¿Qué diagnóstico considera MENOS probable?:

a. Púrpura trombocitopénica idiopática. b. Hiperesplenismo. c. Coagulación intravascular diseminada. d. Síndrome mielodisplásico.

121. Un paciente de 65 años, presenta palidez y disnea de esfuerzos. La analítica hematológica muestra: VCM 103 µ3, Hb 10 g/dL, leucocitos 2.100/mmc. En el aspirado de médula ósea se observa hipercelularidad, algunos megacariocitos pequeños y 50% de blastos. ¿Qué diagnóstico sospecharía en primer lugar?:

a. Leucemia mieloblástica aguda. b. Anemia megaloblástica. c. Leucemia mieloide crónica. d. Síndrome mie1odisplásico.

122. Mujer de 15 años que consulta por astenia y fiebre. En la exploración presenta palidez de piel y mucosas, hematomas generalizados y hepatoesplenomegalia moderada. En sangre periférica se observa leucocitosis con 10% de blastos y en médula ósea 40% de blastos, 7 % de ellos mieloperoxidasas positivos. La identificación de los antígenos proteicos con anticuerpos monoclonales, demuestra que los blastos son CD34, CD33 y CD13 positivos, CD15 negativos. El diagnóstico será leucemia:

a. Aguda mieloblástica Mo. b. Aguda mieloblástica M l c. Aguda linfoblástica d. Mieloide crónica en transformación

123. En un paciente con los siguientes hallazgos en la analítica sérica. pH 7.09, HCO3 8 rnmol/L, Na+ 143 rnmol/L, K+ 3.7 rnmol/L y Cl112 rnmol/L, ¿cuál de las siguientes causas de acidosis metabólica es más probable?:

a. Cetoacidosis alcohólica. b. Exceso de aminoácidos catiónicos en nutrición parenteral. c. Acidosis tubular renal proximal. d. Acidosis tubular renal distal.

124. En un paciente con nefropatía secundaria a reflujo vesicoureteral, que presenta proteinuria en rango nefrótico con niveles normales de albúmina sérica, ¿cuál es el patrón histológico glomerular habitual?:

a. Glomerulonefritis membranosa. b. Glomerulonefritis membrana proliferativa. c. Glomeruloesclerosis focal y segmentaria. d. Glomerulonefritis proliferativa mesangiaa.

125. Un paciente de 65 años, con claudicación intermitente en extremidades inferiores, presenta insuficiencia renal progresiva, hipertensión, sedimiento urinario poco alterado y proteinuria escasa. Se debe considerar el diagnóstico de:

a. Glomeru1onefritis rápidamente progresiva. b. Nefropatía intersticial por analgésicos. c. Nefropatía isquémica ateroesclerótica. d. Vasculitis de arterias medias.

126. La uropatía obstructiva en el adulto se puede presentar con cualquiera de los síntomas siguientes EXCEPTO uno. Señálelo:

a. Dolor lumbar intermitente. b. Piuria mantenida. c. Proteinuria superior a los 3 g día. d. Masa palpable en hipocondrio.

Page 186: Cuestionario de Urología

127. En una enferma con historia de toma crónica de diuréticos, con el propósito de disminuir su peso; ¿qué dato de la exploración funcional nos hará pensar en una depleción de potasio?:

a. Reducción brusca, superior al 50%, de la filtración glomerular. b. Orina persistentemente alcalina. c. Pérdida de arninoácidos del grupo lisina por la orina. d. HipercaIcemia intermitente.

128. Enfermo de 73 años con cuadro de insuficiencia cardíaca congestiva secundaria a miocardiopatia dilatada. Se plantea el uso de diuréticos ahorradores de potasio. ¿Cuál de los siguientes supuesto NO necesita vigilancia estrecha por ser infrecuente o de poca trascendencia?:

a. Hiperpotasemia severa. b. Insuficiencia renal. c. Descompensación de diabetes Mellitus. d. Acidosis metabólica.

129. La hipematremia suele presentarse en casos de:

a. Uso de diuréticos. b. Fallo cardíaco derecho. c. Secreción inadecuada de ADH. d. Enfermos con bajo nivel de conciencia.

130. Un hiato aniónico ("anion-gap") elevado es congruente con:

a. AIcalosis metabólica. b. Edema pulmonar. c. Diarrea aguda. d. Cetoacidosis diabética.

131. Un paciente de 65 años, con antecedentes personales de anulación funcional del riñón derecho por litiasis coraliforme, presenta un cuadro de fracaso renal agudo oligoanúrico, que cursa con dolor lumbar izquierdo, fiebre, leucocitosis, hematuria y niveles séricos elevados de aspartato-transaminasa y láctico desbidrogenasa (AST y LDH). El diagnóstico más probable es:

a. Pielonefritis aguda izquierda. b. Uropatía obstructiva aguda. c. Nefropatía tubulointersticial aguda alérgica. d. Trombosis aguda de arteria renal izquierda.

132. Un paciente de 40 años con litiasis renal cálcica bilateral recidivante que presenta una calcemia de 11 mg%, fosfatemia de 2 mg%, calciuria de 300 mg/24h y fosfaturia de 2000 mg/24h, con una reabsorción tubular de fósforo (RTP) del 70%, debe hacemos pensar que el origen de su litiasis es debido a:

a. Acidosis tubular. b. Intoxicación vitamina D. c. Hiperparatiroidismo primario. d. Sarcoidosis.

133. Paciente de 63 años, con antecedentes de un episodio de hematuria macroscópica autolimitada y cólico renal derecho de repetición. Radiografía simple de abdomen sin alteraciones. En la urografía intravenosa se objetiva un defecto de replección lacunar a nivel de pelvis renal derecha de 1,5 cm. La ecografía objetiva la misma lesión en pelvis renal derecha, sin signos de hipercogenicidad y sin sombras anecoicas posteriores. ¿Cuál será, de los propuestos, el diagnóstico a considerar en primer lugar?:

a. Litiasis de cistina. b. Litiasis de ácido úrico. c. Carcinoma de células transicionales. d. Quiste simple.

134. Señale cuál de las siguientes enfermedades se asocia frecuentemente con el angiomiolipoma renal:

a. Lupus eritematoso. b. Amiloidosis. c. Lepra. d. Esclerosis tuberosa.

135. En un examen clínico rutinario se le descubre a un paciente, al practicarle un tacto rectal, un nódulo prostático mayor de a. 5 cm de diámetro. La ecografía demuestra su situación intraglandular. Biopsiado resulta ser un adenocarcinoma. El antígeno prostático en sangre (PSA) es de 2 ng/mL, no presentando afectación ósea ni ganglionar abdominopélvica en la gammagrafía ósea y la TAC. ¿Qué estadio tumoral corresponde a los datos descritos en este paciente?:

a. T0 N0 M0. b. T1 N0 M0. c. T2 N0 M0. d. T3 N0 M0.

136. Uno de los siguientes procesos, es debido a una alteración metabólica del zinc. Señálelo:

a. Acrodermatitis enteropática. b. Dermatitis herpetiforme. c. Amiloidosis. d. Candidiasis.

137. Enfermo de 40 años al que, en la zona de la picadura de una garrapata, le aparece una úlcera de pequeño tamaño, de centro necrótico color negro, con fiebre y cefalea. A los pocos días tiene una erupción maculosa y papulosa generalizada. ¿Cuál sería su

Page 187: Cuestionario de Urología

diagnóstico?:

a. Papulosis linfomatoide. b. Fiebre botonosa. c. Leishmaniasis. d. Impétigo vulgar.

138. ¿En cuál de los siguientes procesos se encuentran anticuerpos IgG circulantes contra la sustancia intercelular epitelial?:

a. Pénfigo benigno familiar. b. Pénfigo vulgar. c. Penfigoide ampolloso. d. Dennatitis herpetifonne.

139. Enfermo de 45 años al que, hace unos meses, le aparecen en el dorso de las manos unas lesiones ampollosas, asintomáticas, que se erosionan. Sucesivamente, le aparecen nuevas lesiones en manos y cara. Presenta hipertricosis e hiperpigmentación intensa en cara. ¿Cuál sería su diagnóstico?:

a. Penfigoide. b. Eczema de contacto. c. Porfiria hepatocutánea. d. Pelagra.

140. Enferma de 52 años a la que, desde hace años, le aparecen en piernas lesiones nodulares que evolucionan, con épocas de mejoría y empeoramiento y que a veces se ulceran. En el estudio anatomopatológico se demuestra la existencia de una paniculitis lobulillar. ¿Cuál sería su diagnóstico?:

a. Panarteritis nudosa. b. Eritema indurado de Bazin. c. Eritema nodoso. d. Poliangeitis microscópica.

141. Una paciente de 35 años, con antecedentes familiares de hipoacusia, acude por presentar hipoacusia progresiva bilateral más marcada del oído izquierdo, que se ha agravado a raíz de un embarazo. La otoscopia es normal. En la acumetría encontramos un Rinne en oído izquierdo negativo y un Weber lateralizado a la izquierda. La timpanometría mnestra disminución de la compliance y ausencia de reflejo estapediaa. Su diagnóstico más probables será:

a. Malfonnación de la cadena osicular. b. Hipoacusia neurosensorial hereditaria de expresión tardía. c. Otoesc1erosis. d. Timpanoesc1erosis cerrada.

142. Respecto a la prueba de Weber (lateralización o no del sonido de un diapasón colocado en el vértex), el sonido del diapasón, ¿de qué manera secomporta?:

a. Puede lateralizarse en sujetos totalmente sanos. b. Se lateraliza frecuentemente en la otoesc1erosis bilateraa. c. En la hipoacusia de conducción unilateral se lateraliza al lado enfermo. d. En la hipoacusia de percepción unilateral se lateraliza al lado enfermo.

143. Ante un cuadro de hipoacusia neurosensorial unilateral, que se acompaña de hipoestesia corneal y signo de Hitselberger positivo, el diagnóstico más probable será:

a. Laberintitis aguda. b. Neurinoma del VIII par craneal. c. Enfennedad de Meniere. d. Colesteatoma complicado de oído medio.

144. Paciente de 42 años, sin antecedentes de interés, que acude al Servicio de Urgencias por presentar un cuadro de disfagia y dolor faríngeo de 8 días de evolución, con fiebre elevada y afectación del estado generaa. En la exploración se objetivan unas amígdalas hipe_ rémicas, adenopatías subángulo mandibulares bilaterales e induración y dolor por debajo y a lo largo del músculo esternocleidomastoideo, palpándose un cordón fibroso. La analítica muestra una leucocitosis con marcada desviación izquierda. Indique cuál es el diagnóstico más probable:

a. Flemón periamigdalino. b. Mononuc1eosis infecciosa. c. Angina de Vicent. d. Sepsis postanginosa.

145. Al realizar un diagnóstico diferencial de la otosclerosis, ¿cuál de estas patologías NO tendría en cuenta?:

a. Otitis adhesiva crónica. b. Dislocación de la cadena osicular. c. Laberinitis. d. Osteogénesis imperfecta.

146. ¿Cuál de las siguientes entidades NO es causa habitual de parálisis facial periférica?:

a. Otitis media crónica colesteatomatosa. b. Otitis media aguda. c. Otitis media secretora. d. Otitis externa maligna.

147. Un joven acude a la consulta aquejando picor, secreción mucoide en hebras, enrojecimiento y fotofobia bilateraal. Al explorarlo, encontramos una formación papilar en empedrado en conjuntiva tarsal superior, así como nódulos gelatinosos en la conjuntiva limbar. La intervención clínica más adecuada es administrar:

Page 188: Cuestionario de Urología

a. Antibióticos tras realizar cultivo del exudado conjuntival. b. Clomoglicato y corticoides tópicos. c. Tetraciclinas sistémicas y tópicas. d. Aciclovir y corticoides.

148. En un niño de 16 meses, que presenta enrojecimiento en ojo derecho, con tensión ocular de 35 rnrnHg y medios opacos y desprendimiento de retina, ¿cuál de las siguientes actitndes es la más indicada?:

a. Medida de la lactodehidrogenasa en humor acuoso. b. Hipotensores oculares y cirugía del desprendimiento de retina. c. Fotocoagulación e inyección de gas intravítreo. d. Scanner orbitocerebral.

149. Una mujer con historia de eritema nodoso, consulta por dolor y enrojecimiento del ojo derecho. Tras la exploración oftalmológica es diagnosticada de uveitis anterior. La Radiografía de tórax muestra adenopatías hiliares bilaterales. ¿Cuál es la enfermedad más probable?:

a. Tuberculosis pulmonar. b. Infección postestreptocócica. c. Sífilis. d. Sarcoidosis.

150. Un paciente de 61 años, con una hemianopsia homónima derecha congruente, con pérdida del nistagmus optocinético hacia la izquierda y agnosia visual, tendrá una lesión en:

a. La rodilla anterior de la cápsula interna izquierda. b. La cintilla derecha. c. El lóbulo occipital derecho. d. El lóbulo parietal izquierdo.

151. Una mujer de 40 años que padece artritis reumatoide, tratada con sales de oro, presenta hiperemia e inyección ciliar en ojo derecho, con visión borrosa, miosis derecha irregular y dolor ocular, con tensión ocular normaa. ¿Cuál de los diagnósticos, que a continuación se enumeran, es el correcto?:

a. Desprendimiento de retina. b. Glaucoma crónico simple. c. Conjuntivitis aguda. d. Uveitis anterior.

152. El concepto de estereotipia como síntoma psiquiátrico corresponde a la psicopatología de:

a. La orientación. b. La memoria. c. La psicomotricidad d. El pensamiento.

153. Un paciente de 21 años presenta agitación psicomotriz, ideas delirantes de persecución, alucinaciones auditivas, inabordabilidad, conductas desorganizadas y falta de conciencia de enfermedad. No se objetivan trastornos orgánicos ni datos de intoxicación o toxicomanía. Señale el tratamiento farmacológico de inicio de entre los siguientes:

a. lmipramina. b. Clonazepam. c. Diazepam, d. Clorpromazina.

154. El concepto de alucinación como síntoma psiquiátrico, corresponde a la psicopatología de:

a. La conciencia. b. La atención. c. El pensamiento. d. La percepción

155. Un paciente de 30 años presenta desde hace 3, un cuadro clínico caracterizado por estado de ánimo deprimido habitualmente, cansancio, hiporexia, baja autoestima, dudas frecuentes y apatía. Se acompaña de falta de ilusión y disminución del rendimiento. No se objetiva patología orgánica ni toxicomanía. El diagnóstico más probable entre los siguientes es:

a. Depresión mayor. b. Reacción depresiva. c. Distimia. d. Episodio depresivo recurrente.

156. En el tratamiento de un episodio maníaco, ¿cuál de los fármacos siguientes está CONTRAINDICADO?:

a. Fluoxetina. b. Levomepromazica. c. Haloperidol. d. Carbamazepina.

157. De los siguientes síntomas, ¿cuál es característico de los efectos secundarios del tratamiento con antidepresivos tricíclicos?:

a. Ansiedad. b. Bradicardia. c. Diarrea. d. Náuseas y vómitos.

158. En la evolución de uno de los siguientes trastornos psiquiátricos NO aparecen ideas delirantes. Señálelo:

Page 189: Cuestionario de Urología

a. Afectivo tipo episodio maníaco. b. Afectivo tipo depresión mayor. c. Psicótico tipo esquizofrenia. d. De ansiedad por agorafobia.

159. Un paciente de 25 años consulta por sintomatología de ansiedad y profundo malestar porque en su trabajo tiene una sensación de inseguridad constante por la convicción de no realizar correctamente las tareas que le competen. Esto hace que necesite permanentemente revisarlas una y otra vez, lo que le supone pérdida de tiempo y eficacia. Esta sensación es tan dominante en su psiquismo, que le conduce a la idea de fracaso y a la pérdida de autoestima. El diagnóstico inicial corresponde a trastorno:

a. Por idea delirante. b. Por ansiedad fóbica. c. De personalidad evitativo-fóbica. d. Obsesivo-compulsivo.

160. Un hombre de 25 años, es conducido al Servicio de Urgencias por su familia porque dice que "le persiguen unos asesinos que van a matarle". En la exploración fisica se observan pupilas dilatadas, temperatura de 37,8°C, Pulso arterial: 110 Iatidos por minuto y TA de 160/95 mmHg, sin otros hallazgos. La familia afirma que tiene historia de abuso de drogas. La droga que más probablemente ha producido esta reacción es:

a. Alcohoa. b. Cocaína. c. Diazepam. d. Heroína.

161. Enferma diagnosticada de citoadenocarcinoma seroso ovárico en estadio IAl de la FIGO. Tras salpingo-ooforectomía unilateral del lado afecto, ¿cuál sería el siguiente paso en su tratamiento?:

a. Quimioterapia: cisplatino + paclitaxel. b. Radioterapia intraperitoneal. c. Quimioterapia intraperitoneal. d. Ninguno.

162. Señale, entre las siguientes, la afirmación correcta respecto a una mujer de 27 años, con diabetes mellitus insulino-dependiente de 10 años de evolución, gestante de 8 semanas:

a. El riesgo de aborto espontáneo es elevado, independientemente del grado de control de la glucemia. b. Se debe hace una determinación de Hb glicosilada en la primera visita obstétrica (Hgb Al). c. Se debe hacer amniocentesis en el segundo trimestre para determinar cariotipo y aIfafetoproteína. d. En el segundo trimestre, habrá, que evaluar conjuntamente la aIfafetroproteína en suero materno, B-HCG y el estriol no conjugado.

163. En una paciente de 26 años, gestante de 15 semanas, asintomática, ¿cuál de los siguientes hallazgos se debe considerar patológico?:

a. Galope S3. b. Desdoblamiento amplio de Sa. c. Soplo diastólico grado I-II/IV en vértice cardíaco. d. Soplo sistólico grado II-III/IV en borde esternaa.

164. Señale cuál de las siguientes situaciones en una gestante debe realizarse estudio de la coagulación (determinación de plaquetas, tiempo de protombina, fibrinógeno, productos de degradación del fibrinógeno):

a. Hipertensión al final de la gestación. b. Polihidrarnnios. c. Amenaza de parto pretérmino. d. Desprendimiento de la placenta normalmente inserta (DPPN).

165. ¿Qué exploración o método complementario nos permite hacer un diagnóstico de embarazo de certeza y precoz?:

a. El tacto vagino-abdominal. b. Las reacciones inmunológicas de embarazo dellaboratorio. c. El tacto rectovaginal. d. La ecografía intravaginal.

166. El cariotipo más común de la disgenesia gonadal mixta es:

a. (46 XX). b. (45XO.46XY). c. (46 XXX). d. (45 XY . 45 XX).

167. ¿Qué marcador tumoral se produce en los tumores del seno endodérmico?:

a. Fracción beta de la hormona gonadotropa coriónica. b. CA - 125 c. CA - 54/61 d. Alfa fetoproteína.

168. ¿Cómo se denomina la rotura de la bolsa de las aguas cuando el cuello se encuentra totalmente dilatado?:

a. Oportuna. b. Prematura. c. Tardía.

Page 190: Cuestionario de Urología

d. Precoz.

169. ¿Cuál de los procesos malignos del ovario es contraindicación absoluta para la terapia hormonal sustitutiva en la menopausia?:

a. Disgerminoma. b. Tumor de Brenner. c. Teratoblastoma. d. Carcinoma de células claras.

170. La mayor parte del incremento del peso durante el embarazo no patológico, es atribuible al aumento del:

a. Líquido intracelular. b. Anabolismo lipídico. c. Tamaño del útero y su contenido y de las mamas. d. Anabolismo hidrocarbonado.

171. ¿En cuál de las siguientes malformaciones es más difícil hacer el diagnóstico prenatal?:

a. Las del sistema nervioso central. b. Las del aparato digestivo. c. Las renales. d. Las cardíacas.

172. Señale cuál de las siguientes anormalidades se produce en los prematuros con distress respiratorio idiopático (Síndrome de las membranas hialinas), como consecuencia de las alteraciones fisiopatológicas que el mismo produce:

a. Hipoplasia de las cavidades izquierdas. b. Apertura de shunts arteriovenosos cerebrales. c. Hipoplasia de las cavidades derechas. d. Persistencia del ductus arterioso y del foramen oval.

173. Es cierto que la leche materna, comparada con la de vaca contiene:

a. Más sales. b. Menos fenilalanina. c. Más ácidos grasas de cadena corta. d. Más calcio.

174. ¿Cuál de las siguientes afirmaciones es INCORRECTA, respecto al raquitismo común por carencia de vitamina D?:

a. Generalmente la calcemia es normal. b. Los niños de raza negra son más susceptibles. c. Los casos esporádicos son más frecuentes que los familiares. d. Presenta hiperfosforemia que es útil para el diagnóstico.

175. Un niño de ocho años, mantiene una crisis asmática desde hace 50 horas, con pobre respuesta a la medicación broncodilatadora y tiene fiebre de 38,7°C que motiv ó tratamiento con antibióticos.¿ Cuál de las siguientes situaciones de equilibrio ácido-base sería la más propia de su situación?:

a. Normalidad. b. Alcalosis respiratoria pura. c. Acidosis respiratoria pura. d. Acidosis mixta o combinada.

176. ¿Cuál de las siguientes características es más frecuente y peculiar de los vómitos por reflujo gastroesofágico (calasia) en el niño?:

a. Asociación con ferropenia. b. Relación con la ingesta. c. Repercusión sobre el crecimiento. d. Comienzo neonatal y mejoría con la edad.

177. Un niño de 6 años es diagnosticado de meningitis por Haemophilus influenzae tipo b (Hib). La pauta de profilaxis entre los convivientes más corresta es administrar:

a. Vacuna conjugada anti-Hib a todos. b. Vacuna conjugada anti-Hib sólo a los menores de 5 años. c. Garnmaglobulina específica anti-Hib a todos. d. Rifampicina 4 días a todos.

178. Un niño de 14 meses, acude a la consulta con sintomatología digestiva crónica acompañada de hipocrecimiento marcado (<Pc10), desnutrición, hipotonía muscular y una llamativa distensión abdominaa. Entre los diagnósticos que se citan, ¿cuál considera más probable?:

a. Enfermedad celíaca no tratada. b. Intolerancia a proteínas vacunas. c. Alergia a caseína mediada por IgE. d. Deficiencia primaria de lactasa.

179. ¿En cnál de las siguientes situaciones, en la edad pediátrica, la hipoglucemia NO es secundaria al aumento de la secreción de insulina?:

a. Glucogenosis tipo I. b. Hijo de madre diabética. c. Sensibilidad a la leucina. d. Nesidioblastosis.

Page 191: Cuestionario de Urología

180. ¿Cuál de los siguientes hallazgos es MENOS común en un hijo de madre alcohólica?:

a. Anomalías faciales (blefarofimosis, epicantus, micrognatia, etc.). b. Deficiencia metal. c. Retraso de la talla. d. Estenosis del acueducto de Silvio.

181. Un niño de 6 años, con una talla más corta que la que debería corresponder a su edad, aqueja dolores moderados de su cadera derecha y claudicación con la carrera. A la exploración, se aprecia una discreta atrofia del muslo y existe una limitación de los movimientos de rotación interna y abducción. Los estudios de laboratorio son negativos. La radiografía convencional, sólo muestra una discreta osteoporosis del núcleo epifisario de la cabeza femoral. Con este cuadro clínico el diagnóstico debe orientarse hacia:

a. Artritis inespecífica de la cadera. b. Enfermedad de Perthes. c. Quiste óseo esencial del extremo superior del fémur. d. Epifisiolisis femoral superior.

182. Acude a Urgencias un niño de 9 años, tratado dos días antes con un yeso cerrado por una fractura supracondIlea del húmero. Aqueja intenso dolor en el codo, antebrazo y dedos. Los dedos presentan buen relleno capilar y una coloración aceptable, pero están muy hinchados, con poca movilidad activa, y su movilización pasiva produce intenso dolor. La actitud más correcta será:

a. Administrar antiinflamatorios y remitir al paciente a consulta externa. b. Mantener al niño en Urgencias con el yeso elevado e instándole a mover activamente los dedos. c. Retirar el yeso y realizar una osteosíntesis de la fractura. d. Retirar el yeso, vigilar al paciente y, si no mejora rápidamente, fasciotornía.

183. Al Servicio de Urgencias es traído un niño de 5 años por dolor inguinal de 48 horas de evolución. Tres semanas antes ha presentado una infección de vías respiratorias altas tratada con antibióticos. La exploración abdominal es normal y no se palpan hernias. La cadera presenta limitación dolorosa en todos sus movimientos, y el paciente cojea. La analítica y la radiología son normales y la ecografía indica un pequeño derrame articular de cadera probablemente seroso. El primer diagnóstico de sospecha debe ser:

a. Artritis séptica de cadera. b. Artritis reumatoide juvenil. c. Sinovitis transitoria de cadera. d. Enfermedad de Perthes.

184. Un paciente de 18 meses, hijo de una familia de inmigrantes residente en un suburbio marginal de una gran ciudad, es traído a consulta por cojera desde que empezó a caminar a los 16 meses. A la exploración presenta marcha en Trendelenburg. Habrá que sospechar en primer lugar:

a. Luxación congénita de cadera. b. Sinovitis transitoria de cadera. c. Enfermedad de Perthes. d. Artritis séptica de cadera.

185. Paciente de 13 años, con antecedentes de dolor difuso en muslo que, tras un traumatismo banal, presenta una agudización de su sintomatología. ¿Cuál de las siguientes opciones es el diagnóstico más probable?:

a. Enfermedad de Perthes. b. Sinovitis de cadera. c. Epifisiolisis femoral proximaa. d. Fractura de cuello femoral.

186. Una paciente es intervenida por presentar un carcinoma medula de tiroides en el seno de un síndrome MEN tipo 2 A. ¿Cuál es el mejor método para identificar a los familiares de esta paciente que tienen riesgo de desarrollar un carcinoma medular de tiroi-des?:

a. Estudio genético en células de sangre periférica. b. Estudio genérico en células tiroideas obtenidas por punción. c. Determinación de calcitonina en plasma. d. Determinación de tiroglobulina en plasma.

187. Señale cuál de las siguientes NO es una indicación para la quimioprofilaxis de la tuberculosis con isoniacida:

a. Joven de 20 años con Mantoux de 16 mm de induración. b. Niño de 9 años, con Mantoux negativo, cuyo padre acaba de ser diagnosticado de tuberculosis pulmonar. c. Hombre de 45 años, Mantoux positivo, que va a ser sometido a trasplante de corazón. d. Mujer de 60 años, Mantoux positivo, a la que se le va a realizar una histerectornía.

188. Un paciente mayor de edad, competente y bien informado, rechaza un tratamiento que su médico le propone, a pesar de que éste le informa de que está poniendo en riesgo su vida con dicha actitud. El médico debe:

a. Administrar el tratamiento, ocultándoselo al paciente. b. Solicitar autorización familiar para tratarlo. c. Solicitar amparo judicial para su administración. d. Abstenerse de tratar al paciente.

189. Los pacientes que requieren la práctica de estudios diagnósticos o medidas terapéuticas que conllevan riesgos, deben consentir de forma expresa su realización en el formulario "ad hoc" conocido como Consentimiento Informado. La cumplimentación afirmativa de este requisito tiene como objeto:

a. Cambiar la vía penal por la civil, en caso de demanda. b. Evitar las demandas judiciales por cualquier vía. c. Disminuir las indemnizaciones en caso de demanda.

Page 192: Cuestionario de Urología

d. Respetar el principio de autonomía y derecho a estar informado del paciente.

190. Usted dispone de una prueba diagnóstica de sífilis y decide hacer un despistaje de la enfermedad en la población que atiende. Sabemos que la sensibilidad de la prueba es de 0.95 y la especificidad de 0.9d. Si la prevalencia de la sífilis en su población es de 1/1000 aproximadamente, ¿cuál será el valor predictivo positivo?:

a. 0.02 (2%). b. 0.095 (95%). c. a. 0 (100%). d. Con estos datos no se puede saber.

191. Estamos probando una nueva prueba diagnóstica de tuberculosis. Si la aplicamos a 100 tuberculosos conocidos y es negativa en tres de ellos, podemos decir que:

a. Su valor predictivo negativo es de 0.97. b. La especificidad de la prueba es de 0.97. c. La sensibilidad de la prueba es de 0.97. d. Tiene un 3% de falsos positivos.

192. Señale cuál es la etiología más frecuente de la pérdida aguda de conciencia:

a. Bloqueo cardíaco. b. Crisis vasovagal. c. Hipoglucemia. d. Traumatismo craneoencefálico.

193. Respecto a la velocidad de sedimentación (VSG), indique la respuesta FALSA:

a. El valor normal en la mujer es de 20 rom. b. Su aumento está directamente relacionado con la rapidez de agregación y sedimentación de los hematíes. c. Es una prueba inespecífica. d. Un individuo asintomático puede considerarse sano si lo único que hallamos en su analítica es una VSG claramente elevada.

194. Decimos que una prueba diagnóstica es muy específica cuando:

a. Aplicada a una población de enfermos tiene pocos falsos negativos. b. Cuando un resultado negativo es poco probable que corresponda a un enfermo. c. Aplicada a una población de sanos tiene pocos falsos positivos. d. Tiene una alta sensibilidad diagnóstica.

195. Un paciente conocido por usted y diagnosticado endoscópicamente de ulcus duodenal, con brotes estacionales, acude a su consulta solicitando el tratamiento con antibióticos del que ha oído hablar. Usted decide practicar una prueba para Helicobacter pylori que tiene una sensibilidad de 0.95 y especificidad de 0.95. teniendo en cuenta que la prevalencia de H. pylori en el ulcus duodenal es del 95%, si la prueba resulta negativa, ¿qué probabilidad hay de que sea un falso negativo?:

a. Despreciable. b. El 5%. c. El 50%, aproximadamente. d. No se puede saber con estos datos.

196. ¿Cuál de los siguientes es el factor de riesgo más importante en la artrosis?:

a. La edad. b. La obesidad. c. Los traumatismos articulares. d. El grupo étnico.

197. Al realizar el análisis de un estudio clínico-epidemiológico el riesgo relativo indica:

a. La fuerza de la asociación entre una exposición y una enfermedad. b. La diferencia de riesgo entre expuestos y no expuestos. c. La magnitud de un problema de salud en una comunidad. d. La fracción etiológica o atribuible a la exposición.

198. Si desea conocer la probabilidad que tiene de padecer cardiopatía isquémica dentro de 5 años un varón de 50, fumador e hipertenso, usted revisará la literatura buscando preferentemente la:

a. Incidencia acumulada de los fumadores e hipertensos en este grupo de edad. b. Densidad de incidencia de los fumadores e hipertensos en este grupo de edad. c. Riesgo relativo de los fumadores e hipertensos en este grupo de edad. d. Fracción atribuible de los fumadores e hipertensos en este grupo de edad. .

199. ¿Cuál de las siguientes afirmaciones es FALSA con respecto al proceso de cribado ("screening") poblacional?:

a. Identifica un sub grupo de personas con riesgo elevado de padecer enfermedad. b. Se refiere al período asintomático de la enfermedad. c. Requiere confirmación de los resultados negativos en contrados. d. Es diferente de la búsqueda oportunista de casos o "casefinding". ,

200. En un estudio que evalúa la presencia de hipertensión como factor de riesgo del accidente vascular cerebral (A VC), se observa un riesgo relativo de 6 (RR=6 intervalo de confianza 95 %, 4-8). El análisis estratificado en función del sexo, da los siguientes resultados: en mujeres el RR es de 8 (intervalo de confianza 95%,5-11) Y en los hombres de 4 (intervalo de confianza 95% 1-7). ¿Cuál de las siguientes afirmaciones es cierta?:

a. La hipertensión sólo es un factor de riesgo para las mujeres. b. La hipertensión es un modificador del efecto, ya que el RR difiere según estratos.

Page 193: Cuestionario de Urología

c. La hipertensión es un factor de confusión, ya que los valores crudos y estratificados difieren. d. El sexo es un factor modificador de efecto entre la hipertensión y el AVC.

201. Todas las siguientes son recomendaciones dietéticas adecuadas para la prevención de las enfermedades crónicas, EXCEPTO:

a. Limitar el consumo de grasas al 30% o menos del total de calorías diarias. b. Limitar el consumo de colesterol a 300 mg o menos por día. c. Limitar el consumo de sal a 5 gr por día. d. Limitar el consumo de carbohidrato s al 20% del total de calorías diarias.

202. La presencia de nuevos casos de una enfermedad en un número superior al esperado se denomina:

a. Hiperendemia. b. Epidemia. c. Endemia. d. Epizootia.

203. En la comunidad A se diagnostican 40 nuevos casos de cáncer de mama por 100.000 mujeres cada año; en la comunidad B se diagnostican 100 nuevos casos por 100.000 mujeres cada año. ¿Cuál de las siguientes es la explicación más probable?:

a. En la comunidad A se practican más mamografías. b. El tratamiento es mucho más efectivo en la comunidad A. c. Las mujeres de la comunidad A son más jóvenes. d. Las mujeres de la comunidad A son menos fumadoras.

204. Todos los siguientes son factores de riesgo del cáncer testicular, EXCEPTO:

a. Historia de criptorquidia. b. Historia de orquitis secundaria a parotiditis. c. Historia de hernia inguinal en la infancia. d. Historia familiar de cáncer testicular.

205. En un estudio realizado en una muestra de 300 sujetos se informa que su edad media es de 50 años, con una desviación estándar de 10 años. ¿Cuál es el significado de estas cifras?:

a. Las edades de los sujetos de la muestra se sitúan entre 40 y 60 años. b. Las edades de los sujetos de la muestra se sitúan entre 30 y 70 años. c. El 95% de los sujetos de la muestra tienen edades entre 40 y 60 años. d. El 95% de los sujetos de la muestra tienen edades entre 30 y 70 años.

206. Unos investigadores desean diseñar un estudio para comparar dos pautas de tratamiento de la hipertensión arterial, utilizando como medida de eficacia la reducción en las cifras de tensión arterial diastólica (TAD). ¿Cuál de los siguientes NO es un elemento fundamental para calcular el número de sujetos necesarios para dicho resultado?:

a. Error tipo I. b. Error tipo II. c. Mínima diferencia que se desea ser capaz de detectar. d. Duración del seguimiento de pacientes.

207. En un estudio transversal sobre una muestra de sujetos representativos de una comunidad se ha determinado que la cifra media de tensión arterial diastólica (TAD) es de 85 mmHg, con un error estándar de 2,5 mmHg. ¿Cuál de las siguientes afirmaciones es cierta?:

a. El 95% de los sujetos de la muestra tenían cifras de TAD entre 80 y 90 mmHg. b. El 90% de los sujetos de la muestra tenían cifras de TAD entre 82,5 y 87,5 mmHg. c. Se tiene un 95% de confianza de que el intervalo 80-90 mmHg incluya al verdadero valor medio de la TAD de la comunidad. d. Se tiene un 90% de confianza de que el intervalo 82,5 - 87,5 mmHg incluye el verdadero valor medio de la TAD de la comunidad.

208. Un estudio relaciona las concentraciones de colesterol ligado a las lipoproteínas de alta densidad (HDL-Col) con el número de cigarrillos fumados en la actualidad, en una muestra de 100 individuos. El coeficiente de correlación es de -0,15 (p<0.05). ¿Cuál de las siguientes afirmaciones es correcta?:

a. El consumo de tabaco es la causa de que disminuya el HDL-Col. b. El consumo de tabaco no es la causa de la disminución del HDL-Col, porque la correlación es débia. c. El consumo de tabaco es la causa de la disminución del HDL-Col, porque la correlación es estadísticamente significativa. d. Se ha observado una débil correlación negativa, estadísticamente significativa.

209. El consumo de A TP en cada contracción muscular cardíaca y periférica se emplea en:

a. El golpe de remo de la contracción. b. El deslizamiento de las fibras de actina sobre la miosina. c. La separación de la miosina de la actina en la relajación. d. La entrada de Ca++ a través de los canales lentos (L) del sarcolema.

210. En condiciones normales, la ovulación en la mujer se produce por:

a. La activación del eje hipo tálamo hipofisario por la progesterona. b. La acción de retroalimentación negativa de esteroides adrenales sobre la hipófisis. c. La desactivación del hipotálamo por stress debida a mecanismos mal conocidos. d. La liberación de LH y FSH provocada por retroalimentación positiva estrogénica.

211. Respecto al papel que juega el óxido nítrico en procesos fIsiológicos importantes, indique la afirmación INCORRECTA:

a. Interviene en procesos de formación de la memoria en el hipocampo. b. Es liberado en grandes cantidades por la médula adrenal.

Page 194: Cuestionario de Urología

c. Interviene en el mecanismo de erección por vasodilatación de origen parasimpático. d. Puede actuar como molécula mensajera entre células nerviosas.

212. Señale cuál de las afirmaciones siguientes, sobre regulación neuroendocrina, es correcta:

a. Una interrupción completa del tallo hipofisario va acompañada de un panhipopituitarismo. b. La hormona liberadora de tirotropina (TRH) es un tripéptido que controla la secreción de GH y TSH. c. La regulación fisiológica de la secreción de la hormona de crecimiento (GH) se lleva a cabo por 2 péptidos hipotalámicos: GHRH de acción estimuladora y el IGF 1 de acción inhibidora. d. En el hipotiroidismo primario se suele observar una hipertrofia compensadora de las células tirotropas de la adenohipófisis.

213. El potencial de acción de la fibra muscular cardíaca presenta una meseta que dura unos 300 milisegundos (cercana al pico del potencial de acción) y que se debe a la apertura de canales de membrana voltaje-dependientes para el calcio. Este fenómeno es de gran significación fisiológica por su consecuencia directa sobre:

a. La frecuencia cardíaca. b. La contractilidad de la fibra cardíaca. c. El retardo auriculo-ventricular. d. El retorno venoso.

214. Señale la relación INCORRECTA relativa a la motricidad en los primates:

215. En relación con el funcionamiento de la sinapsis, indique el mecanismo que directamente termina la transmisión sináptica:

a. Unión del neurotransmisor (NRT) con los receptores citosólicos. b. Apertura de la vesícula sináptica al espacio intersináptico. c. Recaptación del NRT por la terminación presináptica. d. Activación de la corriente presináptica de calcio voltaje dependiente.

216. Los mecanismos de retroalimentación ("feed back") utilizados frecuentemente por los grandes sistemas de control de la homeostasis (nervioso y endocrino) son:

a. Negativos y positivos porque ambos regulan la constancia de la variable sea cual sea el signo de su desviación. b. Negativos o positivos en función de la desviación de la variable con respecto a su nivel de referencia. c. Sólo positivos para corregir desviaciones de la variable por debajo del nivel de referencia. d. Negativos en la mayoría de los casos.

217. La Miastenia gravis se produce por:

a. Decremento de la actividad eléctrica presináptica. b. Bloqueo de los receptores colinérgicos por nicotina. c. Disminución de la síntesis de aceti1colina. d. Presencia de anticuerpos para receptores colinérgicos.

218. Un paciente sufre una lesión punzante en el borde cubital de la base del tercer dedo de una mano. En la exploración del mismo se aprecia déficit de flexión de la articularción interfalángica distal y anestesia de su borde cubital. Se debe sospechar.

a. Lesión del tendón flexor superficial y del nervio colateral cubital del dedo. b. Lesión del tendón flexor profundo y del nervio colateral cubital del dedo. c. Lesión de ambos tendones flexores y del nervio colateral cubital del dedo. d. Lesión de los tendones de la musculatura intrínseca y del nervio colateral cubital del dedo.

219. Señale cuál de las siguientes afirmaciones sobre la anatomía funcional del corazón es FALSA:

a. El ápex del corazón se dirige hacia abajo, hacia delante y hacia la izquierda. b. Las cavidades derechas ocupan una posición anterior y derecha, respecto las izquierdas. c. Las aurículas contribuyen con su contracción al 15-25% del llenado ventricular. d. El tabique interauricular es muscular, excepto en su posición basal.

220. El conducto torácico desemboca en:

a. Arteria subclavia izquierda. b. Tronco braquiocefálico venoso izquierdo. c. Vena yugular anterior derecha. d. Vena yugular externa izquierda

221. Un paciente es sometido a tiroidectomía total por presentar una neoplasia de tiroides. En el postoperatorio el paciente refiere fatiga de voz y pérdida de timbre de la misma, sin otras manifestaciones acompañantes. ¿Cuál de las siguientes estructuras es más probable que haya sido lesionada en el curso de la intervención?:

a. Rama externa del nervio laríngeo superior. b. Rama interna del nervio laríngeo superior. c. Nervio recurrente laríngeo. d. Nervio glosofaríngeo.

222. Un trabajador de la construcción, tras realizar un brusco esfuerzo, empieza a desarrollar un cuadro de lumbociática con dolores que se proyectan a lo largo del miembro inferior izquierdo. Tras la exploración convencional, se le manda guardar reposo y tomar AINE y analgésicos. Al no ceder el cuadro, se realiza una RM apreciándose en los diferentes cortes una imagen de prolapso discal situada a nivel del espacio L4-L5 y de proyección izquierda. ¿Cuál de las siguientes alteraciones encajarían con la lesión de

Lesión Alteración

a. Cordones posteriores Propiocepción

b. Amplia de núcleos cerebelosos Ataxia

c. Ganglios basales Acinesia y movimientos involuntarios

d. Vía corticoespinal Parálisis espástica

Page 195: Cuestionario de Urología

la raíz que corresponde a tal espacio?:

a. Déficit motor de extensores de la rodilla y reflejo rotuliano débil. b. Inexistencia de déficit motor. Debilidad del reflejo aquíleo. c. Déficit motor de la extensión del primer dedo. Reflejos normales. d. Hipoestesia plantar. Reflejo rotuliano débia.

223. Paciente de 25 años que presenta, tras sufrir un accidente deportivo, una fractura en la diáfisis humeral de su brazo izquierdo, con incapacidad para realizar la extensión de la muñeca y de los dedos de su mano izquierda, al igual que imposibilidad de separar el primer dedo de la mencionada mano. Señale cuál de los siguientes mecanismos explica estos signos:

a. Retracción isquémica de Volkman. b. Lesión del plexo braquial. c. Lesión de la arteria humeral. d. Parálisis del nervio radial.

224. Si se introducen sales de oro en el tratamiento de un paciente con artritis reumatoide, es preciso realizar con frecuencia uno de los siguientes controles:

a. Determinación de la V.S.G. b. Examen oftalmológico. c. Determinación del factor reumatoide. d. Hemograma y sedimento urinario.

225. Paciente con fibrilación auricular en tratamiento con digoxina, en quien persiste una frecuencia ventricular muy rápida, por lo que se debe añadir un fármaco que enlentezca la conducción aurículoventricular. De los siguientes NO se debe administrar:

a. Verapamia. b. Dilitiazem. c. Propranoloa. d. Quinidina.

226. Un paciente es diagnosticado de miocardiopatía dilatada en grado funcional 1, con disfunción sistólica incipiente del ventrÍculo izquierdo. El tratamiento de la insuficiencia cardíaca comenzaría con:

a. Digoxina. b. Diuréticos. c. Inhibidores del enzima conversar de la angiotensina. d. Bloqueantes de los canales del calcio.

227. Paciente de 50 años en tratamiento anticoagulante con dicumarínicos. ¿Cuál de las siguientes pruebas pediría para medir la actividad del anticoagulante?:

a. Fibrinógeno. b. Tiempo de trombina. c. Tiempo de protrombina. d. Tiempo de tromboplastina parciaa.

228. Los diuréticos pueden producir todas EXCEPTO una de estas manifestaciones:

a. Insuficiencia renal prerrenal. b. Alcalosis metabólica. c. Hiponatremia con volumen extracelular normal. d. Depleción salina.

229. Para su actividad inductora del sistema del citocromo P450, la rifampicina disminuye los niveles séricos e interfiere la actividad de todos los fármacos siguientes MENOS uno. Señale cuál:

a. Cic1osporina. b. Penicilina G. c. Glucocorticoides. d. Anovuladores orales.

230. Una paciente de 30 años, tiene infecciones .urinarias de repetición por lo que toma crónicamente antibióticos. Su médico, debido a tos seca y disnea progresiva, le realiza una radiografía de tórax donde objetiva infiltrados intersticiales. ¿Cuál de los siguientes fármacos es el causante más probable?:

a. Cotrimoxazol. b. Ácido pipemídico. c. Nitrofurantoína. d. Cefaloridina.

231. ¿En qué grupo de la clasificación de fármacos antiarrítmicos de Vaughan-Williams incluiría la quinidina? :

a. IA. b. IC. c. II. d. ID.

232. En un paciente con infarto agudo de miocardio de localización inferior, aparece bradicardia e bipotensión tras la administración de Nitroglicerina. ¿Cuál de los siguientes fármacos debe utilizarse como tratamiento inmediato por vía i.v.?:

a. Digoxina. b. Atropina. c. Propranoloa. d. Lidocaina.

Page 196: Cuestionario de Urología

233. ¿Cuál de las siguientes medidas terapéuticas puede usarse cuando, por cualquier circunstancia, es necesario elevar rápidamente las plaquetas?:

a. Corticoesteroides. b. Garnmaglobulina i.v. c. Danazol. d. Azatioprina.

234. ¿Cuál de las siguientes drogas NO es útil en el tratamiento de la hiperpotasemia?:

a. Calcitonina. b. Resinas de intercambio iónico. c. Furosemida. d. Salbutamol

235. La presencia de una biopsia intestinal de macrófagos con glucoproteínas PAS-positivas, es diagnóstico de:

a. Abetalipoproteinemia. b. Linfoma MALT. c. Esprúe tropical. d. Enfermedad de Whipple.

236. ¿Cuál de las siguientes situaciones se produce en la última fase del proceso de cicatrización de las heridas, a partir de la cuarta semana?:

a. Aumenta el número de fibroblastos, así como la red capilar neoformada. Las fibras de colágena no se han desarrollado todavía. b. Predominio fundamental de redes de fibrina, así como presencia abundante de leucocitos polimofonuc1eares y monocitos. c. Hipervascularización, con predominio de la red capilar neoformada. Disminución de los fibroblastos y de las fibras de colágena. d. Disminución del número de fibroblastos y de la red capilar neoformada, predominando las fibras de colágena Y la sustancia fundamentaa.

237. ¿Cuál de las siguientes enfermedades NO está causada por priones?:

a. Panencefalitis esclerosante sub aguda. b. Enfennedad de Creutzfeld-Jakob. c. Kuru. d. Insomnio fatal familiar.

238. Mujer de 40 años, sin antecedentes personales de interés, que es remitida a consulta por obesidad troncular de reciente diagnóstico, amenorrea y depresión. En la exploración física se objetiva una facies redondeada, con hirsutismo moderado, TA de 160/100 mmHg y edemas en miembros inferiores. El estudio hormonal fue el siguiente: Tirotropina (TSH) 0.7 µU/mL (N: 0,4-5,0), T4 libre 16.8 pmol/L (N: 9,0-23,0) cortisollibre en orina 12 µg/24h (N: 20-100), cortisol plasmático 3 µg/dL (N: 5-25) y corticotropina (ACTH) 9 pmol/L (N: inferior a 52). ¿Cuál de las siguientes situaciones es más probable que dé lugar a este cuadro?:

a. Hipotiroidismo subclínico. b. Enfennedad de Cushing. c. Síndrome de Cushing ACTH dependiente. d. Administración exógena de glucocorticoides.

239. En un paciente con insuficiencia renal crónica y en programa de hemodiálisis, ¿qué alteración metabólica es determinante para el desarrollo de un hiperparatiroidismo secundario?:

a. Retención de fosfato y disminución de hidroxilación de vitamina D. b. Disminución de la excrección renal del calcio. c. Aumento de la actividad de los osteoclastos. d. Disminución de la fosfatemia.

240. Señale la asociación correcta, entre las siguientes, de causas y mecanismos fisiopatológicos de hipertensión portal:

241. Una paciente diabética de 89 años tiene una escara profunda en el talón del pie derecho con pus, donde se aisla Staphylococcus aureus meticilin resistente. Dos días después tiene fiebre de 39°C, tiritona y descenso del nivel de conciencia. ¿Qué actitud de las siguientes le parece la más correcta?:

a. Hacer un TAC craneal. b. Tratar empíricamente con ceftriaxona. c. Administrar esteroides i.v. d. Practicar hemocultivos y tratar con vancomicina i.v.

242. ¿Cuál de las siguientes entidades clínicas NO suele aparecer en pacientes con infección precoz por el VIH?:

a. Trombopenia. b. Leucoplasia vellosa oral. c. Linfadenopatía generalizada. d. Toxoplasmosis.

243. ¿Cuál de los siguientes NO es uno de los llamados criterios de Glasgow modificados para predecir un mal pronóstico en el seno de una pancreatitis aguda?:

Causa Mecanismo

Trombosis portal Postsinuosidal

Síndrome de Budd-Chiari Sinuosoidal

EsQuistosomiasis Presinusoidal

Fibrosis congénita Sinusoidal

Page 197: Cuestionario de Urología

a. Edad superior a los 55 años. b. Leucocitosis >15000/mmc. c. Calcemia <8 mg/dL. d. Amilasemia >5000 Ul/A.

244. Entre las particularidades que se mencionan, ¿cuál es característica de las enfermedades genéticas que se heredan a través del DNA mitocondrial o citoplasmático? :

a. Estas enfennedades las transmiten los varones y las sufren las mujeres. b. Las manifestaciones clínicas habitualmente ya son patentes en el recién nacido. c. Presentan un árbol genealógico similar al de una herencia monogénica gonosómica dominante. d. Generalmente se afecta el cerebro, el ojo o el sistema músculoesquelético.

245. Paciente que ingresa por hemorragia digestiva alta. No hay antecedentes de consumo de AINEs. La endoscopia revela úlcera gástrica en incisura angularis, con un punto de hematina y mínimos restos de sangre oscura en el estómago. Se realizan biopsias del margen de la úlcera y una biopsia antral para prueba rápida de ureasa con resultado positivo. ¿Cuál de las siguientes actitudes es la más correcta? :

a. Esclerosis endoscópica de la úlcera seguida de tratamiento con omeprazol, 20 mg/día durante 28 días. b. Esclerosis endoscópica de la úlcera seguida de tratamiento triple anti-Helicobacter pylori durante 7 días. c. Omeprazol, 20 mgldía durante 1 mes. d. Tratamiento triple anti-Helicobacter pylori durante 7 días, seguido de un antisecretor hasta que se confirme la erradicación del germen.

246. Acude a Urgencias un hombre de 71 años refiriendo la presencia de sangre en heces hace 48 horas; en las dos últimas deposiciones no había sangre. Se realiza una colonoscopia hasta ciego que demuestra la presencia de una lesión sugerente de angiodisplasia en el colon descendente ¿Cuál de las siguientes afirmaciones es correcta?:

a. La localización de la lesión es la típica. b. El paciente es demasiado anciano para tener una angiodisplasia. c. El tratamiento debe ser quirúrgico, con resección de la zona enferma. d. En algunos de estos pacientes el tratamiento con estrógenos y progestágenos puede ser útil en la prevención de la recidiva hemorrágica.

247. Paciente de 29 años con datos histológicos de hepatitis crónica en la biopsia hepática y los datos analíticos siguientes: GOT y GPT dos veces por encima del límite alto de la normalidad, bilirrubina 0.2 mg/dL; serología virus de la hepatitis B: HbsAg (-), HBsAc (+), HBcAc (+). Anticuerpos anti virus C (+); ceruloplasmina y Cu sérico normales. ¿Cuál es, entre las signientes, la causa más probable de la hepatitis crónica? :

a. Infección por el virus de la hepatitis C. b. Infección por el virus de la hepatitis B. c. Enfermedad de Wilson. d. Hemocromatosis.

248. El estudio analítico y serológico de un enfermo con astenia e ictericia aporta los siguientes datos: GOT>30 veces el límite de lo normal, GPT >40 veces el limite de lo normal, bilirrubina total 6.7 mgldL, HBsAg (+), HbeAg (+) anticuerpos IgM frente al core de virus positivos, anticuerpos antivirus delta (-) y anticuerpos antivirus C (-). ¿Cuál es, entre los siguientes, el diagnóstico más probable? :

a. Hepatitis aguda por virus B b. Hepatitis crónica por virus B agudizada por virus delta. c. Hepatitis aguda por virus e en portador de virus B. d. Hepatitis aguda por virus B y C.

249. Enferma de 62 años con diabetes mellitus tipo 2, que ingresa por dolor en hemiabdomen superior e hiperamilasemia, y es diagnosticada de pancreatitis aguda. El estudio realizado no demuestra etiología de la misma. A las 3 semanas de evolución de la enfermedad, encontrándose asintomática, la ecografia abdominal evidencia una colección líquida, bien delimitada, de unos 35 x 30 mm de diámetro, con características inequívocas de pseudoquiste pancreático. ¿Qué conducta de las siguientes es más adecuada? :

a. Intervención quirúrgica. b. Drenaje percutáneo de la colección líquida. c. Actitud expectante y seguir la evolución clínica en espera de la resolución espontánea. d. Punción-aspiración percutánea dirigida por ecografía para el análisis del líquido de la colección.

250. Enfermo de 51 años con cirrosis hepática por virus C, en estadio funcional B-8 de la clasificación de Child-Pugh, sin antecedentes de hemorragia digestiva alta. La endoscopia demostró varices esofágicas de gran tamaño con "signos rojos" en su superficie ¿Cuál de las siguientes medidas es más adecuada para prevenir el primer episodio hemorrágico por varices esofágicas? :

a. Escleroterapia endoscópica de las varices. b. Administración de calcioantagonistas. c. Prescribir bloqueadores beta no selectivos. d. Derivación porto-cava profiláctica.

251. Paciente de 75 años, sin antecedentes clínicos de interés, que consulta por un episodio de hematoquecia reciente. La colonoscopia total demuestra un pólipo único pediculado, de unos 25 mm de diámetro, en sigma, con una ulceración en su cúspide. ¿Qué actitud de las siguientes es más adecuada? :

a. Seguimiento cada 6 meses e intervención quirúrgica si aumenta de tamaño. b. Indicar intervención quirúrgica. c. Biopsia endoscópica del pólipo y decidir según resultado. d. Realizar polipectomía endoscópica y estudio anatomopatológico del pólipo.

252. Paciente de 68 años que consulta por pirosis, disfagia leve ocasional y episodios compatibles con regurgitación nocturna

Page 198: Cuestionario de Urología

desde hace 2 semanas. Refiere desde hace unos 3 años, temporadas anteriores de pirosis y regurgitación. La endoscopia alta practicada demuestra una esofagitis erosiva grave. ¿Qué tratamiento farmacológico, entre los siguientes, es el más adecuado? :

a. Inhibidores de la bomba de protones. b. Antagonistas de los receptores H2. c. Sucralfato. d. Tratamiento combinado con anti-H2 y sucralfato.

253. El diagnóstico de certeza de la hepatitis alcohólica se basa en:

a. Datos clínicos. b. Cociente AST/ALT superior aL c. Garnmaglutamil transpeptidasa >1000 VIL. d. Biopsia hepática.

254. Señale cuál de las siguientes características NO es propia de la enfermedad de Crohn:

a. Afectación continua del colon. b. Presencia de inflamación transmural. c. Presencia de fisuras. d. Presencia de fistulas.

255. Indique cuál de las siguientes afirmaciones sobre la acalasia es FALSA:

a. Existe degeneración y disminución de las células ganglionares del plexo de Auerbach. b. La presentación clínica típica es la disfagia para líquidos y sólidos. c. En la manometria esofágica el esfinter esofágico inferior se relaja completamente en respuesta a la deglución. d. Se debe realizar endoscopia para descartar la presencia de lesiones orgánicas esofágicas.

256. La presentación brusca de disfagía, dolor torácico, tos paroxística e hipersalivación tras la ingesta en un anciano, sugíere:

a. Aspiración alimentaria. b. Divertículo de Zencker. c. Impactación esofágica de cuerpo extraño. d. Fístula traqueoesofágica.

257. Señale cuál de las siguientes entidades clínicas NO se asocia a enfermedad inflamatoria intestinal:

a. Nefrolitiasis. b. Pioderma gangrenoso. c. Esteatosis hepática. d. Retinitis.

258. Un enfermo de 14 años presenta dientes supernumerarios, un osteoma en la mandíbula, algunos fibromas y pólipos en colon ¿Qué tratamiento de los siguientes se debe prescribir?:

a. Colectomía total. b. Quimioterapia. c. Radioterapia. d. Terapia génica.

259. Señale cual de las siguientes respuestas es correcta en relación con el virus de la hepatitis B (VHB):

a. La positividad de los anticuerpos antiHBs indica inmunización frente al VHB. b. La presencia de HBsAg es diagnóstica de hepatitis aguda B. c. La aparición de anticuerpo s antiHBe indica elevada infecciosidad. d. Los sujetos con antiHBc positivo deben vacunarse frente al VHB.

260. ¿Cuál de las siguientes afirmaciones, relativas al divertículo de Meckel, es INCORRECTA?:

a. En algunos casos presenta mucosa ectópica gástrica. b. Es un vestigio del conducto onfalomesentérico o vitelino. c. Es un divertículo falso, pues su pared carece de túnica muscular. d. Se localiza en Íleon, casi siempre a menos de un metro de la válvula ileocecal.

261. ¿Cuál de los siguientes signos radiográficos sugiere íleo biliar?:

a. Niveles hidroaéreos en intestino delgado. b. Borrarniento de la línea renopsoas. c. Imagen en "cuentas de rosario". d. Aerobilia.

262. ¿En cuál de las siguientes poliposis es más frecuente la aparición de un cáncer de colon?:

a. Síndrome de Peutz-Jeghers. b. Poliposis adenomatosa familiar. c. Síndrome del Cronkhite-Canada. d. Poliposis juvenil.

263. Señale cuál de las siguientes es la causa más frecuente de isquemia mesentérica aguda:

a. Bajo gasto cardíaco. b. Trombosis arterial. c. Trombosis venosa. d. Embolia arterial.

264. Con respecto al tumor carcinoide, señale cuál de las siguientes afirmaciones es correcta:

Page 199: Cuestionario de Urología

a. Su localización más frecuente es el apéndice y el Íleon. b. En el 90% de los casos se acompaña de síndrome carcinoide. c. Es un tumor siempre benigno. d. Es un tumor mesenquimal.

265. La operación quirúrgica que establece una comunicación entre la luz gástrica y la superficie cutánea abdominal recibe el nombre de:

a. Gastrotomía. b. Gastrostomía. c. Gastrectomía. d. Gastropexia.

266. En el contexto de las enfermedades pleurales, señale cuál de las siguientes aseveraciones es INCORRECTA:

a. En la tuberculosis pleural, la biopsia tiene mayor rentabilidad diagnóstica que la baciloscopia del líquido pleuraa. b. Las neoplasias pleurales cursan siempre con derrame. c. La neoplasia que más frecuentemente afecta a la pleura es el adenocarcinoma metastático. d. La biopsia pleural con aguja es habitualmente más rentable en tuberculosis que en neoplasias pleurales.

267. ¿Cuál de las siguientes pruebas sirve para confirmar el diagnóstico de asma en un paciente con clínica de episodios recurrentes de broncoespasmo?:

a. Reacción dérrnica positiva a determinados alergenos. b. Eosinofilia en esputo. c. Hiperinsuflación pulmonar en la radiografía de tórax. d. Obstrucción reversible en la espirometría.

268. Mujer de 21 años no fumadora, que consulta por tos seca, febrícula, astenia, artralgias y lesiones cutáneas sugerentes de eritema nodoso. La radiografía de tórax muestra adenopatías hiliares bilaterales y en región paratraqueal derecha. La fibrobroncoscopia evidencia inflamación difusa de la mucosa bronquial y el lavado broncoalveolar una linfocitosis del 32 % con cociente linfocitos T4 (colaboradores)/T8(supresores) superior a 3,d. ¿Cuál es, entre los siguientes, el diagnóstico más probable y la actitud a seguir?:

a. Linfoma pulmonar y debe tratarse con MOPP. b. Tuberculosis pulmonar, por lo que hay que iniciar tratamiento tuberculostático inmediato a la espera del resultado del cultivo de Lowenstein. c. Alveolitis alérgica extrínseca, lo que obliga a obtener una muestra de tejido pulmonar para descartarla. d. Sarcoidosis en estadío 1, no siendo necesario iniciar tratamiento de su enfermedad respiratoria.

269. En el postoperatorio de una apendicitis, una mujer de 35 años presenta un cuadro de mareo, parestesias e hiperventilación. La gasometría arterial muestra: pH 7.51; pCO2 28 mmHg; PO2 103 mmHg; HCO3 25 mEq/L Y PO2 (A-a) 1a. ¿Cuál es, entre los siguientes, el diagnóstico más probable?:

a. Tromboembolismo pulmonar. b. Síndrome de ansiedad. c. Acidosis láctica. d. Neumonía.

270. Un paciente de 65 años, sin antecedentes de interés, excepto cirrosis hepática por virus C, consulta por disnea de esfuerzo. La gasometría muestra: pH 7.4; pCO2 35 mmHg; PO2 58 mmHg y PO2 (A-a) 48, no mejorando tras oxigenoterapia, La radiografía de tórax es normal. ¿Cuál es entre los siguientes, el trastorno subyacente responsable?:

a. Alteraciones de la-pared torácica. b. Shunt intrapulmonar. c. Enfermedad intersticial pulmonar. d. Enfermedad vascularpulmonar.

271. La afectación pulmonar del lupus eritematoso diseminado presenta las siguientes características EXCEPTO una, Señálela:

a. Puede ser la manifestación inicial de la enfermedad. b. Potencialmente es muy grave. c. Presenta un auto anticuerpo específico. d. Debe descartarse que se trate de una infección. d. No siempre se acompaña de síntomas.

272. Un hombre de 50 años acude al médico con asma, febrícula, astenia de dos meses de evolución e infiltrados en la radiogafía de tórax que son bilaterales, de tipo alveolar y situados periféricamente junto a las axilas. Las únicas alteraciones analíticas son una cifra de 15 % de eosinófilos y una VSG de 100 mm a la primera hora. El diagnóstico más probable, entre los siguientes, es:

a. Granulomatosis de Wegener. b. Aspergilosis broncopulmonar alérgica. c. Neumonía eosinófila crónica. d. Síndrome de Loeffler.

273. ¿Cual de las siguientes aseveraciones relativas a la insuficiencia respiratoria es correcta?:

a. Conceptualmente se considera que existe cuando la PO2 es inferior a 70 mmHg. b. Su mecanismo fisiopatológico más frecuente es una discordancia entre la ventilación y la perfusión. c. Se produce con más frecuencia por una dificultad de la capacidad de difusión alveolo-capilar del 02. d. Conceptualmente se considera que existe cuando la PCO2 es superior a 42 mmHg.

274. En un paciente diagnosticado de neumonía por aspiración, se aisla un bacilo Gram negativo anaerobio en el esputo, ¿Cuál de los siguientes es el tratamiento antimicrobiano de elección?:

Page 200: Cuestionario de Urología

a. Penicilina G. b. Cotrimoxazoa. c. Metronidazoa. d. Eritrol11icina.

275. Una paciente diagnosticada de hipertensión pulmonar primaria en clase funcional n, muestra mínima reducción en la resistencia vascular tras la administración de adenosina intravenosa. ¿Cuál es, entre los siguientes, el tratamiento médico de elección?:

a. Prostaciclina. b. Anticoagulantes. c. Diltiazem. d. Carvedilol.

276. Paciente diagnosticado de esclerosis lateral amiotrófica que tiene afectado el diafragma, presentando crónicamente un cuadro de insuficiencia respiratoria ¿Cuál de las siguientes aseveraciones es correcta?:

a. La diferencia alveolo-arterial de 02 (P AO2-PaO2) suele ser normal. b. La PO2 puede estar en los límites de la normalidad. c. La PCO2 suele ser normal. d. El tratamiento de elección es la administración de oxigeno por máscara tipo Venturi.

277. Ante un cáncer broncopulmonar (no de "células en avena") de 3 cm de diámetro, situado periféricamente, invadiendo la pleura visceral y con afectación hiliar homolateral, la indicación más adecuada, entre las siguientes, es:

a. Radioterapia sobre la afectación biliar y después cirugía. b. Quimioterapia con MVP (mitomicina, vinblastina, cisplatino) y después cirugía. c. Quimioterapia con cisplatino, gemcitabina y vinorelbina y después cirugía. d. Cirugía sin neoadyuvancia (tratamiento prequirúrgico).

278. ¿Cuál, entre las siguientes, es la causa más frecuente de síndrome de la vena cava superior?:

a. Bocio retroesternal. b. Carcinoma papilar de tiroides. c. Carcinoma broncogénico, d. Teratoma mediastínico.

279. Un hombre de 60 años consulta por dolor torácico severo y progresivo que no mejora con analgésicos ni opiáceos. En una radiografía de tórax se observa un importante engrosamiento pleural derecho. ¿Sobre cuál de los siguientes contactos laborales debemos interrogarle más directamente?:

a. Canarios. b. Ovejas. c. Polvo de carbón. d. Amianto.

280. Hombre de 25 años que, hace un mes, sufrió un traumatismo craneal y precisó ventilación mecánica durante cinco días. Fue dado de alta sin secuelas neurológicas pero, dos semanas después, presenta dificultad respiratoria progresiva con episodios de estridor inspiratotio. No ha mejorado con salbutamol inhalado. El diagnóstico más probable, entre los siguientes, es:

a. Tromboembolismo. b. Asma alérgico extrínseco. c. Insuficiencia cardíaca izquierda. d. Estenosis traqueal inflamatoria.

281. La complicación más frecuente en el neumotórax espontáneo es:

a. El derrame hemático asociado. b. La evolución hacia el neumotórax hipertensivo. c. La infección pleural. d. La recurrencia.

282. Un paciente de 57 años, fumador de 20 cigarrillos diarios, con historia de tos y expectoración matutina habitual, consulta por disnea de mínimos esfuerzos y ortopnea de dos almohadas. Exploración física: TA 180/100 mmHg, presión venosa normal, auscultación pulmonar con crepitantes bibasales, auscultación cardíaca rítmica a 120 Ipm con soplo sistólico efectivo en foco aórtico y tercer ruido. ECG: ritmo sinusal y criterios de hipertrofia ventricular izquierda ¿Cuál, de los siguientes, es el diagnóstico más probable?:

a. Insuficiencia cardíaca congestiva en paciente con EPOC. b. Estenosis aórtica en insuficiencia cardíaca. c. Cardiopatía isquémica con disfunción sistólica. d. Cardiopatía hipertensiva en insuficiencia cardíaca.

283. Los grupos de fármacos antihipertensivos más avalados en grandes estudios clínicos, que han demostrado ser capaces de reducir la morbimortalidad, son:

a. IECAs y betabloqueantes. b. Diuréticos y antagonistas del calcio. c. Diuréticos e IECAs. d. Diuréticos y betabloqueantes.

284. En un paciente hipercolesterolémico, que ha sufrido un infarto agudo de miocardio, el objetivo a conseguir, entre los siguientes, es mantener el colesterol:

a. Total < 250 mg/dA.

Page 201: Cuestionario de Urología

b. Total < 230 mgldL. c. HDL> 50 mg/dL y LDL < 130 mg/dL. d. LDL < 100 mglClL.

285. Un paciente de 66 años, fumador de 20 cigarrillos diarios, con criterios clínicos de bronquitis crónica y antecedentes de hiperplasia prostática benigna, gota e hipercolesterolemia, consulta por medias de TA de 168/96 mmHg, a pesar de restricción salina ¿Cuál sería, de los siguientes, el tratamiento de elección para su hipertensión arterial? :

a. Inhibidor de enzima de conversión de la angiotensina. b. Calcioantagonista. c. Betabloqueante. d. Alfabloqueante.

286. ¿Cuál de las siguientes es una indicación preferente de digoxina?:

a. Fibrilación auricular con respuesta ventricular rápida. b. Disfunción diastólica sintomática. c. Taquicardia sinusal con signos de insuficiencia cardíaca. d. Fallo ventricular izquierdo agudo.

287. La angina de pecho se diagnostica por:

a. Ecocardiografía. b. Hemodinámica. c. Electrocardiografía. d. La clínica.

288. Señale en cuál de las siguientes situaciones existe indicación de profilaxis de endocarditis infecciosa antes de una extracción dentaria:

a. Enfermedad coronaria. b. Presencia de marcapasos. c. Coartación aórtica. d. Prolapso mitral sin regurgitación.

289. El colapso X profundo en el pulso venoso es típico de:

a. Ductus arteriosus b. Estenosis mitral. c. Insuficiencia aórtica. d. Taponamiento pericárdico.

290. Un hombre de 34 años, previamente asintomático, se somete a una manipulación por un podólogo con ulterior infección (panadizo) en dedo gordo del pie. Diez días más tarde, comienza con afectación del estado general y fiebre de 38.5 °C, por lo que ingresa. A la exploración se encuentra una insuficiencia aórtica moderada. Tres días después aqueja disnea creciente, fiebre de 39,5 °C, 30 rpm y TA de 130/50 mmHg. Los tres hemocu ltivos tomados a su ingreso son positivos con crecimiento de Staphilococcus aureus. Señale cuál de los siguientes hallazgos exploratorios es menos probables encontrar:

a. Soplo de Austin Fiínt. b. Aumento en la intensidad del primer ruido. c. Tercer ruido. d. Glié sistólico de eyección.

291. Todas las situaciones siguientes quitan valor diagnóstico a la elevación de la CPK en el infarto agudo de miocardio, EXCEPTO una. Señálela:

a. Inyección intramuscular. b. Traumatismo muscular. c. Postcirugía. d. Presencia de taquicardia.

292. Los siguientes ruidos cardíacos ocurren en la diástole EXCEPTO uno. Señálelo:

a. Chasquido de apertura. b. Extratono pericárdico. c. Chasquido de la prótesis mitral de Starr-Edwards. d. Clic de la válvula de Starr-Edwards en posición aórtica.

293. Un soplo diastólico de llenado puede oírse en las siguientes situaciones, EXCEPTO en:

a. La insuficiencia pulmonar. b. La insuficiencia tricúspide. c. El bloqueo completo. d. La insuficiencia mitral.

294. ¿Cuáles son, entre los siguientes, los criterios diagnósticos electrocardiográficos de hemibloqueo de la división súpero-anterior de la rama izquierda?:

a. ÁQRS de -45º ó más negativo. b. ÁQRS de +90º ó más positivo. c. QRS > 0,12 + ÁQRS izquierdo. d. QRS > 0,12 + ÁQRS derecho.

295. ¿Cuáles son los criterios diagnósticos electrocardiográficos de preexcitación tipo WPW?:

a. PR corto. b. PR corto más onda δ.

Page 202: Cuestionario de Urología

c. Onda δ con PR largo. d. Imagen de BRD más PR largo.

296. Señale, entre las siguientes, cuál es la arritmia fmal en la mayoría de los casos de muerte súbita:

a. Bloqueo AV b. Fibrilación ventricular primaria. c. Taquicardia ventricular sostenida - fibrilación ventricular. d. Torsades de pointes - fibrilación ventricular

297. ¿Cuál es, entre las que se citan, la enfermedad asociada más frecuente en la muerte súbita en el joven?:

a. Cardiopatia isquémica. b. Síndrome de WPW. c. Miocardiopatía hipertrófica. d. Va1vulopatía aórtica.

298. ¿Cuál es el significado de que, en un paciente con estenosis aórtica significativa de larga evolución, se compruebe por ecocardiografía una disminución del desnivel de presión transaórtico?:

a. El paciente mejora, probablemente por una ligera rotura de la zona fusionada. b. Se está produciendo fracaso del ventrículo izquierdo. c. El gasto cardíaco ha mejorado. d. El gasto se mantiene igual, pero la aorta, por encima de la válvula, se ha dilatado.

299. Un alumno de cuarto curso de Medicina, adquiere su primer esfigmomanómetro y toma la tensión a toda su familia, observando que su hermano de 15 años tiene una TA de 180/80 mmHg en tres ocasiones distintas. Se trata con mayor probabilidad de una:

a. Hipertensión secundaria a una nefropatía. b. Estenosis aórtica congénita. c. Hipertensión emocional. d. Coartación aórtica.

300. Señale la afirmación correcta respecto a la fibrilación auricular, que acompaña con frecuencia a la enfermedad reumática estenosante de la válvnla mitral:

a. Es exclusivamente molesta. b. Produce una importante disminución del gasto cardíaco, síntomas desagradables y embolias frecuentes. c. No afecta al gasto cardíaco o, si lo hace, es de forma mínima. d. Puede producir embo1ias, pero no son frecuentes.

301. A un paciente de 70 años de edad, con buen estado general y diabetes mellitus tratada con antidiabeticos orales, le ha sido indicada la sustitución valvular aórtica por estenosis severa. Señale la afirmación correcta respecto a la válvula protésica:

a. La indicada sería exclusivamente una biológica por su edad. b. Sería mejor una mecánica, que no necesita anticoagulación. c. Se puede optar por la biológica o por la mecánica, pues en ambas es imprescindible la anticoagulación. d. Se puede optar por la biológica o por la mecánica, si no hay contraindicación para la anticoagulación.

302. Mujer de 18 años, con 6 episodios al mes de cefalea hemicraneal, de un día de duración, pulsátil, acompañada de vómitos, fotofobia y sonofobia y con examen físico normal. El tratamiento preventivo de elección, entre los siguientes, es:

a. Carbamacepina. b. Carbonato de litio. c. Ergotamina. d. Propranolol.

303. ¿Cuál de las siguientes aseveraciones es correcta respecto al estudio genético para asegurar el diagnóstico de Corea de Huntington?:

a. Debe hacerse al paciente exclusivamente. b. Debe hacerse al paciente y sus hermanos. c. Debe hacerse al paciente y sus padres. d. Debe hacerse al paciente, a sus padres ya sus hermanos.

304. Las lesiones difusas o focales del cerebelo determinan, o pueden causar, todos los síntomas o signos siguientes, EXCEPTO uno. Señálelo:

a. Dismetría en extremidades. b. Ataxia de tronco. c. Disdiadococinesia en movimientos alternativos. d. Hipertonía generalizada.

305. Una mujer de 73 años que comienza, de forma insidiosa, con pérdida de memoria progresiva que le impide realizar sus compras habituales y le restringe su capacidad de salir sola a la calle, presenta, asimismo, sintomatología depresiva (tristeza, insomnio, anorexia). Este cuadro es progresivo y persiste durante un año. Señale, entre los siguientes, el diagnóstico MENOS probable:

a. Enfermedad de Alzheimer. b. Tumor del lóbulo temporal izquierdo. c. Depresión grave. d. Enfermedad de Parkinson.

306. Enfermo de 59 años, cuyo único padecimiento es HTA desde hace lO, tratada y controlada con un betabloqueante (atenolol). Hace un año, comienza con temblor progresivo en miembro superior izquierdo. En el examen neurológico las únicas anomalías

Page 203: Cuestionario de Urología

patentes son: el referido temblor, que se observa en reposo y actitud, leve hipertonía y lentitud de movimientos repetitivos en las cuatro extremidades. Elija, entre las siguientes, la explicación más probable para este cuadro:

a. Efecto adverso del atenolol. b. Parkinsonismo arteriosclerótico. c. Parkinsonismo idiopático. d. Proceso expansivo de ganglios basales izquierdos.

307. Un paciente de 28 años consulta por un cuadro, iniciado hace 48 horas, con dolor lumbar y parestesias en cara posterior de muslos y piernas. Progresivamente, imposibilidad para caminar. En la exploración destaca parálisis de miembros inferiores y debilidad proximal de miembros superiores. Exploración sensorial y pares craneales normales. Reflejos miotáticos universalmente abolidos y respuestas plantares ausentes. No refiere antecedentes de interés, salvo gastroenteritis aguda hace 15 días. Señale, entre las siguientes, la actitud más importante en el manejo de este paciente:

a. Vigilancia estrecha de la función respiratoria y ventilación mecánica en caso de deterioro. b. Descompresión quirúrgica inmediata de la médula cervical. c. Resonancia magnética de columna cervical desde C3 hacia abajo. d. Punción lumbar inmediata para descartar hiperproteinorraquia.

308. Un paciente de 40 años acude a consulta porque, desde hace dos o tres días, padece dolores intensos alrededor del ojo derecho, que duran de 30 a 60 minutos y se acompañan de lagrimeo y congestión nasal. Los tiene después del almuerzo, si ingiere bebidas alcohólicas y por la noche. El diagnóstico más probable, entre los siguientes, es:

a. Neuralgia de la primera rama del trigémino. b. Cefalea en racimos. c. Migraña sin aura. d. Neuralgia del glosofaríngeo.

309. Señale la afirmación INCORRECTA. entre las siguientes, acerca de la oftalmoplejía internuclear:

a. Su aparición bilateral en un paciente joven suele deberse a esclerosis múltiple. b. Se debe a una lesión del fascículo longitudinal mediaa. c. Hay una parálisis completa de la motilidad extraocular. d. En el paciente anciano, la causa habitual es la enfermedad cerebrovascular.

310. Ante un paciente con pérdida aguda de fuerza y trastornos de sensibilidad en ambas piernas, uno' de los siguientes diagnósticos NO debe plantearse. señálelo:

a. Infarto en el territorio de la cerebral media. b. Polirradiculoneuropatía aguda. c. Mielitis transversa. d. Infarto medular.

311. Un paciente de 65 años consulta porque, desde hace unos 7-8 años, presenta temblor en ambas manos cuando come, al escribir o al abrocharse los botones de la camisa. El temblor es más acentuado en el lado derecho y ha aumentado de amplitud en los últimos años. Su exploración neurológica es normal, excepto temblor de actitud bilateral, más evidente en el lado derecho. No refiere otros síntomas. No hay antecedentes farmacológicos destacables. La sospecha diagnóstica más verosímil, entre las siguientes, es:

a. Enfermedad de Parkinson. b. Temblor por hipertiroidismo. c. Temblor por un estado de ansiedad d. Temblor esencial.

312. La miastenia gravis es una enfermedad de la unión neuromuscular, cuyo defecto fundamental es:

a. La reducción de la cantidad de acetilcolina en las vesículas de la terminación presináptica. b. La disminución de la actividad de las acetilcolinesterasas. c. Un trastorno de la membrana de la célula muscular. d. La disminución de receptores postsinápticos disponibles.

313. En relación con el cono de presión tentorial o herniación transtentorial, una de las siguientes afirmaciones es FALSA. Señálela:

a. Se produce cuando un proceso expansivo intracraneal de cualquier localización, empuja el lóbulo temporal contra el tronco cerebral. b. El primer signo clínico suele ser la dilatación pupilar ipsilateral a la masa expansiva. c. La dilatación pupilar puede ocurrir a veces en el lado contralateral. d. Es muy improbable que ocurra en ancianos con atrofia cerebral de ambos hemisferios cerebrales.

314. Un niño de 12 años, sufre pérdida de conciencia breve tras caerse de una bicicleta, al llegar a urgencias, está orientado y presenta signos de impacto en región parietal derecha. Dos horas más tarde aqueja cefalea de intensidad rápidamente creciente, seguida de alteración del nivel de conciencia, La causa más probable del deterioro, entre las siguientes, es:

a. Contusión cerebral parietal derecha. b. Hipertensión intracraneal aguda secundaria a edema cerebral vasogénico. c. Hematoma extradural agudo. d. Hidrocefalia

315. El tratamiento más adecuado, entre los siguientes, para un meningioma intracraneal, es la:

a. Extirpación quirúrgica. b. Radioterapia. c. Cirugía seguida de radioterapia. d. Quimioterapia.

Page 204: Cuestionario de Urología

316. El hallazgo de una calcemia de 11.2 mg/dL (normal: 8.5-10.5), confirmada tras repetición, en un varón asintomático de 5O años, fumador y que se hace anualmente una analítica y chequeo general en su empresa, sugiere como patología subyacente más probable entre las siguientes:

a. Carcinoma pu1monar con hiperca1cemia. b. Carcinoma prostático o pancreático. c. Hiperparatiroidismo primario por dos o más adenomas de paratiroides. d. Hiperparatiroidismo primario por adenoma único de paratiroides.

317. ¿Cuál de las siguientes opciones es la indicada ante un nódulo tiroideo de 3 cm de diámetro, gammagráficamente frío, cuya P AAF (punción aspiración con aguja fina) indica proliferación folicular no bien caracterizada con algún depósito de sustancia amiloide? :

a. Administrar levotiroxina y ver si el nódulo desaparece. b. Practicar hemitiroidectornía lo antes posible. c. Hacer análisis de ca1citonina y catecolaminas. d. Practicar tiroidectornía total sin esperar más resultados.

318. Un paciente de 52 años, es diagnosticado de acromegalia y sometido a cirugía transesfenoidal. Seis meses después, en una revisión, la concentración de GH es de 6 y 7 ng/mL (normal < 2) en ayunas. Tras completar el estudio, ¿cuál de los siguientes hechos permite afirmar que está curado?:

a. Ausencia de diabetes, bocio y cefaleas. b. Cifras de GH > 10 ng/rnL tras estimulo con TRH. c. IGF-l por encima de lo normal. d. GH < 2ng/rnL tras sobrecarga oral de glucosa.

319. Un paciente de 55 años, acude al médico por presentar, desde hace tres meses, dolor epigástrico, sin relación con las comidas, que se calma con alcalinos. Las heces han sido negras como la pez. El paciente refiere anorexia y su peso, que habitualmente era de 78 Kg, es ahora de 62. En la analítica encontramos anemia microcítica (Hb 7.5 g/dL), hierro sérico y ferritina anormalmente bajos, albúmina de 2.9 g/dL (normal 2: 3.5), colesterol 125 mg/dL (normal 150-200) y triglicéridos SS mg/dL En la valoración nutricional de este paciente, el dato más importante, entre los siguientes, es:

a. La pérdida de un 20% del peso corporal. b. La presencia de anemia. c. Los valores de la albúmina. d. El hierro y la ferritina bajos.

320. Un paciente de 35 años, bebedor moderado, sufre un accidente de tráfico con fractura de pelvis, fémur, tibia y peroné de pierna izquierda y rotura de bazo que requiere esplenectomía. En la valoración, a los 5 días del ingreso, encontramos: Hb 8.7 g/dL, colesterol 125 mg/dL, triglicéridos 60 mg/dL, SGOT 84 UIL, SGPT 96 UIL (valores normales hasta 40), albúmina 1.6 g/dA. El paciente se encuentra hemodinámicamente estable, con TA de 110/60 mmHg, y en la exploración se observa ligera ictericia. Señale, de las siguientes, la afirmación correcta:

a. La ictericia, las alteraciones en las transaminasas y la hipoalbuminemia, indican la presencia de una hepatopatía crónica. b. La cifra de Hb indica la necesidad de una transfusión urgente de concentrado de hematíes. c. Las cifras de colesterol, triglicéridos y albúmina señalan que el paciente está desnutrido y se debe comenzar tratamiento con nutrición parenteral. d. La cifra de albúmina, en este caso, no sirve para conocer el estado nutricional del paciente.

321. ¿Qué circunstancia, de las siguientes, indica la presencia de TSH elevada, en un paciente con tiroiditis de Hashimoto? :

a. Presencia de un linfoma. b. Coexistencia con una enfermedad de Graves. c. Necesidad de administrarle levotiroxina. d. Presencia de anemia perniciosa.

322. En relación con el carcinoma papilar de tiroides, ¿cuál de las siguientes afirmaciones NO es correcta? :

a. Se propaga frecuentemente por vía hematógena. b. En muchas ocasiones es multicéntrico. c. El pronóstico está en función del tamaño del tumor. d. La afectación ganglionar cervical no se acompaña de mayor mortalidad.

323. Señale cuál es, de los siguientes, el tratamiento más adecuado de un insulinoma benigno de 1.5 cm de diámetro, localizado en el cuerpo del páncreas y lejano al conducto de Wirsung:

a. Duodenopancreatectornía parcial. b. Pancreatectornía distal. c. Embolización del tumor. d. Enucleación del tumor.

324. En un paciente con insuficiencia renal crónica e hiperparatiroidismo secundario, el tratamiento inicial de su hiperparatiroidismo debe incluir:

a. Aumento de la ingesta de fósforo. b. Restricción del aporte de vitamina D. c. Paratiroidectomía total. d. Restricción del fósforo en la dieta.

325. ¿Cuál de las siguientes afirmaciones es correcta respecto a la tirotoxicosis facticia? :

a. Con frecuencia cursa con exoftalmos marcado. b. La TSH sérica está suprimida. c. Es habitual el bocio visible. d. Para su diagnóstico es necesario realizar garnmagrafía.

Page 205: Cuestionario de Urología

326. Respecto al diagnóstico y tratamiento del feocromocitoma, ¿cuál de las siguientes afirmaciones es INCORRECTA?:

a. La fenoxibenzamina debe administrarse al menos los 10 14 días previos a la cirugía. b. Antes de la cirugía, hay que administrar dieta rica en sal para aumentar el volumen plasmático. c. Los bloqueantes beta-adrenérgicos deben administrarse después de haber inducido un bloqueo ex d. La garnmagrafía con metayodobencilguanidina (MIBG), es útil para detectar los de localizaciones atípicas. e. Se debe administrar solución de lugol 10 días antes de la cirugía.

327. Señale cuál de las siguientes afirmaciones es INCORRECTA, respecto a los anticuerpos que pueden encontrarse en el lupus eritematoso generalizado:

a. Los antinucleares (ANA) aparecen en el 95- 98% de los pacientes, pero no son diagnósticos. b. Los anti-Sm son específicos, pero sólo aparecen en el 30% de los casos. c. Los anti-histona son más frecuentes en el inducido por drogas. d. Los antifosfolipídicos están siempre presentes y se relacionan con la actividad de la enfermedad.

328. Una mujer de 78 años presenta dificultad para subir escaleras y levantarse de una silla. Refiere anorexia y pérdida de 10 Kg de peso desde un tiempo que no precisa. Ha estado deprimida desde el fallecimiento de su marido, saliendo poco de casa. A la exploración se observa la debilidad muscular señalada y dolor a la presión de la tibia, calcio sérico 8.8 mg/dL (N: 8.5-10.5) con 4 g/dL de albúmina (normal); fósforo 2.2 mg/dL (normal 2.54r5); fosfatasa alcalina 312 VIL (N < 120). ¿Qué prueba diagnóstica, de las siguientes, hay que seleccionar para orientar a la enferma?:

a. 25 hidroxicolecalciferol. b. 1,25 dihidroxicolecalciferol. c. Prueba de supresión con dexametasona. d. Hormona tireotropa hipofisaria.

329. Un paciente de 22 años, refiere una historia de 15 meses de dolor lumbar bajo y en nalgas, que le despierta a media noche y es más intenso al despertarse por la mañana. Radiológicamente se aprecia ensanchamiento, erosiones y esclerosis en ambas articulaciones sacroilíacas. Señale cuál de las siguientes afirmaciones es INCORRECTA en relación con este cuadro:

a. El factor reumatoide puede ser positivo. b. Puede presentar uveitis aguda. c. Puede presentar monoartritis de rodilla. d. El reposo no suele mejorar estos síntomas.

330. Un hombre de 69 años, sin antecedentes médicos de interés, presenta una monoartritis aguda de rodilla. La radiografía simple de rodilla muestra calcificación de ambos meniscos. ¿Cuál de las siguientes exploraciones complementarias hay que solicitar en primer lugar para establecer la etiología de la artritis? :

a. Determinación de ácido úrico en sangre. b. Artroscopia de rodilla. c. Examen del líquido sinovial con microscopio de luz polarizada. d. Resonancia magnética nuclear de la rodilla.

331. Señale cuál de las siguientes afirmaciones es INCORRECTA respecto a la fiebre mediterránea familiar:

a. Es una enfermedad autosómica recesiva. b. Cursa clínicamente en forma de crisis periódicas c. La colchicina previene las crisis. d. La colchicina no previene la aparición de arniloidosis.

332. Una paciente diabética de 46 años, acude a consulta por presentar dolor intenso, impotencia funcional y tumefacción de tobillo derecho desde el día anterior, con fiebre de 38.4°C. La exploración confirma la presencia de u n derrame. Determinaciones analíticas: 17.300 leucocitos, Hb 13.5 gIL, VSG 44 mm, proteína C reactiva 14 mgldL (normal 1.2), glucemia 330 mg/dL, urea 34 mg/dL, creatinina 0.7 mg/dL, Na 138 y K 3.6. Indique cuál de las siguientes conductas es prioritaria:

a. Iniciar tratamiento antibiótico por vía general. b. Cultivar el líquido sinovial. c. Evacuar el derrame sinovia1 y hacer lavado articular. d. Indicar artrotomía y drenaje articular.

333. Acude a consulta un paciente de 33 años, quejándose de que su rodilla derecha está hinchada, mostrando su exploración un signo de la oleada positivo. No refiere síntomas articulares a ningún otro nivel. Señale cuál de las siguientes pruebas permitirá determinar si se trata de un proceso articular inflamatorio:

a. Examen físico adecuado. b. Rm de la rodilla. c. Observación rnacroscópica del líquido sinovial. d. Determinación del factor reumatoide, ANA y HLA B27.

334. Paciente de 58 años, remitido a la consulta por objetivarse, en una analítica rutinaria, una cifra de ácido úrico de 9 mg/dL (normal en hombres hasta 7 por el método de la urlcasa), siendo el resto de la analítica, incluida función renal, normaL. No refiere antecedentes personales de interés, excepto que es fumador de medio paquete de tabaco al día, No bebe alcohol ni ha padecido episodios de litiasis renal ni dolores articulares La exploración física es normal y la T A de 120/7O mmHg. ¿Qué actitud terapéutica, de las siguientes, es la más indicada? :

a. Iniciar tratamiento con alopurinol. b. Iniciar tratamiento con alopurino1 y colchicina. c. Iniciar tratamiento con fármacos uricosúricos. d. No realizar tratamiento alguno.

335. Hombre de 86 años con historia de dolor en rodilla derecha, A la exploración se observa que la pierna de ese lado es 3 cm más corta que la izquierda y que la tibia está ligeramente arqueada. Las pruebas de laboratorio son normales, excepto una fosfatasa aIcalina de 382 U/L (normal <120 U/L). La radiografía de rodilla muestra cambios degenerativos y la de la tibia constata el arqueamiento y revela una cortical engrosada, con zonas esclerosas y radiotransparentes entremezcladas En la gammagrafia

Page 206: Cuestionario de Urología

ósea captan la rodilla y la tibia. Entre las siguientes, la opción terapéutica más apropiada es:

a. Antiinflamatorios no esteroideos. b. Antiandrógenos. c. A1endronato. d. Calcio y vitamina D.

336. Paciente de 28 años que sufre una descarga eléctrica en su domicilio presentando dolor en hombro derecho, brazo en adución y rotación interna y bloqueo de la rotación externa del mismo. La radiología anteroposterior no parece mostrar alteraciones. ¿Qué lesión, entre las siguientes, es más probable? :

a. Luxación anterior de hombro. b. Luxación posterior de hombro. c. Parálisis del nervio circunflejo. d. Parálisis del nervio supraescapular.

337. Una tumoración ósea de localización metafisaria, vecina de los cartílagos fértiles de los huesos largos, que puede dar signos de bursitis, resorte o compresión vasculonerviosa puede ser:

a. Quiste óseo esencial. b. Quiste óseo aneurismático. c. Fibroma condromixoide. d. Osteocondroma.

338. Paciente de 15 años, que presenta dolor y tumefacción en pierna izquierda, de dos meses de evolución, sin antecedentes traumáticos ni de otro tipo. Radiológicamente se observa reacción perióstica que afecta a la práctica totalidad de la diáfisis del peroné en forma de escamas de cebolla, así como espÍculas radiadas a partir del córtex óseo, en forma de púas de peine. ¿En cuál de los siguientes diagnósticos debe pensarse en primer lugar? :

a. Fibrosarcoma. b. Sarcoma parostal. c. Osteosarcorna. d. Sarcoma de Ewing.

339. Un niño de 8 años, presenta una fractura mínimamente desplazada de cúbito y radio izquierdos, que es tratada mediante inmovilización con yeso cerrado, manteniendo el codo en 90° de flexión. A las pocas horas, es traído a Urgencias debido a que se encuentra irritable, refiriendo dolor en el antebrazo. A la exploración, el relleno capilar es normal. En la radiología no se han producido cambios a nivel del foco de fractura, pero presenta intenso dolor a la extensión pasiva de los dedos. ¿Cuál es, entre las siguientes, la actitud a seguir? :

a. Mantener el miembro elevado y esperar. b. Abrir el yeso 1ongitudinalmente. c. Abrir la porción dista! del yeso. d. Retirar el yeso totalmente y elevar el miembro

340. Un paciente de 78 años, al que se le ha colocado prótesis total de cadera hace seis meses, acude a la consulta aquejando dolor prácticamente constante en dicha cadera, en los últimos tres días. El paciente niega fiebre u otros síntomas. En relación con la eventual infección de la prótesis, señale cuál de las siguientes afirmaciones es FALSA:

a. Estos síntomas pueden ser su única manifestación. b. Los estreptococos en su conjunto son los gérmenes más frecuentemente implicados. c. La forma de diagnosticarla fiab1emente es obtener líquido articular y conseguir un cultivo positivo. d. La garnmagrafía con tecnecio sólo es de cierta utilidad diagnóstica pasados seis o más meses de la intervención.

341. Un paciente de 25 años, acude al hospital por fiebre alta, de 48 horas de evolución, acompañada de escalofríos, quebrantamiento, tos, hemoptisis y dolor pleurítico. Afirma ser usuario de drogas por vía parenteral. ¿Cuál de las siguientes entidades es MENOS probable encontrar?

a. Endocarditis de la válvula tricúspide. b. Absceso en antebrazo. c. Aspergilosis pulmonar. d. Embolismos sépticos pulmonares.

342. Un paciente de 46 años, que viajó a la India hace 4 meses, acude al hospital con fiebre y dolor abdominal de tres días de evolución. A la exploración física, el dolor parece localizarse en el hipocondrio derecho. En la analítica general tan sólo destaca leucocitosis. La ecografia abdominal muestra una imagen de 5 cm en lóbulo hepático izquierdo, sugerente de absceso. ¿Cuál de las siguientes afirmaciones es FALSA? :

a. Una muestra de heces para parásitos y cultivo convencional nos podría dar el diagnóstico. b. Si en el aspirado del absceso el líquido es esposo y acelular, la primera posibilidad diagnóstica es Entamoeba histolytica. c. El tiempo transcurrido desde su viaje a la India, apoya el diagnóstico de absceso hepático amebiano. d. El aspirado es necesario para realizar el diagnóstico.

343. Una mujer de 24 años, presenta un cuadro de fiebre, adenopatías cervicales y un rash máculopapular generalizado. Refiere que, hace tres semanas, mantuvo una relación sexual que ella considera de riesgo para infectarse por el virus de la inmunodeficiencia humana (VIH). Señale, entre las siguientes, la afirmación correcta:

a. Una serología VIH1/NIH2, negativa por el método de ELISA, descarta la infección por VIH. b. El cuadro clínico no es compatible con infección aguda por VIH. c. Si el ELISA es negativo se le puede realizar una carga viral por el método de la PCR. d. El antígeno p24 es menos sensible que la detección de anticuerpo s para el diagnóstico de infección por VIH.

344. Un paciente de 31 años, usuario de drogas por vía parenteral hasta hace 10, es traído a Urgencias por su familia con un cuadro de tres días de evolución de fiebre, agitación y, en las últimas horas, disminución del nivel de conciencia. Los familiares refieren que es portador de anticuerpos antiVIH, pero que nunca ha querido tratarse. Entre otras exploraciones se realiza una TAC craneal, que muestra una imagen que capta contraste en anillo. Señale cuál de las siguientes afirmaciones es correcta:

Page 207: Cuestionario de Urología

a. La primera posibilidad es linfoma cerebral primario. b. No es preciso iniciar tratamiento hasta conocer su cifra de linfocitos CD4. c. Lo más frecuente es que se trate de una infección aguda por Toxoplasma gondii. d. Es adecuado iniciar tratamiento lo antes posible con sulfadiacina - pirimetamina.

345. Un paciente acude a consulta por presentar, desde hace una semana, una erupción máculopapular que afecta fundamentalmente a tronco, muslos y palmas, de forma simétrica. Las lesiones son rojas claras. Está afebril, pero asténico, con dolor de garganta y cefalea leve. Tras realizarle, entre otras pruebas, RPR y FTAABS, ambas son positivas en suero, diagnosticándose de sífilis. ¿Cuál de las siguientes afirmaciones es correcta? :

a. Se trata de una sífilis primaria. b. El tratamiento de elección es penicilina G benzatina. c. En todos los casos de sífilis, se debe realizar punción lumbar. d. Si el paciente es VIH positivo, la actitud no cambia.

346. Un hombre de 28 años, previamente sano, acude a consulta por presentar exudado blanquecino por uretra, sin fiebre y con importante disuria. Se le toma muestra del exudado y se realiza una tinción de Gram. Señale, entre las siguientes, la afirmación correcta:

a. La tinción de Gram del exudado es poco sensible en la detección de Neisseria gonorrhoeae. b. La muestra del exudado debe tomarse después de la micción. c. La presencia de diplococos Gram negativos extracelularmente, es menos predictiva de gonorrea que el observarIos en el interior de los polimorfonucleares. d. Si en la tinción de Gram se observan diplococos Gram negativos intracelulares, se excluye el diagnóstico de uretritis no gonocócica.

347. Un paciente indigente y alcohólico, de 68 años, ingresa por tuberculosis pulmonar. Señale, entre las siguientes, la afirmación FALSA:

a. Se debe iniciar tratamiento con cuatro drogas hasta tener el antibiograma. b. Hay que evaluar su situación hepática antes del tratamiento. c. Al alta, debería realizar un tratamiento directamente observado. d. El tratamiento es suficiente con tres meses de ingreso hospitalario.

348. Una paciente de 17 años, que estuvo de excursión por el campo hace algunas semanas, presenta fiebre, cefalea, mialgias, escalofríos y fotofobia con una lesión en la espalda de 15 cm de diámetro, papulosa, anular y con palidez centraa. La etiología más probable, entre las siguientes, es:

a. Salmonella typhi. b. Streptococcus grupo A c. Borrelia burgdorferi. d. Spirillum minus.

349. Un paciente de 18 años, ingresa por un cuadro de bronconeumonía con afectación en ambos lóbulos inferiores Señale cuál de los siguientes tratamientos NO estaría indicado:

a. Ampicilina-sulbactam. b. Cefuroxima. c. Ceftriaxona + eritromicina. d. Etambutol + c1aritromicina.

350. Ante un paciente de 24 años, que presenta fiebre alta con dolor, inflamación y enrojecimiento testicular izquierdo, ¿cuál de las siguientes afirmaciones es INCORRECTA?:

a. El diagnóstico más probable es el de epididimitis. b. Los patógeno s más frecuentes son Chlamydia trachomatis y Neisseria gonorrhoeae. c. El tratamiento de elección es vancomicina - gentamicina. d. El tratamiento de elección puede ser ofIoxacino.

351. Señale cuál de las siguientes afirmaciones es correcta en relación con el manejo de los hemocultivos en adultos:

a. Se deben practicar en todos los pacientes que acudan a urgencias con fiebre. b. La extracción a través de catéter venoso es más fácil y rentable. c. Se deben extraer, con un intervalo de 30-60 minutos, tres muestras de 10-30 ml. d. Una vez extraídos, se deben guardar en nevera hasta su procesamiento.

352. Una mujer de 40 años, diabética en tratamiento con insulina, ingresa por cetoacidosis. Unos días después de su recuperación metabólica, comienza con fiebre, dolor facial, cefalea, disminución del nivel de conciencia y enrojecimiento nasal, con lesión negruzca en fosa nasal derecha, ¿Cuál de estos diagnósticos es más probable? :

a. Enfermedad de Wegener. b. Endocarditis por S. aureus. c. Infección por Mucor. d. Carcinoma epidermoide.

353. Señale, entre las siguientes, la afirmación INCORRECTA respecto a las alteraciones neurológicas de la carencia de la vitamina B12:

a. Pueden consistir en una degeneración combinada subaguda de la médula espina!. b. Pueden manifestarse como una demencia. c. Pueden ser una de las formas de comienzo del cuadro carencia!. d. Se acompañan siempre de anemia macrocítica.

354. ¿Cuál es, de las siguientes, la variedad de síndrome mielodisplásico con peor pronóstico? :

Page 208: Cuestionario de Urología

a. Anemia refractaria simple. b. Anemia refractaria sideroblástica. c. Anemia refractaria con exceso de blastos. d. Anemia refractaria con exceso de blastos en transformación.

355. Indique cuál de las siguientes afirmaciones es INCORRECTA, en relación con un paciente asintomático que reúne los criterios de una garnmapatía monoclonal de significado incierto:

a. La plasmocitosis medular es inferior al 10%. b. Puede asociarse con un síndrome nefrótico por amiloidosis AL. c. El índice de timidina tritiada es inferior al 1 %. d. La incidencia aumenta con la edad.

356. Señale la afirmación correcta respecto a una leucemia linfoide crónica:

a. Existen blastos en sangre periférica al comienzo de la enfermedad. b. Se produce crisis blástica final en el 5% de los pacientes. c. No siempre hay infiltración linfoide de la médula ósea. d. Es frecuente la ausencia de síntomas al hacer el diagnóstico.

357. Todas las siguientes son causas de eritrocitosis, EXCEPTO una. Señálela:

a. Hemoglobinopatías con alta afinidad para el oxígeno. b. Admínistración exógena de eritropoyetina. c. Síndromes mielodisplásicos. d. Hipernefroma.

358. En una paciente joven, con buen estado general, que presenta una anemia ferropénica que no responde al tratamiento con hierro oral, sin ninguna evidencia de sangrado, ¿cuál de las siguientes pruebas diagnósticas debe practicarse en primer lugar? :

a. Arteriografía abdominal. b. Cuantificación del hierro en orina. c. Estudio de la médula ósea. d. Determinación de anticuerpos anti-endomisio y antigliadina (Ig G e Ig A).

359. ¿Cuál de los siguientes estudios es el más adecuado para conocer, de manera cierta y rápida, si existe ferropenia en un paciente con hepatopatía difusa y ferritina sérica elevada? :

a. Cuantificación repetida de la ferritina sérica. b. Cuantificación de los niveles de protoporfirina eritrocitaria. c. Realizar una biopsia hepática. d. Estudiar el contenido de hierro en la médula ósea.

360. Señale cuál de las siguientes es la combinación de factores pronósticos, presentes en el momento del diagnóstico, que mejor predice la respuesta al tratamiento y la evolución en la leucemia aguda mieloblástica:

a. Edad, subtipo FAB, grado de anemia. b. Edad, antecedente de mielodisplasia, citogenética. c. Citogenética, cifra de leucocitos, grado de anemia. d. Presencia de organomegalias, edad, estado general.

361. Indique qué afirmación, de las siguientes, sobre la enfermedad de Hodgkin es INCORRECTA:

a. La forma histológica más frecuente, en adolescentes y adultos jóvenes, es la esclerosis nodular. b. El estadio lA es curable con radioterapia local. c. El virus de Epstein-Barr se ha relacionado con la variedad de celularidad mixta. d. La laparatomía exploradora está indicada en todos los casos con estadio clínico I y II.

362. En el mieloma múltiple, ¿cuál es el factor pronóstico más importante entre los que se citan?:

a. La cifra de componente M. b. El grado de infiltración medular. c. La sensibilidad al tratamiento citostático. d. La edad.

363. En un paciente de 75 años, sin antecedentes de interés, con anemia (Hb: 9 g/dL) macrocítica VCM: 106 fL, reticulopenia (0,2%) y concentraciones séricas de vitamina B12, ácido fólico y hormonas tiroideas, dentro de límites normales, lo más indicado, entre lo siguiente, es realizar:

a. Gastroscopia con biopsia gástrica, para descartar anemia perniciosa. b. Tratamiento oral con un complejo multivitarnínico, que incluya vitamina B6 y control de la evolución clínica. c. Tratamiento con esteroides, por tratarse de una anemia hemolítica autoinmune. d. Examen morfológico de la medula ósea, para descartar síndrome mielodisplásico.

364. En un paciente de 65 años con lumbalgia, anemia (Hb: 8 g/dL), elevación importante de la VSG (120 rnm/1ª hora), hipercalcemia y componente monoclonal IgG-kappa de 6 g/dL en el proteinograma, ¿cuál de las siguientes afirmaciones es correcta? :

a. Es importante descartar una hernia discal, mediante resonancia magnética nuclear. b. La macroglobulinemia de Waldenstrom es un diagnóstico muy probable. c. El examen morfológico de la médula ósea es fundamental para el diagnóstico. d. Hay que iniciar ya, tratamiento con calcitonina y calcio.

365. Respecto a la hemoglobina, señale cuál de las siguientes es la afirmación correcta:

a. Es, a partir de la pubertad, más baja en varones que en mujeres.

Page 209: Cuestionario de Urología

b. Difiere en los dos sexos en la infancia. c. No varía con la gestación. d. Es más alta en los sujetos que fuman más de una cajetilla diaria.

366. En una poliglobulia, todos los datos siguientes concuerdan con una policitemia vera, EXCEPTO uno. Señálelo:

a. Aumento de la masa de hematíes. b. Esplenomegalia. c. Leucocitosis y trombocitosis. d. Eritropoyetina sérica alta.

367. Todas las nefropatías primitivas, enumeradas a continuación, se manifiestan típicamente como síndrome nefrótico, EXCEPTO una. Señálela:

a. Glomerulonefritis de mínimos cambios. b. Glomeruloesclerosis segmentaria y focaa. c. Glomerulonefritis proliferativa focal. d. Glomerulonefritis membranosa.

368. Un paciente de 58 años, acude al hospital por dolor abdominal y malestar general. En sus antecedentes destaca que se le realizó una angioplastia coronaria hace un mes. Exploración física: TA 190/100 mmHg, livedo reticularis en muslos y varios dedos azules en ambos pies; pulsos pedios conservados. Analítica: creatinina 6.5 mg/dL, leucocitosis con eosinofilia y microhematuria en sedimento urinario. El diagnóstico de sospecha más probable, entre los siguientes, es:

a. Glomerulonefritis proliferativa en relación a endocarditis bacteriana tras intervención intravascular. b. Trombosis de arteria renal principal. c. Necrosis tubular aguda secundaria a contrastes yodados. d. Fracaso renal agudo secundario a ateroembolismo de colesterol.

369. Señale cuál de las siguientes manifestaciones clínicas generales del síndrome urémico es la única que mejora habitualmente con el tratamiento renal sustitutivo:

a. Disfunción sexual. b. Hipertrigliceridemia. c. Prurito. d. Anorexia.

370. Un paciente de 45 años, es remitido a consulta nefrología ante el hallazgo, por su médico de atención primaria, de una insuficiencia renal severa con acidosis metabólica y potasemia de 5 mEq/L (normal 3,5 5,0). Mantiene una vida activa, trabajando en una pollería y no refiere sintomatología alguna, salvo algún episodio de monoartritis ocasionaa. Previamente, había presentado algún cólico nefrítico. En la exploración fisica, destacaba una TA de 165/100 mmHg. Elija, entre las siguientes, la opción prioritaria:

a. Solicitar ecografia renal y completar estudio analítico para establecer si la insuficiencia renal es aguda o crónica. b. Programar ingreso, para prepararle e iniciar tratamiento renal sustitutivo. c. Realizar ingreso urgente, para comenzar diálisis cuantoantes. d. Iniciar dieta hipoproteica pobre en potasio.

371. ¿En cuál de las siguientes enfermedades es excepcional la aparición de nefrocalcinosis medular como complicación?

a. Enfermedad de Bartter. b. Intoxicación por vitamina D. c. Hiperparatiroidismo primario. d. Acidosis tubular proxirnal tipo II.

372. Un paciente de raza negra, de 40 años, es llevado a Urgencias al haber sufrido deterioro del grado de conciencia con crisis epiléptica en su domicilio. Al ingreso, se objetiva una TA de 240/130 mmHg y, en analítica de urgencias, cifras de urea 4 veces el valor normal y creatinina 6 veces el valor normal, leucocitos: 15.000/mm3, Hb: 7 gIdL y plaquetas: 60.000/mmc. En orina Na bajo y K elevado, con microhematuria y cilindruria en el sedimento. La exploración de fondo de ojo evidencia hemorragias y exudados difusos y edema de papila, ¿Qué exploración, de las siguientes, hay que utilizar para llegar a un diagnóstico de certeza? :

a. Frotis de sangre periférica. b. Ecografía renal. c. TAC craneal. d. Biopsia renal.

373. Respecto a la anemia de la insuficiencia renal crónica, todas las afirmaciones siguientes son correctas EXCEPTO una. Señálela:

a. Es normocrómica normocítica. b. Se trata eficazmente con eritropoyetina humana recombinante. c. A menudo, requiere para su corrección la administración de hierro oral o parenteral. d. No son necesarios suplementos vitamínicos para su manejo adecuado.

374. En un paciente diabético tipo I, de 28 años, al que se le detecta por primera vez en una analítica de primera orina de la mañana microalbuminuria, señale la actuación siguiente que se debe realizar:

a. Iniciar tratamiento con IECAs para enlentecer la progresión a nefropatía diabética establecida. b. Confumar este hallazgo, repitiendo la determinación dos veces en los tres meses siguientes. c. Remitir a neCrología para estudio de confmnación de nefropatía diabética. d. Repetir periódicamente esta determinación, como mínimo cada tres meses, por la progresión inminente a macroalbuminuria.

375. Niño de 5 años, con gastroenteritis de tres días de evolución. En Urgencias se aprecia anemia intensa con hematíes fragmentados, hipertensión severa y elevación de la creatinina sérica, ¿Qué diagnóstico, de los siguientes, es el más probable? :

a. Deplección hidrosalina. b. Fracaso renal agudo por necrosis tubular.

Page 210: Cuestionario de Urología

c. Síndrome urémico hemolítico. d. Glomerulonefritis extracapilar.

376. Ante la sospecha de una torsión de testículo, ¿cual es, entre las siguientes, la prueba diagnóstica de elección? :

a. Ecografía-doppler. b. Garnmagrafía isotópica. c. Ecografía. d. Radiografía escrotaa.

377. Señale, entre las siguientes, la afirmación correcta respecto a la sensibilidad de la ecografía en el diagnóstico de la litiasis renal:

a. Es de alrededor del 0.95 para los cálculos vesicales. b. Para los cálculos ureterales, es mayor cuando se localizan en la porción sacroilíaca del uréter. c. No varía con el tamaño de los cálculos. d. Varía según la composición de los cálculos.

378. Un paciente de 62, años con alteración de la función renal y crisis de hematuria, presenta una masa abdominal palpable en flanco derecho. Se le realiza una TAC, detectándose una masa de carácter sólido, de 8 cm de diámetro, en riñón derecho. En la anamnesis, destaca que el paciente es fumador de 35 cigarrillos al día, ¿ Cuál es, entre los siguientes, el diagnóstico de presunción más probable? :

a. Nefroblastoma. b. Liposarcoma. c. Angiomiolipoma. d. Adenocarcinoma.

379. Un paciente de 20 años, acude a consulta por ausencia del testículo izquierdo. Según refiere, el testículo nunca ha sido palpable en escroto. En la exploración fisica, el testículo derecho es normal y no se palpa el teste izquierdo en el conducto inguinaa. En la TC practicada, se aprecia un rudimento testicular intraabdominal cercano al anillo inguinal interno. ¿Cuál es, entre las siguientes, la conducta más adecuada? :

a. Seguimiento periódico con T AC y alfafetoproteína. b. Exploración quirúrgica y descenso del teste a bolsa escrotal. c. Exploración quirúrgica y extirpación del testículo. d. Colocación de prótesis testicular si el paciente lo desea.

380. Una mujer de 23 años, consulta por presentar desde hace varios meses, unas pápulas y vesículas agrupadas, localizadas en codos, rodillas, nuca y glúteos. La realización de una inmunofluorescencia directa objetiva depósitos granulares de IgA en las papilas dérmicaso ¿Cuál de los siguientes enunciados es cierto en esta paciente? :

a. Debe aplicarse una crema acaricida (lindane, permetrina) todas las noches. b. El tratamiento de elección es un corticoide tópico. c. Es aconsejable que realice una dieta sin gluten. d. El mejor tratamiento es el yoduro potásico.

381. Una mujer joven, consulta porque, desde hace unas dos semanas, le han aparecido de forma eruptiva unas máculas y placas eritematosas en el tronco. Refiere hubo una lesión más grande que precedió a las demás. Las lesiones presentan una descamación fina en la periferia y son discretamente pruriginosas. No existe afectación palmo-plantar. La serología luética es negativa, ¿Qué tipo de pitiriasis, entre las siguientes, es el más probable? :

a. P. versicolor. b. P. rubra pilaris. c. P. liquenoide y variolíforme aguda d. P. rosada.

382. Un hombre de 60 años que presenta unas placas eritematosas en el tronco, es diagnosticado de micosis fungoide. Con dicho diagnóstico entenderemos que el paciente se halla afecto de:

a. Una variante de psoriasis. b. Un linfoma no-Hodgkin de fenotipo T. c. Una infección por el hongo Mycosporumfungoides. d. Un linfoma no-Hodgkin de fenotipo B.

383. La púrpura de Schoenlein-Henoch suele cursar con las siguientes alteraciones, EXCEPTO:

a. Artritis. b. Lesiones cutáneas purpúricas. c. Síntomas neurológicos. d. Síntomas gastroidiestinales.

384. ¿En cuál de los siguientes procesos está absolutamente CONTRAINDICADO el etretinato? :

a. Inmunosupresión. b. Diabetes tipo ll. c. Obesidad mórbida. d. Embarazo.

385. ¿Cuál de las siguientes sustancias, es la que con mayor frecuencia origina dermatitis de contacto alérgicas en los albañiles? :

a. Parafenilenodiamina. b. Thiomersal. c. Dicromato potásico. d. Sulfato de níquea.

Page 211: Cuestionario de Urología

386. Un paciente de 42 años, acude a la consulta por presentar ptosis palpebral bilateral de larga evolución y pérdida de visión debida a catarata bilateral de prodominio corticaa. ¿Cuál de las siguientes opciones es más útil para establecer una hipótesis diagnóstica? :

a. Realizar funduscopia. b. Realizar tonometría ocular. c. Consultar al endocrinólogo. d. Observar cómo nos estrecha la mano.

387. Señale, entre las siguientes, la afirmación correcta respecto a la obstrucción congénita de las vías lacrimales:

a. Se trata por medio de un sondaje lacrimal. b. Con frecuencia da lugar a una blefaritis. c. Es una enfermedad hereditaria. d. Se da con frecuencia en niños con peso elevado

388. Un paciente de 64 años fue intervenido de cataratas mediante facoemulsificación e implantación de lente intraocular endosacular plegable, hace 2 años en ojo derecho (OD). y 3 en ojo izquierdo ( O.I ). Acude a consulta por pérdida de visión progresiva en O.I . de 6 meses de evolución. Agudeza visual actual corregida O.D.: 0,8/ O.I .:0,3 . ¿Cuál es, de los siguientes, el diagnóstico más probable? :

a. Edema macular cistoide post-extracción de cristalino. b. Opacificación de la cápsula posterior. c. Endoftalmitis crónica. d. Desprendimiento de retina regmatógeno.

389. Ante un paciente que acude a la consulta de Oftalmología, por presentar un exoftalmos de comienzo muy brusco, ¿cuál, de los siguientes, es el diagnóstico más probable? :

a. Enfennedad de Basedow. b. Pseudotumor inflamatorio. c. Miopía alta. d. Hemorragia orbitaria.

390. Ante una parálisis facial y un ojo rojo, hay que pensar en:

a. Queratitis por lagoftalmos. b. Conjuntivitis aguda. c. Glaucoma. d. Fístula carótido-cavernosa.

391. La exploración campimétrica de un paciente de 56 años, que presenta cefalea de 2 meses de evolución, muestra una cuadrantanopsia bitemporal superior. El diagnóstico más probable, entre los siguientes, es:

a. Craneofaringioma b. Adenoma de hipófisis. c. Meningioma supraselar. d. Neuropatía óptica isquémica arterítica.

392. Señale, de las siguientes, la afirmación FALSA respecto a la hipertensión ocular (HTO) y al glaucoma:

a. La HTO se define glaucoma, cuando se constatan lecturas repetidas de presión intraocular > 21 mmHg durante más de 6 meses, independientemente del estado de la papila y del campo visual. b. La trabeculoplastia son láser se realiza con láser argón y no con láser YAG. c. El procedimiento más común en el tratamiento del glaucoma es la trabeculectomía. d. El glaucoma crónico de ángulo abierto es casi siempre indoloro.

393. En relación al edema de Reinke, señale cuál de las siguientes respuestas es correcta:

a. Es una forma de laringitis aguda. b. Se manifiesta por disfagia. c. No influye el abuso vocal. d. Se desarrolla entre el epitelio y el músculo vocal.

394. En una mujer de 45 años, que presenta, desde hace 3 meses, hipoacusia y ruidos continuos en oído derecho y crisis vertiginosas que se acompañan de cortejo vegetativo, ¿cuál, de los siguientes, es el diagnóstico más probable? :

a. Síndrome cervical. b. Hipotensión ortostática. c. Epilepsia de lóbulo temporaa. d. Hidrops endolinfático.

395. Un paciente de 68 años, consulta por hipoacusia bilateral, que ha ido progresando en los últimos años. Refiere oír pero no entender, especialmente cuando hay ruido ambiente. La audiometría tonal, muestra una hipoacusia neurosensorial bilateral y casi simétrica para ambos oídos, por afectación de los tonos agudos. Se comprueba una disminución de la inteligibilidad en el audiograma verbal ¿Cuál es, de los siguientes, el diagnóstico más probable? :

a. Enfennedad de Méniere bilateraa. b. Presbiacusia. c. Laberintitis bilateral. d. Colesteatoma bilateral.

396. Señale cuál de las siguientes observaciones clínicas sobre la parálisis facial periférica, es INCORRECTA:

a. Puede no ser completa. b. Puede afectarse la lacrimación.

Page 212: Cuestionario de Urología

c. Puede afectarse la rama frontal contralateraa. d. Puede afectarse el sentido del gusto.

397. La disfonía debida a nódulos en ambas cuerdas vocales, debe ser considerada como una lesión:

a. Precancerosa. b. Funcionaa. c. lnflamatoria. d. Degenerativa.

398. Señale cuál de las siguientes afirmaciones relativas a las infecciones óticas, sus complicaciones y su tratamiento quirúrgico es correcta:

a. Las mastoiditis son frecuentemente consecuencia de las otitis medias. b. Las otitis medias del adulto se propagan frecuentemente por la fisura petroescamosa. c. El nervio facial nunca se lesiona al operar las mastoiditis. d. No se suele realizar la mastoidectomía a través del conducto auditivo externo.

399. Un paciente de 65 años, sin antecedentes psiquiátricos previos, presenta desde hace un mes aproximadamente, un cuadro clínico de inquietud, desasosiego, insomnio y verbalizaciones de tipo hipocondriaco, que la propia familia califica de absurdas, del tipo de: "no puedo comer porque no tengo estómago", llegando en alguna ocasión a expresar su temor a estar muerto. El diagnóstico de presunción, entre los siguientes, es:

a. Psicosis esquizofrénica. b. Depresión psicótica. c. Inicio de una demencia. d. Neurosis hipocondríaca

400. ¿Cuál de estas manifestaciones NO corresponde a la anorexia nerviosa? :

a. Comienzo siempre después de la pubertad. b. Pérdida significativa de peso (índice de Quetelet menor de 17.5). c. La pérdida de peso está originada por el propio enfermo a través de mecanismos diversos. d. Distorsión de la imagen corporaa.

401. Un enfermo, manifiesta sensación de dificultad respiratoria, mareo con sensación de inestabilidad o desfallecimiento, palpitaciones, temblor, sudoración, sensación de ahogo, náuseas, despersonalización, parestesias, sofocación que se alterna con escalofríos, opresión precordial, miedo a morir o a perder el control sobre sí mismo. ¿Cuál es, entre los siguientes, el diagnóstico más probable? :

a. Brote esquizofrénico. b. Depresión endógena. c. Trastorno obsesivo compulsivo. d. Crisis de angustia.

402. Un paciente diagnosticado de un primer episodio de un trastorno esquizofreniforme presenta una buena respuesta al tratamiento psicofarmacológico con antipsicóticos en el plazo de un mes, objetivándose una remisión total de la sintomatología psicótica. Llegados a este punto, la actitud más recomendable, entre las siguientes, es:

a. Suspender el tratamiento psicofarmacológico, puesto que el episodio ha cedido y el riesgo de recidiva es bajo. b. Suspender el tratamiento psicofarmacológico e iniciar un tratamiento psicoterapeútico específico que minimice el riesgo de recidiva a medio o largo plazo. c. Buscar la mínima dosis eficaz de antipsicótico que mantenga al paciente asintomático y mantener el tratamiento durante un mínimo de un año, momento en el cual se puede plantear su interrupción gradual. d. Buscar la mínima dosis eficaz de antipsicótico que mantenga al paciente asintomático y mantener el tratamiento de forma indefinida, puesto que la esquizofrenia es una enfermedad crónica y muy desestructurante.

403. Señale cuál de las siguientes características NO es propia del trastorno límite de la personalidad:

a. Impulsividad en, al menos, dos áreas, por ej.: accesos incontrolables de ira, gastos excesivos, abuso de sustancias tóxicas, conducción temeraria, etc. b. Falta de remordimientos, como lo demuestra la indiferencia o la justificación de haber dañado o maltratado a otros. c. Sentimiento crórtic.b de vacío, desplegando un esfuerzo frenético por evitÁi'el abandono real o imaginario. d. Ideación paranoide transitoria o síntomas disociativos graves relacionados con el estrés.

404. Una de las siguientes afirmaciones, respecto al síndrome confusional agudo o delirium, NO es correcta. Señálela:

a. Es un cuadro muy frecuente que afecta al 30% de los pacientes mayores de 65 años ingresados en un hospital general. b. El comienzo del cuadro es brusco y su duración habitualmente es inferior a un mes. c. Los síntomas empeoran durante la noche, apareciendo el ritmo sueño-vigilia claramente desestructurado. d. Los síntomas delirantes transitorios y las alucinaciones visuales son excepcionales.

405. Enfermo de 81 años, con larga historia de psicosis bipolar, que venía siendo tratado con 30 mg/día de oxacepam. Ante la aparición gradual de síntomas depresivos, se le prescriben 20 mg/día de paroxetina. Una semana después de iniciar este tratamiento, se observa mejoría de algunos síntomas depresivos, pero el paciente se queja de una gran inquietud interna, que se agrava al tener que acostarse o sentarse quieto; va y viene de un lado a otro todo el día ¿Cómo hay que interpretar los síntomas descritos y qué conducta hay que seguir? :

a. Los síntomas son premonitorio s de la inversión del ciclo y hay que agregar carbonato de litio. b. Son síntomas propios de un cuadro ansioso que se asocia con frecuencia a los cuadros depresivos. Es mejor esperar y observar la evolución. c. Es un cuadro de acatisia y debemos suprimir la paroxetina. d. Pueden ser síntomas de un cuadro orgánico de origen vascular. Serían necesarias exploraciones neurológicas complementarias.

406. ¿Cuál de los siguientes síntomas NO es típico de la esquizofrenia paranoide? :

Page 213: Cuestionario de Urología

a. Ideas delirantes de tipo persecutorio. b. Respuestas paradójicas. c. Alucinaciones auditivas en forma de voces. d. Alucinaciones visuales en forma de rnicrozoopsias.

407. El tratamiento electroconvulsivo (TEC) sigue siendo el tratamiento de elección (primera alternativa) en:

a. La psicosis maníaco-depresiva. b. La esquizofrenia paranoide. c. La depresión mayor. d. La depresión delirante con gran agitación y riesgo suicida.

408. Un niño de 6 años, asintomático cardiológicamente, presenta un soplo sistólico eyectivo en borde esternal izquierdo con un desdoblamiento fijo del segundo tono y, en el ECG, un patrón rSR' en precordiales derechas. El diagnóstico más probable, entre los siguientes, es:

a. Comunicación interventricular. b. Comunicación interauricular tipo ostium secundum. c. Soplo inocente. d. Ductus arterioso persistente.

409. Lactante de 10 meses, que comienza hace 3 días con fiebre de hasta 38.7°C, vómitos y rechazo de l as tomas. No presenta síntomas catarrales. En la exploración no se objetiva ningún foco infeccioso. En el hemograma existe una leucocitosis con desviación izquierda y la proteína C reactiva muestra unos valores 10 veces por encima de lo normal. En el análisis de orina hay nitritos y leucocitos positivos, con 15-20 leucocitos por campo en sedimento urinario. ¿Qué actitud de las siguientes hay que adoptar en este momento? :

a. Diagnosticar una infección urinaria y administrar antibióticos orales durante 10 días. b. Realizar una punción lumbar para estudio del líquido cefalorraquídeo. c. Recoger un urocultivo y comenzar tratamiento antibiótico hasta ver sus resultados. d. Tratar con antitérrnicos y ver evolución sin hacer ninguna prueba más de momento.

410. Niña de 24 meses, que es traída a consulta por presentar desde el día anterior dificultad respiratoria que ha ido en aumento, con taquipnea y, según la madre, pitidos con la respiración. No ha presentado fiebre ni síntomas catarrales. A la exploración presenta, en el hemitórax derecho, hipoventilación y sibilancias diseminadas. ¿Qué exploración, entre las siguientes, hay que solicitar en primer lugar? :

a. Radiografía lateral de tórax. b. Electrolitos en sudor. c. Hemograma con fórmula leucocitaria. d. Radiografía anteroposterior de tórax en inspiración y en espiración.

411. En un niño sano de 3 meses, el valor de la hemoglobina es:

a. 13-17 g/dL b. 9,5-14 g/dL c. 15 g/dL d. < 9 g/dL

412. Niño de 3 años, que comienza con síntomas catarrales y, unas horas después, presenta un episodio de pérdida de conocimiento, movimientos tonicoclónicos de extremidades y revulsión ocular, de una duración aproximada de 2 minutos. A la exploración presenta T 39°C, exploración neurológic a normal, excepto tendencia al sueño, faringe muy congestiva con amígdalas hipertróficas y tímpanos hiperémicos. ¿Qué actitud, entre las siguientes, hay que adoptar en ese momento? :

a. Iniciar tratamiento con antitérrnicos y vigilancia postenor. b. Realizar una punción lumbar para análisis del líquido cefalorraquídeo. c. Solicitar un electroencefalograma urgente. d. Iniciar tratamiento con diazepán intravenoso.

413. Recién nacido de 36 semanas de edad gestacional, con 7 días de vida, que desde el tercero presenta una ictericia que ha ido en aumento. La madre es primigesta, tiene un grupo sanguíneo A (Rh negativo) y el niño es O (Rh positivo). El 7° día tiene una bilirrubina total de 12 mg/dL, a expensas de la fracción indirecta, El niño tiene buen estado general y los valores de hematocrito, hemoglobina y reticulocitos son normales. ¿En quécausa de hiperbilirrubinemia, entre las siguientes, hay que pensar en primer lugar? :

a. Ictericia fisiológica. b. Hepatitis neonatal. c. Enfermedad hemolítica Rh. d. Atresia de vías biliares.

414. Lactante de 5 meses, que es traído a consulta por presentar desde 3 días antes, fiebre, rinorrea acuosa y esto mudos, comenzando el día de la consulta con tos y dificultad respiratoria. En la exploración presenta taquipnea, tiraje subcostal, alargamiento de la espiración, y crepitantes y sibilancias diseminados. ¿Qué prueba, de las siguientes, hay que solicitar para determinar la etiología?

a. Hemograma. b. Radiografía de tórax. c. Electrolitos en sudor. d. Búsqueda de virus respiratorio sincitial en exudado nasofaríngeo.

415. Niño de 4 años que, dos semanas después de presentar un proceso catarral, comienza con dolor abdominal de tipo cólico, lesiones eritematosas puntiformes en extremidades inferiores, que se han hecho purpúricas en unas horas, y dolor con tumefacción en tobillo derecho. ¿Cuál es el diagnóstico más probable entre los siguientes? :

a. Púrpura trombocitopénica idiopática.

Page 214: Cuestionario de Urología

b. Púrpura anafilactoide. c. Trombastenia. d. Artritis reumatoide juveniL

416. La hipoglucemia del recién nacido, hijo de madre diabética, se presenta en:

a. Las primeras 6 horas. b. Las primeras 24 horas. c. La primera semana. d. El primer mes.

417. ¿Cuál de las siguientes afirmaciones es FALSA respecto a la ictericia fisiológica del recién nacido? :

a. Es más intensa entre el 3° y 4° día de vida. b. La cifra de bilirribina total suele ser inferior a 13 mg/dA. c. Dura alrededor de un mes. d. El estado general es bueno.

418. ¿Cuál de las siguientes vacunas puede administrarse a la madre durante el embarazo? :

a. Sarampión. b. Poliomielitis. c. Tétanos. d. Rubéola.

419. ¿Cuál de los siguientes parámetros NO está incluido en la prueba de Apgar? :

a. Frecuencia cardíaca. b. Tensión arterial. c. Tono. d. Reflejos.

420. ¿Cuál de estos fármacos es esencial en la parada cardíaca en Pediatría? :

a. Adrenalina. b. Atropina. c. Calcio. d. Dopamina.

421. Sobre la Leishmaniasis visceral o Kala-azar en el niño es FALSO que:

a. Clínicamente se caracteriza por fiebre, esplenomegalia y linfadenopatías b. Afecta únicamente a niños inmunocomprometidos. c. Analíticamente se acompaña de pancitopenia. d. Para el diagnóstico es útil la punción de médula ósea.

422. La endocarditis bacteriana es excepcional en:

a. Cardiopatías reumáticas. b. Prolapso de la válvula mitral. c. Prótesis cardíaca. d. Comunicación interauricular tipo ostium secumdum.

423. La técnica quirúrgica para el tratamiento del megacolon agangliónico en el niño, consiste fundamentalmente en:

a. Resecar una mínima parte del colon. b. Realizar una plicatura colorrectal. c. Hacer una colóitornía proximal. d. Realizar una resección total del segmento agangliónico.

425. La indicación quirúrgica es obligada en un paciente afectado de enterocolitis necrotizante cuando presente uno de estos signos clínicos:

a. Heces mucosanguinolentas. b. Vómitos biliosos. c. Distensión abdominal. d. Neumoperitoneo.

426. La falta de desarrollo del neuroblasto en el embrión y feto, dará lugar a la ausencia de los plexos mientéricos de Meissner y Auerbach, hallazgos histológicos que están presentes en uno de estos procesos:

a. Invaginación intestinaa. b. Íleo meconial. c. Estreñimiento crónico. d. Megacolon agangliónico.

427. Paciente con hipertensión crónica bien controlada y gestación de 8 semanas, en tratamiento con IECAs. Entre las siguientes, la conducta más adecuada es:

a. Mantener el tratamiento dado el buen control tensionaa. b. Mantener el tratamiento y asociar alfametildopa para disminuir los riesgos fetales de los IECAs. c. Mantener el tratamiento y asociar hidralazina para disminuir los riesgos matemos de los IECAs. d. Suspender los IECAs, dado el riesgo que presentan para el feto.

428. Paciente diagnosticada de síndrome de Marfan, con un diámetro en raíz aórtica de 6 mm, que plantea la posibilidad de quedarse embarazada. Se le debe informar que:

Page 215: Cuestionario de Urología

a. El riesgo de aborto es del 60-80%. b. El embarazo carece de riesgo si se aplica tratamiento de inicio con betabloqueantes y anticoagulantes. c. El riesgo de muerte materna es del 25-50%. d. No tiene ningún riesgo con adecuado control ecocardiográfico y electrocardiográfico.

429. Señale cuál de los siguientes hechos se considera una modificación fisiológica como consecuencia del embarazo:

a. Aumento de la motilidad intestinaa. b. Incremento del filtrado glomerular. c. Disminución de la secreción de prolactina. d. Disminución del volumen plasmático.

430. En relación con los procesos que se mencionan, señale qué vacuna está CONTRAINDICADA durante el embarazo:

a. Fiebre tifoidea. b. Tétanos. c. Rubéola. d. Hepatitis B.

431. Señale la afirmación correcta respecto a los fármacos betamiméticos durante la gestación:

a. Son el tratamiento de elección en los casos de amenaza de aborto. b. Constituyen la primera opción terapéutica para la relajación uterina en los casos de desprendimiento prematuro de placenta. c. Tienen su principal indicación en el tratamiento de la amenaza de parto prematuro. d. Se utilizan preferentemente en la inducción del parto del embarazo a término.

432. Paciente que presenta amenorrea de 7 semanas, asintomática, sin evidencia, mediante ecografia vaginal, de útero ocupado. Se determina la fracción beta de la gonadotropina coriónica humana, obteniéndose una cifra de 2500 mU/mA. Ante estos datos, en primer lugar habrá que pensar en:

a. Gestación de evolución normal correspondiente a la amenorrea. b. Gestación normal con menor tiempo de evolución del correspondiente a su amenorrea. c. Gestación ectópica de evolución asintomática. d. Aborto precoz completo con expulsión total de restos ovulares intrauterinos. .

433. Paciente de 38 años, con nódulo mamario indoloro, de bordes imprecisos. La mamografia revela imagen nodular, con espículas en todos sus márgenes, y 10 microcalcificaciones finas, agrupadas en su interior. El diagnóstico más probable, entre los que se citan, es:

a. Fibroadenoma. b. Quiste. c. Displasia mamaria. d. Carcinoma.

434. Una mujer diagnosticada de lupns eritematoso sistémico (LES), desea quedar embarazada y solicita información. Señale, entre las siguientes, la contestación INCORRECTA:

a. El LES es contraindicación absoluta de gestación. b. La gestación puede desencadenar un brote de la enfermedad. c. La gestación en el LES tiene un mayor riesgo de aborto. d. La gestación contraindica el tratamiento con ciclofosfamida. .

435. Señale cuál de las siguientes afirmaciones, respecto a las vulvovaginitis, es INCORRECTA:

a. Es un síndrome clínico común que se diagnostica en más del 25% de las mujeres en las consultas de enfermedades de transmisión sexual. b. La etiología más frecuente es por Cándida albicans. c. El sobrecrecimiento de hongos está favorecido por niveles altos de estrógenos. d. El pH vaginal en la vulvovaginitis por Candida sp es muy elevado.

436. Una mujer de 20 años, embarazada de 28 semanas y que ha presentado hiperemesis gravídica, es traída al hospital porque desde hace una semana, se han intensificado los vómitos, no tolera la ingesta oral de alimentos y presenta un cuadro progresivo de desorientación, apatía, somnolencia, alteraciones visuales y dificultad para la deambulación. En la exploración, destaca deterioro del nivel de conciencia, oftalmoplejía, nistagmo y ataxia severa. El diaguóstico más probable, entre los siguientes, es:

a. Preeclampsia. b. Hipopotasemia por vómitos. c. Mielinosis centropontina. d. Encefalopatía de Wernicke.

437. ¿Cuál de los siguientes hechos, se admite que es producido por los anticonceptivos hormonales combinados? :

a. Disminución del riesgo de cáncer de cérvix. b. Aumento del riesgo de cáncer de ovario. c. Disminución del riesgo de adenocarcinoma de endometrio. d. Disminución del riesgo de accidentes tromboembólicos.

438. En el tratamiento de las convulsiones de la eclampsia, el fármaco de elección, entre los siguientes, es:

a. Sulfato de magnesio. b. Diazepan. c. Fenitoína. d. Hidrato de cloral.

439. ¿Cuál de los siguientes métodos permite hacer el diagnóstico del embarazo más precozmente? :

Page 216: Cuestionario de Urología

a. El tacto vaginal. b. La determinación de gonadotropina coriónica en sangre. c. La ecografía transvaginal. d. La radiografía.

440. Para el diagnóstico prenatal de la trisomía 21, el método que permite el diagnóstico más precoz es la:

a. Amniocentesis precoz. b. Biopsia corial. c. Cordocentesis. d. Determinación de alfafetoproteína en sangre materna.

441. En un estudio de casos y controles, el riesgo de infarto de miocardio (1M), en los individuos con hipertensión arterial (HTA), es de 1.5 con un error tipo 1 (a) de 0.06. La interpretación de estos datos es:

a. Los individuos con HTA tienen un riesgo superior de 1M comparado con los no hipertensos, con una probabilidad de error de aproximadamente 1 en cada 16 veces. b. Los individuos con HTA tienen un aumento significativo, de al menos el 50%, en el riesgo de 1M. c. Estos resultados nos permiten rechazar la hipótesis nula de falta de asociación entre la HTA yellM. d. El aumento en el tamaño de la muestra del estudio no es necesario para incrementar la precisión de los resultados.

442. ¿Cuál de las siguientes afirmaciones es la que mejor define el estado de portador de una enfermedad infecciosa? :

a. Individuo infectado por el agente infeccioso, aunque no tiene los síntomas clínicos de la enfermedad y no la transmite. b. Individuo infectado por un agente infeccioso, aunque no presenta ni presentará síntomas de la enfermedad, pero es capaz de transmitir la infección. c. Individuo infectado que es capaz de transmitir el agente infeccioso y presentar síntomas de la enfermedad en un momento dado. d. Se caracterizan por tener niveles de anticuerpos específicos contra el patógeno, superiores a los de los enfermos.

443. ¿Cuál de los siguientes, es el denominador en la tasa de mortalidad materna, en la población estudiada? :

a. Número de nacidos vivos por año. b. Número de mujeres embarazadas por año. c. Número de mujeres en edad fértil por año. d. Número total de nacimientos (vivos más muertos) por año.

444. El mejor tipo de estudio epidemiológico para evaluar la asociación causal entre un factor de riesgo y una enfermedad determinada, es el:

a. Ecológico. b. Címico aleatorio. c. Transversal. d. De cohortes.

445. ¿Cuál de las afirmaciones siguientes, es cieda con respecto a los ensayos de campo? :

a. Los sesgos de selección ocurren frecuentemente. b. Por su diseño, y a diferencia de otros estudios epidemiológicos, el tamaño de la muestra no es relevante. c. Las consideraciones éticas deben tenerse en cuenta. d. Habitualmente, son poco costosos y fáciles de ejecutar.

446. ¿Cuál de los siguientes ejemplos NO corresponde a la prevención primaria de una enfermedad? :

a. Inmunización contra las enfermedades del calendario vacunal en los niños. b. Campañas de prevención anti-tabaco entre adolescentes. c. Recomendación del uso de guantes en el manejo de productos tóxicos. d. Tratamiento hipolipemiante en individuos con historia de angor.

447. Señale la respuesta FALSA con respecto a los estudios de los casos y controles:

a. Son relativamente baratos y fáciles de ejecutar, comparados con otros estudios analíticos. b. Permiten calcular directamente la tasa de incidencia de la enfermedad en los individuos expuestos. c. Es el mejor tipo de estudio analítico para evaluar enfermedades de baja incidencia. d. Permiten investigar múltiples factores de riesgo de una misma enfermedad.

448. ¿Cuál de los siguientes criterios NO es necesario para estudiar una relación de causa-efecto? :

a. Credibilidad biológica. b. Fuerza de asociación. c. Generalizabilidad. d. Consistencia de los resultados.

449. ¿Cuál es el numerador de la tasa de mortalidad neonatal precoz? :

a. Número de niños muertos de menos de 28 días de vida. b. Número de fetos viables que fallecen antes del parto. c. Número de fallecidos entre 1 y 12 semanas de vida. d. Número de fallecidos hasta el día 7 de vida.

450. Con respecto a los estudios de cohortes, indicar lo FALSO:

a. Permiten examinar múltiples efectos de un único factor de riesgo. b. Permiten calcular directamente tasas de incidencia de la enfermedad en los expuestos. c. El mayor problema es la pérdida de individuos durante el período de seguimiento. d. La asignación de los individuos es aleatoria.

Page 217: Cuestionario de Urología

451. ¿Cuál de las siguientes es la definición del período de incubación de una enfermedad infecciosa? :

a. El tiempo desde la infección hasta el comienzo de la transmisibilidad de la enfermedad. b. El tiempo entre la exposición al agente infeccioso y la aparición de fiebre termometrada. c. El tiempo desde el comienzo de la infectividad hasta la aparición de la sintomatología clínica. d. El tiempo entre la invasión por el agente infeccioso y la aparición del primer signo o síntoma de la enfermedad.

452. El valor predictivo positivo de una prueba diagnóstica para una enfermedad se estima por:

a. La proporción de individuos, que al aplicarles la prueba, dan positivo. b. La proporción de enfermos, que al ap1icarles la prueba, dan positivo. c. La proporción de positivos entre los enfermos. d. La proporción de enfermos entre los positivos.

453. La quimioprofilaxis específica de la meningitis meningocócica, en una mujer embarazada, se realiza con:

a. Isoniazida. b. Rifampicina. c. Eritromicina. d. Espiramicina.

454. Señale cuál es el método aceptado para evitar la infección del recién nacido por Streptococcus agalactiae (estreptococo grupo B):

a. Administrar penicilina o ampicilina a las madres portadoras al iniciarse el parto. b. Dar antibióticos a las madres portadoras durante el tercer trimestre del embarazo. c. Administrar inmunoglobulinas especificas al recién nacido. d. Adoptar rigurosas medidas de asepsia durante el parto.

455. El virus de la gripe puede ser causa de pandemias con alta morbi-mortalidad. Esta situación cabe preverla ante:

a. Aislamiento a partir de animales, de cepas de virus influenza tipo B, con variaciones mayores en la estructura de su hemaglutinina. b. Una variación menor en cepas de virus influenza tipo B, aisladas en personas, por mutaciones puntuales en el gen que codifica la neuraminidasa. c. Aislamiento en personas de cepas de virus influenza tipo A, con una variación mayor en la estructura de hemaglutinina o neuraminidasa. d. Aislamiento en Hong Kong, exclusivamente a partir de aves, de una cepa de influenza A, diferente a cualquiera de las que se conoce que han afectado a personas.

456. ¿Cuál es el método más fiable, entre los siguientes, para detectar en una embarazada el Streptococcus agalactiae (estreptococo del grupo B):

a. Cultivo de exudado endocervical. b. Detección de antígeno en exudado vaginaa. c. Cultivo de muestra vaginal y anorrectaa. d. Detección de anticuerpos en suero.

457. ¿Qué fármaco se utiliza como primera elección en la profilaxis del paludismo por Plasmodiumfalciparum resistente a cIoroquina? :

a. Proguanil. b. Primaquina. c. Doxiciclina. d. Mefloquina.

458. Señale cuál de las siguientes afirmaciones, relativas a la prevención de las hepatitis víricas, es correcta:

a. Debe establecerse la inmunización activa frente a la hepatitis C de todos los usuarios de drogas por vía parenteraa. b. Los recién nacidos de madres infectadas, portadoras del antígeno de superficie del virus de la hepatitis B, deben recibir exclusivamente inmunización pasiva. c. La vacuna contra la hepatitis B, debe administrarse a personas que trabajan de forma continuada en servicios quirúrgicos. d. Tras una inoculación accidental con productos patológicos que contienen antígeno de superficie de la hepatitis B, debe iniciarse tratamiento farmacológico como medida preventiva.

459. Entre las siguientes exploraciones, aplicadas al estudio de la extensión del cáncer broncopulmonar, señale la que NO está justificada:

a. TAC craneal en el adenocarcinoma y el indiferenciado. b. Garnmagrafia ósea si hay síntomas o fosfatasa alcalina alta. c. Toracoscopia ante derrame pleural exudado con citología negativa. d. Mediastinoscopia si aparece parálisis completa del nervio recurrente.

460. ¿Cuál de los siguientes fármacos empleados en el tratamiento de la artritis reumatoide puede producir retinopatía? :

a. Metotrexato. b. Sales de oro. c. D-penicilamina. d. Cloroquina.

461. De los siguientes fármacos empleados en el tratamiento de la artritis reumatoide, ¿cuál puede producir toxicidad hematológica importante, que revierte con la administración de ácido folínico intravenoso?

a. Aurotiomalato sádico. b. Azatioprina. c. Ciclofosfamida.

Page 218: Cuestionario de Urología

d. Metotrexato.

462. Con respecto a los aminoglucósirlos, señale cuál de las siguientes afirmaciones es FALSA:

a. Actúan inhibiendo la síntesis de proteínas bacterianas. b. La dosis debe ajustarse en caso de insuficiencia renaa. c. Carecen de efecto frente a la Pseudomona aeruginosa. d. Pueden administrarse en una dosis única.

463. En el tratamiento corto de la tuberculosis, la pirazinamida se debe administrar durante:

a. Todo el tiempo. . b. Los dos últimos meses. c. Los cuatro primeros meses. d. Los dos primeros meses.

464. Señale, entre las siguientes, la afirmación INCORRECTA referente al tratamiento farmacológico del sangrado agudo por varices esofágicas:

a. La somatostatina y el octreótido son más efectivos que el placebo para controlar la hemorragia. b. La somatostatina y el octreótido tienen menos efectos colaterales que la vasopresina. c. No hay constancia clara de que la somatostatina o el octreótido aumenten la supervivencia. d. Hay constancia de que la vasopresina aumenta la supervivencia a pesar de los efectos colaterales.

465. La biopsia de un recién nacido con ictericia colestática, muestra abundantes hepatocitos con glóbulos hialinos citoplasmáticos PAS positivos. ¿Cuál es el diagnóstico más probable entre los siguientes? :

a. Lúes congénita. b. Enfermedad de Wilson. c. Déficit de alfa l-antitripsina. d. Enfermedad de Gaucher.

466. La biopsia intestinal, realizada por un cuadro de malabsorción en un adulto joven, muestra abundantes macrófagos PAS positivos en la lámina propia ¿Cuál es el diagnóstico más probable? :

a. Enfermedad celiaca del adulto. b. Enfermedad de Whipple. c. Abetalipoproteinemia. d. Agarnmaglobulinemia.

467. ¿Cuál de estas afirmaciones es correcta en relación con la apoptosis? :

a. Es una necrosis fisiológica, b. Mecta a pequeños grupos celulares. c. Apenas hay cambios morfológicos nucleares. d. No produce inflamación circundante.

468. Señale cuál de los siguientes síndromes neuro-musculares paraneoplásicos es el más frecuente:

a. Encefalitis subaguda. b. Degeneración cerebelosa. c. Neuropatía periférica. d. Síndrome miasténico de Lambert-Easton.

469. Señale cuál de las siguientes afirmaciones es correcta:

a. La malignización de tumores benignos es un fenómeno muy frecuente. b. Los tumores benignos nunca se malignizan. c. El seguimiento de pacientes no ha permitido documentar la malignización de pólipos adenomatosos intestinales, lo que no permite apoyar la hipótesis del potencial maligno de estos tumores benignos. d. Los pólipos adenomatosos intestinales representan ejemplos bien documentados del potencial maligno de algunos tumores benignos.

470. Qué afirmación, de las siguientes, relativas al nódulo tiroideo "frío" es FALSA? :

a. La punción-aspiración con aguja fina (PAAF) es útil en su estudio. b. Alrededor del 90% de los nódulos son benignos. c. No es adecuado operarIos sin haber practicado la PAAF d. La presencia de ganglios regionales palpables no orienta para el diagnóstico.

471. En la mayoría de los casos, se ha demostrado que la enfermedad de Hodgkin es una neoplasia derivada de:

a. Linfocitos B. b. Linfocitos T. c. Macrófagos. d. Células dendríticas.

472. Una metástasis de carcinoma en una adenopatía cervical, en la que se demuestra la presencia del virus de Epstein-Barr, es muy sugerente de:

a. Mononuc1eosis infecciosa. b. Asociación con enfermedad de Hodgkin. c. Carcinoma nasofaríngeo. d. Estado de inmunodepresión.

473. ¿Cuál de las siguientes prácticas deportivas expone más a padecer infección por Francisella tularensis? :

Page 219: Cuestionario de Urología

a. La pesca. b. La caza. c. La natación. d. El montañismo.

474. En un lactante con cardiopatía congénita, que sufre infección grave por virus respiratorio sincitial, el tratamiento antivírico, de los siguientes, que hay que recomendar es:

a. Gancic1ovir oraa. b. Acic1ovir i.v. c. Acic1ovir oraa. d. Ribavirina en aerosol.

475. De las siguientes pruebas serológicas, ¿cuál hay que realizar para el diagnóstico de la neurosífilis?:

a. Examen en campo oscuro de líquido cefalorraquídeo (LCR). b. FTA-ABS en LCR. c. VDRL en LCR. d. Test de inmovilización de T. Pallidum en LCR (test de Nelson).

476. Señale la asociación INCORRECTA:

a. Acinetobacter sp Ilinfección nosocomial. b. Haemophilus influenzae II infección de transmisión sexual. c. Streptococcus agalactiae II infección neonataa. d. Mycobacterium avium complex II SillA.

477. ¿Cuáles son los gérmenes más habitualmente responsables de no absceso pulmonar de origen extrahospitalario?

a. Gram negativos. b. S. pneumoniae. c. Legionella sp. d. Anaerobios.

478. La unión de qué ramos y de qué nervios forman, en la mayoría de los casos, el plexo braquial? :

a. Los ventrales de C5, C6, C7, C8 y parte de DI. b. Los ventrales de C4, CS y C6. c. Los ventrales de C6, C7, C8, DI y D2. d. Los ventrales de C3, C4, CS, C6 y C7.

479. Un individuo, participa en una pelea entre varias personas. Horas después consulta por erosiones y equimosis múltiples, heridas superficiales por arma blanca en ambas manos y déficit de extensión activa de la interfalángíca distal del 4° dedo de la mano izquierda, No presenta alteraciones de sensibilidad y las radiografías son normales. ¿Cuál de las siguientes opciones diagnósticas debe considerarse en primer lugar? :

a. Lesión de la musculatura extrínseca extensora del 4° dedo. b. Artritis traumática de la interfalangica distal del 4° dedo. c. Arrancamiento de la inserción del extensor del 4° dedo en la falange distal. d. Arrancamiento de la inserción de los extrínsecos en la falange distal.

480. ¿Cuál de los siguientes músculos extraoculares NO se origina en el vértice de la órbita? :

a. Recto superior. b. Recto externo. c. Recto inferior. d. Oblicuo inferior.

481. Un varón adulto, presenta un cuadro de dolor y rigidez de cuello, con irradiación del dolor a extremidad superior derecha a través de la cara dorsal del antebrazo y del tercer dedo, comprobándose, al mismo tiempo, debilidad de los flexores de la muñeca y disminución del reflejo tricipitaa. Tras la radiología convencional y resonancia magnética, se establece el diagnóstico de hernia discal cervicaa. El disco herniado es él comprendido entre:

a. CI-C2 b. C2-C3 c. C3-C4 d. C6-C7.

482. La función principal del complejo mayor de histocompatibilidad es:

a. La presentación del antígeno a las células responsables de la respuesta inmune. b. Servir de barrera interespecies. c. Favorecer la aceptación de los trasplantes de órganos. d. Proporcionar mecanismos de defensa ante tumores.

483. Los anticuerpos IgG están formados por dos cadenas idénticas pesadas y dos idénticas ligeras, en las que, a su vez, se distinguen regiones variables (Fab) y constantes (Fe). ¿A qué parte de la IgG une el complemento?:

a. Las dos cadenas ligeras. b. Una cadena ligera. c. Las regiones Fab.' d. Las regiones Fc.

484. ¿Cuál de los siguientes fenómenos NO es característico de la disregulación del sistema inmune en los ancianos?

a. Mayor incidencia de enfermedades autoinmunes.

Page 220: Cuestionario de Urología

b. Disminución de la respuesta a vacunas. c. Aumento de la presencia de autoanticuerpos. d. Frecuencia aumentada de la incidencia de tumores.

485. Un paciente de 62 años, con artritis reumatoide seropositiva de 23 años de evolución, en tratamiento exclusivamente con prednisona y piroxicam, consulta por aparición de edemas en miembros inferiores. La analítica muestra VSG de 110 mm, hipoalbuminemia de 2.4 g/L, creatina en sangre de 2.1 mg/dL y proteinuria de 6 g en 24 horas, sin otros hallazgos patológicos en el sedimento urinario. ¿Cuál de las siguientes entidades clínicas es la causa más probable del cuadro que presenta el paciente? :

a. Nefropatía por antiinflamatorios no esteroideos. b. Vasculitis reumatoide. c. Glomerulonefrifis membranosa. d. Amiloidosis secundaria.

486. Señale cuál de las siguientes afirmaciones, respecto al meningioma, es FALSA? :

a. Constituyen el 20% de los tumores intracraneales. b. Se ha recomendado quimioterapia en el tratamiento de los malignos. c. Su localización es el principal factor determinante de la evolución. d. Suelen debutar sintomáticamente en la quinta o sexta década de la vida.

487. Mujer de 35 años, con historia de debilidad muscular y diplopia de un mes de evolución. En una radiografía de tórax se aprecia una masa retroesternal de unos 5 cm de diámetro. Esta masa corresponde más probablemente a un:

a. Teratoma mediastínico. b. Timoma. c. Bocio con crecimiento retroesternaa. d. Adenoma paratiroideo.

488. En un recién nacido cianótico, con sospecha de cardiopatía congénita cianógena, lo prioritario es:

a. Administrar prostaglandina El intravenosa. b. Administrar surfactante pulmonar. c. Iniciar antibioterapia con cefotaxima i.v. d. Hacer un cateterismo diagnóstico.

489. ¿En qué gestantes se considera más aconsejable la práctica de amniocentesis, para el estudio del cariotipo como método diagnóstico, ante posible malformación fetal? :

a. Las muy expuestas a rayos X en el primer trimestre. b. Las epilépticas en tratamiento con hidantoínas. c. Las que han presentado un cuadro de rubéola en el primer trimestre. d. Las de edad superior a los 35 años.

490. El índice de masa corporal o índice de Quetelet se calcula:

a. Peso en Kg/altura en cm. b. Peso en Kg/altura en m. c. Altura en cm/ peso en Kg. d. Peso en Kg/(altura en ml

491. Sobre la angina variante de Prinzmetal NO es cierto que:

a. Hasta tres cuartos de los enfermos, tienen lesiones coronarías fijas. b. El dolor suele ocurrir en reposo. c. Ocurre en pacientes mayores que los que presentan angina arterioesclerótica típica. d. En el ECG se aprecia elevación del segmento STo

492. Paciente de 50 años, que presenta en la radiografía de tórax un patrón intersticial instaurado, tras una clínica de disnea de meses de duración. Para establecer el diagnóstico, se le practica una broncofibroscopia con biopsia transbronquial en la que, entre otras lesiones, se muestran granulomas. El lavado broncoalveolar demuestra un predominio de linfocitos (60%) con un cociente CD4/CD8 bajo. El diagnóstico más probable, entre los siguientes, es:

a. Neumonía intersticiallinfocitaría. b. Neumonitis por hipersensibilidad. c. Sarcoidosis. d. Neumonitis aspergilar.

493. La claudicación intermitente de los gemelos, está causada más frecuentemente por enfermedad oclusiva en la arteria:

a. Aorta. b. llíaca externa. c. Femoral superficial. d. Femoral profunda.

494. En una revisión, a un adolescente diagnosticado de síndrome de Von Recklinghausen, se le descubren cifras altas de tensión arterial sin ninguna otra sintomatología. ¿Qué cuadro patológico acompañante de los siguientes hay que descartar? :

a. Schwannoma perirenal. b. Glioma en región hipotalámica. c. Feocromocitoma intraabdominal. d. Nefropatía mesangial asociada.

495. Un paciente de 45 años de edad es diagnosticado de esófago de Barret largo (segmento de 6 cm) y sigue tratamiento con inhibidores de la bomba de protones (IBP) a dosis estándar. ¿Cuál de las siguientes afirmaciones es cierta?:

a. En su seguimiento se realizará endoscopia sólo si persisten los síntomas.

Page 221: Cuestionario de Urología

b. Mientras se mantenga el tratamiento continuo con IBP se elimina el riesgo de cáncer. c. Se debe asegurar un control adecuado de la secreción ácida con dosis altas de ffiP o preferiblemente ajustando la dosis mediante pHmetría. d. El seguimiento sólo es necesario si existe displasia de alto grado.

496. Señalar en relación con el linfoma gástrico primario, cuál es la respuesta acertada:

a. Ellinfoma gástrico primario es de peor pronóstico que el adenocarcinoma. b. El tratamiento antibiótico de erradicación de Helicobacter Pylori provoca la regresión de un pequeño porcentaje (15%) de los linfomas MALT. c. Los pacientes que responden a la antibioterapia en linfoma MALT deben controlarse mediante endoscopia periódica, debido al desconocimiento de si se elimina el clon neoplásico o simplemente se suprime de forma temporaa. d. La resección gástrica asociada a quimioterapia permite supervivencias de 80 a 90% a los cinco años en pacientes con linfoma de alto grado localizados.

497. Paciente de 68 años, operado hace 15 años de una úlcera gastroduodenal con vagotomía troncal y que consulta por diarrea crónica. Tras el estudio se llega a la conclusión de que es debida a la cirugía previa. De las siguientes respuestas, ¿cuál es la correcta?:

a. Es una compiicación muy rara, menor del 1 %. b. Suele preceder a las comidas. . c. La loperamida no sirve como tratamiento. d. Suele ser debido a un trastorno de la motilidad.

498. Paciente de 23 años diagnosticado de enfermedad celíaca y sometido a dieta sin gluten. Tras ello recuperó peso pero siguió con dos a tres deposiciones al día de heces blandas. Las grasas en heces eran de 10 g/día. Los anticuerpos antiendomisio tipo IgA persistían positivos tras dos años de tratamiento. La causa más probable de esta insuficiente respuesta al tratamiento es:

a. Intolerancia a la lactosa. b. Linfoma intestinal. c. Esprue colágeno. d. Incumplimiento de la dieta sin gluten.

499. En un paciente con metástasis pulmonares y sin afectación hepática por un carcinoma de origen intestinaa. ¿Cuál es la localización más probable del primario?:

a. Recto. b. Sigma. c. Ángulo esplénico del colon. d. Colon transverso.

500. Una paciente de 24 años presenta diarrea con emisión de moco y sangre, y dolor abdominal, sin que existan antecedentes epidemiológicos de interés. ¿Qué diagnóstico sería, entre los siguientes, el más probable y qué exploración realizaría para confirmarlo?:

a. Diverticulosis y enema opaco. b. Angiodisplasia del colon y arteriografía. c. Enfermedad inflamatoria intestinal y colonoscopia. d. Adenoma velloso rectal y rectoscopia.

501. Señale cuál de los siguientes fármacos puede ser eficaz como alternativa a la colectomía en el tratamiento de un paciente con colitis ulcerosa severa, refractaria a tratamiento con esteroides a dosis altas:

a. Ciclosporina. b. Azatioprina. c. Metotrexate. d. Infliximab.

502. Una mujer de 81 años acude por asteuia franca durante el último mes. La analítica revela una anemia microcítica y ferropéuica (hemoglobina 10g/dL, hematocrito 29%, VCM 71 n, y sideremia baja). Es hipertensa por lo cual recibe un inhibidor del enzimaconvertidor de la angiotensina y padece dolores osteomusculares generalizados atribuidos a artrosis que ella trata de forma espontánea con ácido acetilsalicílico. No refiere molestias digestivas, su hábito deposicionales normal y nunca ha visto sangre en las deposiciones. Una prueba de sangre oculta en heces da resultado positivo. Usted indica una endoscopia digestiva alta y una colonoscopia total que no muestra lesión alguna. ¿Cuál sería la conducta más adecuada?:

a. Indicar una exploración del intestino delgado con cápsula endoscópica. b. Indicar una arteriografía selectiva de tronco celíaco y ambas mesentéricas para descartar angiodisplasia. c. Indicar una garnmagrafía con Pertecnetato de Tc 99 para descartar divertículo de Meckea. d. Prohibir el uso de ácido acetilsalicílico u otros AINEs, dar hierro oral y repetir la analítica en un plazo de 2 meses.

503. Enfermo de 35 años diagnosticado de hepatitis C sin confirmación histológica que desde hace 4 años no ha realizado ninguna revisión médica. Bebedor de 100g/día de alcohol desde hace más de 15 años. Acude a la consulta por astenia moderada sin otra manifestación clínica. A la exploración fisica aparecen arañas vasculares, circulación colateral y esplenomegalia. Las exploraciones complementarias presentan los siguientes resultados: Hb 12 g/dL, Leucocitos 3500/mm3, Plaquetas 76000/mm 3, GOT 98 UI/l, GPT 45 UI/l, GGT 175 UI/l, urea Z3 mgfdL, hierro 175 mcg/dL, Ferritina 2300 mcg/L, (límite superior de normalidad 370 mcg/L). Genotipo del virus C: Ib, viremia VHC > 2,5 millones de copias (superior a 700000 UI) HBsAg positivo. Ig G anti CMV positiva. ¿Cuál de las siguientes posibilidades etiológicas podría rechazar con la información disponible?:

a. Hepatitis C. b. Hemocromatosis. c. Hepatopatía alcohólica. d. Hepatitis por CMV.

504. Señale cuál de las siguientes afirmaciones con respecto a la cirrosis hepática es FALSA:

a. El consumo excesivo de alcohol y la hepatitis viral son las causas de la cirrosis en el 90% de los casos.

Page 222: Cuestionario de Urología

b. La cirrosis hepática puede ser una enfermedad asintomática. . c. La esteatohepatitis no alcohólica puede ser causa de cirrosis. d. La presencia de ascitis no es un elemento pronóstico en pacientes con cirrosis.

505. Muchacha de 26 años hospitalizada por una ictericia de instauración reciente, asociada a dolor en hipocondrio derecho. Se detecta hepatomegalia sensible sin esplenomegalia. Hay telangiectasias faciales. Reconoce antecedentes de promiscnidad sexual pero no de consumo de droga intravenosa. Los exámenes de laboratorio muestran bilirrubina 16mg/dl, AST 315 UI/l, ALT 1101UI/I, GGT 680 UI/l, fosfatasas alcalinas 280 UI/l, protrombina 40%, triglicéridos 600 mg/dl, colesterol 280 mg/da. Una ecografía reveló un patrón hiperecogénico del hígado. La serología para virus de la hepatitis A, B Y C fue negativa. ¿Cuál es el diagnóstico más probable?:

a. Hepatitis vírica. b. Hepatitis tóxica. c. Hepatitis alcohólica. d. Hepatitis autoinmune.

506. Enferma de 72 años que consulta porque desde hace al menos 5 años tiene las transaminasas ligeramente elevadas (nunca por encima de 100 UI/ml). Es ligeramente obesa (índice de masa corporal = 28) Y no se encuentra mal. No bebe alcohol, no toma medicamentos de forma habitual. Los marcadores de virus de hepatitis B y C son negativos y la analítica habitual es normal salvo AST (GOT) 82 UI/ml Y ALT (GPT) 107UI/ml, con gammaglobulina de 1,8 g/dl e IgG de 1700 mg/da. Tiene anticuerpos antinucleares positivos a 1/80 y antifibra lisa a 1/160.¿Cuál sería el siguiente paso?:

a. Recomendar que normalice su peso, porque con ello es probable que 1o hagan las alteraciones analíticas. b. Indicar la práctica de una biopsia hepática. c. Iniciar tratamiento con prednisona y azatioprina. d. Iniciar tratamiento con ácido ursodesoxicólico.

507. Paciente colecistectomizada hace 6 años por colelitiasis. Desde hace 6 meses presenta dolor cólico en flanco derecho a temporadas. En la analítica hay valores de colestasis moderada. La ecografía abdominal sugiere coledocolitiasis. ¿Cuál, entre las siguientes, es la prueba indicada para realizar en primer lugar, en este caso?:

a. Tomografía computerizada abdominal con contraste i.v. b. Colangiografía i.v. c. Colangiografía retrógrada endoscópica. d. Colangioresonancia magnética.

508. ¿Cuál es la primera medida terapéutica a adoptar en la pancreatitis aguda?:

a. Aspiración nasogástrica. b. Dieta absoluta. c. Antibioticoterapia de amplio espectro d. Administración de somatostatina.

509. Paciente con vómitos, niveles séricos de amilasa aumentados y de lipasa normales. La amilasuria y el aclaramiento de amilasa y el aclaramiento de creatinina están disminuidos, éste último es del 0,78. Señale, entre los que se mencionan, el diagnóstico más probable:

a. Pancreatitis aguda de más de cinco días de duración. b. Pancreatitis aguda en paciente con hipertrigliceridemia. c. Pancreatitis crónica. d. Macroamilasemia.

510. En una radiografía de abdomen en supino se ve el ligamento falciforme. ¿Cuál es el diagnóstico más probable?:

a. Ascitis. b. Perforación intestinal. c. Peritonitis. d. Oclusión intestinal.

511. En la valoración en el servicio de urgencias hospitalario de un paciente con dolor abdominal agudo, ¿cuál de las siguientes afirmaciones es INCORRECTA?:

a. El inicio, la localización y severidad de dolor son útiles en el diagnóstico diferencial. b. La palpación es el aspecto más importante de la exploración física. c. El recuento de leucocitos puede ser normal en procesos infIamatorios abdominales como la apendicitis. d. No debe administrarse medicación analgésica hasta que el cirujano valore al paciente porque puede oscurecer el diagnóstico.

512. Un paciente de 55 años refiere plenitud post-prandial progresiva desde hace 3 semanas. En la actualidad intolerancia a la ingesta con vómitos de carácter alimentario. La exploración endoscópica muestra restos alimenticios en cavidad gástrica y una gran ulceración (unos 3 cms de diámetro) en la porción distal próxima a la segunda rodilla duodenal. El estudio anatomo-patológico demostró adenocarcinoma. Refiera de las opciones terapéuticas siguientes, cuál es la más correcta:

a. El tratamiento consiste en duodenopancreatectomía cefálica. b. El tratamiento del carcinoma duodenal es paliativo, por lo que debe realizarse gastroyeyunostomía posterior, retrocólica, inframesocólica. c. Se debe realizar resección segmentaria con anastomosis duodenal término-terminal. d. Se practicará instalación de prótesis autoexpandible con radioterapia y quimioterapia sistémica.

513. Un varón de 50 años, cardiópata conocido, en fibrilación auricular crónica, acude al Servicio de Urgencias refiriendo dolor centroabdominal intenso y continuo, irradiado a epigastrio, y de comienzo brusco hace unas 2 horas. A la exploración el paciente está estable y con sensación de mucho dolor abdominal, aunque el abdomen aparece blando y depresible, sin signos de irritación peritoneal. La exploración radiológica simple de tórax y abdomen es normal. Señale, entre las siguientes, la afirmación correcta:

a. La localización y características del dolor permiten descartar una isquemia miocárdica. b. La exploración abdominal normal permite descartar un abdomen agudo quirúrgico.

Page 223: Cuestionario de Urología

c. Se debe administrar analgesia y ver evolución en unas horas. d. Se debe realizar una arteriografía mesentérica urgente para descartar una embolia mesentérica.

514. Paciente de 50 años con antecedentes de cirrosis hepática por virus C de la hepatitis con antecedentes de ascitis controlada con diuréticos. En una ecografía rutinaria se describe la presencia de una lesión ocupante de espacio de 4,5 cm. de diámetro en el segmento VIII. Se realiza una punción aspiración con aguja fina cuyo resultado es compatible con carcinoma hepatocelular. Los análisis muestran un valor de bilirrubina de 2,5 mg/dl, creatinina 0,8 mgldL, INR de 1,9 y alfafetoproteína de 40 UI/A. Se realizará una endoscopia que demuestra la presencia de varices esofágicas de pequeño tamaño. La medición del gradiente de presión venosa hepática refleja un valor de 14mmHg. Señale cuál de las siguientes es la actitud de tratamiento más correcta:

a. Actitud expectante con repetición cada 3 meses de ecografía para la valoración del crecimiento de la lesión. b. Segmentectomía con amplio margen de seguridad. c. Tratamiento paliativo por la excesiva extensión de la enfermedad tumora!. d. Trasplante hepático si el paciente no presenta contraindicaciones para su realización.

515. La auscultación del corazón requiere un estetoscopio con campana y membrana. Señale la respuesta verdadera:

a. La campana es mejor para oír los sonidos graves como el soplo de la estenosis mitral. b. La membrana identifica mejor sonidos graves como el soplo de la Insuficiencia aórtica. c. El primer tono cardíaco sigue al pulso carotídeo y el segundo tono lo precede. d. El primer tono normal es más fuerte y agudo que el segundo.

516. Una mujer de 72 años con antecedentes de diabetes mellitus e infarto de miocardio hace un año, tiene una fracción de eyección ventricular izquierda de 0,30 y está en tratamiento habitual con aspirina, furosemida, (20 mg/día) y captopril, (25 mg/día). Acude a su consulta por disnea de pequeños esfuerzos. La exploración física es compatible con insuficiencia cardiaca, de predo-minio derecho. TA: 140/70 mmHg. FC: 70 Ipm. ¿Cuál de las siguientes intervenciones es de MENOR prioridad desde el punto de vista de mejorar su pronóstico?:

a. Cambiar el captopril por lisinopril (20 g/día). b. Iniciar tratamiento con digoxina (0,25 g/día). c. Iniciar tratamiento con espironolactona (25mg/día). d. Iniciar tratamiento con bisoprolol (1,25 mg/día).

517. Hombre de 67 años, hipertenso y fumador que acude a urgencias por disnea de grandes esfuerzos en el último mes. La auscultación demuestra disminución del murmullo vesicular sin otros hallazgos. La Rx de tórax y el ECG realizados son normales. Los valores en sangre de péptido natriurético tipo B sou de 60 pg/mI. (valores normales <100pg/mI.). ¿Cuál de los siguientes diagnósticos es el MENOS probable?:

a. Disfunción ventricular sistólica. b. Enfennedad pulmonar obstructiva crónica. c. Asma. d. Hipertensión pulmonar primaria.

518. Señale la respuesta correcta respecto a las siguientes valvulopatías:

a. La dilatación auricular izquierda atenúa la elevación de la presión intraauricular izquierda (y, por tanto, capilar pulmonar) en la insuficiencia mitral aguda. b. La dilatación ventricular izquierda atenúa la elevación de la presión teledistólica en la insuficiencia aórtica crónica. c. La dilatación auricular izquierda frena la progresión de la estenosis mitral. d. La disfunción sistólica severa ventricular izquierda producida por la estenosis aórtica contraindica su tratamiento quirúrgico.

519. Mujer de 74 años hipertensa que ingresa en urgencias por episodio sincopal. Su tensión arterial es de 80/40 mmHg y la frecuencia cardiaca de 110 lpm, con una saturación de oxígeno del 91 %. Presenta ingurgitación yugular sin otros hallazgos significativos en la exploración general y neurológica. En el ECG realizado se objetiva taquicardia sinusal con alternancia eléctrica. ¿Cuál de las siguientes pruebas complementarias solicitaría primero?:

a. Garnmagrafía ventilación/perfusión. b. TC torácico. c. Hemograma. d. Ecocardiograma.

520. Una mujer de 70 años ingresa con un infarto agudo de miocardio de localización anterior es tratada con activador tisular del plasminógeno. A las 2 horas de dicho tratamiento refiere intenso dolor precordial y elevación marcada del segmento ST en derivaciones V2, V3, y V4. ¿Cuál de las siguientes exploraciones le parece más indicada?:

a. Una detenninación urgente de troponina. b. Un ecocardiograma transesofágico. c. Una angiografía coronaria. d. Una garnmagrafía de ventilación/perfusión.

521. Paciente de 63 años que refiere disnea progresiva desde hace 6 meses, tiene antecedentes de diabetes mellitus y cirrosis hepática, en la exploración llama la atención una marcada hiperpigmentación cutánea, presión venosa elevada, estertores húmedos pulmonares bilaterales y ritmo de galope. La placa de tórax muestra incipientes signos de edema pulmonar y un tamaño de la silueta. cardíaca aparentemente normal. ¿Cuál de las siguientes cardiopatías se debe sospechar?:

a. Miocardiopatía restrictiva secundaria a amiloidosis. b. Miocardiopatía restrictiva secundaria a hemo-cromatosis. c. Miocardiopatía hipertrófica familiar. d. Miocardiopatía hipertensiva.

522. Una de las formas de taquicardia supraventricular es la taquicardia por reentrada nodal. Indique cuál de las siguientes afirmaciones es FALSA en relacióu con esta arritmia:

a. Representa el mecanismo más frecuente de taquicardia supraventricular y afecta de manera predominante a mujeres. b. Se presenta habitualmente como una taquicardia paroxística, regular, con complejo QRS estrecho. c. Durante la taquicardia la onda P retrógrada aparece si tuada a unos 160 milisegundos después del inicio del complejo QRS.

Page 224: Cuestionario de Urología

d. Clínicamente la característica principal es la sensación de palpitaciones en el cuello debida a la aparición de un reflujo yugular por contracción simultánea de aurículas y ventrículos.

523. Indique cuál de las siguientes afirmaciones es cierta respecto al tratamiento de la hipertensión arterial:

a. En pacientes obesos la reducción del peso por si sola no disminuye la tensión arterial. b. Los inhibidores de la enzima conversora de la angiotensina (lECA) deben añadirse al tratamiento previo con diuréticos sin interrupción de éstos. c. Los antagonistas de los receptores de la angiotensina II no producen hiperpotasemia como efecto secundario, a diferencia de los IECA. d. Los estudios a largo plazo han demostrado que los diuré ticos en el tratamiento de la HTA disminuyen la morbimortalidad.

524. ¿Cuál de los siguientes pacientes hipertensos se beneficia más de un mayor descenso de la TA para prevenir complicaciones cardiovasculares?:

a. Anciano con hipertensión sistólica aislada. b. Varón de edad media con antecedentes de infarto de miocardio. c. Diabético con nefropatía diabética. d. Mujer joven con estenosis de arteria renal.

525. Un paciente varón de 80 años de edad refiere tener dolor lumbar muy intenso, de instauración brusca, en reposo y sin modificación con los movimientos ni la palpación lumbar. En la exploración ÍIsica destaca hipotensión arterial y la existencia de una masa abdominal pulsátia. ¿Cuál de las siguientes afirmaciones son ciertas en relación con el diagnóstico y tratamiento del pacien-te?:

a. El diagnóstico más probable es la existencia de una neoplasia de colon. b. El cuadro clínico sugiere disección a6rtica y debe hacerse de inmediato una aortografía. c. La masa abdominal sugiere un aneurisma aórtico abdominal pero no explica el dolor lumbar del paciente. d. Se debe realizar estudio inmediato con TAC abdominal por probable existencia de aneurisma aórtico abdominal complicado y valoración quirúrgica urgente.

526. Señale la respuesta correcta respecto al shock:

a. En el shock hipovolémico la presión venosa central y la presión de enclavamiento pulmonar están elevadas. b. El shock secundario a insuficiencia suprarenal no precisa de volumen ni vasopresores para su tratamiento. c. El tratamiento inicial del shock séptico debe ser la dobutamina. d. El shock cardiogénico es un fallo primario de bomba que produce disminución del aporte de oxígeno a los tejidos y elevación de las presiones vasculares pulmonares.

527. Mujer de 55 años que es sometida a recambio valvular mitral mediante una prótesis mecánica bivalva. El postoperatorio cursa de forma normal, la paciente es dada de alta al séptimo día en ritmo sinusal y con un ecocardiograma de control que muestra una prótesis normofuncionante y una función ventricular izquierda conservada. ¿Qué régimen de anticoagulación y/o antiagregación recomendaría a largo plazo en dicha paciente?:

a. Anticoagulación oral durante 3 meses para mantener INR entre 2,5-3,5 y posteriormente clopidogrel, 1 comprimido al día, suspendiendola anticoagulación oral. b. Anticoagulación oral para mantener INR entre 4-5 de forma indefinida. c. Anticoagulación oral para mantener INR entre 3-4 de forma indefinida. d. AAS 250mg/24h de forma indefinida.

528. Un paciente de 68 años de edad ha sido diagnosticado de una estenosis de la arteria coronaria derecha (a nivel proximal) siendo tratado mediante angioplastia coronaria transluminal percutánea. Inmediatamente después de la misma, el paciente presenta un cuadro clínico caracterizado por dolor torácico agudo, alteracion_ electrocardiográficas e inestabilidad hemodinámica. ¿'Cuál de las siguientes afirmaciones es INCORRECTA?:

a. Es una complicación infrecuente tras la angioplastia percutánea. b. La sospecha es una disección intimal de la arteria coronaria y la oclusión de la misma. c. Puede ser precisa la cirugía de forma urgente: cortocircuito-bypass-coronario. d. Está contraindicada la realización de una nueva coronariografía urgente para confirmar la sospecha clínica de oclusión arterial.

529. Una paciente de 42 años, fumadora de dos paquetes al día, con clínica de dolor de costado derecho, disnea y tos desde hace 2 meses, presenta, en la exploración fisica, matidez a la percusión en la mitad inferior del hemitórax derecho, plano posterior, con uua línea limitante de la matidez dirigida hacia arriba y afuera. Al auscultar sobre esa zona no se oye murmullo vesicular y existe transmisión de la voz cuchicheada (pectoriloquia áfona). Entre los siguientes posibles hallazgos en la placa de tórax, ¿cuál esperaría encontrar en este caso?:

a. Derrame pleural derecho asociado a condensación pulmonar en lóbulo inferior derecho. b. Un hemitórax derecho blanco con mediastino retraído hacia ese lado. c. Una masa pulmonar derecha, localizada en el lóbulo inferior derecho, con pequeño pinzamiento del seno costofrénico homolateral. d. Un derrame pleural derecho que ocupa el tercio inferior del hemitórax derecho.

530. Se considera como uno de los criterios diagnósticos de Síndrome de Distress Respiratorio del Adulto, uno de los siguientes datos:

a. Presencia de insuficiencia cardíaca. b. Acidemia refractaria. c. Estertores bilaterales intensos. d. Cociente PaO2/FiO2 igual o inferior a 200.

531. Un paciente tiene un volumen espiratorio en el primer segundo, (FEVl )<60% de lo previsto después de broncodilatadores y la relación entre el FEVl y la capacidad vital inspiratoria es del 78%. El patrón espirométrico corresponde a:

a. Tiene una obstrucción leve. b. Tiene una obstrucción moderada.

Page 225: Cuestionario de Urología

c. Tiene una obstrucción severa. d. No tiene obstrucción.

532. Mujer de 21 años de edad, que ha comenzado hace tres meses con sibilancias en el pecho y tos especialmente a la noche. Se han hecho más frecuente y los últimos dias no le dejan descansar. La exploración muestra una discreta disminución del murmullo vesicular y algunos sibilantes, de forma poco intensa en ambos hemitórax, frecuencia cardíaca 86 pulsaciones por minuto. ¿ Cuál será la primera decisión terapéutica?:

a. Prescribir broncodilatadores B2 estimulantes de corta acción cada cuatro horas. b. Prescribir broncodilatadores de larga acción como formoterol asociado a corticoides inhalados. c. Indicar antibióticos asociados a broncodilatadores anticolinérgicos. d. Prescribir N-acetilcisteína, cada 6 horas asociado a amoxicilina.

533. Uno de los siguientes criterios NO es sugestivo de fibrosis pulmonar idiopática:

a. Disminución de la DLco, en presencia de acropaquia y estertores bilaterales. b. Alteración ventilatoria restrictiva. c. Ausencia de hallazgos histopatológicos en biopsia transbronquial o lavado broncoalveolar sugestivos de un diagnóstico alternativo. d. TAC torácico de alta resolución con lesión del tipo "panal de miel", asociada de forma constante a imágenes en "vidrio esmerilado" extensos.

534. Mujer de 70 años hipertensa con insuficiencia renal crónica moderada que acude por disnea súbita con signos de trombosis venosa en miembro inferior derecho. En las pruebas complementarias destaca una hipoxemia de 55 mmHg, hipocapnia de 24 mmHg Y taquicardia sinusal a 115 lpm en el ECG. Las plaquetas y la coagulación están dentro de los límites de referencia. El dimero D es de 981 ng/mI y la creatinina de 3,5 mg/dl .¿Cuál de las siguientes actitudes le parece más adecuada en este momento?:

a. Solicitar TC helicoidal torácico iniciando perfusión con heparina sódica a 1000 Vl/h. b. Iniciar anticoagulación con heparina de bajo peso molecular a dosis de 1 mg/Kg cada 12 horas. c. Administrar 5000 UI de heparina sódica y solicitar gammagrafía pulmonar de ventilación/perfusión. d. Solicitar ecografía con doppler de miembros inferiores para confirmar el diagnóstico para administrar 1 mg/Kg de heparina de bajo peso molecular.

535. Paciente de 50 años que presenta en la radiografía de tórax un patrón intersticial instaurado tras una clínica de disnea de meses de duración. Al no realizarse diagnóstico se le practica una broncofibroscopia con biopsia transbronquial en la que entre otras lesiones se muestran granulomas. El lavado broncoalveolar demuestra un predominio de linfocitos (60%) con un predominio de los linfocitos T supresores citotóxicos (CD8). El diagnóstico más probable es de:

a. Neumonía intersticiallinfocitaria. b. Neumonitis por hipersensibilidad. c. Sarcoidosis. d. Neumonitis aspergilar.

536. Mujer de 45 años que presenta un cuadro de mes y medio de dolor en costado derecho, fiebre de 38° y tos irritativa. La auscultación mostraba una disminución del murmullo vesicular en base derecha. La radiografía de tórax mostraba un velamiento de la base derecha hasta más o menos la mitad del campo pulmonar. El líquido pleural era amarillo y su análisis mostró: neu-trófilos 20 %, linfocitos 51 %, macrófagos 1 %, proteínas pleura/proteínas suero 0,6. Investigación de bacilos de Koch negativo. El diagnóstico más probable, entre los siguientes,es:

a. Quiste hidatídico. b. Tuberculosis. c. Embolismo pulmonar. d. Insuficiencia cardíaca.

537. Un paciente de 30 años adicto a drogas por vía parenteral, se presenta con un cuadro de 3 días de evolución de tiritona, fiebre, dolor torácico y tos con expectoración verdosa con "hilillos" de sangre; en la radiografía de tórax presenta varios infiltrados pulmonares con cavitación central en alguno de ellos. El diagnóstico más probable sería:

a. Neumonía neumocócica. b. Tuberculosis pulmonar. c. Neumonía por anaerobios. d. Neumonía hematógena.

538. La radiografia de tórax de un paciente muestra calcificaciones mediastínicas en "cáscara de huevo". ¿Cuál es el diagnóstico más probable?:

a. Sarcoidosis. b. Tuberculosis. c. Silicosis. d. Histoplasmosis.

539. Un paciente de 70 años, ex-fumador, tiene desde hace 1 mes, expectoración hemoptoica, disfonía, hepatomegalia, con elevación de enzimas hepáticas y una masa hiliar en la radiografía de tórax. ¿Cuál, entre los siguientes, debe ser el diagnóstico de presunción, que permitirá orientar la pauta de estudio más eficiente?:

a. Tuberculosis pulmonar. b. Neumonía por legionella. c. Carcinoma broncogénico T2N0M0. d. Carcinoma broncogénico T2N2M1.

540. Acude al servicio de urgencias un paciente varón de 25 años de edad, refiriendo dolor de hemitórax izquierdo, pleurítico, de instauración brusca y muy intenso, acompañado de disnea de reposo. El paciente es fumador y no tiene otros antecedentes de interés. La TA sistólica es de 80mm Hg, está sudoroso, con signos de hipoperfusión periférica y en la auscultación cardiopulmonar destaca abolición del murmullo vesicular en el referido hemitórax. ¿Cuál de las siguientes afirmaciones es correcta?:

Page 226: Cuestionario de Urología

a. El diagnóstico más probable es la existencia de un embolismo pulmonar. b. La auscultación pulmonar sólo puede indicar la existencia de un derrame pleural izquierdo masivo. c. Lo primero que debe sospechar dado el antecedente de tabaquismo es la presencia de un evento coronario agudo. d. Debe realizarse evaluación radiológica y quirúrgica urgente por probable neumotórax izquierdo a tensión.

541. ¿Cuál de los siguientes parámetros podría ser considerado como criterio de exclusión absoluto para llevar a cabo una bilobectomía en un paciente de 72 años diagnosticado de EPOC y carcinoma broncogénico no microcítico?:

a. Portador de stent coronario por antecedentes de cardiopatía isquémica. b. Estadificación T4 clínica. c. Antecedentes de metástasis cerebral única resecada previamente. d. FEVl preoperatorio de 680cc.

542. Es práctica habitual en los Servicios de Urgencias realizar pruebas de neuroimagen antes de proceder a una punción lumbar diagnóstica para estudio del LC.R., con el fin de evitar el riesgo de una herniación transtentorial. Señale en cuál de las siguientes situaciones NO debe posponerse nunca el estudio de LC.R.:

a. Sospecha de carcinomatosis meníngea. b. Sospecha de meningitis aguda. c. Sospecha de mielitis transversa. d. Sospecha de hemorragia subaracnoidea.

543. Hombre de 57 años que ingresa en el Servicio de Urgencias de nuestro hospital, tras ser encontrado en la calle sin respuesta a estímulos. En la exploración fisica a su llegada destaca coma con escala de Glasgow de 3 puntos, pupilas puntiformes, reflejo corneal abolido, respiración de Kussmaul y sudoración intensa. El cuadro no se modifica tras administración de 0,4 mgr. de Naloxona intravenosa. Señale el diagnóstico más probable, entre los siguientes:

a. Infarto en el territorio de la arteria cerebral media derecha. b. Intoxicación por cocaína. c. Hemorragia pontina. d. Hemorragia talámica izquierda.

544. Un paciente presenta de forma brusca un transtorno del lenguaje caracterizado por lenguaje espontáneo escaso, casi mutismo, dificultad para evocar palabras, muy leve transtorno de la comprensión, repitiendo correctamente. Su primera sospecha diagnóstica será:

a. Cuadro confusional agudo. b. Accidente isquémico frontal profundo izquierdo. c. Hemorragia subaracnoidea. d. Infarto silviano derecho.

545. Mujer de 64 años que consulta por clínica progresiva en los últimos 4 meses de debilidad en la pierna derecha. En la exploración se objetiva una paresia con amiotrofia de miembro inferior derecho y una biperreflexia miotática de dicho miembro. ¿Cuál es su diagnóstico?:

a. Hernia discallumbar deficitaria. b. Síndrome de Guillain-Barré. c. Esclerosis lateral arniotrófica. d. Neuropatía por enfermedad de Lyme.

546. Mujer de 32 años que acude a la consulta por síntomas agudos compatibles con una oftalmoplejia internuclear bilateral. Ante la sospecha de una esclerosis múltiple ¿qué resultado de qué prueba complementaria diagnóstica NO esperaría encontrar?:

a. Potenciales evocados visuales alterados en el ojo izquierdo. b. Presencia de bandas oligoclonales en el LCR y no en el suero. c. Anticuerpos antinucleares negativos en suero. d. LCR con 120 células por microlito.

547. El diagnóstico de la enfermedad de Parkison es fundamentalmente clínico. ¿Cuál de los siguientes hallazgos es muy improbable en esta enfermedad y cuestiona seriamente su diagnóstico?:

a. Seborrea. b. Torpeza en los movimientos altemantes. c. Depresión. d. Limitación en los movimientos de la mirada hacia abajo.

548. Mujer de 82 años sin antecedentes patológicos destacables y que vive sola, refiere en las últimas 10 semanas un cuadro progresivo de anorexia, anhedonia, insomnio, irritabilidad, olvidos y pérdida de peso de unos 5kg. Ha restringido de una manera notable su vida de relación social. La exploración fisica y los complementarios son normales.. En la valoración neuropsicológica se objetiva un Mini-Mental Test de 25 sobre 30 y una escala de Depresión (Geriatric Depresion Scale) de 12 sobre 15. El test del reloj es normal. ¿Cuál es el diagnóstico más probable?:

a. Cuadro confusional agudo. b. Demencia tipo Alzheimer. c. Crisis de ansiedad generalizada. d. Depresión.

549. Un paciente de 35 años refiere pérdida de fuerza progresiva en miembros inferiores, de unos 5 días de evolución, dolores musculares y parestesias en pies y manos. En su exploración se aprecia únicamente debilidad en los cuatro miembros, de predominio distal y en miembros inferiores, y arreflexia generalizada. Probablemente tiene:

a. Una miastenia gravis. b. Una polirradiculoneuritis aguda. c. Un proceso expansivo medular cervical. d. Una mielitis.

Page 227: Cuestionario de Urología

550. Un paciente de 40 años, sin antecedentes relevantes, es traído a urgencias por haber presentado desviación de la cabeza hacia la izquierda, convulsiones que se iniciaron en miembros izquierdos y se generalizaron enseguida a los cuatro miembros, con pérdida de conciencia, incontinencia vesical y estado confusional de una media hora de duración. Independientemente de los hallazgos de la exploración clínica y la analítica clínica de rutina, debería realizarse con premura como primera medida:

a. TAC cerebral. b. Determinación de alcoholemia. c. Determinación de opiáceos en sangre y orina. d. Electroencefalograma.

551. Un paciente de 40 años, que estuvo durante casi 24 horas haciendo la mudanza de su casa, comenzó a notar adormecimiento en cara anterior del muslo izquierdo junto con cansancio y dolorimiento en región lumbar. Acude a consulta porque persisten los síntomas sensitivos al cabo de unos días, cuando el cansancio y la lumbalgia han desaparecido. La exploración neurológica es normal salvo un área extensa de hipoestesia sobre la cara anterolateral del muslo. Este cuadro clínico es probablemente consecuencia de una:

a. Radiculopatía sensorial L4. b. Radiculopatía sensorial L5 (síndrome ciático). c. Mononeuritis crural. d. Afectación del nervio femorocutáneo (meralgia parestésica).

552. Un paciente de 54 años refiere desde hace 10 días, una o dos crisis de dolor de ojo derecho, con lacrimeo, gran nerviosismo, que le despierta por la noche, le obliga a salir de la cama durándole unas dos horas. ¿Cuál de las siguientes medidas entiende que es más eficaz para calmar el dolor?:

a. Oxígeno intranasal. b. Sumatriptan subcutáneo. c. Ibuprofeno oral. d. Tramadol oral.

553. Un hombre de 60 años presenta un cuadro de hemiataxia cerebelosa de dos semanas de evolución. La RM craneal muestra una imagen en el hemisferio cerebeloso derecho que capta contraste en anillo y desplaza el cuarto ventriculo. El diagnóstico más probable es:

a. Metástasis cerebral. b. Glioblastoma multiforme. c. Linfoma cerebral primario. d. Hemangioblastoma cerebeloso.

554. Una mujer de 43 años, acudió a consulta por un cuadro de fiebre, nerviosismo y dolor cervical anterior. La glándula tiroides estaba agrandada y su palpación era dolorosa. La exploración funcional del tiroides mostró una TSH inhibida y T4 libre elevada. Señale la respuesta correcta:

a. El cuadro sugiere una tiroiditis de Hashimoto que se confirmará por la presencia de Anticuerpos antitiroideos. b. Los síntomas relacionados con la situación de Hipertiroidismo mejoran con los fármacos beta-bloqueantes. En esta entidad no está indicado generalmente el uso de antitiroideos. c. La tiroiditis De Quervain conduce indefectiblemente a un estado de hipotiroidismo crónico. d. La tiroiditis subaguda se caracteriza por un aumento homogéneo de la captación de yodo radiactivo por el tiroides.

555. ¿Qué clase de tiroiditis favorece el desarrollo ulterior de un linfoma?:

a. Tiroiditis de De Quervain. b. Tiroiditis silente. c. Tiroiditis de Riedel. d. Tiroiditis de Hashimoto.

556. ¿Cuál de estas afirmaciones es cierta en relación con el síndrome de Cushing?:

a. El carcinoide bronquial puede secretar ACTH y simular clínicamente un Cushing hipofisario. b. El Cushing de origen hipofisario es más frecuente en hombres que en mujeres. c. La enfermedad de Cushing es la causa más frecuente de síndrome de Cushing. d. En el Cushing por secrección ectópica de ACTH, la administración de CRH ("corticotropin releasing hormone") aumenta los niveles de ACTH.

557. Los tumores adrenocorticales:

a. Se diagnostican con frecuencia al realizar un TAC por otros motivos. b. Son malignos si el peso es superior a 20 gr. c. Se asocian a elevación de S-DHA cuando existe un adenoma. d. Son benignos si el diámetro máximo es inferior a 10 cm.

558. El síndrome carcinoide comprende una serie de manifestaciones derivadas de la secreción de serotonina u otras substancias, por determinados tumores. Señale la respuesta correcta:

a. La aparición de tumores carcinoides bronquiales se relacionan con el hábito de fumar. b. Los tumores carcinoides gástricos, con hi-pergastrinemia son la forma más frecuente de tumores carcinoides. c. El síndrome carcinoide aparece generalmente cuando existen metástasis hepáticas, aunque puede presentarse en tumores bronquiales u ováricos localizados, con acceso directo a la circulación sistémica. d. El diagnóstico del síndrome carcinoide lo confirma la cuantificación de metanefrinas.

559. En relación a la Hiperprolactinemia, señalar la opción correcta:

a. La causa más frecuente es la medicamentosa. b. El tratamiento de elección en el prolactinoma es la resección transesfenoidal. c. No afecta a pacientes varones. d. Dentro de los tumores hipofisarios, el prolactinoma es una afectación poco frecuente.

Page 228: Cuestionario de Urología

560. ¿Cuál de las siguientes asociaciones de fármacos antidiabéticos orales actúa fundamentalmente mejorando la sensibilidad a la insulina?:

a. Acarbosa y Miglitol. b. Biguanidas y tiazolidinadionas. c. Glipizida y Glicazida. d. Clorpropamida y Tolbutamida.

561. Una mujer de 18 años, diabética desde los 13 años y en tratamiento regular e intensivo con insulina y con niveles normales de hemoglobina glicosilada y un peso corporal en el límite bajo de la normalidad, comienza a tener crisis de mareo y sudoración al final de la mañana. ¿Cuál debe ser la actitud clínica?:

a. Pensar en que puede estar pasando un período de "luna de miel" y probablemente se puede retirar la insulina. b. Se debe sospechar que hace hipoglucemias en relación con la aparición de un insulinoma. c. Es posible que haga hipoglucemias y puedan resolverse con un suplemento de dieta a mitad de la mañana. d. A esta edad no hay que pretender normalizar las cifras de glucosa y sin duda habrá que bajar todas las dosis de insulina.

562. Ante un paciente de 45 años que en dos ocasiones, separadas en el tiempo, se le objetivan cifras de glucemia basal (en ayunas) de 118 y 135 mg/l00 mi. ¿Qué actitud adoptaría?:

a. Dichas cifras reafirman la existencia de una diabetes y no se justifican más estudio diagnósticos. b. Indicaría medidas dietéticas: reducción de carbohidratos. c. Realizaría una prueba de tolerancia oral de glucosa. d. Solicitaría la determinación de la hemoglobina glucosilada, previa administración (2 días antes) de corticoides.

563. Paciente de 63 años, alcohólico crónico, que acude a Urgencias por malestar general, dolor en una rodilla y gingivorragia. En la exploración clínica llama la atención la delgadez y presencia de púrpura en ambos miembros inferiores. El hemograma muestra ligera anemia macrocítica. Las cifras de leucocitos y plaquetas son normales. La actividad de protrombina es normal. Señale la causa más probable de su trastorno hemorrágico:

a. Déficit de Vitamina K. b. Púrpura trombocitopénica idiopática. c. Déficit de Vitamina A. d. Déficit de Vitamina C.

564. En una mujer de 55 años intervenida de cáncer de mama tres años antes, con buen estado general, se comprueba una hipercalcemia de 11,1 mg/da. ¿Cuál es la primera prueba a realizar?:

a. Determinación de PTH. b. Determinación de 1,25(OH)2 D. c. Determinación de 250HD. d. Gamrnagrafía ósea.

565. Paciente varón de 27 años de edad, que tras ser diagnosticado hace 2 meses de un feocromocitoma, se le descubre un nódulo tiroideo frío en una gammagrafia tiroidea. En la analítica resalta un aumento desproporcionado de calcitonina plasmática. ¿En qué patología habría que pensar?:

a. Adenoma tiorideo. b. Carcinoma medular de tiroides. c. Carcinoma papilar de tiroides. d. Carcinoma folicular de tiroides.

566. Con respecto a la obesidad, una de las siguientes afirmaciones es INCORRECTA:

a. Es un factor de riesgo cardiovascular. b. Se asocia frecuentemente a diabetes tipo 2. c. La obesidad periférica (acúmulo de grasa en extremidades y glúteos) es la que se asocia a un mayor riesgo cardiovascular. d. Se clasifica basándose en el índice de masa corporal.

567. ¿Qué consejo le parece MENOS adecuado para un hombre de 20 años con colesterol total de 320 mg/dl y triglicéridos de 110 mg/ dl?:

a. En el futuro su riesgo cardiovascular va a ser elevado y debe evitar por todos los medios el hábito tabáquico. b. Sus padres y hermanos deben estudiarse el perfil lipídico pues pueden tener alguna forma de hipercolesterolemia familar. c. Su problema no es relevante por ahora, y no modificar sustancialmente sus hábitos, excepto el tabaco, si es que fuma, volviendo a revisiones anuales a partir de los 35 años de edad. d. Debe procurar limitar el consumo de grasas de origen animal y mantener durante toda su vida un grado de ejercicio físico moderado.

568. Paciente mujer de 48 años, sin antecedentes médicos de interés que presenta cuadro de aproximadamente 3 meses de evolución consistente en: poliartritis de manos, muñecas y rodillas, con rigidez matutina de 2 horas y factor reumatoide elevado en la analítica que le realizó su médico de cabecera. Respecto a su enfermedad, ¿cuál de las siguientes afirmaciones resulta INCORRECTA?:

a. Por la clínica que presenta la paciente padece una artritis reumatoide (AR). b. Parece adecuado comenzar tratamiento con AINEs y/o corticoides a bajas dosis para conseguir alivio sintomático. c. Es importante comenzar lo antes posible tra-tamiento con fármacos modificadores de la enfermedad (FME), incluso en combinación. d. Antes de empezar el tratamiento con FME se debe espe rar otros 3 meses a comprobar la respuesta al tratamiento con AINEs y/o corticoides.

569. A un varón de 65 años asintomático, se le encuentra en un estudio de control general una cifra de fosfatasa alcalina dos veces superior a lo normal, con pruebas hepáticas normales. La gammagrafia ósea muestra captación en la mitad superior de la hemi-pelvis derecha, y una radiografia realizada en esta zona pone de manifiesto un patrón de corticales aumentadas de grosor y patrón trabecular grosero. ¿Qué tratamiento es el más apropiado?:

Page 229: Cuestionario de Urología

a. Alendronato, 40 mg diarios. b. Naproxeno, 500 mg dos veces al día. c. Calcio (400 mg.) y vitamina D (400 UI). d. No es preciso tratamiento.

570. Un joven de 21 años viene a la consulta porque tiene desde hace 4 ó 5 meses un dolor constante en la región lumbosacra, que es peor en las primeras horas del día y mejora con la actividad. Recuerda que 2 años antes tuvo un episodio de inflamación en rodilla que se resolvió completamente. También se queja de dolor en la caja torácica con los movimientos respiratorios. El examen fisico demuestra la ausencia de la movilidad de la columna lumbar. ¿Cuál de los siguientes test nos ayudaría a confirmar el diagnóstico del paciente?:

a. El TAC de columna lumbar. b. Las radiografías de las articulaciones sacroilíacas. c. Los niveles de antiestreptolisina 0 (ASLO). d. La radiografía de tórax.

571. En el lupus cutáneo subagudo es caracteríscica la presencia de anticuerpos:

a. Anti-Sm. b. Anti-centrómero. c. Anti-Ro. d. Anti Jo1.

572. Con relación a la sacoidosis es cierto que:

a. La presencia de granulomas no caseificantes no constituye por sí misma una prueba diagnóstica de la enfermedad. b. La reacción de Mantoux es positiva en el 50% de los casos. c. Es muy característica la presencia de pleuritis yadenopatías paratraqueales derechas. d. Cuando afecta al intersticio pulmonar los volúmenes pulmonares están reducidos y la capacidad de difusión del CO es normal.

573. En una esclerosis sistémica progresiva (esclerodermia) el peor pronóstico se asocia con:

a. Una extensa calcinosis cutánea. b. La presencia de disfagia. c. El número de articulaciones inflamadas. d. La presencia de insuficiencia renal.

574. ¿Cuál de estas afirmaciones es FALSA con respecto a las artritis inducidas por microcristales?:

a. Pueden ser producidas por cualquiera de los siguientes cristales: urato monosódico, pirofosfato cálcico, hidroxiapatita cálcica y oxalato cálcico. b. El depósito de cristales puede producir cua-dros clínicos similares a la artritis reumatoide o la espondilitis anquilosante. c. Los cuadros clínicos producidos son específicos para cada uno de los tipos de cristales depositados. d. Para hacer el diagnóstico es imprescindible el estudio del líquido sinovial con microscopio de luz polarizada para identificar el tipo de cristales.

575. ¿Cuál de las siguientes afirmaciones en relación con la enfermedad de Wegener es FALSA?:

a. Es una vasculitis sistémica que afecta sobre todo a vasos de mediano calibre. b. En ausencia de tratamiento cursa de manera progresiva y con frecuencia mortal. c. Presenta con frecuencia afectación renal, que es histológicamente indistinguible de la glomerulonefritis necrótica con semilunas. d. En presencia de afectación pulmonar y/o renal, el uso de ciclofosfamida vía oral es casi siempre imprescindible para obtener un buen control de la enfermedad.

576. Ante un paciente varón de 41 años, que acude a urgencias con un dolor muy intenso en región lumbar baja, de 12 horas de evolución, y cuya exploración física general es normal. ¿Qué actitud terapéutica está CONTRAINDICADA?:

a. Reposo absoluto en cama durante 10 días. b. Educación postural. c. Control del dolor con analgésicos y/o AINES. d. Ejercicios suaves.

577. Una paciente obesa de 75 años consulta por dolor intenso en la rodilla de 2 semanas de evolución, sin antecedente traumático. Presenta varo bilateral de rodillas, mínimo derrame articular, movilidad completa pero dolorosa, y no aprecian inestabilidades. ¿Cuál de las siguientes, será la etiología más probable?:

a. Meniscopatía. b. Osteocondritis. c. Fractura por estrés de meseta tibial. d. Gonartrosis.

578. Paciente de 27 años de edad con buen estado general y con fracturas desplazadas de tercio medio de cúbito y radio. ¿Cuál es el tratamiento a seguir?:

a. Reducción de las fracturas con anestesia local y yeso durante dos meses. b. Reducción de las fracturas con anestesia general y yeso durante dos meses. c. Osteosíntesis estable y movilización precoz de las articulaciones. d. Osteosíntesis estable y yeso protector. e. Reducción de la fractura con anestesia y vendaje funcional precoz.

579. Un varón de 50 años atropellado respira adecuadamente, no presenta sintomatología torácica, presenta dolor en abdomen inferior, está consciente y. orientado, y su TAC abdominal sólo demuestra una fractura de pelvis con inestabilidad "en libro abierto". Progresivamente inicia frialdad, sudoración, palidez, hipotensión y taquicardia. La actitud más correcta será:

Page 230: Cuestionario de Urología

a. Arteriografía urgente y embolización selectiva de vasos pélvicos con sangrado activo. b. Inmovilización de fractura mediante tracciones y cesta pélvica en cama de arco. c. Perfusión inmediata de cristaloides y expansores del plasma, seguida de estabilización de la fractura pélvica mediante fijador externo. d. Laparotomía. exploradora con ligadura de grandes vasos pélvicos.

580. La pseudoartrosis del foco de fractura es una complicación típica de las fracturas proximales de fémur:

a. Extracapsulares pertrocantéreas. b. Extracapsulares basicervicales. c. Intracapsulares tratadas mediante osteosínteses. d. Intracapsulares tratadas mediante artroplastia de cadera.

581. Un enfermo de 16 años de edad presenta una imagen radiográfica radiolúcida y expansiva de 3 cm de diámetro en el pedículo de la vértebra T1b. Le produce dolor que no se calma con aspirina, ¿en qué lesión tumoral habrá que pensar?:

a. Mieloma. b. Metástasis de cáncer de pulmón. c. Encondroma. d. Osteoblastoma.

582. Un paciente de 21 años, que ha sufrido una caída vertical sobre la pierna manteniendo el tobillo en varo y rotación interna, ha sido diagnosticado de esguince externo del tobillo derecho. ¿Cúal sería la técnica exploratoria más adecuada de las que se relacionan, para evaluar la gravedad del cuadro?:

a. Una Tomografía Axial de la pinza bimaleolar. b. Una Resonacia Nuclear del tobillo. c. Radiografías dinámicas del tobillo previa anestesia local o regional. d. Repetir la radiografía estandar pasados 7 días.

583. Un albañil sufre un accidente laboral precipitándose desde 6 metros de altura. Presenta un importante dolor a nivel lumbar y déficit de extensión contra gravedad de los dedos del pie derecho. Habrá que pensar que puede tener:

a. Una lesión de la raíz L-3 b. Una lesión de la raíz L-4. c. Una lesión de la raíz S-l d. Una lesión de la raíz L-5

584. Señale la respuesta correcta referida a las alteraciones del examen de la orina:

a. La presencia de cilindros hialinos es siempre patológica. b. Los cilindros granulosos contienen albúmina e Inmnoglobulinas. c. Los cilindros leucocitarios son típicos de Glomerulonefritis postestreptocócica. d. Los cilindros hemáticos se presentan en cualquier discrasia sanguínea.

585. Paciente de 68 años diabética, que consulta por malestar general. En la gasometría venosa destaca pH 7.25, Bicarbonato 15 mmolJl (normal 24-28 mmol/l). Hiato anionico (anión GAP): 11 mmol/l (normal 10-12 mmol/l). ¿Cúal de las siguientes entidades NO descartaría como diagnóstico?:

a. Cetoacidosis diabética. b. Insuficiencia renal crónica. c. Acidosis tubular renal. d. Ingesta de salicilatos.

586. Cuando un paciente bajo tratamiento diurético con tiazidas o furosemida incumple la dieta y come más sal de la prescrita, el resultado análitico esperable es:

a. Mayor hipematremia. b. Mayor hiponatremia. c. Mayor hiperpotasemia. d. Mayor hipopotasemia.

587. En la uremia pre-renal:

a. El sodio en orina es superior a 60 mEq/l, la osmolaridad urinaria es superior a 500 mOsm/ kg H2O y la relación urea en orina/urea en plasma es superior a 8. b. El sodio en la orina es inferior a 20, la osmolaridad urinaria es inferior a 200 mOsm/kg H2O y la relación entre urea en orina / urea es plama es inferior a 2 c. El sodio en orina es inferior a 20 mEq/l, la osmolaridad en orina es inferior a 200 mOsm/Kg H2O y la relación urea en orina / urea en plama es superior a 8. d. El sodio en orina es inferior a 20 mEq/l, la osmolaridad urinaria es superior a 500 mOsm/Kg H2O, y la relación entre la urea en orina y la urea en plasma es superior a 8.

588. Un hombre, adicto a drogas por vía parenteral, está ingresado por endocarditis infecciosa. Durante su enfermedad presenta un cuadro de glomerulonefritis aguda. ¿Cuál de las respuestas es INCORRECTA?:

a. Suele ser debida a inmunocomplejos. b. No suele presentar piuria. c. El complemento está descendido. d. A veces produce síndrome nefrótico.

590. Uua mujer de 68 años acude al Servicio de Urgencias por malestar general que ha ido progresaudo en los últimos 15 días, a partir de uu episodio gripal. Ha notado disminución progresiva del volumen de diuresis, edemas maleolares y dificultad respiratoria. Es hipertensa. En la analítica destaca una creatinina plamática de 5 mg/dl, urea 180 mg/dl, Na 138 mEq/l, K 4.9 mEq/a. Las cifras de complemento son normales. Los anticuerpos anti-membrana basal son negativos. En la orina presenta cilindros hemáticos,

Page 231: Cuestionario de Urología

proteinuria de 1 g/l y microhematuria. Aporta una analítica de un mes antes, sin alteraciones. ¿Cuál de los siguientes diagnósticos es más probable?:

a. PAN microscópica. b. Brote lúpico. c. Glomerulonefritis aguda postinfecciosa. d. Crioglubulinemia.

591. Con respecto a la nefropatía diabética, señalar la respuesta FALSA:

a. La diabetes tipo 2 es la etiología más frecuente de insuficiencia renal terminal en el mundo occidental. b. Más del 90% de los diabéticos tipo 1 desarrollan nefropatía a los 30 años del diagnóstico de diabetes. c. La alteración renal más temprana es la hiperfiltración. d. La existencia de microalbuminuria predice el desarrollo de nefropatía clínica.

592. Un paciente de 75 años de edad, con historia antigua de hipertensión arterial, de hábito asténico, consulta en la Urgencia del Hospital por un cuadro de dolor en fosa lumbar derecha con hematuria de 6 horas de evolución. La tensión arterial es del 120/80 mmHg, y el paciente está agitado, nauseoso y sudoroso, con el pulso irregular a 80 latidos por minuto. Analíticamente tiene una urea de 5O mg/dl, creatinina 1 mg/dl, GOT 120 mU/mI, GPT 35 mU/mI, amilasa 120 mU/mI, LDH 1100 mU/mI. En el sedimento urinario hay microhematuria. La radiografia de torax muestra una discreta cardiomegalia a expensas de cavidades izquierdas y el electrocardiograma muestra una frecuencia auricular de 250 latidos minuto, con una respuesta ventricular irregular a 75 latidos por minuto. ¿Cuál es, entre las siguientes, la decisión más terminante?:

a. Calmar ante todo el dolor y la ansiedad. b. Hacer una ecografía abdominal. c. Hacer un TAC espiral con contraste. d. Acidificar la orina.

593. Señale lo que considere INCORRECTO en la consideración del emparejamiento donante receptor de trasplante renal:

a. En España se realizan trasplantes de donante cadáver fundamentalmente. b. El donante debe hallarse libre de infecciones virales activas y neoplasias potencialmente transmisibles para poder utilizar los riñones. c. Para la realización de un trasplante renal debe exigirse una compatibilidad igual o superior a cuatro identidades sobre los 6 antígenos HLA. d. No se deben realizar trasplantes renales con incompatibilidad ABO.

594. Con respecto a la litiasis renal, una de las siguientes parejas es INCORRECTA:

a. Acido úrico-radiotransparente. b. Oxalato cálcico-radioopaca. c. Sulfarnidas-radiotransparentes. d. Indinavir-radioopaca.

595. Un varón de 68 años presenta dolor flanco izquierdo. La Urografia intravenosa demuestra un riñón derecho normal y anulación funcional del izquierdo. La tomografia computarizada (TC) muestra una masa renal sólida de 8 cm en la región hiliar del riñón izquierdo con probable presencia de trombo en la vena renal. La radiografía de tórax y la bioquímica sanguínea son nor-males. El siguiente tiaso será:

a. Venografía seguida de cavografía. b. Arteriografía renal selectiva. c. Pielografía retrógrada d. Resonancia magnética nuclear.

596. En un paciente con cáncer de próstata confinado en la glándula prostática con Gleason inferior a 6 y PSA inferior a 10, son opciones terapeúticas indicadas las siguientes EXCEPTO:

a. Prostatectornía radicaa. b. Braquiterapia. c. Radioterapia externa. d. Bloqueo androgénico.

597. ¿Cuál de las siguientes enfermedades no produce elevación en la alfafetoproteina sérica?:

a. Carcinoma embrionario testicular. b. Tumor del seno endodérmico testicular. c. Ataxia-Telangiectasia. d. Seminoma testicular.

598. Varón de 78 años sexualmente activo que acude a nuestra consulta por impotencia de reciente comienzo. Sus erecciones han sido progresivamente menos intensas hasta impedir la penetración. Tiene antecedentes de cardiopatía isquémica e hipertensión y ha estado tomando aspirina y atenolol desde hace años. No es diabético. ¿ Cuál de las siguientes es la causa más probable de la disfunción eréctil de este paciente?:

a. Atenolol. b. Descenso de los niveles de testosterona. c. Neuropatía. d. Enfermedad vascular.

599. ¿Cuál de los siguientes apartados es FALSO en relación a la anemia perniciosa?:

a. Se produce por un déficit de Factor Intrínseco. b. Con frecuencia aparecen alteraciones neurológicas. c. La prueba diagnóstica de elección en la "prueba de Schilling". d. En el hemograma encontramos anemia severa con VCM elevado y reticulocitos altos.

Page 232: Cuestionario de Urología

600. Un paciente de 63 años con leucemia linfática crónica en estadio A diagnosticada hace seis meses, acude a Urgencias por un cuadro de ictericia desde hace 48 h Y cansancio. La exploración fisica sólo revela algunas microadenopatías cervicales y un leve soplo sistólico polifocal. El hemograma revelaba Leucocitos: 36.100/mm 3 , (Linfocitos 87%), Hb: 6,7 g/di, VCM: 105, Plaquetas: 216.000/mm 3 , Bilirrubina total: 5,3 mg/dl, Bilirrubina directa: 0,7 mg/dl, LDH:I050 UI/l, ALT: 37 UI/l, GGT:39 UI/l, Fosfatasa alcalina: 179 UI/a. Indique cúal de las siguientes pruebas o combinación de pruebas serán de mayor utilidad diagnóstica:

a. Ecografía hepática. b. Sideremia, ferritina, B12, y ácido fólico. c. Test de Coombs directo. d. Niveles de reticulocitos.

601. ¿Cuál de las siguientes afirmaciones es cierta en el trasplante hematopoyético?:

a. La morbi-mortalidad del trasplante alogénico es menor que la del autólogo. b. El rechazo del implante es la principal complicación del trasplante alogénico. c. En el trasplante autólogo se producen más recidivas de la enfermedad de base que en el alogénico. d. Casi el 80% de los pacientes dispone de un donante compatible.

602. Los pacientes con Púrpura Trombopénica Autoinmune, se tratan inicialmente exclusivamente con corticosteroides, pero en situaciones especiales se asocia al tratamiento altas dosis de garnmaglobulina por vía endovenosa. ¿En cuál de las siguientes situaciones puede estar indicado el uso de garnma-globulina?:

a. Pacientes mayores de 60 años. b. Cuando la cifra de plaquetas es inferior a 5 x 10 9 /L. c. En las pacientes gestantes con Púrpura Trombopénica Autoinmune. d. Brotes hemorrágicos graves.

603. ¿Cuál es el oncogén implicado en la resistencia a la apoptosis que participa en la patogénesis del linfoma folicular y se activa mediante la traslocación t(14,18)?:

a. BCL 1 b. BCL 2 c. BCL 6 d. BAD

604. El tratamiento de inducción de la leucemia promielocítica aguda está basado en la siguiente combinación:

a. Arabinósido de citosina y una antracic1ina. b. Arabinósido de citosina, antracic1ina y etopósido. c. Ácido retinoico yantracic1ina. d. Vincristina-antracic1ina y prednisona.

605. En relación con las alteraciones genéticas asociadas al desarrollo de patología trombótica, es decir los denominados Estados de Hipercoagulabilidad o Trombofilias, señale la afirmación FALSA:

a. La deficiencia de la Antitrombina ID es el estado de trombofilia más frecuente de la población occidental. b. La deficiencia de Proteína C y la deficiencia de Proteína S pueden ir asociadas en ocasiones. c. La ingesta de contraceptivos orales incrementa en muchas de estas situaciones el riesgo tromboembólico. d. El polimorfismo responsable del cambio Arg/Glu 506 en el factor V se conoce como Factor V Leiden.

606. Un paciente de 35 años de edad acude al servicio de urgencias por presentar fiebre elevada de hasta 39°C, junto con confusión mental. En la exploración física destaca la existencia de lesiones cutáneas en pie izquierdo a nivel distal, maculares, de milímetros de diámetro, de aspecto isquémico hemorrágico .La auscultación cardiopulmonar es normal. A los pocos día se obtiene crecimiento de Staphylococo aureus meticilin sensible en tres hemocultivos de los tres extraídos. ¿Cuál de las siguientes sería la actitud correcta a seguir en ese momento?:

a. Considerar el resultado de los hemocultivos como probable contaminación. b. Pautar de inmediato tratamiento antibiótico con penicilina y gentamicina durante 10 días. c. Comenzar tratamiento con cloxacilina y gentamicina y realizar estudio ecocardiográfico por la existencia probable de endocarditis aguda. d. Descartar la existencia de endocarditis por la ausencia de soplos en la auscultación cardiaca y buscar focos de posible osteomielitis.

607. Señale la cierta, en relación a la encefalitis herpética:

a. Está producida casi siempre por el virus del herpes simple tipo 2. b. La clínica característica es la presencia de fiebre con síntomas focales del lóbulo occipital. c. La PCR en el líquido cefalorraquídeo no es útil para establecer el diagnóstico. d. Las secuelas neurológicas son frecuentes.

608. ¿Cuál de los siguientes cuadros clínicos NO se asocia con toxoplasmosis?:

a. Abscesos cerebrales en personas con SillA. b. Miocarditis en transplantados cardiacos. c. Coriorretinitis en adultos inmunocompetentes. d. Colitis asociada a inmunodeficiencia humoral.

609. Señale la afirmación INCORRECTA con respecto a Haemophilus influenzae:

a. Es un parásito obligado de las mucosas humanas. b. Es un bacilo grarnnegativo pequeño y pleomórfico. c. La vacunación es de poca utilidad porque no cubre el serotipo B, que es el más prevalente en nuestro ambiente. d. Las infecciones no sistémicas (otitis, bronquitis, conjuntivitis,...) están producidas generalmente por serotipos diferentes al B.

610. Señale la cierta respecto a la Fiebre Q:

Page 233: Cuestionario de Urología

a. Se transmite por inhalación de partículas contaminadas. b. Los hemocultivos son positivos en la fase inicial. c. La mancha negra se observa en el 60% de los casos. d. El exantema suele afectar palmas y plantas.

611. Varón de 47 años de edad, fumador de 20 cigarrillos al día, que consulta por fiebre, expectoración purulenta y dolor pleurítico de dos días de evolución. La Rx de tórax muestra una condensación lobar derecha y en el hemocultivo se aisla un "Coco Gram positivo en cadena". Se inicia tratamiento con ceftriaxona 2g/24 i.v. A las 72 h. el paciente persiste febril a 39°C y con afectación del estado general. ¿Cuál es la causa más probable de la mala evolución?:

a. Dosis de ceftriaxona insuficiente. b. Bacteria responsable (probablemente Streptococcusp neumoniae) resistente al tratamiento. c. Proceso complicado por una abscesificación de la condensación. d. Presencia de un empiema pleural.

612. Una paciente de 42 años consulta por presentar, tres horas después de la ingesta de un pastel de crema en un restaurante, un cuadro de vómitos y deposiciones diarreicas sin productos patológicos ni fiebre. Mientras que la paciente es visitada, su marido inicia un cuadro similar. ¿Cuál de los siguientes microorganismos es probablemente el responsable del cuadro clínico?:

a. Salmonella enteriditis. b. Shigella sonnei. c. Staphylococcus aureus. d. Campylobacter jejuni.

613. Día: sábado. Lugar: urgencias de un hospital comarcal. Paciente con síndrome febril y sopor a la vuelta de Mali. El laboratorio es incapaz de hacer con garantías una prueba para diagnóstico de paludismo. La actitud a seguir más lógica sería:

a. Dar un tratamiento sintomático y esperar el lunes a que lo vea alguien más experto. b. Tratar como si fuera una infección por P.Vivax. c. Administrar una cefalosporina de 3" generación y esperar la evolución. d. Hacer una gota gruesa para enviar a centro especializado y tratar como si fuera una infección por P.falciparum.

614. Un paciente infectado por el VIH recibe tratamiento antirretroviral con AZT + 3TC + Efavirenz, desde hace 14 meses. La última determinación de linfocitos CD4 y carga viral era de 350/mm3 y <200 copias/mi respectivamente. Consulta por disnea y palidez y se constata una anemia microcítica (Hemoglobina: 7,8 gr/dl, VCH 68 fl). El diagnósticomás probable es:

a. Crisis aplástica por Parvovirus B. b. Leishmaniasis diseminada. c. Infección diseminada por Mycobacterium avium intracellulare. d. Hemorragia digestiva crónica.

615. Un paciente de 40 años diagnosticado de infección por VIH hace 10 años que no sigue tratamiento antirretroviral presenta síntomas compatibles con candidiasis esofágica y además refiere un cuadro de 10 días de evolución de cefalea, fiebre, vómitos. En las últimas 24 horas presenta disminución del nivel de conciencia; la exploración fisica muestra confusión y rigidez de nuca, el TAC de cráneo es normal y en la punción lumbar existe una presión de apertura elevada, no se ven células y las proteinas son del 300 mgIda. El cuadro es compatible con:

a. Hipertensión intracraneal benigna. b. Hidrocefalia. c. Meningitis tuberculosa. d. Meningitis criptocócica.

616. Señalar la FALSA respecto a la leucoencefalitis multifocal progresiva en paciente con infección por VIH

a. Ocurre en un 4% de pacientes con SIDA. b. El agente causal es el virus del papiloma humano. c. No se conoce ningún tratamiento específico. d. Los pacientes suelen presentar defectos focales múltiples sin alteraciones de la conciencia.

617. Una mujer de 67 años con antecedentes de polineuropatía periférica ha sido recientemente diagnosticada de Arteritis de células gigantes y puesta en tratamiento con 60 mg al día de prednisona. La radiograf'rn de tórax es normal y el test cutáneo con 5U de PPD es de 18 mm de induración. ¿Qué actitud tomaría con esta paciente?:

a. Hay que tratar con isoniacida durante 3meses. b. Esperar la finalización del tratamiento esteroideo y después realizar quimioprofilaxis. c. No está indicado utilizar quimioprof1laxis. d. Está indicado utilizar isoniacida más piridoxina entre 6 y 9 meses.

618. Acerca del tratamiento de la tuberculosis:

a. El régimen de elección es isoniacida +rifampicina+etambutoa. . b. En la mayoría de los pacientes, son mejor 4 fármacos que 3 fármacos. c. Todos los tratamientos' que duren menos de 9 meses se asocian con una tasa muy elevada de recidivas. d. La asociación de pirazinamida durante los dos primeros meses, permite acortar la duración del tratamiento a 6 meses.

619. Varón de 30 años de edad, sexualmente activo que presenta disuria .y secreción uretral purulenta. En la tinción de esta última se observan diplococos gramnegativos intracelulares. ¿Cuál cree que sería el tratamiento de elección?:

a. Penicilina benzatina. b. Doxiciclina. c. Azitromicina. d. Ceftriaxona.

620. La radioterapia con quimioterapia simultánea o concurrente NO es el tratamiento de elección en estados localmente avanzados de:

Page 234: Cuestionario de Urología

a. Carcinoma epidermoide de canal anaa. b. Carcinoma epidermoide de esófago. c. Adenocarcinoma de próstata. d. Carcinoma no microcítico de pulmón.

621. Una mujer de 90 años demenciada, incontinente e incapacitada por hemiparesia, presenta una úlcera sacra.. En la exploración vemos que está en la cama sobre una almohadilla húmeda y con una sonda de alimentación que está bien colocada. Está afebril y tiene un pulso y una tensión arterial normales. La úlcera sacra de 4x4 cm se extiende hacia la fascia, con exudado verde y piel normal que rodea a la úlcera. ¿Cuál es la primera prioridad en los cuidados de esta paciente?:

a. Empezar tratamiento con antibióticos. b. Cultivar el exudado del decúbito. c. Aplicar vendajes semihúmedos de solución salina tres veces al día. d. Hacer cambios posturales a la paciente cada dos horas.

622. Un paciente de 52 años de edad con cáncer de pulmón metastásico en situación terminal recibe tratamiento habitual con morfina oral de liberación retardada 30 mg cada 12 horas. En este momento se encuentra en situación de agonía y no puede deglutir. ¿Cuál sería la dosis adecuada de morfina subcutánea en esta situación, teniendo en cuenta que su función renal previa era normal?:

a. 30 mg cada 12 horas. b. 5 mg cada 4 horas. c. 10 mg cada 8 horas. d. 15 mg cada 6 horas.

623. En el manejo de los "estertores de la agonía" está indicado:

a. El empleo de la fisioterapia respiratoria. b. Forzar la hidratación con sueroterapia. c. El empleo de la aspiración nasofaríngea continuada. d. La administración de anticolinérgicos.

624. ¿Cuál de las siguientes lesiones melanocíticas benignas muestra un mayor riesgo de degenelr en melanoma maligno?:

a. Un nevo azul celular. b. Un nevo melanocítico de palmas o plantas. c. Un nevo melanocítico congénito gigante. d. Un nevo melanocítico muy hiperpigmentado.

625. ¿Cuál de las siguientes afirmaciones sobre la alopecia areata es FALSA?:

a. Es una alopecia cicatricial irreversible. b. Se asocia a enfermedades autoinmunes. c. Puede afectar a las uñas. d. Puede afectar cualquier zona pilosa.

626. Una mujer de 64 años, hipermétrope y con cataratas en ambos ojos, acude a su consulta con dolor intenso en ojo izquierdo, de unas horas de evolución. La exploración de ese ojo pone de manifiesto una tensión ocular de 40 mmHg, reacción hiperémica cilioconjuntival, midriasis y edema comeal. ¿Cuál sería la actitud inmediata más correcta de las que se enumeran a continuación? :

a. Manito1 intravenoso asociado a tratamiento miótico y corticoides tópicos. b. Practicar una trabecu1ectomía. c. Corticoides tópicos asociados a tratamiento midriático. d. Operar con urgencia la catarata de ese ojo, desencadenante del cuadro.

627. Una niña de 7 años, que presenta ceguera nocturna (hemeralopia) en la exploración oftalmológica se observa constricción del campo visual con escotoma anular, pérdida de la agudeza y electrorretinograma anómalo. ¿Qué enfermedad ocular, de las que a continuación se relacionan, puede presentar?:

a. Persistencia de vítreo primario. b. Catarata congénita o infantil. c. Aniridia bilateral. d. Retinosis pigmentaria (retinitis pigmentaria).

628. Un paciente de 15 años acude a la consulta aquejando dolor en ojo derecho. A la inspección se detecta la existencia de blefarospasmo en ojo derecho. ¿Cuál de las que a continuación se relacionan, será más probablemente la enfermedad que padece?:

a. Enfermedad de Graves-Basedow. b. Neuropatía óptica derecha. c. Queratitis. d. B1efaritis.

629. Varón de 28 años que presenta en su ojo izqnierdo cuadro de vasculitis retiniana severa y edema macular con una agudeza visual O.L El paciente refiere historia de altas bucales y genitales recidivantes. En base a los hallazgos clínicos y a la historia del paciente ¿cuál considera, de los signientes, que es el diagnóstico más probable?:

a. Sarcoidosis. b. Enfermedad de Behçet. c. Esclerosis múltiple. d. Tuberculosis.

630. Varón de 65 años, con antecedentes de hipertensión arterial en tratamiento, queacu-de a la consulta por notar pérdida brusca de agudeza visual en su ojo izqnierdo hace 24 horas. A la exploración oftalmológica la agudeza visual es de percepción de luz y a la oftalmoscopia se evidencia cuadro sugestivo de embolia de la arteria central de la retina. ¿Qué exploraciones complementarias

Page 235: Cuestionario de Urología

efectuaría al paciente desde el punto de vista sistémico para completar el estudio diagnóstico?:

a. Resonancia nuclear magnética cerebral. b. Punción lumbar. c. Doppler carotídeo y ecocardiograma. d. Arteriografía cerebral.

631. Señale la afirmación correcta en referencia a los tumores de glándulas salivales:

a. Los tumores que asientan en la glándula parótida tienen mayor probabilidad de ser malignos que los que asientan en las glándulas salivales menores. b. Es frecuente que los tumores benignos de parótida produzcan una paresia o parálisis facial debido a la elongación del nervio, que se relaciona estrechamente con la glándula en parte de su trayecto. c. El tumor de Warthin es un cistoadenocarcinoma papilífero que afecta preferentemente la glándula submaxilar con preponderancia en el sexo femenino. d. El adenoma pleomorfo es el tumor benigno más frecuente en la glándula parótida.

632. Varón de 57 años, que consulta por sensación de cuerpo extraño faríngeo de semanas de evolución, disfagia y otalgia derecha. A la palpación cervical, no se aprecian adenopatías, y mediante laringoscopia indirecta se aprecia una neoformación ulcerada a nivel del repliegue aritenoepiglótico derecho, con movilidad conservada de las cuerdas vocales. Se solicita una TAC donde se conf'mna la citada lesión que invade la cara laringes de la epiglotis y el repliegue aritenoepiglótico derecho, sin apreciarse adenopatías sospechosas de malignidad. Señale la respuesta correcta:

a. En los países anglosajones predominan los carcinomas de localización supraglótica sobre los de glotis y subglotis. b. La probabilidad de que este paciente presenta adenopatías metastásicas es inferior al 5%. c. La presencia de metástasis ganglionares cervicales es el principal factor pronóstico en los pacientes con carcinoma epidermoide de cabeza y cuello. d. En este paciente (estadio T2) la laringuectomía horizon tal supraglótica, conllevaría peores tasas de control local de la enfermedad que la laringuectomía total.

633. La presencia de una otitis secretora unilateral en un adulto en el que aparece una adenopatía cervical alta ipsilateral dura, de 2,5 cm. di diámetro, obliga a realizar inicialmente:

a. Biopsia de la adenopatía. b. Vacunación anticatarral. c. Exploración del cavum. d. Estudio alergológico.

634. ¿Cuál es lugar de origen más frecuente de los tumores malignos de nariz y senos paranasales?:

a. Tabique nasal. b. Fosa nasal. c. Seno esfenoidal. d. Seno maxilar.

635. ¿Cuál es el principal tratamiento psicofarmacológico del trastorno obsesivo-compulsivo?:

a. Antipsicóticos. b. Benzodiacepinas. c. Antidepresivos inhibidores de la recaptación de noradrenalina. d. Antidepresivos inhibidores de la recaptación de serotina.

636. ¿Cuál de los siguientes aspectos NO es característico del tratamiento con sales de litio?:

a. Precisa para su control de determinaciones de litemia. b. Es eficaz en el tratamiento del episodio maníaco. c. Eleva los niveles de uricemia. d. Es eficaz en la profIlaxis del trastorno bipolar.

637. Una joven de 23 años acude al servicio de urgencias con varios cortes superficiales en la cara interna de ambos antebrazos, y en un estado de somnolencia y torpor que hace suponer la ingesta reciente de psicofármacos o substancias psicoactivas. La paciente alega que estaba muy nerviosa ("a punto de explotar") y que había ido tomando tranquilizantes sin encontrar mejoría, hasta que acabó autoinflingiéndose los cortes para paliar la tensión interna. Un episodio así es frecuente en:

a. La esquizofrenia. b. El trastorno esquizotípico de la personalidad. c. El retraso mental. d. El trastorno límite de la personalidad.

638. En la esquizofrenia desorganizada, una de las siguientes afirmaciones es INCORRECTA:

a. Es sinónimo de hebefrenia. b. Tiene mejor pronóstico a largo plazo que la esquizofrenia paranoide. c. Existe desorganización en el lenguaje. d. La afectividad está alterada y básicamente es aplanada.

639. En relación al tratamiento de la depresión, una de las siguientes afirmaciones es INCORRECTA:

a. La terapia electroconvulsiva no tiene indicación en las depresiones neuróticas. b. El litio se utiliza en los cuadros bipolares. c. Los IMAOS están indicados en depresiones atípicas. d. El tiempo medio de mantenimiento de la medicación antidepresiva es de 6 semanas.

640. Hablamos de "depresión doble" cuando:

a. El paciente presenta un trastorno orgánico severo asociado a su depresión.

Page 236: Cuestionario de Urología

b. Existe una comorbilidad del cuadro depresivo con un trastorno por dependencia a sustancias tóxicas. c. Se sobreimponen episodios depresivos mayores sobre un trastorno distímico. d. Existe un riesgo de viraje a fase maníaca ya que en realidad se trata de un trastorno afectivo bipolar.

641. ¿Cuál de las siguientes alteraciones NO aparecen en la anorexia nerviosa?:

a. Aumento de los niveles de colesterol que no se relaciona con la ingesta de grasas. b. Aceleración del vaciado gástrico. c. Disminución del aclaramiento de creatinina secundario a la hipovolemia. d. Alteraciones del EKG que guardan relación con las pérdidas de potasio.

642. ¿Cuál de las siguientes afirmaciones sobre los trastornos somatomorfos NO es correcta?:

a. El trastorno de somatización suele desencandenarse en personas con dificultad para expresar verbalmente sus emociones. b. El trastorno por dolor es una enfermedad que puede llegar a ser incapacitante. c. La hipocondría es el miedo a contraer o padecer una enfermedad. Si esta idea cumple criterios de idea delirante no se diagnosticaría de hiponcondría sino de trastorno delirante. d. Al igual que el trastorno facticio, los trastornos somatoformos se caracterizan por una producción deliberada de uno o un conjunto de síntomas.

643. Un hombre de 85 años de edad que vive con su hija y yerno, es traído al servicio de urgencias por alteración del nivel de conciencia. El yerno refiere que lleva encarnado 2 días y que en las últimas semanas ha estado cada vez más paranoico con ideas de envenenamiento. La exploración física demuestra úlceras en sacro avanzadas y equimosis bilateral en ambos brazos. ¿Cúal de los siguientes es el diagnóstico más importante que debe reconocerse en este caso?:

a. Sepsis urinaria. b. Abuso del anciano. c. Demencia. d. Síndrome de abstinecnia de medicación previa.

644. Una mujer de 22 años de edad es traída al servicio de urgencias en estado comatoso después de una crisis convulsiva, con TA de 80/40 y PA 148 l/m. En el ECG, la duración del QRS es 280 mseg. Había estado deprimida y comenzado a tomar nortriptilina 2 semanas antes. ¿Cúal de los siguientes es el tratamienteo inicial más apropiado?:

a. Coger vía venosa y administrar bicarbonato sódico intavenoso. b. Coger vía venosa, lavado gástrico, y diazepam para controlar las convulsiones. c. Control de la vía aérea, coger vía venosa y administrar carbón activado por sonda nasogástrica. d. Control de la vía aerea y ventilación mecánica, coger vía venosa y administrar bicarbonato sódico intravenoso.

645. Los gemelos unidos se forman cuando:

a. La división del óvulo fertilizado ocurre en la fase de mórula. b. Entre el 4° y 8° día postfecundación antes de la diferenciación de las células del arnnios. c. La división ocurre después de la formación del disco embrionario. d. Si la división ocurre cuando el arnnios ya está establecido, 8 días después de la fertilización.

646. ¿Cuál de las siguientes afirmaciones es FALSA en relación con la transmisión vertical al feto/recién nacido por parte de la gestante con infección HIV:

a. Aumenta el riesgo de malformaciones fetales. b. Tiene lugar sobre todo durante el parto. c. La lactancia aumenta el riesgo de transmisión entre un 10 y un 20%. d. Disminuye con tratamiento antirretroviral durante el embarazo y especialmente durante el parto.

647. Gestante de 24 semanas que acude a la consulta porque a su hijo de 4 años le diagnosticaron hace 5 días la varicela. La paciente no recuerda si padeció la enfermedad en la infancia, pero sí sabe que no fue vacunada y está muy preocupada por la posible afectación fetal. ¿Qué actuación sería la correcta?:

a. Administrar la vacuna específica. b. Solicitar cuantificación de Ig G, y si fuera negativo, administrar la garnmaglobulina específica. c. Tranquilizar a la paciente informándole de la ausencia de riesgos fetales. d. Administrar aciclovir oral a dosis de 800 mg, 5 veces al día, durante 5-7 días.

648. En la utilización de los corticoides en la Rotura Prematura de las Membranas, antes de las 34 semanas, las siguientes afirmaciones son ciertas MENOS UNA. Indique cual:

a. Aumenta la mortalidad perinatal por infección. b. Disminuye la aparición del distrés respiratorio del RN. c. Disminuye el riesgo de hemorragia intra/periventricular. d. Disminuye el riesgo de enterocolitis necrotizante.

649. ¿Dónde se produce la 16-alfahidroxilación de la hormona DehidroepiandrosteronaSulfato, paso necesario para la síntesis de Estriol por la placenta?:

a. En el sincitiotrofoblasto. b. En el citotrofoblasto. c. En las glándulas suprarrenales fetales. d. En el hígado fetal.

650. Respecto al la incompatibilidad materno-fetal en el sistema ABO de grupos sanguíneos, es cierto que:

a. Afecta a primogénitos. b. Es más grave que la isoinmunización D. c. La detección prenatal es muy importante porque se asocia a anemia fetal severa. d. La prueba de Coombs es positiva.

Page 237: Cuestionario de Urología

651. En el manejo de la Eclampsia son correctas todas EXCEPTO:

a. Administración de Sultato de Magnesio i.v. b. Colocar a la paciente en decúbito lateral. c. Establecer una vía aérea y administrar oxígeno. d. Parto inmediato mediante cesárea.

652. La utilización de anticonceptivos orales combinados de estroprogestágenos puede producir los siguientes efectos beneficiosos, EXCEPTO uno:

a. Mejoría de la anemia secundaria a pérdida menstrual abundante. b. Disminución del riesgo de padecer cáncer de ovario. c. Disminución del riesgo de enfermedades de transmisión sexual de origen bacteriano. d. Disminución del riesgo de cáncer de cérvix.

653. Una paciente de 19 años, nuligesta, acude a la consulta por irritabilidad, inestabilidad emocional, cefaleas, sensibilidad y distensión mamaria. La sintomatología se iuicia una semana antes de la mestruación y desaparece tras presentarse la regla. Una vez comprobada la relación con el ciclo menstrual mediante un diario de síntomas, decidimos pautar tratamiento, pues la paciente refiere que la sintomatología interfiere con su trabajo y sus relaciones sociales. ¿Qué aconsejamos como primera elección terapeútica?:

a. Aumento de ingesta de cafeína y azúcares refinados en segunda fase del ciclo menstrual. b. Progesterona 200-300 mg/día 14 al 23 del ciclo. c. Evitar ejercicio físico y tomar suplemento de vitamina Bl2 durante sintomatología a una dosis de 150 mg/día. d. Alparazolam 0,25 mg/8 horas, desde el día 20 del ciclo hasta el 2° día de la regla, disminuyendo des pués a una toma al día.

654. Paciente de 74 años que tras hemorragia uterina postmenopáusica es diagnosticada de adenocarcinoma de endometrio. Se realizó tratamienteo quirúrgico consistente en histerectomía abdominal total con salpingo-oo-forectomía bilateral más toma para citología peritoneal. Tras estudio anatomopatológico se determina que la paciente se encuentra en un estadio I con grado de diferenciación histológico 1 . ¿Cuál sería la actitud correcta?:

a. Administrar radioteraparia pélvica (4500-5000 cGy dosis total). b. Administrar quimioterapia con progestágenos. c. Realizar controles periódicos sin tratamiento adicional. d. Completar cirugía con linfadenectomía pélvica.

655. El aumento de la resistencia a la insulina es un problema asociado frecuentemente a:

a. Mioma uterino. b. Endometriosis. c. Adenorna hipofisario productor de prolactina. d. Síndrome de ovarios poliquísticos, debido al exceso de síntesis de andrógenos.

656. Señale, de las que a continuación se relacionan, qué otra patología puede presentar ligada a alteraciones genéticas, una paciente afecta de carcinoma ovárico familiar o hereditario:

a. Ovario poliquístico. b. Ca. de pulmón. c. Endometriosis. d. Cáncer colorrectal familiar sin pólipos.

657. Señale, de los que a continuación se refieren, cuál es el oncogén implicado en la patogénesis del cáncer de mama que se utiliza actualmente como una diana terapéutica mediante un anticuerpo monoclonal específico frente a este oncogén:

a. BRCA1. b. ERBB2 ó HER-2. c. ATM. . d. K-RAS.

658. Una mujer de 60 años ha sido intervenida mediante tumorectomía de un carcinoma de mama. Se trataba de un tumor de 2 cm. con ganglios libres y receptores hormonales negativos. ¿Cúal de las siguientes opciones de tratamiento recomendaría?:

a. Quimioterapia con un taxano. b. Radioterapia adyuvante. c. Tamoxifén adyuvante. d. Revisiones anuales los dos primeros años.

659. ¿Cuál es, de los siguientes, el más útil y eficaz factor de pronóstico en el cáncer de mama?:

a. Negatividad de los receptores de estrógenos en las células tumorales. b. Presencia de focos de componente intraductal en puntos distantes al tumor. c. Invasión de ganglios linfáticos. d. Presencia de mutación en el gen BRCA1.

660. Un recién nacido con grave distress respiratorio presenta imágenes aéreas circulares que ocupan hemitórax izquierdo. El diagnóstico más probable es:

a. Pulmón poliquístico. b. Malformación adenomatoidea. c. Quiste pulmonar multilocular. d. Hernia diafragmática.

661. En el estreñimiento funcional del niño, ¿cuál de las siguientes afirmaciones es FALSA?:

a. Los pacientes presentan con frecuencia rectorragia leve. b. La asociación con encopresis en infrecuente.

Page 238: Cuestionario de Urología

c. La desimpactación de las heces es necesaria al inicio del tratamiento. d. Los niños retienen voluntariamente las heces para evitar el dolor de la defecación.

662. Niño de 8 años con cianosis, acropaquias, disnea, soplo cardíaco y corazón pequeño en la Radiografía de tórax. El diagnóstico es:

a. Coartación de aorta. b. Conducto arterioso persistente. c. Tetralogía de Fallot. d. Comunicación interauricular.

663. De las siguientes alteraciones del desarrollo en niños, señale cuál es la de mayor prevalencia:

a. Parálisis cerebral. b. Trastorno visual. c. Déficit de atención/trastorno de hiperactividad. d. Retraso mental.

664. En la quimioprofllaxis de la enfermedad meningocócica, ¿cuál el el fármaco alternativo a la Rifampicina en el niño?:

a. Ceftriaxona. b. Amoxicilina -clavulánico. c. Eritromicina. d. Vancomicina.

665. La enfermedad celíaca es una intolerancia permanente al gluten. ¿Cuál de las siguientes afirmaciones es INCORRECTA en relación con esta enfermedad?:

a. La lesión intestinal está mediada por mecanismos inmunológicos. b. Suele insistir un intervalo libre de síntomas entre la introducción del gluten en la dieta y el comienzo de la clímea. c. La determinación de anticuerpos antigliadinaes la prueba serológica más específica en esta enfermedad. d. La infestación por lamblias puede dar un cuadro clínico similar.

666. Chico de trece años que refiere episodios de dolor abdominal no filiados, y artralgias erráticas en codos, rodillas, codos y muñecas. En las últimas 24 horas le han aparecido manchas rojizas en muslos. Lo más destacado de la exploración física es la existencia de púrpura palpable en nalgas y muslos. No presenta anemia, las plaquetas son normales, la IgA está elevada y el aclaramiento de creatinina es normal. Se objetiva proteinuria de 1 gr/24 horas y 50-70 hematíes por campo en la biopsia renal se observa proliferación mesangial y depósitos de IgA (+++) e IgG( +). El diagnóstico más probable es

a. Vasculitis tipo PAN microscópica. b. Lupus eritematoso sistémico. c. Enfermedad de Wegener. d. Síndrome de Schonlein-Henoch.

667. Niño de 10 meses con cuadro febril de 3 días de duración, sin otra sintomatología acompañante salvo irritabilidad con los períodos de hipertemia. El cuarto día presenta aparición de exantema en tronco y desaparición de la fiebre. Respecto al cuadro Clínico citado ¿cual de las siguientes afirmaciones es verdadera?:

a. El diagnóstico más probable es una infección por virus del sarampión. b. El signo físico diagnóstico es la presencia de una amigdalitis exudativa. c. El tratamiento indicado es amoxicilina oraa. d. Se asocia a una infección por virus herpético humano tipo 6.

668. La presencia de alopecia en un niño con raquitismo grave debe hacerle pensar en:

a. Déficit carencial de vitamina D. b. Tubulopatía primaria asociada. c. Déficit de 25 hidroxilasa hepática. d. Déficit funcional del receptor de vitamina D.

669. Un lactante de 15 días de vida presenta ambigüedad genital desde el nacimiento. Comienza con cuadro de vómitos, deshidratación y tendencia al colapso cardiocirculatorio. Desde un punto de vista analítico presenta acidosis metabólica, hiponatremia y natriuresis elevada. ¿Que enfermedad pre-senta este paciente?:

a. Un trastorno de la esteroidogénesis suprarrenaa. b. Una disgenesia gonadal. c. Una estenosis hipertrófica del píloro. d. Una anomalía del receptor androgénico.

670. Niño de 11 meses que a los 2 meses de vida empieza a tener muguet de repetición, diarrea e incapacidad para ganar peso. A los 10 meses tuvo una neumonía por Neumocystis carinii. En la analítica, hipogammaglobulinemia, linfopenia severa con ausencia de linfocitos T y de células NK y elevados linfocitos B. ¿De que diagnóstico se trata?:

a. Síndrome de hiper IgM ligado al cromosoma X. b. Infección por VIH. c. Inmunodeficiencia combinada severa ligada al cromosoma X. d. Síndrome de Wiscott-Aldrich.

671. Un chico de 12 años en ,la fase de crecimiento rápido presenta asimetría de hombros, escápulas y flancos, giba costal al flexionar el tronco, y dorso plano, pero no se aprecia báscula pélvica. El diagnóstico más probable es:

a. Actitud escoliótica secundaria a dismetría de miembros inferiores. b. Escoliosis idiopática del adolescente. c. Escoliosis congénita. d. Escoliosis neuromuscular.

Page 239: Cuestionario de Urología

672. Lactante de tres meses de vida que desde hace un mes presenta episodios intermitentes de distensión abdominal, dolores de tipo cólico y algunos vómitos. Tendencia al estreñimiento. Entre sus antecedentes personales hay que destacar que fue prematuro, pesó 900gr. al nacimiento y tuvo dificultad respiratoria importante que precisó ventilación asistida durante 15 días. ¿Cual es el diagnóstico más probable de su cuadro clínico?:

a. Estenosis colónica secundaria a Enterocolitis necrotizante. b. Megacolon congénito. c. Vólvulo intestinal intermitente. d. Enteritis crónica por rotavirus.

673. Para tratar de establecer una relación causal entre el consumo de benzodiacepinas durante el embarazo y el riesgo de fisura palatina en el recién nacido se seleccionaron madres de recién nacidos con fisura palatina y se compararon con madres de recién nacidos sanos en cuanto a los antecedentes de toma de benzodiacepinas. ¿Cuál es el tipo de diseño de estudio empleado?:

a. Casos y controles. b. Estudio de cohortes. c. Ensayo clínico aleatorizado. d. Estudio ecológico.

674. Un estudio publicado establece una asociación entre la "renta per cápita" de diferentes países y la incidencia de accidentes de tráfico. ¿De qué tipo de diseño de estudio se trata?:

a. Estudio de casos y controles anidado. b. Estudio transversal. c. Estudio de cohortes. d. Estudio ecológico.

675. ¿A qué tipo de estudio corresponde el estudio de Framingham, que empezó en 1949 para identificar factores de riesgo de enfermedad coronaria, en el que de entre 10.000 personas que vivían en Framingham de edades comprendidas entre los 30 y los 59 años, se seleccionó una muestra representativa de ambos sexos, reexaminándose cada dos años en busca de señales de en-fermedad coronaria?:

a. Ensayo clínico b. Estudio de cohortes. c. Estudio de prevalencia. d. Estudio de casos y controles.

676. Indique la afirmación INCORRECTA en relación con los estudios con controles históricos:

a. Existe mayor riesgo de que la información para la evaluación adecuada de la respuesta no esté disponible. b. Puede haber un sesgo en la homogeneidad de los grupos en estudio debido a cambios históricos en los criterios diagnósticos. c. Los estudios con controles históricos tienden a infravalorar la eficacia del fármaco experimental. d. Los estudios con controles históricos necesitarían el mismo número de pacientes que un ensayo concurrente.

677. Desea llevar a cabo el diseño de un ensayo clínico en el que es muy importante que en los dos grupos en comparación la presencia de dos factores concretos de riesgo se distribuyan de igual manera en ambos grupos de estudio. Usted diseñaría un ensayo clínico con:

a. Aleatorización simple. b. Aleatorización por bloques. c. Aleatorización estratificada. d. Aleatorización central.

678. En relación con el análisis e interpretación de los resultados de un ensayo clínico, señale la correcta:

a. La definición de sub grupos de pacientes durante el análisis permiten conocer la eficacia de un fármaco en estos subgrupos con plena validez si existe un error alfa suficientemente bajo. b. Las comparaciones múltiples aumentan la eficiencia de los ensayos clínicos porque aumentan la posibilidad de encontrar diferencias entre los tratamientos en comparación. c. Los ensayos clínicos que no encuentran diferencias entre los tratamientos se deben interpretar como demostración de la igualdad de eficacia entre los mismos. d. El denominado "análisis por protocolo" puede incurrir en sesgos debidos a las pérdidas durante el estudio.

679. En cuanto a los análisis intermedios, en el análisis estadístico de un ensayo clínico, ¿cuál de las siguientes afirmaciones es FALSA?:

a. La realización de múltiples análisis intermedios aumenta el riesgo de cometer un error de tipo 1, es decir, de obtener un resultado falso positivo. b. El número de análisis intermedios que se van a realizar debería estar previsto en el protocolo del estudio. c. Los análisis intermedios están justificados por motivos éticos, económicos y prácticos. d. Se debe hacer el máximo número de análisis intermedios posible para la variable principal porque así se aumenta la probabilidad de obtener un resultado positivo en el ensayo.

680. De las siguientes características ¿cuál es la que mejor define a un ensayo clínico?:

a. Prospectivo. b. Experimental. c. Paralelo. d. Tamaño muestral predefinido.

681. En un estudio en fase ID para evaluar la eficacia bacteriológica, en pacientes con neumonía, de un antibiótico, ¿cuál de los siguientes diseños considera más adecuado?:

a. Secuencial. b. Paralelo, ciego, controlado con placebo.

Page 240: Cuestionario de Urología

c. Paralelo, aleatorizado, controlado con otro antibiótico. d. Cruzado, aleatorizado, ciego, controlado con otro antibiótico.

682. En la evaluación de un ensayo clínico que pretenda comparar la eficacia de un nuevo antiepiléptico en dosis crecientes individualizadas, frente a fenitoina en dosis fijas de 150 mgldía en adultos con crisis generalizadas, ¿cuál es el principal problema?:

a. Relevancia clínica. b. Validez externa. c. Validez interna. d. Reclutamiento.

683. Señale la respuesta correcta de entre las siguientes, referidas a los ensayos clínicos:

a. Cuando en un estudio de Fase III no encontramos diferencias entre el fármaco en estudio y el de control, podemos asumir que son equivalentes. b. Está justificado realizar un ensayo clínico que no resuelva ninguna duda científica, siempre que no haga daño a nadie. c. El error alfa se refiere a la posibilidad de no encontrar diferencias cuando realmente existen. d. Solo es correcto utilizar variables intermedias o sustitutas, cuando estén validadas.

684. El representante de un laboratorio farmacéutico le informa de la reciente comercialización de un nuevo antibiótico para el tratamiento de la neumonía que permite una administración menos frecuente (una vez al día) que los actualmente existentes y le resalta la importancia de cambiar el antibiótico que actualmente utiliza por el que presenta. Como demostración de su eficacia le presenta un ensayo clínico en el que el riesgo relativo (experimental/referencia) en la tasa de curaciones entre ambos es del 0.97 (intervalo de confianza: 0.60-1.30; no significativo). Basándose en estos resultados los autores concluyen que ambos tratamientos tienen una eficacia similar. Su opinión sobre la eficacia de dicho fármaco y la posibilidad de cambiar de antibiótico sería:

a. Los resultados del ensayo y la opinión de los investigadores, que tienen la experiencia con el fármaco, le llevaría a aceptar su conclusión sobre la eficacia del fármaco y a utilizarlo. b. Cree que efectivamente ambos antibióticos pueden considerarse similares a efectos prácticos, ya que la diferencia de eficacia entre ambos (3% de curaciones) es muy pequeña y clínicamente irrelevante. c. La autorización del fármaco por las autoridades es suficiente garantía para aceptar su eficacia y sustituir el antibiótico que estaba utilizando hasta ahora. d. La posible diferencia de eficacia entre ambos tratamientos incluye cifras demasiado amplias para que puedan considerarse equivalentes terapéuticos.

685. ¿Cuál de las siguientes caracterÍSticas del diseño cruzado de un ensayo clínico es FALSA?:

a. Necesita menor número de pacientes que uno paralelo. b. La variabilidad interindividual de la respuesta es menor que en uno paralelo. c. El estudio se encuentra más influido por los abandonos que uno paralelo. d. Se utilizan técnicas estadísticas para datos apareados.

686. Uno de los principales inconvenientes de los estudios de casos y controles es:

a. Que a menudo requieren muestras de gran tamaño. b. Su mayor susceptibilidad a incurrir en sesgos respecto a otros estudios epidemiológicos. c. Que no son adecuados para estudiar enfermedades raras. d. Que suelen ser caros y de larga duración.

687. Para valorar la eficacia de un nuevo tratamiento en el mantenimiento de la remisión en pacientes con un brote moderado o grave de colitis ulcerosa, se ha diseñado un estudio cuya variable principal es la recaída de enfermedad un año después de iniciado el tratamiento. Si el estado actual del conocimiento de esta enfermedad indica la existencia de un tratamiento eficaz en esta indicación, señale cuál es el diseño ética y metodológicamente más adecuado para responder a la pregunta de investigación planteada:

a. Estudio de casos y controles (casos constituidos por enfermos nuevos, controles constituidos por pacientes históricos recogidos del archivo de historias clínicas). b. Ensayo clínico de distribución aleatoria con dos grupos que comparan el fármaco experimental con placebo. c. Estudio de cohortes anterógrado en el que se analiza la evolución de la enfermedad en dos grupos de pacientes tratados respectivamente con el tratamiento reconocido y el tratamiento experimental, sin intervención por parte del investigador en la distribución de los pacientes a cada grupo. d. Ensayo clínico de distribución aleatoria con dos grupos en los que se compara el tratamiento experimental con el tratamiento eficaz previamente reconocido.

688. En un estudio de casos y controles se encontró una asociación entre la ingesta de alcohol y el desarrollo de cáncer esofágico. Dado que el consumo de tabaco se encontró asociado tanto a la ingesta de alcohol como al desarrollo de cáncer de esófago, el consumo de tabaco en este estudio debe considerarse como:

a. Un factor (sesgo) de confusión. b. Un sesgo de información. c. Un sesgo de memoria. d. Un sesgo de selección.

689. El sistema español de farmacovigilancia ha recibido varias notificaciones espontáneas de lesiones hepatocelulares, algunas de ellas graves, asociadas a un medicamento que lleva dos años comercializado. Durante la realización de ensayos clínicos antes de comercializarlo no se describió ningún caso, ¿Cuál de las afirmaciones siguientes cree que es correcta ante esta situación?:

a. Hay que evaluar la posibilidad de una alerta, por lo que hay que conocer la cantidad de medicamento que se ha vendido, como aproximación al número de pacientes que han podido estar expuestos. b. La existencia de una notificación adversa grave, ya es motivo suficiente para retirar el fármaco del mercado. c. El que se hayan recibido más de un caso implica que existe una fuerte asociación causa-efecto entre la administración del medicamento y la hepatitis. d. La asociación es debida al azar, ya que la hepatitis no se detectó durante la realización de los ensayos clínicos.

Page 241: Cuestionario de Urología

690. Un investigador está interesado en determinar si existe una asociación entre las cifras de tensión arterial diastólica (medida en mm de Hg) y los niveles de colesterol (medidos en mgr/mI). Para ello, ha realizado estas mediciones a 230 voluntarios. ¿Qué prueba estadística es la más apropiada para examinar esta asociación?:

a. Regresión logística. b. Prueba de la t c. Prueba de chi cuadrado. d. Correlación de Pearson.

691. Un investigador desea determinar la prevalencia de tabaquismo en niños de 12 años en una zona urbana mediante un estudio descriptivo. Para calcular el tamaño muestral que necesita para su estudio ya posee los siguientes datos: tamaño de la población de 12 años, porcentaje de pérdidas, la precisión con la que desea dar la prevalencia que obtendrá (por ejemplo 5%) y ha seleccionado un nivel de confianza (por ejemplo 95%). ¿Qué otro dato le falta?:

a. Estimar el error beta. b. Estimar la proporción esperada de tabaquismo. c. Estimar la desviación estándar del tabaquismo. d. Estimar el erroralfa.

692. Se pretende comparar la frecuencia de complicaciones de dos preparados distintos de un mismo fármaco. Se observó un 5 % de complicaciones con un preparado y un 3 % con el otro, siendo esta diferencia estadísticamente significativa (p=O,045). La interpretación correcta de este resultado es:

a. Si ambos preparados tuvieran la misma frecuencia de complicaciones, la probabilidad de encontrar una diferencia igual o mayor a la observada es 0,045. b. La probabilidad de que ambos preparados tengan la misma frecuencia de complicaciones es de 0,045. c. Los dos preparados tienen distinta frecuencia de complicaciones. d. Los dos preparados no tienen la misma frecuencia de complicaciones.

693. En un ensayo clínico se comparan 3 tratamientos (p.e. placebo, tratamiento establecido y un tratamiento nuevo). La variable respuesta es contínua (p.e. nivel de glucosa en sangre). Aceptando que la variable tiene una distribución normal, el test correcto para comparar la respuesta es:

a. La t de Student. b. El test de Wilcoxon. c. Análisis de la varianza. d. El test de Kruskal-Wallis.

694. En el Análisis Coste Beneficio aplicado a la evaluación económica de las tecnologías sanitarias. ¿En qué términos o clase de unidades vienen expresados los resultados del empleo de tales tecnologías?:

a. Calidades de vida. b. Años de vida ajustados por calidad. c. Físicas. d. Monetarias.

695. ¿Cuál de entre los siguientes estudios de evaluación económica de tecnologías sanitarias es el que trata de determinar la alternativa más eficiente, considerando solo los costes de las mismas, porque la diferencia de sus resultados no es clínicamente relevante, ni estadísticamente significativa?:

a. De impacto presupuestario. b. Costes de la enfermedad. c. Análisis de costes. d. Análisis de minimización de costes.

696. Con respecto a los ensayos clínicos en fase ll, señale la respuesta INCORRECTA:

a. También se llaman exploratorios porque su objetivo principal es explorar la eficacia terapéutica de un producto en pacientes para la indicación determinada. b. Un objetivo importante de estos ensayos clínicos es determinar la dosis y la pauta posológica para los estudios en fase III o confimatorios. c. En estos ensayos se administra el medicamento por primera vez en humanos. d. La escalada de dosis es un diseño habitual en los ensayos en fase II.

697. Los Grupos Relacionados con el Diagnóstico (GRDs) son un Sistema de Clasificación de Pacientes utilizados en gestión hospitalaria que tienen como principal criterio de clasificación de los pacientes:

a. La igualdad de síntomas (iso-síntomas) b. El conjunto de variables que pueden determinar diferencias entre pacientes con un mismo diagnóstico principal y que caracterizan los diversos procesos con un mismo diagnóstico (iso-enfermedad). c. Iso-complejidad de los diagnósticos al alta hospitalaria. d. Iso consumo de recursos.

698. Todos los siguientes son grupos que deben inmunizarse anualmente con la vacuna de la gripe SALVO:

a. Niños y adolescentes (6 meses a 18 años) en tratamiento crónico con aspirina. b. Adultos y niños con dolencias crónicas pulmonares cardiovasculares (incluido el asma) c. Médicos enfermeros y otro personal sanitario. d. Mujeres en el primer trimestre del embarazo durante la época epidémica de gripe.

699. Acude a la consulta de su médico de familia una mujer de 54 años por cuadro .catarral. Aprovecha la visita para conocer su opinión sobre la necesidad de un chequeo general y sobre la necesidad de realizar revisiones ginecológicas. Entre sus antecedentes personales destacan 3 gestaciones con partos eutócicos, menopausia hace dos años sin sangrado posterior. No fuma, no bebe, ni lo ha hecho nunca, no toma ningún medicamento, ni presenta ninguna patología activa o crónica conocida. No antecedentes familiares de interés. Última revisión ginecológica hace 18 años. ¿Qué actividades preventivas son las más recomendadas de inicio en esta mujer?:

Page 242: Cuestionario de Urología

a. Se trata de una mujer sana y por lo tanto no hay que hacer nada. b. Citología vaginal cada 3 años, indicar búsqueda de atención médica si sangrado vaginal y mamografía bienal. c. Cribado de HTA, hipercolesterolemia, obesidad, explorar actividad física y dieta, vacunación de difteria y tétanos o dosis de recuerdo cada diez años, citología vaginal anual en 2 años consecutivos, indicar búsqueda de atención médica si sangrado vaginal y mamografía bienal. d. Anualmente medición de TA, de colesterol total, perfil tifoideo, densitometría, peso, talla, !MC, radiografía de tórax, explorar actividad física y dieta, vacunación de difteria y tétanos y dosis de recuerdo cada 10 años, citología vaginal anual en 2 años consecutivos, indicar búsqueda de atención médica si sangrado vaginal y mamografía bienal.

700. ¿Cuál de los siguientes fármacos puede provocar un cuadro de intoxicación de mayor gravedad?:

a. Amitriptilina. b. Fluvoxamina. c. Venlafaxina. d. Sertralina.

701. ¿Cuál de los siguientes fármacos NO se utiliza en la actualidad como terapia inmunosupresora a largo plazo en el trasplante cardíaco?:

a. Ciclosporina. b. Micofenolato Mofetilo. c. Azatioprina. d. Anticuerpos Monoclonares OKT3.

702. ¿Cuál de las siguientes circunstancias NO condiciona un incremento del riesgo de toxicidad renal por aminoglucósidos?:

a. Uso simultáneo de Furosemida. b. Uso simultáneo de Antiinflamatorios no esteroideos. c. Hipovolemia. d. Uso simultáneo de Fluoxetina.

703. ¿Cuál de los siguientes fármacos NO aumenta los niveles plasmáticos de potasio?:

a. Captopril. b. Ibuprofeno. c. Amiloride. d. Nifedipino.

704. Uno de los siguientes fármacos debe su eficacia terapéutica en el tratamiento de la hipertrofia benigna de próstata a la inhibición directa del enzima 5 α-Reductasa. Indíquelo:

a. Etinilestradiol. b. Finasterida. c. Alfuzoxina. d. Leuprolide.

705. Todos los signientes virus tienen como célula diana la que se indica en cada caso, SALVO uno. Indíquela:

a. El virus de Epstein Barr los linfocitos B. b. Los rinovirus las células epiteliales (receptor ICAM-l, proteína de adherencia de la superfamilia de las inmunoglobulinas). c. El virus de la rabia las neuronas (receptor de acetilcolina). d. Los reovirus las células epiteliales (receptor sialil oligosacáridos).

706. Una de las siguientes afirmaciones referidas al.lipopolisacárido (LPS) de la membrana externa de la pared celular de las bacterias gram negativas es cierta:

a. Es una toxina termolábil que no resiste la esterilización en autoclave. b. Tiene una actividad endotóxica que está asociada con el lípido A. c. Contiene el antígeno O que es esencial para la viabilidad celular. d. Su concentración en sangre no está directamente relacionada con la mortalidad por shock irreversible y colapso cardiovascular.

707. Paciente politraumatizado ingresado en UCI. En un absceso cutáneo y en tres muestas de hemocultivo se aisla una cepa de Staphyiococcus aureus resistente a meticilina. Señale el tratamiento antimicrobiano de elección:

a. Amoxicilinalacido clavulánico. b. Cefotaxima. c. Ceftazidima. d. Vancomicina.

708. La diferencia entre Entamoeba histoiytica y Entamoeba dispar (patógena y comensal respectivamente) se hace en los laboratorios de microbiología clínica por:

a. Estudio de las diferencias morfológicas (microscópica) con ayuda de una tinción permanente. b. Estudio de las diferencias metabólicas (Galeria metabóliea). c. Estudio de sus diferencias antigénicas mediante pruebas inmunológicas. d. Estudio de las características diferenciales de la movilidad (pseudópodos).

709. Señala cuál de los siguientes hechos NO forma parte de la primera fase de la cascada metastásica de los tumores malignos:

a. La fijación de las células neoplásicas a la laminina y fibronectina de la matriz extracelular. b. La pérdida de adherencia entre las células neoplásicas. c. La neoangiogénesis de vasos linfáticos en el campo de desarrollo del tumor. d. La migración de las células tumorales tras de la degrada ción de la membrana.

710. La triada: alteración espongiforme de las neuronas con aspecto microvacuolado de la sustancia gris, astrogliosis y placas

Page 243: Cuestionario de Urología

amiloides se presenta en una de las siguientes entidades:

a. Enfermedad de Parkinson. b. Enfermedad de Alzheimer. c. Enfermedad de Pick. d. Enfermedad de Creutzfeldt-Jakob.

711. La necrosis coagulativa es característica de:

a. Infección bacteriana. b. Hipoxia-isquemia celular. c. Citolisislisosómica. d. Traumatismos.

712. De las siguientes enfermedades, ¿cuál es la que con mayor frecuencia produce ulceraciones múltiples en las primeras porciones (duo-deno-yeyuno) del intestino delgado?:

a. Antiinflamatorios no-esteroideos (AINES). b. Enfermedad de Crohn. c. Tuberculosis intestinal. d. Ingesta de sustancias cáusticas.

713. Una mujer de mediana edad presenta diarrea acuosa crónica, sin sangre. La colonoscopia muestra mucosa normal. ¿Qué hallazgo morfológico, entre los siguientes, cabe esperar en la biopsia?:

a. Erosiones superficiales. b. Grupos de células epitelioides intramucosos. c. Engrosamiento colagénico de la membrana basal bajo el epitelio de superficie. d. Atrofia glandular con pérdida de la capacidad mucígena y microabscesos crípticos.

714. Las nefronas del riñón post-natal derivan del:

a. Pronefros. b. Mesonefros. c. Metanefros. d. Conducto de Wolff.

715. En relación con la disposición de los elementos vásculo-biliares en el ligamento hepatoduodenal, refiera cuál de las afirmaciones siguientes es la correcta:

a. La vena porta sigue un trayecto anteromedial con respecto a la arteria hepática. b. La arteria hepática derecha se divide en dos ramas que abrazan el conducto hepático derecho o el conducto cístico. c. La vía biliar principial es postero-lateral derecha con respecto a la vena porta. d. La arteria hepática izquierda nace de la arteria coronaria o de la arteria gastroduodenal.

716. ¿Qué músculo, componente del cuádriceps crural, se inserta como tendón rotuliano en la tuberosidad anterior de la tibia?:

a. El vasto interno. b. El vasto intermedio o músculo crural. c. El recto anterior. d. El vasto externo.

718. El nervio laríngeo recurrente derecho, punto de referelicia importante en la Cirugía de la glándula tiroides, pasa por debajo de una de las siguientes arterias:

a. Arteria cervical transversa. b. Arteria tifOidea superior. c. Arteria tifOidea ina. d. Arteria subclavia derecha.

719. En el momento actual, una de las herramientas principipales en los estudios inmunológicos es la proteómica. Señale cuál de las respuestas siguientes es cierta con respecto a esta técnica:

a. Estudia los genes a nivel molecular. b. Está basada en la utilización de la transcriptasa reversa. c. Se apoya en el uso de la reacción en cadena de la polimerasa pero de una manera semicuantitativa. d. Es el análisis de las proteínas y utiliza el espectrómetro de masas.

720. Señale cuál de las siguientes respuestas es cierta en relación a las células madre:

a. Las únicas caracterizadas son las células madre hematopoyéticas. Se aislan mediante su molécula de membrana CD34 y sirven para el tratamiento de los tumores hematológicos. b. Su aislamiento sólo es posible a partir de tejido fetal o del cordón umbilical. c. Son células pluripotenciales, son capaces de diferenciarse en distintos tipos de tejido y/o estirpes celulares. d. El proceso para obtenerlas es muy laborioso, dado que no son capaces de crecer en cultivos in vitro.

721. En el desarrollo actual de nuevos agentes inmnnomoduladores para las enfermedades autoinmunes y los cuadros tumorales, se estudian fármacos cuya diana es inmunológica. Señale cuál de los siguientes NO es una diana inmunológica de este tipo de terapia en la actualidad:

a. Enzima tirosina kinasa. b. Factor de necrosis tumoral alfa. c. lnterleukina-l. d. Interleukina-l0.

722. Señala la respuesta INCORRECTA en relación a la respuesta inmunológica del linfocito T:

Page 244: Cuestionario de Urología

a. El receptor de antígeno del linfocito T es el encargado de reconocer el péptido extraño en el contexto de moléculas del complejo mayor de histocompatibilidad tanto de clase I como de clase II. b. Si el reconocimiento de antígeno se produce en ausencia de una señal coestimuladora se desencadena un fenómeno de tolerancia inmunológica. c. Puede conseguirse una tolerancia activa del linfocito T a través del bloqueo de la vía B7/CD28. d. Las células dendríticas de Langerhans son una subpoblación de células T activadas presentes en la piea.

723. Señale cuál de las siguientes respuestas es cierta en relación a un superantígeno:

a. Se unen a la porción lateral de la cadena alfa del receptor de la célula T y a la cadena beta de las moléculas del complejo mayor de histocompatibilidad de clase il. b. Son moléculas de naturaleza proteíca capaces de activar hasta un 20% de los linfocitos T de sangre periférica. c. Están implicados en el desencadenamiento del síndrome del aceite tóxico. d. En su mayor parte son agentes nutricionales.

724. Todas las substancias que a continuación se enumeran tienen la acción fisiológica que se indica en cada caso SALVO una. Señálela:

a. Endotelina - vasoconstricción. b. Péptido intestinal vasoactivo - vasodilatación. c. Serotonina - vasoconstricción. d. Oxido nítrico - vasoconstricción.

725. ¿A nivel de bioquímica molecular, qué se entiende por mecanismo de acción indirecto de las radiaciones ionizantes?:

a. La alteración de la estructura terciaria de las proteínas. b. La formación de radicales libres, capaces de reaccionar con moléculas biológicamente importantes. c. La destrucción del RNA mensajero. d. La lesión directa de la molécula de DNA.

726. En relación con la composición de las tipo-proteínas, indique la respuesta correcta:

a. La Apo B100 es un componente de los quilomicrones. b. La Apo AI es componente principal de las LDL. c. La Apo E se encuentra en los quilomicrones, VLDL, IDL yHDL. d. La Apo B48 es el componente principal de las HDL.

727. En las reacciones fisiológicas de la cascada de la coagulación, una de las respuestas siguientes es INCORRECTA:

a. El factor X se activa únicamente por el factor VIII activado. b. El factor VIII se activa a través de la fase intrínseca o de contacto de la coagulación. c. El factor VII se activa por el factor tisular. d. El factor V es inactivado por el sistema proteína C proteína S.

728. En relación con los mecanismos de absorción intestinal, una de las siguientes respuestas es INCORRECTA:

a. La absorción de hierro tiene lugar en la parte más proximal del intestino delgado. b. La absorción de vitamina B12 requiere una sustancia segregada en las células parietales del estómago. c. La absorción adecuada de calcio requiere la presencia de vitamina D. d. Los ácidos grasos de cadena media requieren la lipólisis pancreática.

729. Señale la respuesta FALSA en relación con el H. Pylori:

a. Es, junto con el consumo de AlNEs, el factor etiológico de la mayoría de las úlceras gástricas y duodenales. b. La prueba de detección más sensible y específica es la serología. c. No parece tener un papel esencial en la enfermedad por reflujo gastroesofágico. d. La erradicación del H. Pylori en pacientes con ulcus péptico se asocia a una drástica reducción de recaídas ulcerosas, en comparación con el tratamiento antisecretor.

730. ¿Cuál de las siguientes asociaciones referidas a la exploración cardiovascular NO es correcta?

a. Onda a del pulso yugular - Cuarto ruido. b. Clicks de apertura - Protosístole. c. Arrastre presistólico - Ritmo sinusal. d. Aumento de intensidad del soplo con inspi-ración - Origen izquierdo.

731. ¿Cuál de las siguientes entidades NO se acompaña de eosinofIlia?:

a. Asma intrínseca. b. Infectaciones por helmintos. c. Enfermedad de Hodgkin. d. Fiebre tifoidea.

732. Respecto a las fIlariosis, señale la respuesta INCORRECTA:

a. Se transmiten por invasión directa de larvas parasitarias a la piel desde tierras húmedas al andar descalzo. b. Onchocerca volvulus produce nódulos subcutáneos, prurito y afectación ocular (queratitis, retinitis). c. Loa Loa produce edemas transitorios subcutáneos y conjuntivitis. d. Wuchereria bancrofti produce varicocele y faringitis.

733. En la cuarta edición del DSM-IV la demencia se defme como:

a. Una alteración de la conciencia y de la cognición por un breve período de tiempo b. Un deterioro de múltiples funciones cognoscitivas, incluida la alteración de la memoria, pero no de la conciencia. c. Un deterioro progresivo de la memoria que puede desarrollarse en ausencia de otros trastornos significativos. d. Un trastorno neuropsiquiátrico complejo de índole orgánica, que incluye deterioro, delirio y alucinaciones.

Page 245: Cuestionario de Urología

734. Un estudio informa qne la mediana de supervivencia de los pacientes sometidos a cierta intervención quirúrgica es de 7 años. Ello quiere decir que:

a. El valor esperado de tiempo de supervivencia es 7 años. b. La mitad de los pacientes sobreviven más de 7 años. c. No hay ningún paciente que sobreviva menos de 7 años. d. La mitad de los pacientes sobreviven aproximadamente 7 años.

735. ¿Cuál de los siguientes cánceres invasivos es el más frecuente en España?:

a. El cáncer de endometrio. b. El cáncer de ovario. c. El cáncer de cérvix. d. El cáncer de vagina.

736. Según las pautas de prática clínica generalmente aceptadas, la Oxigenoterapia Crónica domiciliaria, está indicada en pacientes con enfermedad pulmonar obstructiva crónica (EPOC) cuya presión parcial arterial de oxí-geno (PaO2) sea:

a. Inferior a 55 mmHg en situación clínica aguda. b. Entre 55 y 60 mmHg en sirtuación clínica aguda, pero con historia de agravarnientos frecuentes más severos, por debajo de 55mmHg. c. Inferior a 55 mmHg en situación clínica estable. d. Entre 55 y 60 mmHg en situación clínica estable, pero con historia de agravamientos frecuentes más severos, por debajo de 55 mmHg.

737. Un paciente varón de 60 años acude al hospital aquejando hematuria indolora. La exploración demuestra una tumoración vesical de cara lateral derecha sin afectación del meato ureteral que se reseca endoscópicamente. Señale cuáles de los siguientes datos tienen la significación pronóstica más relevante y deciden el tratamiento:

a. El grado, el tipo histológico y el estadiaje o nivel infiltrativo local. b. El tipo histológico y su asociación con áreas de cistitis y de carcinoma in situ. c. El sexo y edad del paciente y su asociación, en varones, con hiperplasia prostática benigna con obstrucción urinaria. d. El tipo histológico y la duración de la hematuria.

738. No podrá Vd. normalizar la saturación arterial de oxígeno mediante oxigenoterapia si un post-operado presenta:

a. Anemia importante. b. Crisis asmática severa con hipoxemia. c. Síndrome de distress respiratorio del adulto. d. Tres costillas fracturadas con movilidad respiratoria restringida.